You are on page 1of 348

Danh Sách 35 Đề Thi Học Sinh Giỏi môn Tiếng Anh Lớp 12 Có Lời Giải Chi Tiết

1. Đề thi HSG môn Tiếng Anh lớp 12 cấp tỉnh năm 2020-2021 có đáp án - Sở GD&ĐT Quảng Trị
2. Đề thi HSG môn Tiếng Anh lớp 12 cấp tỉnh năm 2020-2021 có đáp án - Tp HCM
3. Đề thi HSG môn Tiếng Anh lớp 12 cấp tỉnh năm 2020-2021 có đáp án - Sở GD&ĐT Bắc Ninh
4. Đề thi HSG môn Tiếng Anh lớp 12 cấp tỉnh năm 2020-2021 có đáp án - Sở GD&ĐT Bạc Liêu
5. Đề thi HSG môn Tiếng Anh lớp 12 cấp tỉnh năm 2020-2021 có đáp án - Sở GD&ĐT Bắc Ninh
6. Đề thi HSG môn Tiếng Anh lớp 12 cấp tỉnh năm 2020-2021 có đáp án - Sở GD&ĐT Vĩnh Phúc
7. Đề thi HSG môn Tiếng Anh lớp 12 cấp tỉnh năm 2020-2021 có đáp án - Sở GD&ĐT Hải Dương
8. Đề thi HSG môn Tiếng Anh lớp 12 cấp tỉnh năm 2020-2021 có đáp án - Sở GD&ĐT Thanh Hóa
9. Đề thi HSG môn Tiếng Anh Quốc Gia năm 2020-2021 có đáp án
10. Đề thi HSG môn Tiếng Anh lớp 12 cấp tỉnh năm 2020-2021 có đáp án - Trường THPT Quế Võ số 1
11. Đề thi HSG môn Tiếng Anh lớp 12 cấp tỉnh năm 2020-2021 có đáp án - Sở GD&ĐT Quảng Nam
12. Đề thi HSG môn Tiếng Anh lớp 12 cấp tỉnh năm 2020-2021 có đáp án - Sở GD&ĐT Đồng Nai

13. Đề thi HSG môn Tiếng Anh lớp 12 cấp tỉnh năm 2020có đáp án - Tỉnh Quảng Nam
14. Đề thi chọn HSG môn Tiếng Anh lớp 12 cấp tỉnh năm 2020có đáp án - Tp Hồ Chí Minh
15. Đề Thi KSCL HSG môn Tiếng Anh lớp 12 cấp tỉnh năm 2020có đáp án -Trường Nông Công I

16. Đề thi HSG môn Tiếng Anh lớp 12 cấp tỉnh năm 2019-2020 có đáp án - trường Đồng Đậu – Vĩnh Phúc
lần 1
17. Đề thi HSG môn Tiếng Anh lớp 12 cấp tỉnh năm 2019-2020 có đáp án - trường Đồng Đậu – Vĩnh Phúc
lần 2
18. Đề thi HSG môn Tiếng Anh lớp 12 cấp tỉnh năm 2020 có đáp án - Tỉnh Quảng Nam
19. Đề Thi HSG môn Tiếng Anh lớp 12 cấp tỉnh năm 2019-2020 có đáp án - Chuyên Quảng Nam
20. Đề Thi HSG môn Tiếng Anh lớp 12 cấp tỉnh năm 2019-2020 có đáp án - Trường THPT Liễn Sơn
21. Đề thi HSG môn Tiếng Anh lớp 12 cấp tỉnh năm 2019-2020 có đáp án - Sở GD&ĐT Hải Phòng
22. Đề thi HSG môn Tiếng Anh lớp 12 cấp tỉnh năm 2019-2020 có đáp án - Sở GD&ĐT Hải Dương
23. Đề thi HSG môn Tiếng Anh lớp 12 cấp tỉnh năm 2019-2020 có đáp án - Trường THPT Yên Lạc 2, Vĩnh
Phúc

24. Đề thi HSG môn Tiếng Anh lớp 12 cấp tỉnh năm 2018-2019 có đáp án - Sở GD&ĐT Quảng Nam
25. Đề Thi HSG môn Tiếng Anh lớp 12 cấp tỉnh năm 2019 có đáp án - Chuyên Quảng Nam
26. Đề Thi HSG môn Tiếng Anh lớp 12 cấp tỉnh có đáp án – Đề 1
27. Đề Thi HSG môn Tiếng Anh lớp 12 cấp tỉnh có đáp án – Đề 2
28. Đề thi HSG môn Tiếng Anh lớp 12 cấp tỉnh năm 2018-2019 có đáp án - trường Lưu Nhân Chú – Thái
Nguyên
29. Đề Thi Chọn HSG môn Tiếng Anh lớp 12 cấp tỉnh năm 2018-2019 có đáp án - Trường THPT Liễn Sơn
30. Đề Thi Chọn HSG môn Tiếng Anh lớp 12 cấp tỉnh năm 2018-2019 có đáp án - Trường Chuyên Vĩnh Phúc

31. Đề Thi HSG môn Tiếng Anh lớp 12 cấp tỉnh năm 2017-2018 có đáp án - Tỉnh Quảng Nam
32. Đề Thi HSG môn Tiếng Anh lớp 12 cấp tỉnh năm 2017-2018 có đáp án - Trường THPT Liễn Sơn
33. Đề thi HSG môn Tiếng Anh lớp 12 cấp tỉnh năm 2017-2018 có đáp án - Tỉnh Quảng Bình
34. Đề Thi Chọn HSG môn Tiếng Anh lớp 12 cấp tỉnh năm 2017-2018 có đáp án - TP Hà Nội
35. Đề thi Môn Tiếng Anh 12 Năm 2017-2018 Có Đáp Án – Tỉnh Nam Định
SỞ GIÁO DỤC VÀ ĐÀO TẠO KỲ THI HỌC SINH GIỎI VĂN HOÁ LỚP 12 THPT
QUẢNG TRỊ Khóa ngày 06 tháng 10 năm 2020
Môn thi: TIẾNG ANH
ĐỀ THI CHÍNH THỨC
Thời gian làm bài: 180 phút (không kể thời gian giao đề)

(Đề thi này gồm 11 trang)

Họ và tên học sinh: Số báo danh

Ngày sinh:
Phòng thi số:
Hội đồng thi:
Họ, tên và chữ ký Họ, tên và chữ ký Mã phách
GIÁM THỊ 1 GIÁM THỊ 2 (Thí sinh không viết vào ô này)

HƯỚNG DẪN THÍ SINH L ÀM BÀI:

● Thí sinh làm toàn bộ bài thi trên đề thi theo yêu cầu của từng phần. Thí sinh phải
viết câu trả lời vào phần trả lời được cho sẵn ở mỗi phần (Your answers here). Trái
với điều này, phần bài làm của thí sinh sẽ không được chấm điểm.

● Đề thi gồm có 11 trang (không kể trang phách). Thí sinh phải kiểm tra số tờ đề thi trước khi làm
bài.

● Phần NGHE, mỗi Part thí sinh được nghe 2 lần.

● Thí sinh không được ký tên hoặc dùng bất cứ dấu hiệu gì để đánh dấu bài thi ngoài việc
làm bài theo yêu cầu của đề ra. Không được viết bằng mực đỏ, bút chì, không viết
hai thứ mực trên tờ giấy làm bài. Phần viết hỏng, ngoài cách dùng thước để gạch
chéo, không được tẩy xóa bằng bất kỳ cách gì khác (kể cả bút xóa màu trắng). Trái với
điều này bài thi sẽ bị loại.

● Thí sinh nên làm nháp trước rồi ghi chép cẩn thận vào phần bài làm trên đề thi. Giám thị sẽ
không phát giấy làm bài thay thế đề và giấy làm bài do thí sinh làm hỏng.

● Giám thị không giải thích gì thêm về đề thi.


SECTION I: LISTENING COMPREHENSION (5 POINTS)
Part 1: Listen to a conversation and complete the form below with NO MORE THAN TWO
WORDS and/or A NUMBER for each answer. Write your answers in the corresponding
numbered boxes.
Student Union Registration Form
Name : Stefan Unger
Degree programme: (1) ______________________
Department : (2) ______________________
Leisure activities: (3) ______________________
Language(s) (apart from English): (4) ______________________
Type of accommodation: (5) ______________________
Contact number: 0295069003
Part 2: Listen to part of a radio programme. Decide which of the sentences are true (T), and which
are false (F) according to the recording. Write your answers in the corresponding numbered boxes.
1. Everybody in the Outback has at least a telephone to keep in touch with the world.
2. In an emergency, doctors use jeeps to get to the patient.
3. It doesn’t take the doctors more than ninety minutes to get to any place in the Outback.
4. Children living in the Outback do not go to school at all.
5. The children in the Outback can communicate with their teachers by two- way radio and post.
Part 3: Listen to an interview with a woman called Amy Rowntree, who works as a fashion designer
and choose the correct option (A, B, or C). Write your answers in the corresponding numbered boxes.
1. Amy decided to become a fashion designer while ________.
A. she was working in a shop
B. she was still at school
C. she was helping run a fashion show
2. Amy says the most important skill for a designer is ________.
A. remaining patient in difficult situations
B. understanding how clothes are made
C. knowing about different materials
3. What does Amy particularly like about her work?
A. being able to sell her designs
B. having her name recognised
C. seeing her clothes on display
4. What helps Amy find new ideas for designs?
A. going to visit museums
B. looking at photos of clothes
C. seeing things around her
5. What project is Amy working on at the moment?
A. a fashion show in New York
B. a collection of clothes for a film
C. a new range for a London store
Part 4: Listen to a lecture about student orientation and complete the table below.
Write NO MORE THAN THREE WORDS for each answer. Write your answers in the
corresponding numbered boxes.

Orientation Schedule
Monday
Part I: Campus Tour meet at door of (1) __________________ at 9 a.m.
visit (2) ___________________ in the afternoon.
Tuesday
Computer Centre: to get (3) ____________________
(4) ____________________: to apply for membership
Wednesday
Part meet at Auditorium in the west campus (5)
II: Course Arrangement _______________
course requirement
(6) ___________________ and optional courses
assessments:
(7) ___________________ 80%
assignment
(8) ___________________ group work
exam: open-book
Thursday
go to department office to:
get a (9) ____________________
hand in optional course form
Part III: Welcome Party Friday Time: 5 p.m.
Venue: (10) _____________________ on the third floor

Your answers here


Part 1:
1. 2. 3. 4. 5.

Part 2:
1. 2. 3. 4. 5.

Part 3:
1. 2. 3. 4. 5.

Part 4:
1. 2. 3.

4. 5. 6.

7. 8. 9. 10.
SECTION II: LEXICAL & GRAMMAR (3.5 POINTS)
Part 1: Choose the best answer (A, B, C, or D) to each of the following questions and write your
answers in the corresponding numbered boxes.
1. If Jack had done his homework last night, he ______ a bad mark now.
A. wouldn’t get B. won’t get C. wouldn’t have got D. would get
2. It is necessary that Dave _______ to see the doctor right now.
A. come B. comes C. to come D. is coming
3. Everyone hardly finds out a quiet place here, ______?
A. don’t they B. do they C. does she D. doesn’t she
4. Even when John was angry, he would never _______ to violence.
A. resolve B. recourse C. exert D. resort
5. Tom’s parents were really pleased when they read his school ______.
A. report B. paper C. diploma D. account
6. Susan says that she feels less nervous since she ______ on tea and coffee.
A. stopped off B. turned back C. cut out D. cut down
7. Mike: What a nice hat you have on! ~ Paul: ______
A. You’re welcome. B. Yes, you like it?
C. Oh, you must be kidding. D. Thanks. I bought it at Kelly’s.
8. This room is decorated in a _____ combination of colours.
A. delicious B. sweet C. tasteful D. tasty
9. My decision to drop out of university after a year is the one I now ______ regret.
A. keenly B. painfully C. heavily D. harshly
10. It was decided that the cost of the project would be ______ and so it was abandoned.
A. restrictive B. prohibitive C. repressive D. exclusive
Your answers here
1. 2. 3. 4. 5.
6. 7. 8. 9. 10.

Part 2: Identify 10 mistakes in the following passage in any order and correct them. (0) in Line 1
has been done as an example. Write your answers in the corresponding numbered boxes.
Line 1 An endangered species is a population of an organism who is at risk of
Line 2 becoming extinct because it is either few in numbers, or threatened by
Line 3 changed environmental or predation parameters. An endangered species
Line 4 is usually a taxonomic species, but may be other evolutionary significant
Line 5 unit, The World Conservation Union (IPCN) has calculated the percent
Line 6 of endangered species as 40 percent of all organisms based at the
Line 7 samples of species that have been evaluating through 2006. Many
Line 8 nations have laws offering protection to this species: for example,
Line 9 forbidding hunting, restricting land development or create reserves.
Line 10 Only a few of the many species at risk of extinct actually make it to the
Line 11 lists and obtain legal protection. Much more species become extinct, or
Line 12 potential will become extinct, without gaining public notice.
Your answers here
Line Mistake Correction Line Mistake Correction
Line 1 (0) who which
1. 6.
2. 7.
3. 8.
4. 9.
5. 10.

Part 3: Complete each of the following sentences with one appropriate preposition/particle. Write
your answers in the corresponding numbered boxes.
1. Fred is really mature. He is completely independent ____________ his parents.
2. Make ____________ your mind now or we will go without you.
3. Kelly must be responsible ____________ the director for what she has just said.
4. Nowadays many people have been suffering ____________ cancer.
5. He really gets ____________ my nerves. He never stops complaining.
6. No student can answer the professor’s question because it is too hard to take ____________.
7. Sofia worked very hard and her success went ____________ my expectation.
Your answers here
1. 2. 3. 4. 5. 6. 7.

Part 4: Supply the correct form of the words given in CAPITALS to complete the sentences. Write
your answers in the corresponding numbered boxes.
Meditation
People are often put off meditation by what they see as its many mystical associations. Yet
meditation is a (1) ______ (STRAIGHT) technique which merely involves sitting and resting the
mind. In addition to its (2) ______ (SIMPLE), meditation offers powerful help in the battle against
stress. Hundreds of studies have shown that meditation, when (3) ______ (TAKE) in a principled
way, can reduce hypertension which is related to stress in the body. Research has proved that certain
types of meditation can substantially decrease key stress symptoms such as anxiety and (4) ______
(IRRITABLE). In fact, those who practice meditation with any (5) ______ (REGULAR) see their
doctors less and spend, on average, seventy percent fewer days in hospital. They are said to have
more stamina, a happier disposition and even enjoy better relationships.
When you learn to meditate, your teacher will give you a (6) ______ (PERSON) ‘mantra’ or
word which you use every time you practice the technique and which is (7) ______ (SUPPOSE)
chosen according to your needs. Initial classes are taught individually but subsequent classes usually
consist of a group of students and take place over a period of about four days. The aim is to learn how
to slip into a deeper state of (8) ______ (CONSCIOUS) for twenty minutes a day. The rewards speak
for themselves.
Your answers here
1. 2. 3. 4.
5. 6. 7. 8.
SECTION III: READING COMPREHENSION (5 POINTS)
Part 1: Read the text below and think of ONE word which best fits each gap. Write your answers in
the corresponding numbered boxes.
Always a sure source of affection, my grandparents (1) _______hugely important figures in
my life. They would shower my sisters and me with sweets, indulgences and stories, (2) _______tales
about my parents as naughty children. When the last of (3) _______ died, we all wondered who
would hold the family together.
People have relied on grandparents in Britain since the industrial Revolution, (4) _______
whole families moved into cities from the country to get work in the new factories, taking
grandmother along to look after the children. (5) _______ the fact that more grandmothers are
working now, grandparents are still the backbone of childcare in Britain. They provide 44% of full-
time care for pre-school children, which (6) _______ you wonder how the country would manage
without them.
The traditional image of a grandparent is a smiling old person surrounded by a cohort of happy
children, but this doesn’t match the facts. (7) _______ we have now is the so-called ‘beanpole
family’, thinly stretched over several generations, with fewer family members in each and with
growing (8) _______ of single-parent families. Grandparents are getting younger - more than 50% of
grandparents have already had their first grandchild by the age 54.
For many of them, grandparenthood means juggling a job, involvement with grandchildren
and, sometimes, the care of their own parents. It is up to us to balance the demands we make on them
if we don’t want to wear them (9) _______. Grandparents are (10) _______ a valuable part of the
family that we just cannot do without them.
Your answers here
1. 2. 3. 4. 5.
6. 7. 8. 9. 10.

Part 2: Read the following passage and decide which option (A, B, C, or D) best fits each gap.
Write your answers in the corresponding numbered boxes.
From Nigel’s point of view, a love of travelling began with what is called a ‘gap year’. In (1)
_____ with many other British teenagers, he chose to take a year out before (2) _____ to study for his
degree. After doing various jobs to raise some money, he left home to gain some experience of life in
different cultures, visiting America and Asia. The more adventurous the young person are, the (3)
_____ the challenge they are likely to (4) _____ themselves for the gap year, and for some, like
Nigel, it can (5) _____ in a thirst for adventure.
Now that his university course has (6) _____ to an end, Nigel is just about to leave on a three-
year trip that will take him (7) _____ around the world. What’s more, he plans to make the whole
journey using only means of transport which are (8) _____ by natural energy. In other words, he’ll be
relying mostly on bicycles and his own legs; and when there’s an ocean to cross, he won’t be taking a
short cut by climbing aboard a plane, he’ll be joining the crew of a sailing ship (9) _____.
As well as doing some mountain climbing and other outdoor pursuits along the way, Nigel hopes to
pass on to the people he meets the environmental (10) _____ that lies behind the whole idea.
1. A. term B. sense C. common D. conclusion
2. A. settling down B. getting up C. taking over D. holding back
3. A. stronger B. wider C. deeper D. greater
4. A. place B. set C. aim D. put
5. A. lead B. result C. cause D. create
6. A. come B. turned C. made D. brought
7. A. complete B. just C. whole D. right
8. A. powered B. charged C. forced D. pulled
9. A. anyway B. alike C. instead D. otherwise
10. A. notice B. message C. tour D. voyage
Your answers here
1. 2. 3. 4. 5.
6. 7. 8. 9. 10.

Part 3: Read the following passage and choose the best answer (A, B, C or D). Write your answers
in the corresponding numbered boxes.
Animation traditionally is done by hand-drawing or painting successive frames of an object,
each slightly different from the preceding frame. In computer animation, although the computer may
be the one to draw the different frames, in most cases the artist will draw the beginning and ending
frames and the computer will produce the drawings between the first and the last drawing. This is
generally referred to as computer-assisted animation, because the computer is more of a helper than
an originator.
In full computer animation, complex mathematical formulas are used to produce the final
sequence of pictures. These formulas operate on extensive databases of numbers that define the
objects in the pictures as they exist in mathematical space. The database consists of endpoints, and
colour and intensity information. Highly trained professionals are needed to produce such effects
because animation that obtains high degrees of realism involves computer techniques for three-
dimensional transformation, shading, and curvatures.
High-tech computer animation for film involves very expensive computer systems along with
special colour terminals or frame buffers. The frame buffer is nothing more than a giant image
memory for viewing a single frame. It temporarily holds the image for display on the screen.
A camera can be used to film directly from the computer’s display screen, but for the highest
quality images possible, expensive film recorders are used. The computer computes the positions and
colours for the figures in the picture, and sends this information to the recorder, which captures it on
film. Sometimes, however, the images are stored on a large magnetic disk before being sent to the
recorder. Once this process is completed, it is repeated for the next frame. When the entire sequence
has been recorded on the film, the film must be developed before the animation can be viewed. If the
entire sequence does not seem right, the motions must be corrected, recomputed, redisplayed, and
rerecorded. This approach can be very expensive and time consuming. Often, computer- animation
companies first do motion tests with simple computer-generated line drawings before selling their
computers to the task of calculating the high-resolution, realistic-looking images.
1. What aspect of computer animation does the passage mainly discuss?
A. The production process B. The equipment needed
C. The high cost D. The role of the artist
2. According to the passage, in computer-assisted animation the role of the computer is to draw the
______.
A. first frame B. middle frames
C. last frame D. entire sequence of frames
3. The word “they” in the second paragraph refers to ______.
A. formulas B. databases C. numbers D. objects
4. According to the passage, the frame buffers mentioned in the third paragraph are used to ______.
A. add colour to the images B. expose several frames at the same time
C. store individual images D. create new frames
5. According to the passage, the positions and colours of the figures in high-tech animation are
determined by ______.
A. drawing several versions B. enlarging one frame at a line.
C. using computer calculations D. analyzing the sequence from different angles
6. The word “once” in the fourth paragraph is closest in meaning to ______.
A. before B. since C. after D. while
7. According to the passage, how do computer-animation companies often test motion?
A. They experiment with computer-generated line drawings.
B. They hand-draw successive frames.
C. They calculate high-resolution images.
D. They develop extensive mathematical formulas.
8. Which of the following statements is supported by the passage?
A. Computers have reduced the costs of animation.
B. In the future, traditional artists will no longer be needed.
C. Artists are unable to produce drawings as high in quality as computer drawings.
D. Animation involves a wide range of technical and artistic skills.
Your answers here
1. 2. 3. 4.
5. 6. 7. 8.

Part 4: Read the passage and choose the best answer (A, B, C or D) to the following questions. Write
your answers in the corresponding numbered boxes.
The oceans are so vast and deep that until fairly recently, it was widely assumed that no matter
how trash and chemicals humans dumped into them, the effects would be negligible. Proponents of
dumping in the oceans even had a catchphrase: "The solution to pollution is dilution."
Today, we need look no further than the New Jersey–size dead zone that forms each summer
in the Mississippi River Delta, or the thousand–mile–wide swath of decomposing plastic in the
northern Pacific Ocean to see that this "dilution" policy has helped place a once flourishing ocean
ecosystem on the brink of collapse.
There is evidence that the oceans have suffered at the hands of mankind for millennia. But
recent studies show that degradation, particularly of shoreline areas, has accelerated dramatically in
the past three centuries as industrial discharge and run–off from farms and coastal cities have
increased.
Pollution is the introduction of harmful contaminants that are outside the norm for a given
ecosystem. Common man–made pollutants reaching the oceans include pesticides, herbicides,
chemical fertilizers, detergents, oil, sewage, plastics, and other solids. Many of these pollutants
collect at the ocean's depths, where they are consumed by small marine organisms and introduced
into the global food chain.
Many ocean pollutants are released into the environment far upstream from coastlines.
Nitrogen–rich fertilizers applied by farmers inland, for example, end up in local streams, rivers, and
groundwater and are eventually deposited in estuaries, bays, and deltas. These excess nutrients can
spawn massive blooms of algae that rob the water of oxygen, leaving areas where little or no marine
life can exist.
Solid wastes like bags, foam, and other items dumped into the oceans from land or by ships at
sea are frequently consumed, with often fatal effects, by marine mammals, fish, and birds that
mistake them for food. Discarded fishing nets drift for many years, ensnaring fish and mammals. In
certain regions, ocean currents corral trillions of decomposing plastic items and other trash into
gigantic, swirling garbage patches. One in the North Pacific, known as the Pacific Trash Vortex, is
estimated to be the size of Texas.
Pollution is not always physical. In large bodies of water, sound waves can carry undiminished
for miles. The increased presence of loud or persistent sounds from ships, sonar devices, oil rigs, and
even from natural sources like earthquakes can disrupt the migration, communication, and
reproduction patterns of many marine animals, particularly aquatic mammals like whales and
dolphins.
1. What does the passage mainly discuss?
A. The end of the “dilution” era
B. Marine pollution and its many forms
C. Various kinds of harmful pollutants
D. Noise and its disruptive effects on marine life
2. The word “negligible” in paragraph 1 is closest in meaning to ______.
A. unpredictable B. serious C. insignificant D. positive
3. It can be inferred from paragraph 2 that the “dilution” policy is related to _____.
A. helping the ecosystem of the oceans flourish
B. neglecting the effects of dumping trash into the oceans
C. treating harmful materials in the oceans properly
D. dealing with the problems of water pollution
4. The word “they” in paragraph 4 refers to ______.
A. ocean’s depth B. the oceans
C. marine organisms D. man–made pollutants
5. The word “spawn” in paragraph 5 can be best replaced by ______.
A. produce B. appear C. prevent D. limit
6. According to the passage, nitrogen–rich fertilizers _______.
A. cannot be found inland
B. do not relate to the disappearance of marine life
C. are created by massive blooms of algae
D. cause a shortage of oxygen in the ocean water
7. Which of the following statements is NOT supported in the passage?
A. The oceans in the past were more contaminated than they are now.
B. Industrial wastes and agriculture run–off are blamed for the degradation of the oceans.
C. It is apparent that the oceans have been polluted for a long time.
D. Many pollutants deposited in the oceans finally become part of the global food chain.
8. Whales and dolphins are mentioned in the final paragraph as an example of marine creatures that
_____.
A. can survive earthquakes because of their large bodies
B. suffer from loud or persistent sounds at sea
C. are forced to migrate because of water pollution
D. can communicate with each other via sound waves
Your answers here
1 2 3 4
5 6 7 8

Part 5: Read through the following text and then choose from list A-H the best phrase or sentence
to fill each of the blanks. One suggested answer does not fit at all. Write your answers in the
corresponding numbered boxes.
A. The local people are used to it
B. It does not say much for the quality control in the military weapon factories of Britain and
Germany
C. Only after positive identification of their country of origin can the shells be made safe
D. Once cleaned and classified, the shells are placed in wooden boxes, separated by sand
E. That makes 400 million unexploded shells
F. Army personnel try to identify all types of shells
G. Over the years they have grown to treat them with a certain indifference
H. Shortly afterwards, huge explosions rock the sea
Twice a day throughout the summer in a field outside the small Belgian town of Poelkapelle, a
strange ritual takes place. First a siren sounds. Then a number of boxes are lowered into specially
prepared pits. (1) _______, throwing clouds of earth into the air. (2) _______; it is only another
consignment of World War I shells exploding 75 years late.
Bomb disposal experts at Poelkapelle will be hard at work for many years to come. (3)
_______, but best estimates suggest that of 1.5 billion shells fired on the Western Front between 1914
and 1918, about 30% failed to explode on impact. (4) _______, most of which are still out there. In
the countryside around Poelkapelle, farmers plough up these deadly souvenirs almost daily. (5)
_______; after unearthing the shells, they leave them by the roadside to be collected by an army jeep.
The shells, however, remain potentially unstable and lethal as most of them are badly corroded after
so many decades in the ground.
(6) _______. This is initially difficult because they are covered with rust and dirt. Officers
used to clean them by hand in the open air. Now they use a high-pressure water jet or, if the dirt
proves too stubborn, they remove it with a remote-control machine. (7) _______. Shells over 50kg
have to be exploded at sea; the remainders are stored, ready to be detonated at the ritual hours of
11.45am and 3.30pm.
Your answers here
1 2 3 4 5 6 7

SECTION IV - WRITING (4.5P)


Part 1: Finish each of the following sentences in such a way that it means exactly the same as the
sentence printed before it. Write your answers in the space provided.
1. She regrets not studying hard enough for the final examination.
 She wishes _____________________________________________________________.
2. They believe that the room was badly decorated.
 The room _____________________________________________________________.
3. I fully intend to find out who is responsible for the graffiti.
 I have every ____________________________________________________________.
4. I didn’t stop worrying about the wild animals until we were safe inside camp.
 It was only when we reached ______________________________________________.
5. It’s sad, but unemployment is unlikely to go down this year
 Sad __________________________________________________________________.
6. The only thing that prevented the passing of the bill was the death of the Prime Minister.
 Had __________________________________________________________________.
7. You may be disqualified if you don’t obey the regulations.
 Failure ________________________________________________________________.
8. His condition improved so rapidly that he went home four days after the operation.
 There _________________________________________________________________.
9. He is proud of his loyalty to his friends.
 He prides ______________________________________________________________.
10. I was not surprised to hear that Harry had failed his driving test.
 It came ________________________________________________________________.

Part 2: Complete the second sentence so that it has a similar meaning to the first sentence, using
the word given. Do not change the word given. You must use between three and six words,
including the word given. Write your answers in the space provided.
1. She can’t stand her toothache, so she goes to see the dentist. (PUT)
 She goes to see the dentist as she can’t ___________________________ her toothache.
2. He couldn’t understand the instruction of this task. (SENSE)
The instruction of this task ____________________________ him.
3. A foreign manager is running our restaurant at the moment. (RUN)
 Our restaurant ____________________________ a foreign manager at the moment.
4. The fast food Linda eats affects her health badly. (EFFECT)
 The fast food Linda eats ____________________________ her health.
5. Nora described the trip to us in detail. (DETAILED)
 Nora ____________________________ the trip.
6. On Monday last week I met one of my friends by chance at the stadium. (CAME)
 On Monday last week I ____________________________ mine at the supermarket.
7. Robert offered her a lift in his new car, but she didn’t accept. (TURNED)
 She ____________________________ offer of a lift in his new car.
8. Tony said that she would never talk to anyone else about the matter. (DISCUSS)
 Tony promised never ____________________________ anyone else.
9. She really hates it when people speak to her like that! (OBJECTS)
 She really ____________________________ to like that!
10. Kevin hates tennis, so don’t try to persuade him to play. (WORTH)
 It ____________________________ Kevin to play tennis because he hates it.

Part 3: Writing an essay (about 200 words) to express your opinion on the following topic:
The world grows more connected through social networks. Therefore, many people think that school
students should be allowed to use mobile phones in class.
Do you agree with the above point of view?

----------------------------------------------------------------------------------------------------------------
----------------------------------------------------------------------------------------------------------------
----------------------------------------------------------------------------------------------------------------
----------------------------------------------------------------------------------------------------------------
----------------------------------------------------------------------------------------------------------------
----------------------------------------------------------------------------------------------------------------
----------------------------------------------------------------------------------------------------------------
----------------------------------------------------------------------------------------------------------------
----------------------------------------------------------------------------------------------------------------
----------------------------------------------------------------------------------------------------------------
----------------------------------------------------------------------------------------------------------------
----------------------------------------------------------------------------------------------------------------
----------------------------------------------------------------------------------------------------------------
----------------------------------------------------------------------------------------------------------------
----------------------------------------------------------------------------------------------------------------
----------------------------------------------------------------------------------------------------------------
----------------------------------------------------------------------------------------------------------------
----------------------------------------------------------------------------------------------------------------
----------------------------------------------------------------------------------------------------------------
----------------------------------------------------------------------------------------------------------------
----------------------------------------------------------------------------------------------------------------
----------------------------------------------------------------------------------------------------------------
----------------------------------------------------------------------------------------------------------------
----------------------------------------------------------------------------------------------------------------
----------------------------------------------------------------------------------------------------------------
----------------------------------------------------------------------------------------------------------------
----------------------------------------------------------------------------------------------------------------
----------------------------------------------------------------------------------------------------------------
----------------------------------------------------------------------------------------------------------------
----------------------------------------------------------------------------------------------------------------
----------------------------------------------------------------------------------------------------------------
----------------------------------------------------------------------------------------------------------------
----------------------------------------------------------------------------------------------------------------
----------------------------------------------------------------------------------------------------------------
----------------------------------------------------------------------------------------------------------------
----------------------------------------------------------------------------------------------------------------
----------------------------------------------------------------------------------------------------------------
----------------------------------------------------------------------------------------------------------------
----------------------------------------------------------------------------------------------------------------
----------------------------------------------------------------------------------------------------------------
----------------------------------------------------------------------------------------------------------------
----------------------------------------------------------------------------------------------------------------
----------------------------------------------------------------------------------------------------------------
----------------------------------------------------------------------------------------------------------------
----------------------------------------------------------------------------------------------------------------
----------------------------------------------------------------------------------------------------------------
----------------------------------------------------------------------------------------------------------------
----------------------------------------------------------------------------------------------------------------
----------------------------------------------------------------------------------------------------------------
----------------------------------------------------------------------------------------------------------------
----------------------------------------------------------------------------------------------------------------
----------------------------------------------------------------------------------------------------------------
SỞ GIÁO DỤC VÀ ĐÀO TẠO QUẢNG TRỊ
HƯỚNG DẪN CHẤM ĐỀ THI CHÍNH THỨC MÔN TIẾNG ANH
KỲ THI HỌC SINH GIỎI VĂN HOÁ LỚP 12 THPT
Khóa ngày 06 tháng 10 năm 2020
(Hướng dẫn chấm gồm 02 trang)

SECTION I: LISTENING COMPREHENSION (5 POINTS)


Part 1: (0.2/ea)
1. postgraduate 2. engineering 3. computer games 4. German 5. (in) hall
Part 2: (0.2/ea)
1. F 2.F 3.T 4.F 5.T
Part 3: (0.2/ea)
1.A 2. B 3. B 4. C 5. B
Part 4: (0.2/ea)
1. Student Union 2. (the) library 3. username and password
4. (the) Sports Centre 5. in the morning 6. compulsory (courses)
7. attendance 8. presentation 9. curriculum 10. Common Room

SECTION II: USE OF LANGUAGE (3.5 POINTS)


Part 1: (0.1/ea)
1. A 2.A 3.B 4.D 5.A
6. D 7. D 8.C 9. B 10. B
Part 2: (0.1/ea; 0,05 for any correct half of the answer )
Line Mistake Correction Line Mistake Correction
Line 1 (0) who which
Line 3 1. changed changing Line 8 6. this these
4
Line 4 2.other another Line 9 7. create creating
5Line 5 3. percent percentage Line 10 8. extinct extinction
Line 6 4.at on Line 11 9. much many
Line 7 5.evaluating evaluated Line 12 10.potential potentially
Part 3: (0.1/ea)
1. of /from 2. up 3. to 4. from 5. on 6. in 7. beyond
Part 4: (0.1/ea)
1.straightforward 2. simplicity 3. undertaken 4. irritability
5. regularity 6. personal 7. supposedly 8. consciousness

SECTION III: READING COMPREHENSION (5 POINTS)


Part 1: (0.1/ea)
1. were 2. telling/inventing/concocting 3.them 4.when 5.despite
6.makes 7.what 8.numbers 9.out 10.such
Part 2: (0.1/ea)
1. C 2. A 3.D 4.B 5.B
6.A 7.D 8.A 9. C 10. B
Part 3: (0.1/ea)
1.A 2. B 3. D 4. C
5. C 6. C 7. A 8. D
Part 4: (0.1/ea)
1.B 2. C 3. B 4. D
5. A 6. D 7. A 8. B
Part 5: (0.2/ea)
1. H 2.A 3.B 4.E 5.G 6.C 7.D
SECTION IV - WRITING (4.5 POINTS)
Part : (0.15/ea)
1. She wishes she had studied hard enough for the final examination.
2. The room is believed to have been badly decorated.
3. I have every intention of finding out who is responsible for the graffiti.
4. It was only when we reached the safety of/inside the camp that I stopped worrying about the wild
animals.
5. Sad though/as it is, unemployment is unlikely to go down this year.
6. Had it not been for the death of the Prime Minister, the passing of the bill would/ could not have
been prevented. / the bill would have been passed.
7. Failure to obey the regulations may lead to/ result in disqualification.
8. There was such a rapid improvement in his condition that he went home four days after the
operation
9. He prides himself on his loyalty to his friends.
10.It came as no surprise to me that Harry had failed his driving test.
Part 2: (0.15/ea)
1. She goes to see the dentist as she can’t …… put up with …..her toothache.
2. The instruction of this task … did not make (any/much) sense to….. him.
3. Our restaurant …… is being run by …..a foreign manager at the moment.
4. The fast food Linda eats …… has a bad effect on …. her health.
5. Nora …… gave us the/a detailed description of …… the trip.
6. On Monday last week I …… came across a friend of .……mine at the supermarket.
7. She …… turned down Robert’s …….. offer of a lift in his new car.
8. Tony promised never ……… to discuss the matter with …….anyone else.
9. She really …… objects to being spoken ……to like that!
10. It … is not worth trying to persuade….Kevin to play tennis because he hates it.
Part 3: Write an essay (1.5 pts)
Criteria/scale 0.0 0.25 0.5 0.75 1.0 1.25 1.5
Content
Communicative achievement
Organization
Language
Note: Điểm của bài luận là điểm trung bình cộng của 4 tiêu chí, làm tròn đến một chữ số thập phân

------------ THE END ------------


SỞ GIÁO DỤC VÀ ĐÀO TẠO KỲ THI HỌC SINH GIỎI VĂN HOÁ LỚP 12 THPT
HỒ CHÍ MINH NĂM HỌC 2020-2021
Môn thi: TIẾNG ANH
ĐỀ THI CHÍNH THỨC
Thời gian làm bài: 180 phút (không kể thời gian giao đề)

SECTION A: GRAMMAR & VOCABULARY (40 POINTS)

I. Choose the word or phrase (A, B, C or D) which best completes each sentence. (20points)

1. He_________ his son of the dangers of driving too fast in his new car
A. warned B. remembered C. threatened D. concerned
2. The child was_________ by a lorry on the safety crossing in the main street.
A. knocked out B. run across C. run out D. knocked down
3. The independent arbitrator managed to_________ the confrontation between the union and
the employers.
A. refuse B. confuse C. refute D. defuse
4. When I heard the footsteps behind me I was_________ that I would be attacked.
A. horrified B. terror-struck C. terrorized D. terrified
5. His illness made him_________ of concentration.
A. incompetent B. unable C. incapable D. powerless
6. Medieval travelers’ tales of fantastic creatures were often fascinating but not
always________.
A. credible B. creditable C. credulous D. imaginable
7. An almost________ line of traffic was moving at a snail’s pace through the town.
A. continuous B. constant C. continual D. stopping
8. Somebody ran in front of the car as I was driving. Fortunately I________ just in time.
A. could stop B. could have stopped C. managed to stop D. must be able to stop
9. You are being thoroughly________ in refusing to allow this ceremony to take place.
A. unrequited B. unrepresentative C. unreliable D. unreasonable
10 The sudden resignation of the financial director put the company in a
very_________position.
A. weak B. unsteady C. vulnerable D. collapsed
11. David: Would you like fish or meat? Mary: I_________ fish, please.
A. would rather B. would prefer C. suppose D. believe
12. Many teenagers show signs of anxiety and_________ when being asked about their future.
A. depress B. depression C. depressed D. depressing
13. . A part – time job gives me the freedom to_________ my own interests.
A. pursue B. chase C. seek D. catch
14. The new road currently under_________ will solve the traffic problems in the town.
A. design B. progress C. construction D. work
15. - Daisy: “What a lovely house you have!” - Mary: “_______________.”
A. Lovely, I think so B. Thank you. Hope you will drop in
C. Of course not, it’s not costly D. No problem
16. We were shocked to hear the news of your ________.
A. having fired B. being fired C. having been fired D. to have been fired
17. _______ as taste is really a composite sense made up of both taste and smell.
A. That we refer to B. What we refer to
C. To which we refer D. What do we refer to
18. They are happily married although, of course, they argue _______.
A. most times B. from day to day
C. every now and then D. on the occasion
19. I don’t know French, but I’ll ________.
A. get Tom to translate it B. have it translate
C. have Tom to translate it D. make it translate
20. Doctors advise people who are deficient __________ vitamin C to eat more fruit and
vegetables.
A. from B. of C. in D. for
II. Use the word in capitals at the end of these sentences to form a word that fits in

the blank space. (10 points)

1. The main goals of the Association of Southeast Asian Nations are to promote peace
and________ in the region.
STABLE
2. The security of the earth can be threatened by________ groups.
TERROR

3. I don’t care if you had had too much to drink. Your behaviour last night was______.
DEFEND

4. Her son is always mischievous and________ which annoys her very much.
OBEY

5. The Americans are much more concerned than the Indians and the Chinese with
physical________ when choosing a wife or a husband.
ATTRACT

6. You can never be sure what my sister is going to do. She is so________.
PREDICT

7. He is completely________. Not only is he lazy but he is dishonest too.


EMPLOY

8. His boss told him off because he had behaved________


RESPONSIBLE

9. He won the discus event at the Olympic Games but was later________ when a medical

check proved that he had been taking drugs.


QUALIFY

10. The trouble with Mr. Brown is that he’s so________. One minute he goes mad when you
come late; the next he says nothing. You never know where you are!
CONSIST

III. Find one mistake in each sentence below by choosing the letter A, B, C or D. (10 pts)?

1. Jill mustn’t have arrived yet, otherwise she would have phoned me
A B C D
2. Not many people realize that apples have been cultivating for over 3,000 years
A B C D
3. The building manager is having all the windows and doors replace on the second and third
floor as well as in the restaurant A B
C D
4. Having live here for seven years, my friend is used to speaking English with all her
classmates.
A B C D

5. If only we knew all this information about the market many weeks ago
A B C D
6. Peter apologized me for not working hard for the final exam.

A B C D

7. John had so interesting and creative plans that everyone wanted to work with him.

A B C D

8. Species become extinct or endangered for the number of reasons, but the primary cause
A B C
is the destruction of habitat by human activities .
D
9. Were she be invite to their wedding nniversary, she would be very happy .
A B C D
10. Not until the end of prehistoric times that did the first wheeled vehicles appear.
A B C D

SECTION B: READING (40 POINTS)

I. Read the text below and fill in each blank with ONE suitable word. (10 pts)

TSUNAMI IN JAPAN

Japan's most powerful earthquake since records began has struck the north-east
coast,triggering a massive tsunami. Cars, ships and buildings were (1)________ away by a
wall ofwater after the 8.9 - magnitude tremor, which struck about 400 kms (250 miles) north-
east ofTokyo. A state of emergency has been declared at a nuclear power plant,
(2)________pressure has exceeded normal levels. Officials say more than 10,000 people are
dead and about 5,000 (3)________, but it is feared the final death toll will be (4)________
higher. Inone ward alone in Sendai, a port city in Miyagi prefecture, 200 to 300 bodies were
found.“The quake has been the fifth-largest in the world (5)________ 1900 and nearly
8,000(6)________ stronger than the one which devastated Christchurch, New Zealand, last
month”, said scientists. Thousands of people (7)________ near the Fukushima nuclear power
plant have been ordered to evacuate. Japanese nuclear officials said that pressureinside a
boiling water reactor at the plant was running much higher than normal after the cooling
system failed. Officials said they might need to deliberately (8)________ some radioactive
steam to relieve pressure, but that there would be no health risk. US Secretary of State Hillary
Clinton had earlier said the US Air Force had flown emergency coolant to the site. But US
officials later said (9)________ coolant had been handed over because the Japanese had
decided to handle the situation (10)________.The UN's nuclear agency said four nuclear
power plants had been shut down safely.

1._____________ 2._____________ 3._____________ 4._____________


5.____________ 6._____________ 7._____________ 8._____________

9._____________ 10.____________

II. Read the passage and choose the best option for each of the following blanks. (10 pts)

Women nowadays have more (1)________ than those in the past. For example, ourgreat
grandmothers were not allowed to go to school or to work to earn their own
living.(2)________, they had to depend on their husbands financially. Modern women, on
thecontrary, can get good education, have their own careers, and (3)________ their
interests.They can even take good positions in politics if they are competent (4)________ it.
However,women living in our modern society have their (5)________ too. Today women
work harderthan their great grandmothers so that they can gain the (6)________ between
working lifeand family life. Many people predict that by 2032, most (7)________ positions at
work willbe taken by women. Then, it is possible that women will have more (8)________ life
because, (9)________ in a very modern society, the women can’t (10)________ their role
inthe family.

1. A. advances B. advantages C. benefits D. conveniences

2. A. Therefore B. However C. As a result D. Although

3. A. pursue B. support C. promote D. stimulate

4. A. to B. at C. with D. of

5. A. obstacles B. disputes C. profits D. problems

6. A. equality B. stables C. balance D. steadiness

7. A. senior B. junior C. inferior D. superior

8. A. sheltered B. healthy C. strenuous D. active

9. A. though B. even C. ever D. never

10. A. perform B. adopt C. fulfill D. neglect

III. Read the following passage and choose the option that indicates the correct answer to
each of the following questions.(10 pts)

Over the past 600 years, English has grown from a language of few speakers to become the
dominant language of international communication. English as we know it today emerged
around 1350, after having incorporated many elements of French that were introduced
following the Norman invasion of 1066. Until the 1600s, English was, for the most part,
spoken only in England and had not extended even as far as Wales, Scotland, or Ireland.
However, during the course of the next two centuries, English began to spread around the
globe as a result of exploration, trade (including slave trade), colonization, and missionary
work. That small enslaves of English speakers became established and grew in various parts
of the world. As these communities proliferated, English gradually became the primary
language of international business, banking, and diplomacy. Currently, more than 80 percent
of the information stored on computer systems worldwide is in English. Two thirds of the
world‟s science writing is in English, and English is the main language of technology,
advertising, media, international airports, and air traffic controllers. Today there are 700
million English users in the world, and over half of these are nonnative speakers, constituting
the largest number of nonnative users of any language in the world.

1. What is the main topic of the passage?

A. The number of non-native users of English.

B. The French influence on the English language.

C. The expansion of English as an international language.

D. The use of English for science and technology.

2. English began to be used beyond England approximately.............................

A. in 1066 B. around 1350 C. before 1600 D. after 1600

3. According to the passage, all of the following contributed to the spread of English around
the world EXCEPT .....................................

A. the slave trade B. the Norman invasion C. missionaries. D. colonization

4. Which of the following statements is NOT true?

A. Most of the information stored on computer systems is in English.

B. Only one thirds of the world‟s science writing is in languages other than English.

C. English is the only language used in technology, and advertising.

D. International airports and air controllers use mostly English.

5. According to the passage, approximately how many non-native users of English are there in
the world today?

A. A quarter million B. Half a million C. 350 million D. 700 million.

Sustainable architecture - lessons from the ant

Termite mounds were the inspiration for an innovative design in sustainable living
Africa owes its termite mounds a lot. Trees and shrubs take root in them. Prospectors mine
them, looking for specks of gold carried up by termites from hundreds of metres below. And
of course, they are a special treat to aardvarks and other insectivores.

Now, Africa is paying an offbeat tribute to these towers of mud. The extraordinary Eastgate
Building in Harare, Zimbabwe's capital city, is said to be the only one in the world to use the
same cooling and heating principles as the termite mound.

Termites in Zimbabwe build gigantic mounds inside which they farm a fungus that is their
primary food source. This must be kept at exactly 30.5°C, while the temperatures on the
African veld outside can range from 1.5°C at night- only just above freezing - to a baking hot
40°C during the day. The termites achieve this remarkable feat by building a system of vents
in the mound. Those at the base lead down into chambers cooled by wet mud carried up from
water tables far below, and others lead up through a flue to the peak of the mound. By
constantly opening and closing these heating and cooling vents over the course of the day the
termites succeed in keeping the temperature constant in spite of the wide fluctuations outside.

Architect Mick Pearce used precisely the same strategy when designing the Eastgate Building,
which has no air conditioning and virtually no heating. The building - the country's largest
commercial and shopping complex - uses less than 10% of the energy of a conventional
building its size. These efficiencies translated directly to the bottom line: the Eastgate's
owners saved $3.5 million on a $36 million building because an air-conditioning plant didn't
have to be imported. These savings were also passed on to tenants: rents are 20% lower than
in a new building next door.

The complex is actually two buildings linked by bridges across a shady, glass-roofed atrium
open to the breezes. Fans suck fresh air in from the atrium, blow it upstairs through hollow
spaces under the floors and from there into each office through baseboard vents. As it rises
and warms, it is drawn out via ceiling vents and finally exits through forty-eight brick
chimneys.

To keep the harsh, high veld sun from heating the interior, no more than 25% of the outside is
glass, and all the windows are screened by cement arches that jut out more than a metre.

During summer's cool nights, big fans flush air through the building seven times an hour to
chill the hollow floors. By day, smaller fans blow two changes of air an hour through the
building, to circulate the air which has been in contact with the cool floors. For winter days,
there are small heaters in the vents.

This is all possible only because Harare is 1600 feet above sea level, has cloudless skies, little
humidity and rapid temperature swings - days as warm as 31°C commonly drop to 14°C at
night. 'You couldn't do this in New York, with its fantastically hot summers and fantastically
cold winters,' Pearce said. But then his eyes lit up at the challenge. 'Perhaps you could store
the summer's heat in water somehow .... '
The engineering firm of Ove Arup & Partners, which worked with him on the design,
monitors daily temperatures outside, under the floors and at knee, desk and ceiling level. Ove
Arup's graphs show that the temperature of the building has generally stayed between 23°C
and 25°C, with the exception of the annual hot spell just before the summer rains in October,
and three days in November, when a janitor accidentally switched off the fans at night. The
atrium, which funnels the winds through, can be much cooler. And the air is fresh - far more
so than in air-conditioned buildings, where up to 30% of the air is recycled.

Pearce, disdaining smooth glass skins as 'igloos in the Sahara', calls his building, with its
exposed girders and pipes, 'spiky'. The design of the entrances is based on the porcupine-quill
headdresses of the local Shona tribe. Elevators are designed to look like the mineshaft cages
used in Zimbabwe's diamond mines. The shape of the fan covers, and the stone used in their
construction, are echoes of Great Zimbabwe, the ruins that give the country its name.

Standing on a roof catwalk, peering down inside at people as small as termites below, Pearce
said he hoped plants would grow wild in the atrium and pigeons and bats would move into it,
like that termite fungus, further extending the whole 'organic machine' metaphor. The
architecture, he says, is a regionalised style that responds to the biosphere, to the ancient
traditional stone architecture of Zimbabwe's past, and to local human resources.

Choose the correct answer, A, B, C or D.

1 Why do termite mounds have a system of vents?

A to allow the termites to escape from predators

B to enable the termites to produce food

C to allow the termites to work efficiently

D to enable the termites to survive at night

2 Why was Eastgate cheaper to build than a conventional building?

A Very few materials were imported. C Its tenants contributed to the costs.

B Its energy consumption was so low. D No air conditioners were needed.

3 Why would a building like Eastgate not work efficiently in New York?

A Temperature change occurs seasonally rather than daily.

B Pollution affects the storage of heat in the atmosphere.

C Summer and winter temperatures are too extreme.

D Levels of humidity affect cloud coverage.


4 What does Ove Arup's data suggest about Eastgate's temperature control system?

A It allows a relatively wide range of temperatures.

B The only problems are due to human error.

C It functions well for most of the year.

D The temperature in the atrium may fall too low.

5 Pearce believes that his building would be improved by

A becoming more of a habitat for wildlife.

B even closer links with the history of Zimbabwe.

C giving people more space to interact with nature.

D better protection from harmful organisms.

The accidental rainforest

According to ecological theory, rainforests are supposed to develop slowly over millions of
years. But now ecologists are being forced to reconsider their ideas

When Peter Osbeck, a Swedish priest, stopped off at the mid-Atlantic island of Ascension in
1752 on his way home from China, he wrote of 'a heap of ruinous rocks' with a bare, white
mountain in the middle. All it boasted was a couple of dozen species of plant, most of them
ferns and some of them unique to the island.

And so it might have remained. But in 1843 British plant collector Joseph Hooker made a
brief call on his return from Antarctica. Surveying the bare earth, he concluded that the island
had suffered some natural calamity that had denuded it of vegetation and triggered a decline in
rainfall that was turning the place into a desert. The British Navy, which by then maintained a
garrison on the island, was keen to improve the place and asked Hooker's advice. He
suggested an ambitious scheme for planting trees and shrubs that would revive rainfall and
stimulate a wider ecological recovery. And, perhaps lacking anything else to do, the sailors set
to with a will.

In 1845, a naval transport ship from Argentina delivered a batch of seedlings. In the following
years, more than 200 species of plant arrived from South Africa. From England came 700
packets of seeds, including those of two species that especially liked the place: bamboo and
prickly pear. With sailors planting several thousand trees a year, the bare white mountain was
soon cloaked in green and renamed Green Mountain, and by the early twentieth century the
mountain's slopes were covered with a variety of trees and shrubs from all over the world.
Modern ecologists throw up their hands in horror at what they see as Hooker's environmental
anarchy. The exotic species wrecked the indigenous ecosystem, squeezing out the island's
endemic plants. In fact, Hooker knew well enough what might happen. However, he saw
greater benefit in improving rainfall and encouraging more prolific vegetation on the island.

But there is a much deeper issue here than the relative benefits of sparse endemic species
versus luxuriant imported ones. And as botanist David Wilkinson of Liverpool John Moores
University in the UK pointed out after a recent visit to the island, it goes to the heart of some
of the most dearly held tenets of ecology. Conservationists' understandable concern for the
fate of Ascension's handful of unique species has, he says, blinded them to something quite
astonishing - the fact that the introduced species have been a roaring success.

Today's Green Mountain, says Wilkinson, is 'a fully functioning man-made tropical cloud
forest' that has grown from scratch from a ragbag of species collected more or less at random
from all over the planet. But how could it have happened? Conventional ecological theory
says that complex ecosystems such as cloud forests can emerge only through evolutionary
processes in which each organism develops in concert with others to fill particular niches.
Plants co-evolve with their pollinators and seed dispersers, while microbes in the soil evolve
to deal with the leaf litter.

But that's not what happened on Green Mountain. And the experience suggests that perhaps
natural rainforests are constructed far more by chance than by evolution. Species, say some
ecologists, don't so much evolve to create ecosystems as make the best of what they have. 'The
Green Mountain system is a man-made system that has produced a tropical rainforest without
any co-evolution between its constituent species,' says Wilkinson.

Not everyone agrees. Alan Gray, an ecologist at the University of Edinburgh in the UK,
argues that the surviving endemic species on Green Mountain, though small in number, may
still form the framework of the new ecosystem. The new arrivals may just be an adornment,
with little structural importance for the ecosystem.

But to Wilkinson this sounds like clutching at straws. And the idea of the instant formation of
rainforests sounds increasingly plausible as research reveals that supposedly pristine tropical
rainforests from the Amazon to south-east Asia may in places be little more than the
overgrown gardens of past rainforest civilisations.

The most surprising thing of all is that no ecologists have thought to conduct proper research
into this human-made rainforest ecosystem. A survey of the island's flora conducted six years
ago by the University of Edinburgh was concerned only with endemic species. They
characterised everything else as a threat. And the Ascension authorities are currently turning
Green Mountain into a national park where introduced species, at least the invasive ones, are
earmarked for culling rather than conservation.
Conservationists have understandable concerns, Wilkinson says. At least four endemic species
have gone extinct on Ascension since the exotics started arriving. But in their urgency to
protect endemics, ecologists are missing out on the study of a great enigma.

'As you walk through the forest, you see lots of leaves that have had chunks taken out of them
by various insects. There are caterpillars and beetles around,' says Wilkinson. 'But where did
they come from? Are they endemic or alien? If alien, did they come with the plant on which
they feed or discover it on arrival?' Such questions go to the heart of how rainforests happen.

The Green Mountain forest holds many secrets. And the irony is that the most artificial
rainforest in the world could tell us more about rainforest ecology than any number of natural
forests.

Do the following statements agree with the information given in Reading Passage 3? In
boxes 1-6 on your answer sheet write

TRUE if the statement agrees with the information

FALSE if the statement contradicts the information

NOT GIVEN if there is no information on this

1 When Peter Osbeck visited Ascension, he found no inhabitants on the island.

2 The natural vegetation on the island contained some species which were

found nowhere else.

3 Joseph Hooker assumed that human activity had caused the decline in the island's plant life.

4 British sailors on the island took part in a major tree planting project.

5 Hooker sent details of his planting scheme to a number of different countries.

6 The bamboo and prickly pear seeds sent from England were unsuitable for Ascension.

Complete each sentence with the correct ending A-G from the box below.

Write the correct letter A-G in boxes 7-10 on your answer sheet.

7 The reason for modern conservationists' concern over Hooker's tree planting

programme is that

8 David Wilkinson says the creation of the rainforest in Ascension is important

because it shows that

9 Wilkinson says the existence of Ascension's rainforest challenges the theory that
10 Alan Gray questions Wilkinson's theory, claiming that

A other rainforests may have originally been planted by man.

B many of the island's original species were threatened with destruction.

C the species in the original rainforest were more successful than the

newer arrivals.

D rainforests can only develop through a process of slow and complex


evolution.

E steps should be taken to prevent the destruction of the original ecosystem.

F randomly introduced species can coexist together.

G the introduced species may have less ecological significance than

the original ones.

SECTION C: WRITING (20 POINTS)

I. Finish the second sentence in such a way that it means exactly the same as the sentence
printed before it. (10 pts)

1. I am sure it wasn’t Mrs. Brown you saw yesterday because she had gone abroad.
It can’t have been Mrs. Brown you saw yesterday because she had gone abroad.
2. Some scientists report that dolphins have a brain capacity larger than human beings’

Dolphins are reported to have a brain capacity larger than human beings.
3. They think that someone started the fire on purpose

The fire is thought to have started been started on purpose.


4. After Louie had written his composition, he handed it to his teacher.
Having written his composition, Louie handed it to his teacher.
5. If only I had studied hard enough to pass the final exam.
I regret not studying hard enough to pass the final exam.

6. John speaks Chinese fluently because he used to live in China for ten years.
Had John not lived in China for ten years, he could not speak Chinese fluently.

7. "How beautiful is the dress you have just bought!" Peter said to Mary.
Peter complimented Mary on her beautiful dress.
8. "You’re always making terrible mistakes," said the teacher.
The teacher complained about his student‟s making terrible mistakes

9. Sue is too slow to understand what you might say.

So slow is Sue that she can’ t understand what you might say.

10. Although it was expected that he would stand for election, he didn’t.

Contrary to what people expected, he didn’t stand for election.

II. Write a new sentence similar in meaning to the given one, using the word given in the
brackets. Do not alter the word in any way. (10 pts)

1. You looked tired. Why don’t you go to bed early tonight? (better)

You had better go to bed early tonight as you looked tired.

2. Zoe has a job which makes her feel very stressful. (less)

Zoe wishes she had a less stressful job.

3. Simon wants to be left alone because he’s upset. (rather)

Simon would rather be left alone because he’s upset.

4. He’d rather eat with friends than eat alone.. ( prefers)

He prefers eating with friends to eating alone.

5. He is too irresponsible to run the department. (charge)

He is not responsible enough to be in charge.

6. His arrival was completely unexpected. (took)

His arrival took us by surprise.

7. If I help you now, don’t assume I’ll help you next time.(count)

If I help you now, don’t count on me to help you next time.

8. He owes his life to that surgeon. (indebted)

He is indebted to that surgeon for saving his life

9. Don’t pay any attention when she complains. (notice)

Don’t take any notice of her when she complains.

10. A rejection of their offer would have been unwise. (accepted)

Not to have accepted their offer would have been unwise.


ANSWER KEYS
SECTION A
I.

1.A 5.C 9.D 13.A 17.B

2.D 6.A 10.C 14.C 18.C

3.D 7.A 11.B 15.B 19.A

4.D 8.C 12.B 16.C 20.C

II.

1. STABILITY 5. ATTRACTIVENESS 9. DISQUALIFIED


2. TERRORIST 6. UNPREDICTABLE 10.INCONSISTEN
3. INDEFENSIBLE 7. UNEMPLOYED
4. DISOBEDIENT 8. IRRESPONSIBLY
III.
1.A 3.B 5.B 7.A 9.A
2.C 4.A 6.A 8.C 10.B

SECTION B
I.
1. SWEPT 5. SINCE 9. NO
2. WHERE 6.TIMES 10. THEMSELVES
3. MISSING 7.LIVING
4. MUCH 8. RELEASE
II.
1. B 3. A 5. D 7. A 9. B
2. C 4. B 6. C 8. C 10.D
III. READING COMPREHENSION
PASSAGE1
1. C 2. D 3. B 4. C 5. C
PASSAGE 2

1. B 2. D 3. A 4. C 5. A
PASSAGE 3

1. NG 3. F 5. NG 7. B 9. D
2. T 4. T 6. F 8. F 10.G

UBND TỈNH BẮC NINH ĐỀ THI CHỌN HỌC SINH GIỎI CẤP TỈNH NĂM
SỞ GIÁO DỤC VÀ ĐÀO TẠO HỌC 2020 - 2021
¯¯¯¯¯¯¯¯¯¯¯¯¯¯¯¯ Môn thi: Tiếng Anh 12
Thời gian làm bài: 90 phút (không kể thời gian giao đề) (Đề
ĐỀ CHÍNH THỨC
thi gồm 08 trang, 80 câu trắc nghiệm)

Mã đề 033
Họ và tên học sinh :.................................................................... SBD: ...................

Listen to the recording and choose the correct answer to each of the following questions.
Question 1. According to the talk, what does presidency mean?
A. The office of the president B. The US president’s room
C. A particular president D. The US president’s position and power
Question 2. The U.S. presidency .
A. was not similar to any other position in world C. was created in the 17th century
history D. was created by Jefferson in the 1700s
B. refers to the American Constitution
Question 3. If a country were a monarchy and ruled by a king or queen, _ .
A. a lot of power was given to one person
B. power belonged to an elected group
C. power was shared between king or queen with representatives
D. voters elected representatives
Question 4. What were the writers of the Constitution concerned about?
A. One person had a lot of power in a democracy C. Fighting a war for independence against the British
B. Being under the control of a British king D. Making the country a democracy
Question 5. According to the writers of the Constitution, what could a strong executive become?
A. A curator B. A president C. A tyrant or corrupt D. A king or queen
Question 6. What was the problem of a weak executive branch?
A. The Constitution’s writers argued with each other.
B. It took too much time to ask state legislatures to make national decisions.
C. The problems were resolved.
D. There were conflicting ideas about what the office of the president should be.
Listen to an interview with a health worker about vaccination programmes in less economically developed
countries.
Question 7. The proportion of the world’s children receiving vaccinations is .
A. just over two thirds B. just a half
C. just under three quarters D. just over four fifths
Question 8. One achievement mentioned by the expert is that .
A. there are only 71 measles cases since 2000
B. polio has almost been eradicated
C. there are 200,000 fewer tetanus cases since the 1980s
D. measles cases have fallen by one quarter
Question 9. The expert warns against .
A. both rich and poor children still living in poor conditions
B. investing too heavily in immunisation and neglecting other health issues
C. giving children one vaccination and not delivering a full course of injections
D. assuming that recent achievements are sufficient and withdrawing funding
Question 10. According to the expert, many children do not get immunized if they .
A. are already sick C. can access to health care
B. do not live within an established system D. have proper living conditions
Question 11. Which of the following is NOT mentioned as the reason why leaflets are not an effective
means of advertising immunisation programmes because .
A. some mothers cannot read
B. they do not reach people in isolated areas
C. there is a shortage of health workers
D. the health worker doesn’t have enough time
Question 12. To ensure that vaccinations reach everyone, it is necessary to .
A. have an education programme in place as well
B. teach illiterate women how to read and write
C. pinpoint where the system fails and address these issues
D. only hire health workers who are well trained and trustworthy
Question 13. To solve the storage problem, the expert’s organisation is .
A. finding warm places to keep vaccines
B. training health workers how to store vaccines properly
C. creating vaccines that can be stored at any temperature
D. building cheap fridges that work reliably without electricity
Question 14. The expert mentions that computers are necessary to .
A. keep knowledge up-to-date B. maintain communications
C. manage the production of vaccines D. manage distribution
Question 15. The expert advises against using vaccines which are .
A. cheap B. costly C. unbranded D. not pre-qualified
Question 16. A DALY is .
A. the amount of time lost when a person is ill or dies prematurely
B. the loss of earnings due to ill-health and caring for relatives
C. the amount of money a country spends on its health service
D. the amount of time which is well spent on vaccinations
Mark the letter A, B, C, or D on your answer sheet to indicate the word that differs from the other three in
the position of primary stress in each of the following questions.
Question 17. A. Argentina B. disadvantage C. existential D. nevertheless
Question 18. A. intimacy B. intensity C. Antarctica D. invaluable
Question 19. A. architecture B. manufacture C. agriculture D. literature
Mark the letter A, B, C, or D on your answer sheet to indicate the word or phrase that is OPPOSITE in
meaning to the underlined part in each of the questions.
Question 20. I’ve worked with her for donkey’s years and I still don’t know how to use the photocopier.
A. for good and all B. from the beginning C. from time to time D. for a short time
Question 21. Should the ice caps melt, warm tropical waters will circle the globe and make this a hostile planet.
A. dry B. friendly C. hospitable D. warlike
Mark the letter A, B, C, or D on your answer sheet to show the underlined part that needs correction in each
of the following questions.
Question 22. Quinine, cinnamon and other useful substances are all derived of the bark of trees.
A. other B. substances C. the bark of trees D. derived of
Question 23. Cell membrance, when exposing to electrical impulses, can absorb medicine.
A. can absorb B. electrical C. exposing D. Cell membrance
Question 24. She made some very complementary remarks about my English, which was encouraging to me.
A. complementary B. about C. made D. encouraging
Question 25. In outer space, spacecraft can be maneuvered by means small steering rockets.
A. In B. can be maneuvered C. by means D. steering
Read the following passage and mark the letter A, B, C, or D to indicate the correct answer to each of the
questions.
Composers today use a wider variety of sounds than ever before, including many that were once considered
undesirable noises. Composer Edgard Varese (1883-1965) called this the ‘liberation of sound…the right to
make music with any and all sounds’. Electronic music, for example, made with the aid of computers,
synthesizers, and electronic instruments, may include sounds that in the past would not have been considered
musical.
Environmental sounds, such as thunder, and electronically generated hisses and blips can be recorded,
manipulated, and then incorporated into a musical composition. But composers also draw novel sounds from
voices and non-electronic instruments. Singers may be asked to scream, laugh, groan, sneeze, or to sing
phonetic sounds rather than words. Wind and string players may lap or scrape their instruments. A brass or
woodwind player may hum while playing, to produce two pitches at once; a pianist may reach inside the piano
to pluck a string and then run a metal blade along it. In the music of the Western world, the greatest expansion
and experimentation have involved percussion instruments, which outnumber strings and winds in many
recent compositions. Traditional percussion instruments are struck with new types of beaters; and instruments
that used to be considered unconventional in Western music, tom-toms, bongos, slapsticks, maracas, are
widely used .
In the search for novel sounds, increased use has been made in Western music of Microtones. Non- Western
music typically divides and intervals between two pitches more finely than Western music does, thereby
producing a greater number of distinct tones or micro tones, within the same interval. Composers such as
Krzysztof Penderecki create sound that borders on electronic noise through tone clusters, closely spaced tones
played together and heard as a mass, block, or band of sound . The directional aspect of sound has taken on
new importance as well. Loud speakers or groups of instruments may be placed at opposite ends of the stage,
in the balcony, or at the back and sides of the auditorium. Because standard music notation makes no provision
for many of these innovations, recent music scores may contain graph like diagrams, new note shapes and
symbols, and novel ways of arranging notation on the page.
Question 26. The passage suggests that Edgard Varese is an example of a composer who .
A. criticized electronic music as too noise like
B. modified sonic of the electronic instruments he used in his music
C. wrote music with environmental themes
D. believed that any sound could be used in music
Question 27. The word ‘it’ in paragraph 2 refers to .
A. string B. piano C. blade D. music
Question 28. According to the passage, which of the following would be considered traditional elements of
Western music?
A. Microtones B. Pianos C. Tom-toms and bongos D. Hisses
Question 29. The word ‘thereby’ in paragraph 3 is closest in meaning to .
A. by this means B. in return for C. by the way D. in spite of
Question 30. According to the passage, Krzysztof Pederecki is known for which of the following practices?
A. Combining traditional and nontraditional instruments
B. Using tones that are clumped together
C. Seating musicians in unusual areas of an auditorium
D. Playing Western music for non-Western audiences
Question 31. The word ‘wider’ in paragraph 1 is closest in meaning to .
A. more extensive B. more controversial
C. more distinctive D. more impressive
Question 32. According to the passage, which of the following types of instruments has played a role in much
of the innovation in Western music?
A. brass B. woodwind C. string D. percussion
Question 33. In paragraph 3, the author mentions diagrams as an example of a new way to .
A. indicate how particular sounds should be produced
B. chart the history of innovation in musical notation
C. explain the logic of standard musical notation
D. design and develop electronic instruments
Read the following passage and mark the letter A, B, C, or D to indicate the correct answer to each of the
questions.
THE FUTURE OF NEWSPAPERS
Anybody who says they can reliably forecast the future of newspapers is either a liar or a fool. Look at the raw
figures, and newspapers seem doomed . Since 2000, the circulation of most UK national dailies has fallen by
between a third and a half. The authoritative Pew Research Center in the USA reports that newspapers are now
the main source of news for only 26 percent of US citizens as against 45 percent in 2001. There is no shortage
of prophets who confidently predict that the last printed newspaper will be safely buried within 15 years at
most.
Yet one of the few reliable facts of history is that old media have a habit of surviving. An over- exuberant New
York journalist announced in 1935 that books and theatre ‘have had their day’ and the daily newspaper would
become ‘the greatest organ of social life’. Theatre dully withstood not only the newspaper, but also cinema
and then television. Radio has flourished in the TV age; cinema, in turn, has held its own against videos and
DVDs. Even vinyl records have made a comeback, with online sales up 745 percent since 2008.
Newspapers themselves were once new media, although it took several centuries before they became the
dominant medium for news. This was not solely because producing up-to-date news for a large readership
over a wide area became praticable and economic only in the mid-19th century, with the steam press, the
railway and the telegraph. Equally important was the emergence of the idea that everything around us is in
constant movement and we need to be updated on its condition at regular intervals - a concept quite alien in
the medieval times and probably also to most people in the early modern era . Now, we expect change. To our
medieval ancestors, however, the only realities were the passing of the seasons, punctuated by catastrophes
such as famine, flood or disease that they had no reliable means of anticipating. Life, as the writer Alain de
Botton puts it, was ‘ineluctably cyclical’ and ‘the most important truths were recurring’.
Journalism as a full-time trade from which you could hope to make a living hardly existed before the 19th
century. Even then, there was no obvious reason why most people needed news on a regular basis, whether
daily or weekly. In some respects, regularity of newspaper publication and rigidity of format was, and remains,
a burden. Online news readers can dip in and out according to how they perceive the urgency of events.
Increasingly sophisticated search engines and algorithms allow us to personalise the news to our own priorities
and interests. When important stories break, internet news providers can post minute-by- minute updates.
Error, misconception and foolish speculation can be connected or modified almost constantly. There are no
space restrictions to prevent narrative or analysis, and documents or events cited in news stories can often be
accessed in full. All this is a world away from the straitjacket of newspaper publication. Yet few of any
providers seem alive to the new medium’s capacity for spreading understanding and enlightenment.
Instead, the anxiety is always to be first with the news, to maximise reader comments, to create heat and sound
and fury and thus add to the sense of confusion. In the medieval world what news there was was usually
exchanged amid the babble of the market place or the tarven, where truth competed with rumour, mishearing
and misunderstanding. In some respects, it is to that world that we seem to be returning. Newspapers have
never been very good- or not as good as they ought to be- at telling us how the world works. Perhaps they now
face extinction. Or perhaps , as the internet merely adds to what de Botton discribes as our sense that we live
in ‘an improvable and fundamentally chaotic universe’, they will discover that they and they alone can guide
us to wisdom and understanding.
Question 34. Which phrase in the second paragraph has the same meaning as ‘held its own against’?
A. flourished B. made a comeback C. had their day D. withstood
Question 35. What does the writer suggest is the main advantage of online news sites?
A. The flexibility of the medium B. The ease of access for their users
C. The breadth of their potential readership D. The accuracy of the reporting
Question 36. The word ‘alien’ in the passage is closest in meaning to .
A. acceptable B. ridiculous C. strange D. absurd
Question 37. In the third paragraph, the writer stresses the importance of _ .
A. a shift in people’s attitudes towards the outside world
B. certain key 19th century advances in mechanisation
C. the competition between newspapers and more established media
D. the challenges of news distribution in the pre-industrial era
Question 38. What point is the writer making in the second paragraph?
A. Existing media are not necessarily replaced by new ones.
B. Public enthusiasm for new types of media is often unpredictable.
C. It is inevitable that most media technologies will have a limited life.
D. The best media technologies tend to be the most long-lasting.
Question 39. The word ‘recurring’ in the passage is closest in meaning to .
A. emerging B. happening again C. astonishing D. maintaining
Question 40. In the first paragragh, the writer is presenting .
A. his interpretation of a current trend
B. reasons for the decline in newspaper readership
C. his prediction on the future of print journalism
D. evidence that supports a widespread view
Mark the letter A, B, C or D on your answer sheet to indicate the word CLOSEST in meaning to the
underlined word in each of the following questions.
Question 41. The fighter plane gave pivotal assistance to the ground forces that were surrounded by the enemy.
A. reliable B. significant C. compulsory D. necessary
Question 42. Holding copyright provides the only legal remedy against unauthorised copying.
A. redress B. relief C. treatment D. preparation
Mark the letter A, B, C, or D on your answer sheet to indicate the sentence that is closest in meaning to
each of the following questions.
Question 43. No matter how hard Fred tried to lose weight, he did not succeed .
A. Fred tried very hard to lose weight and succeeded .
B. Try as Fred might, he could not lose weight.
C. It was hard for Fred to lose weight because he never succeeded.
D. It did not matter whether Fred could lose weight.
Question 44. There are several categories of people who do not have to pay the new tax.
A. There are several categories of people who are under obligation to pay the new tax.
B. There are several categories of people who are exempt by paying the new tax.
C. There are several categories of people who are exempt from paying the new tax.
D. There are several categories of people who mustn’t pay the new tax.
Question 45. You should have persuaded him to change his mind.
A. You persuaded him to change his mind but he didn't listen.
B. You didn't persuade him to change because of his mind.
C. You should persuade him to change his mind.
D. It was essential to persuade him to change his mind but you didn’t.
Question 46. The woman was too weak to lift the suitcase.
A. Such was her weakness that the woman couldn’t lift the suitcase.
B. So weak the woman was that she couldn’t lift the suitcase.
C. The woman shouldn’t have lifted the suitcase as she was weak.
D. Weak though the woman was, she could lift the suitcase.
Question 47. He is an authority on primitive religion.
A. He has authority over the people who practise primitive religion.
B. He has official permission to practice primitive religion.
C. He has a great influence on the people who practise primitive religion.
D. He is famous for his vast knowledge of primitive religion.
Question 48. Due to the fact that the demand for tea was very high in the 19th century, its price was astronomical.
A. In the 19th century the price for tea didn’t increase despite the demand .
B. It was its astronomical price which decreased the demand for tea in the 19th century.
C. The demand for tea was so high in the 19th century that its price was enormous.
D. It was not until the 19th century that the demand for tea started to increase.
Mark the letter A, B, C, or D on your answer sheet to indicate the most suitable response to complete
each of the following exchanges.
Question 49. Viva and Bob have been reading about the wonders of the world .
Viva: “Would you say The Great Wall is among the seven man- made wonders of the world?”
Bob: “ ”
A. Do you say so? B. There’s no doubt about that.
C. That’s the least I could do. D. I’m glad you like it.
Question 50. Elizabeth has worked with her colleague on a project for a long time and they feel a little tired.
Laurie: “Then, let’s call it a day, shall we?”
Elizabeth: “ ”
A. All right. See you later.
B. I think we have. Is there anything else to discuss?
C. I’d love to, and I’ve got a pretty tight schedule today.
D. I’m sorry I can’t. Maybe we can finish it tomorrow.
Question 51. Frank and Brandy are talking about Brandy's last vacation.
Frank: "I learned that you had an amazing vacation last summer. Did you go with your parents?"
Brandy: " . I went with my cousins. We had a blast!"
A. That's right B. Of course not
C. Certainly with my friends too D. Oh, no. That's unbelievable
Mark the letter A, B, C, or D on your answer sheet to indicate the sentence that best combines each pair
of sentences in the following questions.
Question 52. Jack has won a jackpot prize. 10% of it was donated to flooded areas.
A. Jack has won a jackpot prize, 10% which was donated to flooded areas.
B. Jack has won a jackpot prize, which was donated 10% to flooded areas.
C. Jack has won a jackpot prize, which of 10% was donated to flooded areas.
D. Jack has won a jackpot prize, 10% of which was donated to flooded areas.
Question 53. I owed Bill a favor. I agreed to help him.
A. If I hadn’t owed Bill a favor, I would have agreed to help him.
B. It was only because I owed Bill a favor that I agreed to help him.
C. Although I owed Bill a favor, I agreed to help him.
D. I only agreed to help Bill because I owed him some money.
Question 54. She gave in her notice. She planned to start her new job in January.
A. Her notice was given in order for her to start her new job in January
B. She gave in her notice with a view to starting her new job in January.
C. Her notice was given in with an aim to start her new job in January.
D. She gave in her notice, plan to start her new job in January.
Question 55. His wife phoned him early in the morning. She didn’t want him to forget to bring along the
document.
A. His wife phoned him early in the morning when she didn’t want him to bring along the document.
B. His wife phoned him early in the morning lest he would forget to bring along the document.
C. His wife phoned him early in the morning so that she wanted him to bring along the document.
D. His wife phoned him early in the morning though she didn’t want him to forget to bring along the
document.
Mark the letter A, B, C or D on your answer sheet to indicate the correct answer to each of the
following questions.
Question 56. The speech was by the music coming from the next room. We couldn’t hear a word she
said .
A. broken out B. drowned out C. pulled out D. dropped out
Question 57. I don't want to ask the question, but it is the big in the room.
A. problem B. animal C. elephant D. matter
Question 58. If the work-force respected you, you wouldn't need to your authority so often.
A. affirm B. inflict C. maintain D. assert
Question 59. The earthquake 6.5 on the Richter scale.
A. measured B. achieved C. counted D. weighed
Question 60. The children by social networks are likely to suffer from depression and other health
problems.
A. obsessing B. who obsessed C. are obsessed D. obsessed
Question 61. The factory is working below because of the shortage of essential materials.
A. scope B. capacity C. range D. denstiy
Question 62. Four miles off the southern coast of Massachusetts , a popular summer resort.
A. where the island of Martha’s Vineyard lie B. lies the island of Martha’s Vineyard
C. the island of Martha’s Vineyard lies there D. does the island of Martha’s Vineyard lie
Question 63. She is reported to be a spectator.
A. an athlete more than B. more an athlete than
C. an athlete of more than D. more of an athlete than
Question 64. They were at the stadium with us last night, so they at the theatre then.
A. needn’t have been B. might have been
C. mustn’t have been D. can’t have been
Question 65. Ebola virus disease which is ongoing in West Africa is a severe, often fatal illness, with a
rate of up to 90%.
A. dying B. mortal C. dead D. fatality
Question 66. The Martins have confirmed their strong to charity actions by donating a lump sum of
money again.
A. assignment B. compliance C. commitment D. reliance
Question 67. Travellers will need some cash in _ currency but they can use their credit cards.
A. local B. current C. real D. area
Question 68. It’s Prime Minister’s right to an election at any time he likes
A. nominate B. submit C. call D. summon
Question 69. You are bound to find information on the stock market crash of 1987 in the newspaper .
A. archives B. files C. records D. collections
Question 70. , we missed our plane.
A. The train is late B. The train being late C. To be late D. The train was late
Question 71. Offices, too, will go with the result that paper will almost completely disappear.
A. electrified B. well C. wrong D. electronic
Question 72. We did our best to fix the broken computer but our efforts bore no .
A. end B. fruit C. luck D. success
Mark the letter A, B, C, or D on your answer sheet to indicate the word whose underlined part differs
from the other three in pronunciation in each of the following questions.
Question 73. A. width B. bathroom C. method D. bathing
Question 74. A. candidate B. considerate C. associate D. adequate
Question 75. A. booked B. wicked C. naked D. beloved
Read the following passage and mark the letter A, B, C or D to indicate the correct word or phrase that best
fits each of the numbered blanks.
The popular writer J. K. Rowling has agreed to end her part in the destruction of the world’s forests by having
her books printed on paper which is environmentally friendly. The multi-millionaire author, whose novels
about a teenage wizard have _(76) 6.5 million trees so far, is one of a number of high-profile authors who
have pledged their support for the environment by stipulating that only recycled paper should be used for their
books. Techniques (77) in Canada mean that, for the first time, paper made from such materials as
office waste paper can be used to make books. The Canadian edition of Rowling’s last book was printed
without chopping down a single tree, saving an (78) 40,000 of them. In the past, it was difficult to
print books on recycled paper because the paper was not strong enough to withstand a lifetime’s reading.
Technological advances mean that paper which is manufactured from waste materials is now just as (79)
as paper made from virgin fiber in terms of quality and strength. Despite the high cost of developing
recycled paper that has the required strength and whiteness needed for books, there will not necessarily be a
price rise for the readers. (80) , publishers are likely to compensate for the higher paper cost by using
cheaper book covers, as happened in Canada.
Question 76. A. exhausted B. erased C. consumed D. absorbed
Question 77. A. led B. inaugurated C. pioneered D. founded
Question 78. A. evaluated B. estimated C. established D. assessed
Question 79. A. durable B. firm C. persistent D. substantial
Question 80. A. Moreover B. Instead C. However D. Therefore
ĐÁP ÁN MÔN TIẾNG ANH
HỌC SINH GIỎI LỚP 12 NĂM HỌC 2020-2021

Câu 543 987 694 033


1 B D C D
2 B B A A
3 B C D A
4 D A A A
5 A A A C
6 C B D B
7 D B B D
8 B D A B
9 C A B D
10 C D B B
11 D B B C
12 A A A C
13 D C B D
14 A D B D
15 C D C D
16 B B A A
17 C C B D
18 D A B A
19 D C B B
20 D D C D
21 A C A C
22 A B C D
23 C C A C
24 A B A A
25 B A A C
26 B A D D
27 B D C A
28 A B B B
29 D A D A
30 B C D B
31 D C B A
32 C B C D
33 A B D A
34 C C D D
35 D B B A
36 D B B C
37 A B D A
38 A D B A
39 C C A B
40 B D D D
41 C C C B
42 D D B A
43 B D C B
44 C D A C
45 A C D D
Câu 543 987 694 033
46 B C C A
47 C D A D
48 A A C C
49 D B C B
50 D B C A
51 D B C B
52 B C C D
53 B B B B
54 D A B B
55 B A D B
56 B D A B
57 C B B C
58 D B B D
59 D D B A
60 B C D D
61 A D C B
62 A D B B
63 B D B D
64 A A C D
65 D D D D
66 D B B C
67 C A C A
68 B A D C
69 B D B A
70 C B B B
71 D A B D
72 A A B B
73 B B B D
74 B B C C
75 B B A A
76 A C B C
77 C A C C
78 C B C B
79 A B A A
80 A A D B

CHỮ KÝ XÁC NHÂN CÁN BỘ RA ĐỀ


SỞ GDĐT BẠC LIÊU KỲ THI CHỌN HSG LỚP 12 VÒNG TỈNH
NĂM HỌC 2020-2021
CHÍNH THỨC
(Gồm 12 trang) * Môn thi: TIẾNG ANH (BẢNG A)
* Ngày thi: 05/11/2020
(Không kể trang phách) * Thời gian: 180 phút (Không kể thời gian giao đề)

Điểm
Chữ ký GK 1 Chữ ký GK 2 Mã phách
Bằng số Bằng chữ

(Thí sinh làm bài trực tiếp trên đề thi)

Part A. LISTENING
I. Listen to Amanda talking to a friend about a birthday party. For each question, circle the best
answer (A, B, C). You will hear the conversation twice. (0) is as an example. (1 point)
Example:
0. How many people can come to the party?
A. 8 B. 11 C.18
1. Which ice cream will they have at the party?
A. coffee B. lemon C. apple
2. What is broken?
A. the CD player B. the cassette recorder C. the guitar
3. Whose birthday is it?
A. Emma's B. Joan's C. Amanda's sister's
4. What present has Amanda bought?
A. a camera B. a video C. a football
5. What time should people arrive at the party?
A. 8 p.m. B. 8.30 p.m. C. 9.30 p.m.

II.Complete the notes below. Write NO MORE THAN TWO WORDS


AND / OR A NUMBER for each answer. You will hear twice. (2 points)

West Bay Hotel - details of job

Example Answer
te…
• Newspaper advert for … mp…or…
ar…
y .. staff

• Vacancies for (1) ..................................


• Two shifts
• Can choose your (2) ................................... (must be the same each we
• Pay: £5.50 per hour, including a (3) ..................................
• A (4) ...............................is provided in the hotel
• Total weekly pay: £231
• Dress: a white shirt and (5) ............................ trousers (not supplied)a (6)…………(supplied)
• Starting date: (7) ....................................
• Call Jane (8) ..........................(Service Manager) before (9) ...................tomorrow (Tel: 832009)
• She'll require a (10) ………......................

Part B. LEXICO AND GRAMMAR


I. Circle the best answer (A, B, C, or D) to complete each unfinished
sentence. (1 point)
1. I love this painting of an old man. He has such a ...............................smile.
A. childhood B. childish C. childless D. childlike
2. You’d better leave for the airport now ………….. there’s a lot of traffic on the way.
A. in fact B. in time C. in order D. in case
3. I would like to thank you,............................my colleagues, for the welcome you have given us.
A. on account of B. on behalf of C. because of D. instead of
4. The poor child was in floods of...................................because his bicycle had been stolen.
A. weeping B. tears C. crying D. unhappiness
5. The soldier was punished for ……………. to obey his commanding officer’s orders.
A. refusing B. regretting C. objecting D. resisting
6. She ………… her husband’s job for his ill health.
A. accused B. blamed C. caused D. claimed
7. It’s time we ………….. this old car and bought a new one.
A. will B. had sold C. have sold D. sold
8. The project was rejected because of ...........................funds.
A. unavailable B. inconsiderable C. incomplete D. insufficient
9. Please leave this space ………… on the enrolment form.
A. absent B. blank C. missing D. undone
10. You shouldn’t have criticized him in front of his friends. It was extremely ………..of you.
A. unfortunate B. insensitive C. insensible D. unconscious
11. Phone me before ten; ……………. I’ll be too busy to talk to you.
A. Unless B. whether C. otherwise D. if
12. According to the ………… of the contract, tenants must give six months’ notice if they intend
to leave.
A. laws B. rules C. terms D. details
13. The injured man was taken to hospital and ………… for internal injuries.
A. cured B. healed C. operated D. treated
14. The door hinges had all been oiled to stop them ……………
A. squeaking B. screechingC. shrieking D. squealing
15. Mary attempted to herself with her new boss by volunteering to take on extra work
A. gratify B. please C. ingratiate D. commend
13. You are under no obligation ………… to accept this offer.
A. indeed B. eventually C. apart D. whatsoever
14. Wasn’t it you yourself ……….. the door open?
A. to be left B. were left C. who left D. that should leave
15. If only motorists …………. drive more carefully!
A. must B. shall C. would D. will
16. It is regretted that there can be no …………… to this rule.
A. exclusion B. alternative C. exception D. deviation
17. Complete the form as …………… in the notes below.
A. insisted B. specified C. implied D. devised

II. In the passage below some lines are correct but some have a word that should
not be there. Indicate the correct lines with a tick (√). For the incorrect lines, write
the words that should not be there. (1 point)
1.................................Many species of animals, birds and even that insects are
in danger 2..................................of disappearing from the earth off. Every day
construction in
3 ………….. the rainforests destroys the habitats of these creatures. They
4 cannot survive in other environments. They rely on about the food 5
....................and shelter being in their own habitats. Each time a
habitat is

6.................................destroyed, the animals must have search for a new place.


Luckily, 7..................................there are some people who they are doing
something about this
8 ………….. situation. Animal protection societies are helping for the world
9 to learn more than about endangered species. When people know 10
more, they can fight to protect and save the animals.

III. Choose the right tenses to complete the following passage. (1 point)
Traditionally, universities (1.carry out) ........................... two main activities: research and
teaching. Many academics would argue that both these activities play a critical role in
serving the community. The fundamental question, however, is: how does the community
want or need (2. serve) ......................................................................?
In recent years, universities have been coming under (3.increase)
...........................pressure from both governments and the public (4.ensure) ........................... that they
do not remain “ivory towers” of study divorced from the realities of everyday life. University
teachers (5.encourage) ..........................., and in some cases constrained, to provide more
courses which
produce graduates with the technical skills (6. require) ........................... for the commercial
sector and research that has practical benefits which are commercially exploitable. If
Aristotle (7.want)..............................................................to work in a tertiary institution in the
UK today, he would have a good chance of teaching computer science but would not be so
readily employable as a philosopher.
A post-industrial society (8. require) ........................... large numbers of computer
programmers, engineers, managers and technicians to maintain and develop its economic
growth but man, as the Bible says “does not live by bread alone”. Apart from requiring
medical and social services which do not directly contribute to economic growth, it (9.be)
................................................an impoverished society that did not value and enjoy literature,
music and the arts. In these cost- conscious times, it even (10.point out)
........................... in justification for the funding of the arts that they can be useful money
earners. A successful musical, for instance, can contribute as much to the Gross Nation
Product through tourist dollars as any other export initiative.

IV. Use the correct form of the word in capitals to fit each gap. (1 point)
Interviews are an imperfect method of choosing the best people
for jobs, yet human (1) ...........................like to examine each other in BE
this way. One of the many problems of (2) ...........................as it is SELECT
commonly practised is that the forms filled in by (3) ........................... APPLY
often fail to show people as they really are. This means that you can
follow all the best (4) ........................... when completing your form and ADVISE
still find that you are (5) ........................... at the next stage - the interview. SUCCESS
(6) ..........................., in the rare cases where interviewers are automatic, a
SIMILAR
candidate with an (7) ........................... form may do surprisingly well.
ADEQUAT
E Of course, your forms need to show that you have (8) ...........................
CONFIDEN
T in your (9) ........................... to do the job, but don’t try to turn yourself
ABLE into
someone else – a person you have to pretend to be at the
interview. Realism and (10) ........................... are definitely the best approach. HONEST

V. Choose the correct prepositional phrase from the box to complete the
following sentences. You may not use one phrase. (1 point)

on good terms independent of tired of out of


ashamed of on the whole capable of at most
for safe keeping out of order for a

1. She was .................................... her poor exam results.


2. He gave his watch and wallet to me .................................................while he went for a swim in the lake.
3. You are ........................................ better work than this.
4. The piano is ........................................ again. It sounds terrible.
5. She is old enough to be ........................................ her parents.
6. Despite many disagreements, they’re now.....................................................with their next- door
neighbours.
7. I’m ..........................................doing the same thing every day.
8. There weren’t many passengers on the bus – twelve ........................................... .
9. Parts of the play were boring, but ......................................... I quite enjoyed.
10. You’ll have to use the stairs. The lift is ...........................................
VI. Insert a, an or the if necessary. Write if there is no article. (1 point)
In (1)………….. summer of 1907, I was living in (2)...........................................little
cottage in (3)………….. country, at (4) …………..small distance from
(5)…………..sea. Half (6)………….. mile from my cottage there was (7) school, The Grables,
where Harold Stackhurst, (8) ………….. headmaster, and several other teachers taught students and
prepared them for various professions. Stackhurst and I went on friendly terms and he was (9) only
man in………….. (10) neighbourhood who sometimes called at my cottage.
PART C. READING
I. Read the following passage. Circle the best answers (A, B, C or D) to complete
the passage. (1 point)
Smoking causes lung cancer, heart disease, and breathing problems. Worldwide, about three million
people die every year because of smoking - that's about one
....(1)...... every ten seconds ! In fact, smoking is the ....(2)...... of almost 20 percent of all deaths in the
developed countries of the world. Smoking doesn't just ....(3). ............................................the
smoker, it hurts other people, too. When a pregnant ....(4)...... smokes, she is hurting
her developing....(5)....... When a man .....(6) at home, his wife and children are also breathing
in smoke and can become sick. Even though most people understand the ....(7). effects of
smoking, they continue to smoke. The....(8) of women and teenagers who smoke is increasing. Cigarette
companies make advertisements that.....(9).......these groups of people so that they...(10). to buy cigaretes.

1. A. death B. end C. loss D. decrease


2. A. origin B. reason C. cause D. basis
3. A. injure B. danger C. destroy D. hurt
4. A. child B. man C. woman D. wife
5. A. baby B. children C. love D. youth
6. A. relax B. enjoy C. cigarettes D. smokes
7. A. beneficial B. harmful C. harmed D.
8. A. number B. amount C. quantity D. total
9. A. interest B. concern C. matter D. worry
10. A. retain B. maintain C. continue D. preserve
Read the passage below. Then circle the best option (A, B, C or D) to answer each question.

(1 point)
Rachel Carson was born in 1907 in Springsdale, Pennsylvania. She studied biology at college
and zoology at Johns Hopkins University, where she received her master’s degree in 1933. In
1936, she was hired by the U.S. Fish and Wildlife Service, where she worked most of her life.
Carson’s first book, Under the Sea Wind, was published in 1941. It received excellent reviews,
but sales were poor until it was reissued in 1952. In that year, she published The Sea Around
Us, which provided a fascinating look beneath the ocean’s surface, emphasizing human history
as well as geology and marine biology. Her imaginary and language had a poetic quality.
Carson consulted no less than 1,000 printing sources. She had voluminous correspondence and
frequent discussions with experts in the field. However, she always realized the limitations of
her non-technical readers.
In 1962, Carson published Silent Spring, a book that sparked considerable controversy. It
proved how much harm was done by the uncontrolled, reckless use of insecticides.She
detailed how they poisoned the food supply of animals, killed birds and fish, and contaminated
human food. At the time, spokesmen for the chemical industry mounted personal attacks
against Carson and issued propaganda to indicate that her findings were flawed. However, her
work was vindicated by a 1963 report of the President’s Science Advisory Committee.

A. The passage mainly discusses Rachel Carson’s work


A. as a researcher B. at college
C. at the U.S. Fish and Wildlife Service D. as a writer
B. According to the passage, what did Carson primarily study at Johns Hopkins
University?
A. oceanography B. history
C. literature D. zoology
C. When she published her first book, Carson was closest to the age of
A. 26 B. 29 C. 34 D. 35
D. It can be inferred from the passage that in 1952, Carson’s book Under the Sea Wind
A. was outdated. C. was praised by critics.
B. became more popular than her D. sold many copies.
other books.
E. Which of the following was Not mentioned in the passage as a source of information
for The Sea Around Us?
A. printed matter B. talks with experts
C. a research expedition D. letters from scientists
F. Which of the following words or phrases is LEAST accurate in describing The Sea
Around Us?
A. highly technical B. poetic
C. fascinating D. well-researched
7.The word “reckless” is closest in meaning to
A. unnecessary B. limited
C. continuous D. irresponsible
8. According to the passage, Silent Spring is primarily
A. an attack on the use of chemical preservatives in food.
B. a discussion of the hazards insects posing to the food supply.
C. a warning about the dangers of misusing insecticides.
D. an illustration of the benefits of the chemical industry.
9. The word “flawed” is closest in meaning to
A. faulty B. deceptive C. logical D. offensive
10. Why does the author of the passage mention the report of the President’ s Science
Advisory Committee?
A. To provide an example of government propaganda.
B. To support Carson’s ideas.
C. To indicate a growing government concern with the environment.
D. To validate the chemical industry’s claims.
II. Read the text below carefully and then do the following exercises. (1 point)
Testing 1, 2, 3,

A. These are testing times. In both education and the field of work, the prevailing wisdom
apears to be: if it moves, test it and if doesn’t, well, test it anyway. I say wisdom, but it
has become rather an absession. In addition to the current obstacles, like GCSEs, A-
levels, GNVQs, ONDs, and HNDs, not to mention the interviews and financial hurdles
that school-leavers have to overcome in order to access higher education, students are
facing the threat of “new tests”, Scholastic Aptitude Tests (SATs)
B. SATs are being imported from the United States, where they have been in use for
nearly a hundred years. As a supplement to A-levels, the tests purport to give students
from poor backgrounds a better chance of entering university. SATs are intended to
remove the huge social class bias that exists in British universities. But, in fact, they
are, no more than an additional barrier for students. The tests, which masquerade as IQ
tests, are probably less diagnostic of student potential than existing examinations, and,
more seriously, are far from free of the bias that the supporters pretend.
B. First of all, as for any other tests, students will be able to take classes to cram for SATs,
which again will advantage the better-off. At a recent conference of the Prefessional
Association of Teachers, it was declared that school exams and tests are
biased toward middle-class children. Further, the content of the tests in question is not
based on sound scientific theory, merely on a pool of Multiple Choice Questions
(MCQs), set up by a group of item writers.
The questions in SATs are tested on a representative sample of children. Those which
correlate with the school grades of the children are kept, and the rest discarded. This is
highly unsatisfactory. There is also evidence that in MCQs tests women are at a
disadvantage, because of the way they think, i.e. they can see a wider picture. And it
is worth noting that MCQs are only as good as the people who write them; so, unless
the writers are highly trained, those who are being tested are being judged against the
narrow limitations of the item writers.
Globalisation has introduced greater flexibility into the workplace, but the educational system
has not been so quick off the mark. But there are signs that times are a changing. Previously,
students took exams at the end of academic terms, or at fixed dates periodically throughout the
year. Now, language examinations like the TOEFL, IELTs and the Pitman ESOL exams can
be taken much more frequently. The IELTs examination, for example, is run a test centre
throughout the world subject to demand. Where the demand is high, the test is held more
frequently. At present, in London, it is possible for students to sit the exam about four times a
week.
C. Flexible assessment like the IELTs has been mooted in other areas. It has been
suggested that the students may in future be able to walk into a public library or
other public building and take assessment test for a range of skills on a computer. The
computer will dispense an instant assessment and a certificate. The beauty of this
system is the convenience.

Questions 1 – 5: This reading passage has 6 paragraphs (A-F). Choose the most suitable
heading for each paragraph from the list of headings below. Write the appropriate
number (1-10) next to the paragraph. One of the headings has been done for you.
Note: There are more headings than paragraphs, so you will not use all of them.
YOU MAY USE ANY HEADING MORE THAN ONCE

0. Paragraph A 9 Paragraph B Paragraph C


Paragraph D Paragraph E Paragraph F

List of Headings
1. Assessment in the future
2. The theory behind MCQs
3. Problem with SATs
4. Misuse of testing in School
5. The need for computer assessment
6. The benefits of SATs
7. Testing in workplace
8. The sortcoming of MCQs
9. Too much testing
10. Flexibility in language tesing

Question 6- 10: Read the passage again and then decide whether the following statements
agree with the information in the Reading or not. Write:
Yes if the statement agrees with the information in the passage.
No if the statement contradicts the information in the passage
No Information if there is no information about the statement in the passage.
Example:
0. SATs is the abbreviation of Scholastic Aptitude Tests .
Answer: Yes

6. In the fields of education and work the prevailing wisdom seems to be to test
everything.
7. Research in genetics refutes the theory that people are predestined to follow certain
careers.
8. Psychometric testing is favoured by headmaster and mistresses in many high
schools.
9. The writer of this article is in favour of testing in general.
10. According to the writer, students get benefits from SATs
Your answer
6. 7. 8. 9. 10.

III. You are going to read an article about the Beatles Band. Ten sentences or
phrases have been removed from the article. Choose from the sentences or
phrases A- J the one which fits each gap (1-10). (1 point)
The Beatles were an English rock band, active throughout the 1960s and………….. (1)
…………... Formed in Liverpool, by 1962 the group consisted of John Lennon (rhythm guitar,
vocals), Paul McCartney (bass guitar, vocals), George Harrison (lead guitar, vocals) and Ringo
Starr (drums, vocals). Rooted in skiffle and 1950s rock and roll…………..(2) …………... The
nature of their enormous popularity, which first emerged as "Beatlemania", transformed as
their songwriting grew in sophistication. They came to be perceived as the embodiment of
ideals of the social and cultural revolutions of the 1960s.

Initially a five-piece line-up of Lennon, McCartney, Harrison, Stuart Sutcliffe (bass) and
Pete Best (drums), ………….. (3) …………... Sutcliffe left the group in 1961, (4)
…………... Moulded into a professional outfit by their manager, Brian Epstein, their musical
potential was enhanced by the creativity of producer George Martin.
………….. (5) …………... Gaining international popularity and acquiring the nickname "Fab
Four" the following year, they toured extensively until 1966. During their subsequent "studio
years", they produced what critics consider some of their finest material including the album
Sgt. Pepper's Lonely Hearts Club Band (1967),
………….. (6) …………... After their break-up in 1970, the band members all found success
in independent musical careers. ………….. (7) …………... McCartney and Starr remain
active.

The Beatles are the best-selling band in history, ………….. (8) .................................They have
had more number one albums on the UK charts, ………….. (9) .................................According
to the RIAA, they have sold more albums in the United States than any other artist, and they
headed Billboard magazine's list of all-time top Hot 100 artists in 2008. They have received 7
Grammy Awards from the American National Academy of Recording Arts and Sciences and
15 Ivor Novello Awards from the British Academy of Songwriters, Composers and Authors.
.................................................................................................(10)
…………...

A. They were collectively included in Time magazine's compilation of the 20th


century's 100 most influential people.
B. the group later worked in many genres ranging from pop ballads to psychedelic
rock, often incorporating classical and other elements in innovative ways.
C. and Best was replaced by Starr the following year.
D. They achieved mainstream success in the United Kingdom in late 1962, with their
first single, "Love Me Do".
E. Lennon was murdered outside his home in New York City in 1980, and
Harrison died of cancer in 2001.
F. one of the most commercially successful and critically acclaimed acts in the history
of popular music.
G. they built their reputation playing clubs in Liverpool and Hamburg over a three-
year period from 1960
H. which was widely regarded as a masterpiece.
I. and they have held the top spot longer than any other musical act.
J. and over four decades after their break-up, their recordings are still in demand.
Your answer:
1. 2. 3. 4. 5.
6. 7. 8. 9. 10.
PART D. WRITING
I. Rewrite the following sentences, so that their meaning stay the same, using
the exact words given. (1 point)
1. He stood no chance of passing his exams. INEVITABLE
……………………………………………………………………………………….
2. I am sure that he missed the eleven o’clock train. CAN’T
……………………………………………………………………………………….
3. Not a word came out of her mouth. LOST
……………………………………………………………………………………….
4. I am not a solitary person, I’m sociable. RATHER
……………………………………………………………………………………….
5. You cannot choose which hotel you stay at on this package holiday. OPTION
……………………………………………………………………………………….
6. I certainly won’t change my mind about resigning. QUESTION
……………………………………………………………………………………….
7. You couldn’t do anything more stupid than to give up your job now. HEIGHT
……………………………………………………………………………………….
8. I really don’t know what you are talking about. FAINTEST
……………………………………………………………………………………….
9. I have hardly done anything today. NEXT
……………………………………………………………………………………….
10. Many people will congratulate her if she wins. SHOWERED
……………………………………………………………………………………….
II. The table below shows the consumer durables (telephones, refrigerators…)
owned in the country of Paraland from 1998 to 2005. Write a report describing
the information shown below.
You should write at least 150 words. (2 points)
Consumer 199 199 200 200 200 200 200 200
Percentage of
households with:
Televisions 93 94 94 95 95 96 97 98
Videos 18 20 22 22 25 30
Washing machines 37 40 43 50 55 60 65 70
Vacuum cleaners 65 65 68 72 73 75 77 79
Dish washers 26 26 28 30 30 32 33 35
Refrigerators 60 62 68 70 73 75 78 85
Telephones 42 50 54 60 66 75 80 92

III. Some people think that family is the most important influence on young
adults. Other people think that friends are the most important influence on
young adults. Which view do you agree with? Use examples to support your
opinion.
You should write at least 250 words. (3points)
............................................................................................................................. ...............
............
............................................................................................................................................
............
............................................................................................................................. ...............
............
............................................................................................................................. ...............
............
............................................................................................................................................
............
............................................................................................................................. ...............
............
............................................................................................................................................
............
............................................................................................................................. ...............
............
............................................................................................................................. ...............
............
............................................................................................................................................
............
............................................................................................................................. ...............
............
............................................................................................................................................
............
............................................................................................................................. ...............
............
............................................................................................................................. ...............
............
............................................................................................................................................
............
............................................................................................................................. ...............
............

............................................................................................................................. ...............
............
............................................................................................................................................
............
............................................................................................................................. ...............
............

---HẾT--
SỞ GDĐT BẠC LIÊU KỲ THI CHỌN HSG LỚP 12 VÒNGTỈNH
NĂM HỌC 2020-2021
CHÍNH THỨC

(Gồm 04 trang) * Môn thi: TIẾNG ANH (BẢNG A)


* Ngày thi: 05/11/2020
* Thời gian: 180 phút (Không kể thời gian giaođề)

HƯỚNG DẪN CHẤM

Part A. LISTENING
I. 1.0p. Each correct answer gets 0.2p
1. C 2. A 3. A 4. B 5. B
II. 2.0ps. Each correct answer gets 0.2p
1. waiter(s) 2. day off 3. break 4. (free) meal 5. dark
(coloured/colored)
6. jacket 7. 28 June 8. Urwin 9. 12.00 (pm)/noon/mid-day 10.
reference
Part B. LEXICO AND GRAMMAR
I. 2.0ps. Each correct answer gets 0.1p
1. D 11. C
2. D 12. C
3. B 13. D
4. B 14. A
5. A 15. C
6. B 16. D
7. D 17. C
8. D 18. C
9. B 19. C
10. B 20. B
II. 1.0p. Each correct answer gets 0.1p
1. that 6. have
2. off 7. they
3. √ 8. for
4. about 9. than
5. being 10. √
III. 1.0p. Each correct answer gets 0.1p
1. have carried out 2. to be served 3. increasing 4. to ensure
5. have been encouraged 6. required 7. wanted 8. requires
9. would be 10. has even been pointed out
IV. 1.0p. Each correct answer gets 0.1p
1. beings 2. selection 3. applicants 4. advice
5. unsuccessful 6. Similarly 7. inadequate 8. confidence
9. ability 10. honesty
V. 1.0p. Each correct answer gets 0.1p
1. ashamed of 2. for safe keeping 3. capable of 4. out of tune
5. independent of 6. on good terms 7. tired of 8. at most
9. on the whole 10. out of order
VI. 1.0p. Each correct answer gets 0.1p
I. the 2. a 3. the 4. a 5. the 6. a 7. a 8. the 9. the 10. the
PART C. READING
I. 1.0p. Each correct answer gets 0.1

1.A 2.C 3.D 4.C 5.A 6.D 7.B 8.A 9.A 10.C
II. 1.0p. Each correct answer gets 0.1p
1.D 2.D 3.C 4.D 5.C 6.A 7.D 8.C 9.A 10.B
III. 1.0p. Each correct answer gets 0.1p
1. Paragraph B 3
2. Paragraph C 3
3. Paragraph D 8
4. Paragraph E 1
5. Paragraph F 7
6. Yes 7. Not given 8. Not given 9. No 10. Yes
IV. 1.0p. Each correct answer gets 0.1p
1. F (one of the most commercially successful and critically acclaimed acts in the history of
popular music).
2. B (the group later worked in many genres ranging from pop ballads to psychedelic rock,
often incorporating classical and other elements in innovative ways).
3. G (they built their reputation playing clubs in Liverpool and Hamburg over a three-year
period from 1960).
4. C (and Best was replaced by Starr the following year).
5. D (They achieved mainstream success in the United Kingdom in late 1962, with their first
single, "Love Me Do").
6. H (which was widely regarded as a masterpiece).
7. E (Lennon was murdered outside his home in New York City in 1980, and Harrison died
of cancer in 2001).
8. J (and over four decades after their break-up, their recordings are still in demand).
9. I (and they have held the top spot longer than any other musical act).
10. A (They were collectively included in Time magazine's compilation of the 20thcentury's
100 most influential people).

PART D. WRITING
1.0p. Each correct answer gets 0.1p
1. It was inevitable that he would fail his exams.
2. He can’t possibly have caught the eleven o’clock train.
3. She was lost -for words. -She had lost -her tongue.
4. I’m sociable rather than a solitary person. Rather than being a solitary person, I’m sociable.
5. There is/ You have no option (concerning/ with regard to/ as to/
regarding about) which hotel you stay at on this package holiday.
6. There is no question of my changing my mind about resigning. My changing my mind
about resigning is out of the question.
7. It would be the height of your stupidity to give up your job now.
8. I don’t have the faintest idea what you are talking about.
9. I have done next to nothing today.
10. She will be showered with congratulations if she wins.
II (2 points) + III (3 points)
The mark given to parts 2 and 3 is based on the following scheme:
1. Content: (40% of total mark) a provision of main ideas and details as appropriate.
2. Organization and Presentation: (30% of total mark) ideas are
organized and presented with coherence, style and clarity appropriate to
the level of English language gifted upper- secondary school students.
3. Language: (30% of total mark) a variety of vocabulary and
structures appropriate to the level of English language gifted upper-
secondary school students.

---HẾT---
UBND TỈNH BẮC NINH SỞ ĐỀ THI CHỌN HỌC SINH GIỎI CẤP TỈNH
GIÁO DỤC VÀ ĐÀO TẠO NĂM HỌC 2020-2021
MÔN THI: TIẾNG ANH - LỚP 12 - THPT
ĐỀ CHÍNH THỨC Thời gian làm bài: 180 phút (không kể thời gian giao đề)
Ngày thi 29 tháng 3 năm 2020
================
Điểm bài thi Giám khảo 1 Giám khảo 2 Số phách
Bằng số: Họ tên: Họ tên:

Bằng chữ: Chữ ký: Chữ ký:

* Ghi chú: Đề thi gồm 07 trang. Thí sinh làm bài trực tiếp vào đề thi.
PART I: PHONETICS (1 point)
I: Find a word in each line whose underlined part is pronounced differently from that of the
other three by circling A, B, C or D. (0.5 p)
1 A allegedly B confusedly C supposedly D wickedly
2 A youths B maps C cigarettes D months
3 A equation B television C mention D decision
4 A rise B rinse C browse D bruise
5 A substitute B muddled C shutter D substantial
II: Find the word with the stress pattern different from that of the other three words in each
question by circling A, B, C or D. (0.5 p)
1 A academic B amphibian C apartheid D aquarium
2 A tuberculosis B mathematician C inheritance D communicatio
3 A casualty B habitual C characterize D ignorance
4 A magnificent B memorial C tobacconist D humanism
5 A trigonometry B explanatory C immediately D democracy
PART II: LEXICO - GRAMMAR (7 points)
I: Choose the word or phrase which best completes each sentence. Circle the letter A, B, C
or D next to the correct word or phrase. (2 p)
1. …… every industry in our modern world requires the work of engineers.
A. Wholly B. Hardly C. Most D. Virtually
2. Jane had a problem with her finances, so we talked …… and now it's fine.
A. over B. it over C. over it D. over and over
3. When the electricity failed, he …… a match to find the candles.
A. rubbed B. scratched C. struck D. started
4. I usually buy my clothes …… . It‟s cheaper than going to the dressmaker.
A. on the house B. off the peg C. in public D. on the shelf
5. My father …… when he found out that I had damaged his car.
A. hit the roof B. saw pink elephants
C. made my blood boil D. brought the house down
6. According to the captain, his special units can take an immediate action against terrorists should such
a need ........... .
A. arise B. originate C. evoke D. experience
7. We were by the officers' decision to divert the whole traffic from the main route.
A. rambled B. baffled C. stumbled D. shuffled
8. The book says that the revolution was...............off by the assassination of the state governor.
A. launched B. cropped C. triggered D. prompted
9. The hijackers have demanded a.............to be paid for releasing the civilian hostages from the plane.
A. currency B. revenue C. deposit D. ransom
10. He‟s...................................work and cannot possibly see you now.
A. up to his ears in B. very interested in
C. not involved with D. concerned with
11. He suddenly saw Sue ..................... the room. He pushed his way ..................the crowd
of people to get to her.
B. across/through B. over/through C. over/along D. across/across
12. She tried to .......................
A. talk out of me the plan B. talk me the plan out of
C. talk me out of the plan D. talk out me of the plan.
13. My cousin obviously didn‟t ..................much of an impression on you if you can‟t
remember meeting her.
A. create B. do C. make D. build
14. She was kept awake for most of the night by the….............of a mosquito in her car.
A. whine B. moan C. groan D. screech
15. Her business must be going rather well,..............by the car she drives.
A. deducing B. deciding C. inferring D. judging
16. He looks very aggressive and threatening, and so his soft, gentle voice is rather………….
A. disembodied B. disconcerting C. dismissive D. discordant
17. If I were you, I would regard their offer with considerable…............., because it seems too
good to be true.
A. suspicion B. doubt C. reservation D. disbelief
18. My sister‟s confidence in her ability to play the piano was badly…............by her last music teacher.
A. subsided B. weakened C. undermined D. loosened
19. Your grandfather is rather tired so do not….............your visit. Let him have a rest.
A. prolong B. lengthen C. delay D. shorten
20. Their eventual choice of the house was… ..............by the time Peter would take to get to the office.
A. related B. consequent C. determined D. dependent
II. Put each verb given in brackets into an appropriate tense or form (1p)
In 1764 Dr. Johnson accepted the contract (1. produce) a dictionary. (2. rent) a garret, he took on a
number of copying clerks, who (3. stand) at a long central desk. Johnson (4. not have) a library
available to him, but eventually produced definitions of 40,000 words ( 5. write) down in 80 large
notebooks. On publication, the Dictionary immediately (6. hail) in many European countries as a
landmark. According to his biographer, James Boswell, Johnson‟s principal achievement was (7.
bring) stability to the English language: “It (8. be) the cornerstone of Standard English, an
achievement which (9. confer) stability on the language of his country”. As a reward for his hard
work, he (10. grant) a pension by the king.
1………..…… 2…………...……….. 3…………..………
4 ………..…… 5………..…………… 6.……..……………
7. ……….. 8. ………..…………… 9. ………..………… 10.
Give the correct form of the words in brackets (1 p)
1. Please (know)...................................our letter of the 25th. We have not had a reply.
2. Eating fish and lots of vegetables greatly increases your life (expect) ......................
3. It is very rude to interrupt someone in ……………………... (sentence)
4. Wow, I'm afraid I am not very (photo)..........................
5. The (forest)..........................................has caused many so-called man-made disasters.
6. All the.................from the last lecture were not allowed to attend the interview for the
coming project. (absence)
7. The road was (pass).................................because of the snow.
8. She spent hours getting the house (spot).............................clean.
9. Paul is a good employee, and is very ..................(conscience).
10. children will not be allowed to cross busy roads. (accompany)

Your answers:
1 ………..………... 2…………...……….. 3…………..………
4 ………..………… 5………..…………… 6.……..……………
7. ………..………... 8. ………..…………… 9. ………..………… 10.

III. The passage below contains 11 mistakes. (0) has been done for you as an example.
IDENTIFY and CORRECT the other ten. (1 p)
0. all complete --> completely
Things started to go wrong as soon as we got to the hotel. We were all complete exhausted after our
long journey and looking forward to shower and a rest. However, we found that our room has not
ready, which was very annoy, although the manager was extremely apologetic. While we were
waiting, we asked about the excursions to places of an interest which we had read about in
brochure. Imagine how we felt when we were told they had all cancelled! Apparently, the person
responsible for organise them had left suddenly and had not been replaced. Then Sally saw a notice
pinning to the door of the restaurant, saying it has closed for redecoration, and Peter discovered that
the swimming pool was empty. When we eventually got to our room we were horrified find that it
was at the back of the hotel, and we had a view of a car park, which seemed to be used as a rubbish
dump. We seriously began to wonder whether or not to stay.
Your answers:
1…………...……… --> . ………………........ 2. ………………… -->. ……....……………
3. ……..…………..... --> . ………….…........ 4. . ……………….. -->. ……………...…....
5. ……………..….. --> . . ………….…........ 6.………...……… --> . ………………........
7. ……………..….… -->. …….....…………… 8. ……..………….. --> . ……….….…........
9. . ……………...….. -->. …………….....….... 10. ……………..…. --> . . ………….…........
IV. Fill in each blank with one suitable preposition or particle (1 p).
1. Dishonesty is foreign.......................his nature.
2. Yuri Gagarin lifted …………… into space aboard the Vostok 1 at 9.07 a.m. Moscow
time…………… 12th April, 1961.
3. She was free to indulge......................leisure activity like reading.
4. Is it OK if I write......................pencil?
5. If we leave ………… the station ……… once, we arrive ………… ten minutes …………hand.
6. Her bright red hair made her stand...............from the others.
Your answers:
1 ………..…… 2…………...… - …………..… 3. ………..…… 4.………..……
5………….….. -…………..… - ………..…… -………..…… 6.. ...................
V. Insert the, a(n) or X (no article) where necessary (1 p).
I had long since prepared my mixture; I purchased at once, from (1)………… firm of wholesale
chemists, (2)……….. large quantity of (3)……….. particular salt, which I knew, from my
experiments, to be (4)........................................last ingredients required, and late one night, I mixed
(5) ………….. elements, watched them boil and smoke together in (6)…………. glass, and
when (7)………….. liquid had cooled, with (8)………… strong glow of (9)… courage,
drank off (10)….....................potion.

Your answers:
1 ………..…… 2…………...… 3…………..… 4 ………..…… 5………..……
6………….….. 7…………..… 8 ………..…… 9………..…… 10. ...................
PART III: READING (6 points)
I: Read the passage and use ONLY ONE suitable word to fill in each gap (2 p).
In a village on the east coast of Scotland, people were waiting for news. Two of fishing- boats had
been caught in the storm which had blown up during the night. In the cottages round the harbor
people stood by their doors (1) worried to talk.
The rest of the fishing fleet had (2) the harbor before dark, and the men from these ships waited
and watched with the wives and families of the missing men. Some had (3)
thick blankets and some flasks of hot drinks, knowing that the men
(4) be cold and tired. When dawn began to break
over in the east, a small point of light was (5) in the darkness of the water and a few minutes later,
(6) was a shout.
(6) long, the two boats were turning in, past the lighthouse, to the
inside of the harbor. The men (8)
helped out of their boats, and (9) they were stiff (10)
cold and tiredness, they were all safe.

Your answers:
1……….. 2…………. 3………… 4………… 5……………
6……….. 7…………. 8………… 9………… 10…………..
II: Read the passage carefully and then choose the best answer to each sentence by circling A,
B, C or D (1.5p)
While many nineteenth–century reformers hoped to bring about reform through education or
by eliminating specific social evils, some thinkers wanted to start over and remark society by
founding ideal, cooperative communities. The United States seemed to them a spacious and
unencumbered country where models of a perfect society could succeed. These communitarian
thinkers hoped their success would lead to imitation, until communities free of crime, poverty, and
other social ills would cover the land. A number of religious groups, notably the Shakers, practiced
communal living, but the main impetus to found model communities came from nonreligious,
rationalistic thinkers.
Among the communitarian philosophers, three of the most influential were Robert Owen, Charles
Fourier, and John Humphrey Noyes. Owen, famous for his humanitarian policies as owner of
several thriving textile mills in Scotland, believed that faulty environment was to blame for human
problems and that these problems could be eliminated in a rationally planned society. In 1825, he
put his principles into practice at New Harmony, Indiana. The community failed economically after
a few years but not before achieving a number of social successes. Fourier, a commercial employee
in France, never visited the United States. However, his theories of cooperative living influenced
many American through the writings of Albert Brisbane, whose Social Destiny of Man explained
Fourierism and its self-sufficient associations or “phalanxes”. One or more of these phalanxes was
organized in very Northern state. The most famous were Red Bank, New Jersey, and Brook Farm,
Massachusetts. An early member of the latter was the author Nathaniel Hawthorne. Noyes founded
the most enduring and probably the oddest of the utopian communities, the Oneida Community of
upstate New York. Needless to say, none of these experiments had any lasting effects on the
patterns of American society.
1. The main topic of the passage is……..
A. nineteen-century schools. B. American reformers
C. the philosophy of Fourierism D. model communities in the
nineteenth.
2. Which of the following is not given in the passage as one of the general goals of
communitarian philosophers?
A. To remake society B. To spread their ideas throughout the United State
C. To establish ideal communities D. To create opportunities through education.
3. The Shakers are mentioned in paragraph 1 as an example of…….
A. a communal religious group B. radical reformers
C. rationalistic thinkers D. an influential group of writers.
4. Which of the following is closest in meaning to the word impetus in paragraph 1?
A. Stimulus B. commitment C. Drawback D. Foundation.
5. The “phalanxes” described in paragraph 2 were an idea originally conceived by….
A. Albert Brisbane B. Robert Owen C. Charles Fourier D. John Humphrey Noyes
6. Why does the author mention Nathaniel Hawthorne in paragraph two?
A. He founded Brook Farm in Massachusetts.
B. He was a critic of Charles Fourier.
C. He wrote a book that led to the establishment of model communities.
D. He was at one time a member of the Brook Farm community.
7. Which of the following communities lasted longest?
A. New Harmony B. The Oneida Community C. Red Bank D. Brook Farm
8. The word oddest in paragraph 2 is closest meaning to which of the following?
A. Earliest B. Most independent C. Largest D. Most unusual
9. The author implies that, for readers, the conclusion of the paragraph is..........
A. obvious B. surprising C. absurd D. practical
10. Why did the author probably divide the passage into two paragraphs?
A. To compare nineteenth-century reforms with twentieth-century reforms.
B. To present an overview of a concept in the first paragraph and specific examples in the second.
C. To contrast the work of utopian thinkers with that of practical reforms.
D. To give the causes for a phenomenon in the first paragraph and its consequences in the second
III: Read the text and decide which word best fits each blank by circling the letter A, B, C or D (1.5p).
United Parcel Service (UPS) believes that its employees should give the firm a fair
day‟s
work for a fair‟s day pay. The package delivery firm seems willing to give more than a fair‟s day
pay. But in (1) , UPS expects maximum output from its employees.
Since 1920s, the firm‟s industrial engineers have been studying every detail of every task (2)
by most UPS employees. From their studies have come time and motion standards that
(3) how those tasks are performed and how long they should take. Drivers, for example, are
expected to walk to a customer‟s door at a speed of exactly three feet per second. They are told to
knock as soon as they get there, rather than (4) time looking for a doorbell.
Work engineers are (5) riding with drivers, timing everything from stops at traffic lights, to wait
at customers‟ doorway, to stairway climbs, to coffee break. And they are not (6) to pointing out
the occasional inefficiency. Additionally, supervisors ride with the least good drivers, noting
how they work and constantly (7) them until their work is up to standard.
The (8) of all this work engineering is efficiency, and UPS has been called one of the most
efficient companies anywhere. It's also a highly profitable company. Most drivers take the
regimentation in stride: many show (9) in meeting the UPS standards each day. Others,
however, feel that they are constantly being pushed, that it is impossible for them to
(10) at work. UPS officials claim that the standards provide accountability. And, they say,
employees who work according to UPS standards should feel less tired at the end of the day.
1: A. fact B. exchange C. return D. short
2: A. hold B. performed C. accepted D.
under 3: A. indicate B. govern C.
demonstrate D. tell 4: A. wasting B.
spend C. spending D. waste
5: A. consistently B. continually C. constructively D. chronically
6: A. impolite B. brave C. intimate D. averse
7: A. scolding B. criticizing C. encouraging D. correcting
8: A. task B. reason C. object D. target
9: A. pride B. passion C. interest D.
pleasure 10: A. rest B. relieve C. relax D. restrain

IV. Read through the following text and then choose the best phrase given below, to fill each of the
gaps. Write one letter (A-I) in each of the numbered gaps. Some of the suggested answers do not fit at
all. (0) has been done for you (1p).
Every teacher knows that not all students are good examinees. Some are too tense, become over- anxious or
too stressed and then perform below expectations just when it matters most.

Teachers try to help by compensating, believing that if they boost a student‟s academic knowledge
they will cure his fear of exams.
So, last year, (0) I_ , I completely rewrote the Business Studies Revision Course at this secondary school. The
central idea of the course is to treat the examination as an event, a challenge, a performance, much like a
sports match, a drama production, or perhaps a major music concert, (1)

and very definitely on the public stage. The idea is to show that the exam is not a test, but an
opportunity to show how good the candidate is.

The objective is to improve students‟ final performance (2) , control and ability to cope. The theme of
„total preparation for performance‟ teaches them that (3) are obviously important, they are only
two of the five skills required, the others being coping strategies, mental skills and management skills. These
additions give a new dimension (4) , increasing enjoyment and motivation. They widen a student‟s
focus and help to convince some of the less confident students

A those not mattering so much B self-confidence and self-esteem


C by increasing self-confidence D relying on my expertise alone
E to a student‟s revision F but a real desire
G while knowledge and examination techniques H but bigger and more important
I drawing on my teaching experience and sports psychology skills

PART FIVE: WRITING (6 points)


I. Rewrite the following sentences in such a way that the second sentence has the
same meaning as the first one (2p)
1. Something must be done quickly to solve the problem of homelessness.
--> Urgent ...............................................................................................................
2. Scientists have tried very hard to find a cure for this disease.
--> Enormous ..........................................................................................................
3. Just thinking about his face at the moment makes me laugh.
--> The very………………………………………………....……………...……....
4. The teachers agreed to introduce the new methods.
--> There was an ........................................................................................................
5. The boy does whatever his father wants in an obedient way.
--> The boy dances ................................................................................................
6. I simply fail to understand some of my colleague‟s attitudes to work.
-->I have some colleagues ........................................................................................
7. I did not realize how much he was influenced by his brother.
-->I did not realize the extent ..................................................................................
8. Mass tourism has been one of the causes of the environmental problems.
-->Mass tourism is ....................................................................................................
9. It was six months since I stopped subscribing to that magazine.
-->I cancelled .............................................................................................................
10. These books are on loan from the British Council library.
-->These books have .....................................................................................................

II: Rewrite the following sentences with the given words in such a way that the second
sentence has the same meaning as the first one. Do not change the form of the word in
brackets (2p)
1. I can‟t find the answer without a calculator. (out)
............................................................................................................................. .........................

2. My friend took no notice of my advice. (deaf )


.....................................................................................................................................................

3. These two makes of computer are practically the same. (hardly)


......................................................................................................................................................
4. His smooth manner didn‟t deceive us. (taken)
............................................................................................................................. .........................
5. Everyone who spoke to the victim is a suspect. (under)
................................................................................................................................. .....................
6. There‟s nothing new about crimes of passion (hills)
......................................................................................................................................................
7. The northwest of Britain has more rain each year than the southeast. (annual)
......................................................................................................................................................
8. From the educational point of view his childhood years had been well spent. (terms)
......................................................................................................................................................
9. Make yourself at home. (ceremony)
............................................................................................................................. .........................
10. She will probably be elected. (stands)
............................................................................................................................. .........................
III: Make all the changes and additions necessary to produce, from the cues given below, a
complete letter (2p)
Dear Rob and Randy,
1. all these weeks/ hospital/ I just/ receive two pieces/ good news.
............................................................................................................................. .........................
2. The doctor/ say/I / can / home / few days.
............................................................................................................................. .........................
3. My wife tell/ me / how you two/ been clearing/ snow / from our driveway and sidewalk.
............................................................................................................................. .........................
4. Have/ such/ good neighbours/ make / very happy.
......................................................................................................................................................
5. My wife/ tell/ you/ refuse / take / money /for your efforts.
......................................................................................................................................................
6. But I/ think/ I find / way round that.
............................................................................................................................. .........................
7. Please accept / enclosed check.
............................................................................................................................. .........................
8. I/ would/ never think/ as payment/ the snow shoveling.
......................................................................................................................................................
9. but as a sincere token/ appreciation/ your thoughtfulness.
......................................................................................................................................................
10. Thank/ again / see/ soon.
............................................................................................................................. .........................

Best regards,

- THE END -
HƯỚNG DẪN CHẤM ĐỀ THI CHON HỌC SINH GIỎI
MÔN TIẾNG ANH 12 - THPT
Total marks: 20

PART I: PHONETICS (1 point)


I: Find a word in each line whose underlined part is pronounced differently from that of the
other three by circling A, B, C or D. (0.5 p)
1B 2A 3C 4B 5D
II: Find the word with the stress pattern different from that of the other three words in each
question by circling A, B, C or D. (0.5 p)
1A 2C 3B 4D 5A
PART II: LEXICO - GRAMMAR (7 points)
I: Choose the word or phrase which best completes each sentence. Circle the letter A, B, C or D
next to the correct word or phrase. (2 p)
1D 2B 3C 4B 5A 6A 7B 8C 9D 10A
11A 12C 13C 14A 15D 16B 17C 18C 19A 20C

II. Put each verb given in brackets into an appropriate tense or form (1p)
1. to produce 2. Having rented 3. stood 4. did not have 5. written
6. was hailed 7. to bring 8. is 9. conferred 10. was

III. Give the correct form of the words in brackets (1 p)


1. acknowledge 2. expectancy 3. mid-sentence 4. photogenic 5. deforestation
6. absentees 7. impassable 8. spotlessly 9. 10.

IV. The passage below contains 11 mistakes. (0) has been done for you as an example.
IDENTIFY and CORRECT the other ten. (1 p) [0.1 p for each both identified and corrected
mistake]
1. all complete--> completely
1. shower a shower 2. has was 3. annoy annoying
4. an interest interest 5. in brochure the brochure 6. cancelled been
7. organise organising 8. pinning pinned 9. has closed was closed
10. horrified find horrified to find

V. Fill in each blank with one suitable preposition or particle (1 p).


1. to 2. off - on 3. in 4. in 5. for - at - with - in 6. out

VI. Insert the, a(n) or X (no article) where necessary (1 p).


1. a 2. a 3. a 4. the 5. the 6. the/a 7. the 8. a 9. 0 10. the

PART III: READING (6 points)


I: Read the passage and use ONLY ONE suitable word to fill in each gap (2 p).
1. too 2. reached/entered 3. brought/got 4. would 5. seen/ visible
6. there 7. Before 8. were 9. although/ though 10. with/ from
II: Read the passage carefully and then choose the best answer to each sentence by circling A, B,
C or D (1.5p)
1.D 2C 3A 4A 5C 6D 7B 8D 9A 10B
III: Read the following text and decide which word best fits each blank by circling the letter A, B,
C or D (1.5 p).
1C 2B 3B 4D 5B 6D 7D 8C 9A 10C

IV. Read through the following text and then choose the best phrase given below, to fill each of
the gaps. Write one letter (A-I) in each of the numbered gaps. Some of the suggested answers do
not fit at all. (0) has been done for you (1p).
0. I 1. H 2. C 3. G 4. E 5. B

PART FIVE: WRITING (6 points)


I: Rewrite the following sentences in such a way that the second sentence has the same
meaning as the first one (2p)
1. Urgent action must be taken to solve the problem of homelessness.
2. Enormous efforts have been made by scientists to find a cure for this disease.
3. The very thought of his face at the moment makes me laugh.
4. There was an agreement among the teachers to introduce the new methods.
5. The boy dances attendance on/upon his father.
6. I have some colleagues whose attitudes to work I simply fail to understand.
7. I did not realize the extent to which he was influenced by his brother.
8. Mass tourism is partly responsible/ to blame for the environmental problems.
9. I cancelled my subscription/subscribing to that newspaper six months ago.
10. These books have been lent by the British Council library.

II: Rewrite the following sentences with the given words in such a way that the second
sentence has the same meaning as the first one. Do not change the form of the word in
brackets (2p)
1. I can‟t work out the answer without calculator.
2. My friend turned a deaf ear to my advice.
3. There is/are hardly any difference(s) between these two makes of computer.
4. We were not taken in by his smooth manner.
5. Everyone who spoke to the victim is under suspicion.
6. Crimes of passion are as old as the hills
7. The annual rainfall in/for the northeast of Britain is higher than that in/for the southeast.
8. In terms of education, his childhood years had been well spent.
9. Don‟t stand on ceremony
10. She stands a (good) chance of being elected.
III: Make all the changes and additions necessary to produce, from the cues given below, a
complete letter (2p)
Dear Rob and Randy,
1. After all these weeks in (the) hospital I (have) just received two pieces of good news.
2. The doctor said/says that I could/can go home in a few days.
3. My wife told me how you two have been clearing the snow from our driveway and sidewalk
4. Having such good neighbours like you makes me very happy.
5. My wife told me you refused to take any money for your efforts.
6. But I think I've found a way round that.
7. Please accept this/ the/my/our enclosed check.
8. I would never think it as payment for the snow shovelling
9. but as a sincere token of our appreciation for your thoughtfulness.
10. Thanks again/Thank you again and see you
soon. Best regards,
SỞ GIÁO DỤC VÀ ĐÀO TẠO
KỲ THI CHỌN HỌC SINH GIỎI CẤP TỈNH
VĨNH PHÚC
LỚP 12 THPT NĂM HỌC 2020-2021
ĐỀ THI CHÍNH THỨC
Môn: TIẾNG ANH
Thời gian: 180 phút, không kể thời gian giao đề.
Ngày thi: 02/11/2020.
Đề thi gồm 05 trang.
PART A: LISTENING.
You are going to listen to this telephone conversation between JON and JANICE and answer
the questions. You should write NO MORE THAN THREE WORDS OR A NUMBER for
each answer.
You will hear the recording TWICE.

Areas dealt with: (1) , North suburbs

Rent: from (2) £ to £ 500 a


month
Depends on: the area

availability (3) .
garage

Properties available : West Park Road

rent (4) £ a
month including (5) bill
Tithe Road

rent £ 380 a month

including (6) rental

Viewing arrangements: meet at office on (7) afternoon at 5.00


pm

Need: letter from bank

reference from your (8) .

Must: give (9) notice of


PART B: PHONETICS.
moving in give deposit of (10)
Identify the word whose main stress is different from that of the others in each group.
1. A. interest
pay for contract B. important C. decision D. relation
2. A. damage B. arrange C. pocket D. nature
3. A. eternal B. essential C. immoral D. practical
4. A. begin B. polite C. attend D.
5. A. popularity B. investigate C. university D.

PART C: VOCABULARY AND GRAMMAR


I. Choose the right word or phrase to complete the sentences.
1. Julie has out of playing with dolls.
A. developed B. moved C. removed D. grown
2. Mike in for sailing after he made friends with Harry.
A. got B. went C. put D. came
3. The other runners were too far ahead for us to up with.
A. catch B. go C. get D. turn
4. This is road I’ve ever driven along.
A. the bumpy B. the bumpiest C. the bumpier D. bumpier
5. Jack better if he had had more time.
A. could have done B. could do C. couldn’t do D. did
6. Keith is taller than her brother.
A. lightly B. delicately C. sparely D. slightly
7. I must go to the dentist and .
A. get my teeth to take care of B. take care of my teeth
C. my teeth be taken care of D. get my teeth taken care of
8. The last time I went to the supermarket I ended buying all the things I didn’t really need atall.
A. in B. to C. by D. up
9. 20,000 people are thought to have attended the concert.
A. As much as B. More C. As many as D. Less than
10. I cross the Channel by boat, I feel seasick.
A. Though B. As soon as C. So that D. Whenever
II. Choose a word or phrase in each of the following sentences that needs correcting.
1. Families who are enough fortunate to own a historic home may be able to get restoration funds
A B C D
From the government.
2. The first things a new international student must do include renting an apartment,
A B
registering for classes, and to get to know the city.
C D
3. When parents allow his children to spend many hours watching television, the children are
A B
not likely to be physically fit.
4. Jessica is only an amateur, but she sings sweeter than most professionals.
A B
5. Mrs. Adams was surprising that her son and his friend had gone to the mountains to ski.
A B C D
III. Give the correct form of the word in blanket.
MOVING HOUSE
Moving house is said to be the third most stressful experience you can have (coming after the (1.
DIE) of a close relative, or a divorce). The reason for this is partly the (2- ORGANISE)
involved, but also the feeling of (3. SECURITY) caused by completely changing
your environment. Of course, a (4. SYSTEM) approach can help ease the difficulties,
especially on the day of (5. REMOVE) . Plan your packing
carefully or, better, employ a (6. RELY) company to pack and move
your things. This will (7. CERTAIN) lessen the amount of damage to your (8. POSSESS)
. It’s also a good idea to take out (9. INSURE) . Some worry is, of course, (10.
AVOID) but try to keep calm and look forward to life in your new home.
PART D: READING
I. Read the passage and fill each gap with ONE suitable word.
They call New York “the Big Apple”. Maybe it’s not (1) like an apple, but it’s certainly very big.
There are too many people, that’s the (2) . The street are always full of cars and trucks,
and you can never find a (3) to park.
If you have enough money, you can take a taxi. New York cabs are yellow. They look all the same.
But the drivers are very (4) . Some were born and
(5) up in New York, but many are (6) to the United States. A
few drive slowly, but most go very, very fast. Cab (7) _ is a difficult job. It can be
dangerous, too. Thieves often try to steal the drivers’ money. Drivers sometimes get hurt.
If you don’t want to take a taxi, you can go by bus or you can take a subway. The (8)
is quick, and it’s cheap, but parts of it are
old (9) dirty. Lights don’t always work and there
are often fires on the track. On some subway lines, there are new, clean, silver trains. But you can’t
see the color of the old trains easily. There is too much dirt and too much graffiti, inside and (10)
.
II. Choose the best answer from the four options marked A, B, C or D to complete each
numbered gap in the passage below.
School exams are, generally speaking, the first kind of tests we take. They find out how much
knowledge we have gained. But do they really show how intelligent we are? After all, isn’t it a (1)
that some people who are very academically successful don’t have any common sense?
Intelligence is the speed (2) which we can understand and react to new situation and it is
usually tested by logic puzzles. Although scientists are now preparing (3)
computer technology that will be able to ‘read’ our
brains, (4) tests are still the most popular ways of measuring
intelligence.
A person’s IQ is their intelligence (5) it is measured by a special test. The most common
IQ tests are (6) by Mensa, an organization that was founded in England in 1946. By 1976, it (7)
1,300 members in Britain. Today there are 44,000 in
Britain and 100,000 worldwide, largely in the US. People taking the tests are judged in (8) to an
average score of 100, and those who score over 148 are entitled to join Mensa. This works out at
percent of the population. Anyone from the age of six can take the tests. All the questions are
straightforward and most people can answer them if (9) enough time. But that’s the problem,
the whole (10) of the tests is that they’re against the clock.

1. A. case B. fact C. circumstance D. truth


2. A. on B. to C. in D. at
3. A. advanced B. forward C. ahead D. upper
4. A. at this age B. for the present C. at the time D. now and
5. A. how B. that C. as D. so
6. A. appointed B. commanded C. run D. steered
7. A. held B. had C. kept D. belonged
8. A. concerned B. relation C. regard D. association
9. A. allowed B. spared C. let D. provided
10. A. reason B. point C. matter D. question

III. Read the following passage and then answer the questions that follow by choosing A, B,
C or D.
Cholera, a highly infectious disease, has resulted in millions of deaths time after time over
centuries. It is caused by the bacterium Vibrio cholerae, first isolated by Robert Koch in 1883.
The organism enters the body through the digestive tract when contaminated food or water is
ingested. The bacteria multiply in the digestive tract and establish infection. As they die, they
release a potent toxin that leads to severe diarrhea and vomiting. This results in extreme
dehydration, muscle cramps, kidney failure, collapse and sometimes death. If the disease is treated
promptly, death is less likely.
In many countries, a common source of the organism is raw or poorly cooked seafood, taken from the
contaminated waters. The disease is especially prevalent after a natural disaster or other destruction that
results in a lack of fresh water. Sewer systems fail and waste travels into rivers or streams; piped water is
not available so people must take their drinking and cooking water from rivers or streams. Because
people frequently develop communities along waterways, the disease can be spread easily from one
community to the next community down streams, resulting in serious epidemics.
1. The word infectious in the first sentence is closest in meaning to _.
A. communicable B. severe C. isolated D. common
2. According to the passage, cholera is caused by .
A. a virus B. a bacterium C. kidney failure D. dehydration
3. All of the following are probable causes of infection EXCEPT .
A. eating food cooked with contaminated water B. eating undercooked seafood
C. eating overcooked pork D. eating raw oysters
4. According to the passage, what is a symptom of the infection?
A. Release of a toxin by the bacteria B. Regurgitation
C. Overeating D. Epidemics
5. Which of the following would be an appropriate title for this passage?
A. Dysentery and its effects B. Water Purification Systems and Their Importance
C. Results of Wars and Natural Disasters D. The Causes and Effects of Cholera
6. The word prevalent in the third paragraph is closest in meaning to .
A. dangerous B. commonplace C. unusual D. organized
7. The word lack in the third paragraph is closest in meaning to .
A. contamination B. multitude C. shortage D. well
8. According to the passage, cholera .
A. is easily passed from one person to another
B. is not a real threat
C. is no more dangerous than the common cold
D. cannot be passed from one to another by casual contact
9. What can you infer from the passage?
A. Careful cooking and hygiene practices can reduce the chance of getting the disease
B. Water mixed with other substances will not pass the disease
C. The respiratory system is the most common area of entrance
D. Kidney disease is the most common cause of the disease
10. The word epidemics at the end of the passage is closest in meaning to _ .
A. studies B. illness C. bacteria D. plagues
PART E: WRITING
I. Complete the second sentence so that it has the similar meaning to the first sentence.
1. I don’t intend to apologize to either of them.
→ I have ………………………………………………………………….
2. She has never been to the ballet before.
→ It is the ………………………………………………………………….
3. The result of the match was never in doubt.
→ At no time ………………………………………………………………
4. Under no circumstances should you phone the police.
→ The last ………………………………………………………………………..
5. “You broke my bicycle, Minh!” said Hoa.
→ Mary accused ………………………………………………………….
6. I didn’t know you were coming, so I didn’t wait for you.
→ If……………………………………………………………………….
7. When I picked up my pen I found that the nib had broken
→ On………………………………………………………………………
8. There is no danger that you will fall from the verandah
→ There is no danger of……………………………………………………
9. The sea was so cold that we couldn’t swim in it.
→ The sea was too …………………………………………………………
10. He felt too ill to get up.
→ He did not………………………………………………………………
II. Topic writing (from 150 to 200 words).
Write a paragraph about “The importance of family in a person’s life”.
Use specific reasons and examples to support your answers.
SỞ GIÁO DỤC VÀ ĐÀO KỲ THI CHỌN HỌC SINH GIỎI CẤP TỈNH LỚP 12
TẠO VĨNH PHÚC THPT NĂM HỌC 2020- 2021

Môn: TIẾNG ANH – THPT HƯỚNG DẪN CHẤM


(Gồm 02 trang)
PART A: LISTENING
1. (the) city centre/ center (itself) 6. (the) telephone/ phone
2. 250 7. Wednesday/ Wed
3. (a) garden 8. employer
4. 325 9. two/ 2 weeks’
5. (the) water 10. one/ 1 month(‘s) rent
PART B: PHONETICS
1. A 2. B 3. D 4. D 5. B

PART C: VOCABULARY & GRAMMAR


I. Choose the right word or phrase to complete the sentences.
1. D 2. B 3. A 4. B 5. A
6. D 7. D 8. D 9. C 10. D

II. Choose one underlined word or phrase that is incorrect.


1. B 2. D 3. A 4. C 5. A

III. Supply the correct form of the word in the blanket


2.
1. death 3. insecurity 4. systematic 5. removal
organisation/
6. reliable 7. certainly 8. possessions 9. insurance 10. unavoidable

PART D: READING (30 points)


I. Read the passage and fill each gap with ONE suitable word
1. exactly 6. newcomers
2. problem 7. driving
3. place 8. subway
4. different 9. and
5. grew 10. outside
II. Choose the best answer from the four options marked A, B, C or D to complete each
numbered gap in the passage below

1. B 2. D 3. A 4. B 5. C
6. C 7. B 8. B 9. A 10. B

III. Read the passage and choose the best answer)

1. A 2. B 3. C 4. B 5. D
6. B 7. C 8. A 9. A 10. D

PART E: WRITING
I. 10 points – 1p/ item
1. I have no intention of apologising to either of them/ I have no intention to apologise to either
of them.
2. It is the first time she has been to the ballet.
3. At no time was the result of the match in doubt.
4. The last thing you should/ ought to/ must do is (to) phone the police.
5. Hoa accused Minh of breaking her bicycle/ Hoa accused Minh of having broken her bicycle
6. If I had known you were coming, I would have waited for you.
7. On picking up my pen, I found that the nib had broken.
8. There is no danger of your falling from the verandah.
9. The sea was too cold for us to swim in.
10. He did not feel well enough to get up.

II. Topic writing:


- Well-organised –
- Few grammatical mistakes
- Various grammatical structures and vocabularies

* Total: 100 points.


SỞ GD - ĐT HẢI DƯƠNG KÌ THI CHỌN HỌC SINH GIỎI TỈNH LỚP 12 THPT
NĂM HỌC 2020-2021 MÔN THI: TIẾNG ANH
ĐỀ CHÍNH THỨC Thời gian làm bài: 180 phút (Không tính thời gian giao đề)
(Đề thi có 08 trang)

Chú ý: Thí sinh làm bài vào tờ giấy thi


Phần trắc nghiệm: Chỉ cần viết đáp án A, hoặc B, C, D Phần tự luận: Viết
đầy đủ theo yêu cầu của bài (Thí sinh không được sử dụng bất cứ tài liệu gì)

A. LISTENING:
Hướng dẫn làm bài Nghe:
Bài Nghe gồm có 04 phần, mỗi phần thí sinh được nghe 02 lần.
Mở đầu và kết thúc phần nghe có tín hiệu nhạc.
Mọi hướng dẫn cho thí sinh (bằng tiếng Anh) có trong bài.
I. Listen to five short conversations and choose the correct answer to each question (5.0 points).
1. How will they book their flights?
A. on the Internet B. at the travel agent C. on the phone
2. What has the daughter forgotten to bring on holiday?
A. toothpaste B. hairbrush C. shampoo
3. What will the man and the woman do on Sunday?
A. do the housework B. go on a picnic C. watch DVDs and read
newspapers
4. Which blouse does the girl decide to buy?
A. a collar, long sleeves B. no collar, short sleeves C. a collar, without sleeves
5. When is the girl having a party?
A. 15th B. 14th C. 13th
II. Listen to the recording and choose the correct answer to each question (6.0 points).
6. What made Holly start writing when she was young?
A. keeping a diary each day C. completing a homework project
B. wanting to remember her dreams
7. Holly tells the story about the starfish to show .
A. how to enjoy wildlife without hurting it
B. how to keep wild animals at home
C. how to find the most interesting animals in the wild`
8. What does Holly think is the most interesting thing about nature?
A. that animals live so close to us. C. that there is so much to learn.
B. that nature is so beautiful.
9. What does Holly say about trees?
A. They have lots of uses. C. They are everywhere
B. They are often unnoticed.
10. Holly hopes that her young readers .
A. go on to study nature at college or university
B. improve their genneral reading skills
C. learn how to discover nature for themselves
11. What kind of books does Holly enjoy readinPgaigne h4eorf s5pare time?
A. classic novels B. cookery books C. field guides
III. Listen to the recording and fill in each blank with NO MORE THAN 3 WORDS /
NUMBERS (5.0 points).

SUNNINGTON SPORTS CAMP

Choice of afternoon activity:


Sign list outside the (12)

Clothes:
Wear track suit, but also bring shorts and a T-shirt

(13) sports shoes

Food:
Lunch served in canteen every half-hour bewteen 12.15 and (14)

IV. Listen to the recording and state whether the statements are true (T) or false (F) (4.0
points).
Snack bar sells drinks, chocolate and biscuits T F
17. Maria would like to travel a lot as part of her job.
Certificate:
18. Maria is confident she will be able to work for an airline.
19. Rolf intends toMarks
do a job connected
given for: with his degree.
20. Maria and Rolf agree it is important to have a good salary
- Attitude: (15) and team-work
B. GRAMMAR – -VOCABULARY
Performance: strength, speeds and skill
– LANGUAGE FUNCTIONS
- Level six certificate to anyone who gets (16) or more.
I. Choose the word, phrase or expression which best completes each sentence (8.0 points).
21. The answer Henry gave was so confusing that his lecturer could hardly make of it at
all.
A. sense B. meaning C. interpretation D. intelligibility
22. Considered America’s first great architect, .
A. many of the buildings at Harvard University were designed by Henry Hobson
Richardson B. it was Henry Hobson Richardson who designed many of
the buildings at Harvard University
C. Henry Hobson Richardson designed many of the buildings at Harvard University
D. Harvard University has many buildings that were designed by Henry Hobson Richardson.
23. “Jack, can you help me push the piano to the corner of the hall to our party?” said
the teacher
A. give place to B. make place for C. take room for D. make room for
24. The surrealistic movement in art in the 1920s and 1930s placed is pictured in the
unconscious and often incorporated dreamlike images.
A. to emphasize it B. an emphasis on what
C. an emphasis on it D. emphasize what
25. “Frankly, Ms. Adamson works that her figures never need ,” said the General
Manager.
A. such efficiently that / to check B. so efficient that / checking
C. such an efficient that / to be checked D. so efficiently that / to be checked
26. It is highly recommended that language learners in general and English learners
should learn the language in the native community so as to master it.
A. in particular B. on the whole C. in short D. on the other hand
27. Lama: “What’s your greatest phobia, Carolina?
Carolina: “ ”
A. I’m afraid not. B. I haven’t made up my mind.
C. What an absurd idea! D. Worms, definitely.
28. Waiter: “How would you like your steak, sir?
Man: “ ”
A. Not too bad B. Very good C. Rare, please D. Yes, of
course I like it
II. Give the correct form of the words in brackets (5.0 points).
29. The (PREDICT) weather in the mountains can make climbing extremely hazardous.
30. Both sides have shown a distinct lack of (ENTHUSIAST) for discussion.
31. The woman tried to break into a security file for which she was not (AUTHORITY) .
32. Life (EXPECT) for both men and women has improved greatly in the past 50 years.
33. Julie Johnson and Mark Thomas, aged 19 and 20 (RESPECT) , are taking part in the
competition.
III.Choose the underlined word or phrase in each sentence that needs correcting (3.0 points).
34. The American architect Frank Lloyd developed a theory of architecture stressed the needs of the
A B C D

people who used it.

35. Children subjected to violence, exploration, abuse, and neglect are in risk of death, poor
A B C

physical and mental health, HIV/AIDS infection, and educational problems.


D
36. The works of early American woodcarvers had many artistic qualities, but these craftsmen
A B
probably did not think of them as artists.
C D

IV.Choose the word that is CLOSEST in meaning to the underlined word in the following
sentences (2.0 points).
37. Sarah stood there, looking at his boyfriend without showing any reaction when he told her
the news.
A. caringly B. indifferently C. heartlessly D. impersonally
38. Martha decided to remain celibate and devoted her life to helping the homeless and orphans.
A. married B. separated C. single D. divorced
V. Choose the word that is OPPOSITE in meaning to the underlined word in the
following sentences (2.0 points).
39. Written sources are considered absolutely indispensable for today’s history teaching.
A. unique B. unnecessary C. inexpensive D. affordable
40. The court concluded from the evidence that Mr. Smith was innocent and released him from prison.
A. guilty B. benevolent C. innovative D. naive
C. READING:
I. There are FIVE blanks in the passage below. From the phrases / sentences in the box,
choose the most suitable for each blank. There is ONE EXTRA option that you DO NOT
need to use (5.0 points)

A. by attracting as many as several hundred thousand international students to their countries


B. so are not classed as international students
C. as defined by the Immigration Control and Refugee Recognition Act
D. in accordance to its own national education system
E. preferred by international students are the United States, United Kingdom, Germany, France,
Canada and Australia
F. by sending hundreds of thousand students to study abroad

The definition of "international student" varies in each country (41) _. In Australia, international
students are defined as those studying onshore only with visa subclasses 570 to 575, excluding
students on Australian-funded scholarships or sponsorship or students undertaking study while in
possession of other temporary visas. New Zealand citizens do not require a visa to study in
Australia, (42) . In Japan, international students are defined as foreign nationals who
study at any Japanese university, graduate school, junior college, college of technology,
professional training college or university preparatory course on a “college student” visa, (43) .
According to the United Nations Educational, Scientific and Cultural Organization (UNESCO) in
their 2009 World Conference on Higher Education report, over 2.5 million students were studying
outside their home country. UNESCO also predicted that the number of international students
might rise to approximately 7 million by the year 2020. The main destinations (44) . Overall,
the number of international students more than doubled to over 2 million between 2000 and 2007.
However, the sharpest percentage increases of international students have occurred in New Zealand,
Korea, the Netherlands, Greece, Spain, Italy and Ireland. In recent years, some Asian and Middle East
countries have started to attract more international students. These regions have entered the market with
declared ambitions to become regional education centers (45) -- .
II. Choose the most suitable heading from the box for each paragraph below. There is
ONE EXTRA heading that you DO NOT need to use (5.0 points).

HEADINGS

A. The amount that American eat


B. Americans even getting fatter
C. Changing meal times everywhere
D. Why Americans love fast food
E. Unhealthy American schools
F. How American schools deal with obesity among
teenagers
46:
In the past year, the adult obesity rate rose in 48 of America’s 52 states. Despite the campaigns to
make people aware of the dangers of over-eating, about 119 million, or 64% of US adults are either
overweight or obese. Worryingly, this figure is predicted to rise to 73% by the end of this year.
What does this mean for Already 300,000 deaths each year are caused by obesity (only smoke
kills more people), and the annual cost to the country is around $100 billion.
47:
Many people blame for the fast food industry, along with sedentary life styles, for the worrying
increase. The average American now consumes about three hamburgers and four portions of French
fries every week. That’s 90 grams of fat and 2.250 calories (the average person needs about 2.000
calories a day). A recent survey found that every month 90% of America children eat at least one
meal in fast food restaurant.
48:
Most Americans know that fast food isn’t good for them. It is high in calories, fat, salt and sugar.
Yet fast-food restaurants are part of American life. After World War II, higher salaries and
productivity, together with technological developments and increased consumerism, made fast food
restaurants popular. It presented a modern lifestyle, and is still a popular place for teenagers to get
together with friends. The food is cheap, and many say they love the taste. The advertising is often
aimed at children, with offers of entertainment and free gifts.
49:
Another in fast food sales – not only in the USA, but all over the world – is our increasingly busy
lifestyles. In the past, families ate a home-cooked meal together at dinner time. These days, meals
can be eaten at any hour of the day, and people rely on easy, instant food – not always nutritious –
that is available “24/7”. America invented the “TV dinner”, and it is something most people around
the world now enjoy.
50:
Finding nutritious food is also difficult in schools, which is often have contracts with fast food
supplies and drinks companies. Teenage boys in the US each drink an average of 868 cans of fizzy
drinks per year. Exercise in school has become less important too. A recent report showed that half
of all US teens aged 12 – 21 got only America’s problem – Europe is also getting faster. It is
estimated that over one million children in the UK are obese and this number has trebled over the
past 20 years.
III. Read the passage below and decide which option A, B, C or D best fits each space (10 points).
Childhood is the time when there are (51) responsibilities to make life difficult. If
a child has good parents, he is fed, looked after and loved, (52) he may do. It is
improbable that he will ever again in his life be given so much without having to do anything in
return. In (53) , life is always presenting new things to the child - things that have lost
their interest for older people because they are too well-known. A child finds pleasure playing in
the rain, or in the snow. His first visit to the seaside is a marvellous adventure. (54) _, a child
has his pain. He is not so free to do as he wishes as he thinks older people are: he is continually
told not to do things, or being punished (55) what he has done wrong. His life is
therefore not perfectly happy.
When a young man starts to (56) his own living, he becomes free from the discipline of school
and parents, but at the same time he is forced to accept responsibilities. He can no longer expect
(57) to pay for his food, his clothes and his room, but he has to
work if he wants to live comfortably. If he spends most time playing about in a day he used to as a
child, he will be hungry. And if he breaks the laws of society as he used to break the laws of his
parents, he may go to prison. If, however, he works hard, keeps out of troubles and has good health,
he can have the greatest happiness of seeing himself make steady (58) in his job
and of building up for himself his own position in society.
Old age has always been thought of as the worst age to be, but it is not necessary for the old to be
unhappy. With old age (59) 5 d the ability to help others with advice wisely
given. The old can have the joy of seeing their children making progress in life: they can watch
their grandchildren growing up around them, and perhaps best of all, they can, if their life has been
a useful one, feel the happiness of having come (60) the battle of life safely and of having
reached a time when they can lie back and rest, leaving others to continue the fight.
51. A. little B. few C. a little D. a few
52. A. whenever B. wherever C. whatever D. whosoever
53. A. short B. term C. reality D. addition
54. A. However B. But C. Therefore D. Moreover
55. A. because B. for C. at D. by
56. A. take B. have C. create D. earn
57. A. other B. another C. others D. someone
58. A. progress B. achievement C. improvements D. accomplishments
59. A. coming B. come C. came D. had come
60. A. out B. across C. through D. back
IV. Read the following passage and choose the best answer to each question (10 points).
Paul Bunyan is perhaps America’s best-known folk hero. A fictional logger of
incredible strength, he was most likely based on an actual nineteenth-century logger
from the northern United States or Canada. As a folk hero, he struck a chord with
Line Americans on some level, perhaps because he was incredibly strong but also because he
(5) was hard-working and capable, ingenious in solving problems, and fun-loving.
Though there is evidence that Paul Bunyan tales were part of oral tradition in the
nineteen century, Paul Bunyan stories did not appear in written form until the early
twentieth century. Journalist James McGillivray included descriptions of Bunyan in a
series of essay entitled “The Round River Drive” which appeared in a number of
(10) Midwestern newspapers between 1906 and 1910. However, it was through an extensive
advertising campaign that Paul Bunyan moved solidly into print.
Recognizing the appeal of Paul Bunyan as a figure for his company’s advertising,
William Laughead, an advertising executive for the Red River Lumber Company,
initiated a campaign that consisted of a series of publications featuring Paul Bunyan.
(15) For several decades, the company distributed these publications free of charge and
made no attempt to obtain a copyright on them. In fact, the company vigorously
encouraged other writers to make use of Paul Bunyan because it felt that the use of this
character enhanced the name recognition of the Red River Lumber Company inasmuch
as the name of the folk hero and the name of the company had become interwoven.
(20) The Bunyan stories published by Red River and further circulated by others were tall
tales of gigantic proportions. In these tales, Banyan is depicted as a man of superhuman
proportions, who is strong, hard-working, entrepreneurial, and innovative. In one story,
for example, Paul is credited with digging the Great Lakes in order to create a watering
hole for his giant ox, Babe. In another of these tales, Paul caused an entire winter of
(25) blue snow to fall by swearing a blue streak after he injured himself by smashing his
thumb with a large hammer. A third story in the series describes Paul’s role in
establishing the Mississippi River.
Fascination with Paul Bunyan has continued to grow, and today he is a standard of
American folklores. The prevalence of Bunyan as a figure of folklore today is
evidenced by references to him in countless stories, cartoons, poems, and songs as well
as the numerous community festivals and logging competitions featuring Paul Bunyan
that can be found throughout the sections of the country where
logging has strong tradition.
61. The purpose of this passage is to .
A. present the actual feats of a real-life
B. provide an overview of American folktales
C. describe logging in North America
D. discuss a “larger than life” folk hero
62. It is NOT stated in the passage that Paul Bunyan is known for his .
A. unusual strength B. dedication to work
C. ingenuity in difficult situations D. serious nature
63. The passage states that Paul Bunyan tales first appeared .
A. in oral stories B. in advertising C. in newspapers D. in a series of essays
64. Which of the following CANNOT be inferred about the Red River Lumber Company’s
advertising campaign featuring Paul Bunyan?
A. It endured for quite a time.
B. The company did not protect its ownership of the stories.
C. The campaign did little to enhance the company’s profitability.
D. The company wanted the name Paul Bunyan to be known as widely as possible.
65. The pronoun “them” in line 14 refers to .
A. publications B. series C. decades D. writers
66. The word “interwoven” in line 17 could be best replaced by .
A. unfashionable B. mixed together C. not compatible D. too separate
67. Where in the passage does the author discuss a weather phenomenon that Paul Bunyan
supposedly caused?
A. Lines 3-5 B. Lines 14-17 C. lines 20-23 D. Lines 25-29
68. The word “countless” in line 27 could be best replaced by the expression .
A. an overestimated number of B. an insubstantial number of
C. a large number of D. a specified number of
69. Which paragraph describes the plots of some of the tales of Paul Bunyan?
A. The second paragraph B. The third paragraph
C. The fourth paragraph D. The fifth paragraph
70. The author’s tone in this passage is .
A. humorous B. neutral C. sarcastic D. pessimistic
D. WRITING:
I. Finish each of the following sentences in such a way that it means exactly the same as the
sentence printed before (5.0 points).
71. Something must be done to help the victims of the earthquake in Mexico.
Urgent
……………………………………………………………………………………………………
……………..………………………………………………………………………………………
……
72. It was careless of you to allow your 16-year-old son to drive your car.
You should
……………………………………………………………………………………………………
…………….……………………………………………………………………………………
73. I shall never lend John any money, no matter what happens.
Under no
………………………………………………….…………………………………………………
………………….…………………………………………………………………………………
74. Someone has suggested abolishing income tax.
It
…………………………………………………………………………………………….………
……………….………………………………………….………………………………………
75. Thank you for reminding me about the timetable, otherwise I would have missed the last
train.
 If……………………..……………………………………………………………………….……
……………………………………………………………….……………………………………………….
……

II. This is a part of a letter that you have received from David, a pen friend from New

I cannot decide whether I should go to Vietnam to work after


my graduation. Can you give me some advice?
Zealand. In about 100 - 120 words, write a letter to answer his questions (10 points).

You are required to begin and finish your letter as followed:


Dear David,
………………………………………………………………………………………………
……………………………………………..………………………………………………………
…………………
………………………………………………………………………………………………
……………………………………………..………………………………………………………
…………………
………………………………………………………………………………………………
……………………………………………..………………………………………………………

Yours, Tran Thuy Hanh
III. Vietnamese society has changed significantly in the past decades. However, many
traditional values and practices have been lost, which is a pity. To what extent do you
agree or disagree with this statement? In about 230 - 250 words, write an essay to support
your opinion (15 points).

=== THE END ===


UBND TỈNH HẢI DƯƠNG ĐÁP ÁN VÀ BIỂU ĐIỂM
SỞ GIÁO DỤC VÀ ĐÀO ĐỀ THI CHỌN HỌC SINH GIỎI TỈNH LỚP 12 THPT
TẠO NĂM HỌC 2020-2021
MÔN THI: TIẾNG ANH
ĐÁP ÁN CHÍNH THỨC

A. LISTENING:
I. (5.0 points): 1.0 point for each correct answer:
1. B 2. C 3. B 4. A 5. C
II. (6.0 points): 1.0 point for each correct answer:
6. A 7. A 8. C 9. B 10. C 11. C
III. (5.0 points): 1.0 point for each correct answer:
12. changing room 13. Two / 2 pairs of
14. 1.45 15. effort 16. 180 marks
IV. (4.0 points): 1.0 point for each correct answer:
17. T 18. F 19. F 20. T
B. GRAMMAR – VOCABULARY – LANGUAGE FUNCTIONS
I. (8.0 points): 1.0 point for each correct answer:
21. A 22. C 23. D 24. B
25. D 26. A 27. D 28. C
II. (5.0 points): 1.0 point for each correct answer:
29. UNPREDICTABLE 30. ENTHUSIASM
31. AUTHORIZED / SED32. EXPECTANCY 33. RESPECTIVELY

III. (3.0 points): 1.0 point for each correct answer:


34. C 35. C 36. D
IV. (2.0 points): 1.0 point for each correct answer: 37. B 38. C
V. (2.0 points): 1.0 point for each correct answer: 39. B 40. A
C. READING:
I. (5 points): 1.0 point for each correct answer:
41. D 42. B 43. C 44. E 45. A
II. (5 points): 1.0 point for each correct answer:
46. B 47. A 48. D 49. C 50. E
III. (10 points): 1.0 point for each correct answer:
51. B 52. C 53. D 54. A 55. B
56. D 57. C 58. A 59. B 60. C
IV. (10 points): 1.0 point for each correct answer:
61. D 62. D 63. A 64. C 65. A
66. B 67. C 68. C 69. C 70. B
D. WRITING:
I. (5.0 points): 1.0 point for each correct answer:
71. Urgent actions must be taken to help the victims of the earthquake in Mexico.
72. You should not have allowed your 16-year-old son to drive your car.
73. Under no circumstances will / shall I lend John any money.
74. It has been suggested that income tax (should) be abolished.
75. If you had not reminded me about the timetable, I would have missed the last train.
II. (10 points):
1. Form: Informal letter (1.0 point)
2. Task fulfillment:
- Task completed with relevant - Well-organized (2.0 point)
3. Language:
- Accurate grammar (2.0 points) - Correct spelling and punctuation (1.0 point)
- Appropriate vocabulary (2.0 points)
III. (15 points):
Score of 14-15: An essay at this level:
 shows very effective writing skills
 is very well organized and well developed
 uses details clearly and properly to support a thesis or illustrate ideas
 displays consistent ability in the use of language
 demonstrates variety in sentence structure and proper word choice
Score of 11 - 13: An essay at this level:
 shows effective writing skills
 is well organized and well developed
 uses details clearly and properly to support a thesis or illustrate ideas
 displays consistent ability in the use of language
 demonstrates variety in sentence structure and proper word choice
Score of 9 - 10: An essay at this level:
 may address some parts of the task more effectively than others
 is generally well organized and developed
 uses details to support a thesis or illustrate an idea
 displays ability in the use of the language
 shows some variety in sentence structure and range of vocabulary
Score of 7 - 8: An essay at this level:
 addresses the writing topic adequately but does not meet all of the goals of the task
 is adequately organized and developed
 uses some details to support a thesis or illustrate an idea
 shows adequate but possibly inconsistent ability with sentence structure
 may contain some usage errors that make the meaning unclear
Score of 5 - 6: An essay at this level may reveal one or more of the following weaknesses:
 inadequate organization or development
 poor choice of details or does not provide enough details to support or illustrate
generalizations
 a noticeably improper choice of words or word forms
 numerous errors in sentence structure and/or usage
Score of 3 - 4: An essay at this level is seriously flawed by one or more of the following weaknesses:
 serious disorganization or underdevelopment
 little or no detail, or irrelevant specifics
 serious and frequent errors in sentence structure or usage
 serious problems with focus
Score of 1- 2: An essay at this level:
 may be incoherent
 may be undeveloped
 may contain severe and persistent writing errors
Score of 0: An essay will be rated 0 if it:
 contains no response
 merely copies the topic is off-topic, written in a foreign language or consists only of keystroke characters
TRƯỜNG THPT NÔNG CỐNG I ĐỀ THI THỬ KHẢO SÁT CHẤT LƯỢNG ĐỘI
TỔ TIẾNG ANH TUYỂN HỌC SINH GIỎI CẤP TỈNH LẦN 4
MÔN THI: TIẾNG ANH 12
ĐỀ CHÍNH THỨC Thời gian làm bài: 180 phút, không kể thời gian
phát đề.
(Đề thi có 12 câu, gồm 4 trang)

Part A : PHONETICS ( 5pts )


I. Choose the word whose underlined part is pronounced differently from the rest in the same
line. (2pts)
1. A. teenage B. dosage C. voyage D. carriage
2. A. monkey B. hunger C. hunter D. banker
II. Identify the word whose stressed pattern is different from that of the others. (3pts)
3. A. instrument B. indispensable C. stimulate D. symphony
4. A. preserves B. tropical C. climate D. temperature
5. A. satisfaction B. mathematics C. opportunity D. mathematician

Part B : LEXICAL AND GRAMMAR ( 45 pts )


I. Choose the option A, B, C or D to indicate the word or phrase which best completes each
sentence.(15pts)
6. The criminal was sentenced to death because of of his crime.
A. the severity B. the complexity C. a punishment D. the importance
7. “ So how are things at school, Tim? “ ”
A. Oh, pretty good, actually . B. Well, I can’t agree with you.
C. It’s my pleasure. D. I was not very good at it.
8. Laura is very thin, her young sister, who is quite heavy.
A. dissimilar to B. unlikely C. unlike D. dislike
9. Please and see us some time. You’re always welcome.
A. come to B. come about C. come away D. come around
10. The proposal has not met with .agreement.
A. voluntary B. universal C. informal D. effective
11. A child's vocabulary .through reading.
A. expands B. expends C. expels D. exposes
12. We friends even after we grew up and left home.
A. became B. made C. struck up D. remained
13. This picture book, the few pages are missing, is my favorite.
A. for which B. of that C. to which D. of which
14. I am _ tired to think about that problem now.
A. much more B. far too C. nearly D. very
15. My neighbour me a cup of coffee.
A. begged B. suggested C. offered D. invited
16. Why don’t the police take measures against crime?
A. effective B. affective C. ineffective D. effecient
17. What happened their car broke down on the motorway so they didn’t get to Jo’s wedding on time.
A. to be that B. being that C. was that D. to that
18. War stole his youth and his home, .
A. as a result B. moreover C. however D. furthermore
19.- “Do you like frog’s legs?” - “ them, I don’t really know.”
A. Never try B. Not to have try C. Never having tried D. Never had tried
20. My friend has for a bargain.
A. a sharp ear B. a keen eye C. a trong head D. a keen ear

II. The following passage contains 10 errors. Find and correct them (10pts)
Who were the people responsible for collection and sending plants from one country to 21
another? And why did they do it? Initially they were travelers with other purposes:
22
traders, colonists, pilgrims and missionaries have all been important in providing new
plants for English gardens. They sent back indigenous wild plants, or sometimes, as in 23
the cases of visitors to China and Japan, plants which have been cultivated and
24
improved for hundreds of years. This worked, of course, in both directions: English
gardens were making in the most unlikely places. Travelers did not always recognize an 25
interesting plant on seen it – interesting, that is, to the collector at home. So in the 16th
and 17th century, attempts were made to collect on a most professional basis, either by 26
patrons sending collections into the field, or by subscriptions to finance local 27
enthusiasts in the most promised areas. By 1611 John Tradescant was traveling and
collecting in France and other parts of Europe. Lately, Peter Collinson, a London 28
merchant, who had seen the richness of the plant material sending back by Tradescant, 29
organized a syndicate to finance the amateur botanist John Bartram. Before long,
special collectors were being dispatched to all parts of the world by institutions such as 30
the Chelsea Physic Garden.
III. Put each verb in brackets into an appropriate form. (10pt )
31. A - Stop! You (not see) the notice?
B.- I see it but I can’t read it. What it (say) ?
32. That man was a little mad. He always (try) to improve that the earth was flat.
33. Something tells me that you (not listen) to a single word I have said in the past ten
minutes.
34. Hi, Paul! What a nice surprise! I (not think) I (run) into you today.
35. I’m so confused that I don’t know what (do)
36. Over the last few months, garages (put) up the price of petrol four times.
37. Smith had a lucky escape. He (kill) _.
38. The two men never (introduce) formally stared at each other across the table.
IV. Put each word in brackets into an appropriate form. (10pt)

Part C : READING ( 30 pts )


I. Read the following passage and then choose the most suitable word or phrase for each
space. (10 pt)
THE LANGUAGE OF TEARS
The ability to weep is a uniquely human form of emotional response. Some scientists have
suggested that human tears are evidence of an aquatic past - but this does not seem very likely. We
cry from the moment we enter this (49) for a number of reasons. Helpless babies
cry to persuade their parents that they are ill, hungry or uncomfortable. As they (50)
they will also cry just to attract parental attention and will often stop when they get it.
The idea that having a good cry can do you (51) is a very old one and now it has scientific (52)
since recent research into tears has shown that they contain a natural
painkiller called enkaphalin. By (53) sorrow and pain this chemical helps you to feel
better. Weeping can increase the quantities of enkaphalin you (54) .
Unfortunately, in our society we impose restrictions upon this naturally (55)
activity. Because some people still regard it as a (56) of weakness in men, boys in
particular are admonished when they cry. This kind of repression can only increase stress, both
emotionally and physically.
Tears of emotion also help the body (57) itself of toxic chemical waste, for there is
more protein in them than in tears resulting from cold winds or other irritants. Crying comforts,
calms and can be very enjoyable - consider the popularity of the highly emotional films which are
commonly (58) "weepies". It seems that people enjoy crying together almost as
much as laughing together.
49. A. world B. place C. earth D. space
50. A. evolve B. change C. develop D. alter
51. A. better B. fine C. good D. well
52. A. validity B. truth C. reality D. reason
53. A. struggling B. fighting C. opposing D. striking
54. A. construct B. achieve C. provide D. produce
55. A. curing B. treating C. healing D. improving
56. A. hint B. symbol C. feature D. sign
57. A. release B. rid C. loosen D. expel
58. A. named B. entitled C. subtitled D. called
II. Supply the most suitable word for each blank. (10 pts)
True relaxation is most certainly not a matter of flopping down in front of the television with a
welcome drink. Nor is it about drifting (59) an exhausted sleep. Useful though these responses to
tension and over-tiredness (60) be, we should distinguish between them and conscious relaxation in
(61) of quality and effect. (62) of the level of tiredness, real relaxation is a state of alert yet at
the same time passive awareness, in which our bodies are (63)
rest while our minds are awake.
Moreover, it is as natural for a healthy person to be relaxed when moving as resting. (64)
relaxed in action means we bring the appropriate energy to everything we do, so as to have a
feeling of healthy tiredness by the end of the day, (65) than one of exhaustion.
Unfortunately, as a result of living in today’s competitive world, we are under constant strain and
have difficulty in coping, (66) alone nurturing our body’s abilities. What needs to be rediscovered
is conscious relaxation. With (67) in mind we must apply ourselves to understanding stress and the
nature of its causes (68) deep-seated.
III. Read the following passage, and then choose the best answer from A, B, C or D.(10pts)
We find that bright children are rarely held back by mixed-ability teaching. On the contrary, both
their knowledge and experience are enriched. We feel that there are many disadvantages in
streaming pupils. It does not take into account the fact that children develop at different rates. It
can have a bad effect on both the bright and the not-so-bright child. After all, it can be quite
discouraging to be at the bottom of the top grade!
Besides, it is rather unreal to grade people just according to their intellectual ability. This is only
one aspect of their total personality. We are concerned to develop the abilities of all our pupils to
the full, not just their academic ability. We also value personal qualities and social skills, and we
find that mixed-ability teaching contributes to all these aspects of learning.
In our classrooms, we work in various ways. The pupils often work in groups: this gives them the
opportunity to learn to co-operate, to share, and to develop leadership skills. They also learn how to
cope with personal problems as well as learning how to think, to make decisions, to
analyze and evaluate, and to communicate effectively. The pupils learn from each other as well as
from the teachers.
Sometimes the pupils work in pairs; sometimes they work on individual tasks and assignments, and
they can do this at their own speed. They also have some formal class teaching when this is
appropriate. We encourage our pupils to use the library, and we teach them the skills they need in
order to do this effectively. An advanced pupil can do advanced work; it does not matter what age
the child is. We expect our pupils to do their best, not their least, and we give them every
encouragement to attain this goal.
69. In the passage, the author’s attitude towards “mixed-ability teaching” is .
A. objective B. questioning C. critical D. approving
70. The words “held back” in 1st paragraph means “ ”.
A. made to lag behind in study B. forced to study in lower classes
C. prevented from advancing D. made to remain in the same classes
71. The author argues that a teacher’s chief concern should be the development of the pupils’
.
A. total personality B. learning ability and communicative skills
C. intellectual abilities D. personal and social skills
72. Which of the following is NOT mentioned in the passage?
A. Group work provides the pupils with the opportunity to learn to be capable organizers.
B. Pupils also learn to develop their reasoning ability.
C. Group work gives pupils the opportunity to learn to work together with others.
D. Pupils also learn how to participate in teaching activities.
73. The author’s purpose of writing this passage is to .
A. recommend pair work and group work classroom activities
B. argue for teaching bright and not-so-bright pupils in the same class
C. emphasize the importance of appropriate formal classroom teaching
D. offer advice on the proper use of the school library
74. According to the passage, which of the following is NOT TRUE?
A. There is no fixed method in teaching pupils to develop themselves to the full.
B. Development of pupils as individuals is not the aim of group work.
C. It’s not good for a bright child to find out that he performs worst in a mixed-ability class.
D.Pupils cannot develop in the best way if they are streamed into classes of different intellectual abilities.
75. According to the passage, which of the following is an advantage of mixed-ability teaching?
A. Pupils can be hindered from an all-round development.
B. Formal class teaching is the important way to give the pupils essential skills such as those
to be used in the library.
C. A pupil can be at the bottom of a class.
D. Pupils as individuals always have the opportunities to work on their own.
76. Which of the following statements can best summaries the main idea of the passage?
A. The aim of education is to find out how to teach the bright and not-so-bright pupils.
B. Children, in general, develop at different rates.
C. Various ways of teaching should be encouraged in class.
D. Bright children do benefit from mixed-class teaching.
77. According to the passage, “streaming pupils” .
A. is the act of putting pupils into classes according to their academic abilities
B. will help the pupils learn best
C. aims at enriching both their knowledge and experience
D. is quite discouraging

78. According to the author, mixed-ability teaching is more preferable because .


A. it doesn’t have disadvantages as in streaming pupils
B. children can learn to work with each other to solve personal problems
C. formal class teaching is appropriate
D. it aims at developing the children’s total personality

Part D : WRITING ( 20 pts )


I. Finish each of the following sentences in such a way that it is as similar as possible in
meaning to the sentence printed before it.(5pts)
79. David said that the accident was his fault. TOOK
……………………………………………………………………………………
80. The stranded climber would never have been rescued if his brother hadn’t had an ingenious
plan. INGENUITY
……………………………………………………………………………………
81. We didn’t think that he would win the price DOUBTFUL
……………………………………………………………………………………
82. She didn’t shed a tear when the story ended in tragedy. NOT
……………………………………………………………………………………
83. They pretended to be enjoying themselves, but they weren’t really. AS
……………………………………………………………………………………
II. Complete each of the following sentences in such a way that it means exactly the same as
the sentence printed before it. (5pts)
84. Children learn a lot about how to behave in a situation like this.
 Only …………………………………………………..…………………………
85. Such a ridiculous proposal isn't worth serious consideration.
 There is ……………………………………………………………………………
86. I certainly won’t change my mind about resigning.
 My changing …………………………………………………………..……………
87. Harriet was upset because she saw Peter with another woman.
 It was ……………………………………………………………….………………
88. When Marry heard the results, he began to feel more confident.
 Since …………………………………………………..……………………………
III. Do you agree or disagree with the following statement? “Games are as important for
children as they are for adults.” Write an essay (about 150 to 200 words) to express your
personal point of view.(10 pts)

The End _
Part A : PHONETICS ( 5pts )
I. Choose the word whose underlined part is pronounced differently from the rest in the same
line. ( 2pts )
1A 2C
II. Pick out the word whose main stressed syllable is different from the rest in the same line.
(3pts )
3B 4A 5D
Part B : LEXICAL AND GRAMMAR ( 45 pts )
I. Choose the option A, B, C or D to indicate the word or phrase which best completes each
sentence.(15pts)
6A 7A 8C 9D 10 B
11 A 12 D 13 D 14 B 15 C
16 A 17 C 18 A 19 C 20 B
II. The following passage contains 10 errors. Find and correct them (10pts)
Who were the people responsible for collection and sending plants from one country to another?
And why did they do it? Initially they were travelers with other purposes: traders, colonists,
pilgrims and missionaries have all been important in providing new plants for English gardens.
They sent back indigenous wild plants, or sometimes, as in the cases of visitors to China and Japan,
plants which have been cultivated and improved for hundreds of years. This worked, of course, in
both directions: English gardens were making in the most unlikely places. Travelers did not always
recognize an interesting plant on seen it – interesting, that is, to the collector at home. So in the 16th
and 17th century, attempts were made to collect on a most professional basis, either by patrons
sending collections into the field, or by subscriptions to finance local enthusiasts in the most
promised areas. By 1611 John Tradescant was traveling and collecting in France and other parts of
Europe. Lately, Peter Collinson, a London merchant, who had seen the richness of the plant
material sending back by Tradescant, organized a syndicate to finance the amateur botanist John
Bartram. Before long, special collectors were being dispatched to all parts of the world by
institutions such as the Chelsea Physic Garden.
Mistake Correction Mistake Correction
21 collection collecting 26 most more
22 cases case 27 collections collectors
23 have had 28 promised promising
24 making made 29 Lately Later
25 seen seeing 30 sending sent
III. Put each verb in brackets into an appropriate form. ( 10pts )
31. Don’t you see/ does it say 32. was always trying
33. haven’t been listening 34. didn’t think/ would run
35. should be done 36. have put
37. could have been killed 38. having never been introduced
IV. Put each word in brackets into an appropriate form. ( 10pts )
39. UNECONOMICAL 40. MISBEHAVIORS/
MISBEHAVIOURS
41. UNACKNOWLEDGED: không được thừa nhận 42. MISGIVINGS
43. INTERACTION 44. HUNG-OVER choáng váng, buồn nôn
45. FORGIVENESS: sự khoan dung. 46. DISCOLORATION sự đổi màu;
sự bẩn màu; sự bạc màu
47. BREATHTAKINGLY. 48. NON-OBSERVATION
Part C : READING ( 30 pts )
I. Read the following passage and then choose the most suitable word or phrase for each
space. (10 pt )

49.A 50. C 51. C 52.A 53. B 54.D 55. C 56. D 57. B 58. D
II. Supply the most suitable word for each blank. (10 pts)
59. into 60. might/ can/ 61. terms 62. regardless/ 63. at
may irrespective
64. Being 65. rather 66. let 67. that/this 68. how

III. Read the following passage, and then choose the best answer from A, B, C or D.(10pts)
69. D 70. C 71. A 72. D 73. B
74. B 75. D 76. D 77. A 78. D
Part D : WRITING ( 20 pts )
I. Finish each of the following sentences in such a way that it is as similar as possible in
meaning to the sentence printed before it.(5pts)
79. David took the blame/responsibility for the accident.
80. But for the ingenuity of his brother’s plan, the stranded climber would never have been rescued.
81. We were doubtful about his change of winning the price.
82. Not a tear did she shed when the story ended in tragedy.
83. They acted as if they were enjoying themselves, but they weren’t really.
II. Complete each of the following sentences in such a way that it means exactly the same as
the sentence printed before it. ( 5pts )
84. in a situation like this can children learn a lot about how to behave.
85. no point in considering such a ridiculous proposal seriously.
86. my mind about resigning is out of question.
87. because she saw Peter with another woman that Harriet was upset / that upset Harriet.
88. hearing the results, Marry has felt more confident.
III. Do you agree or disagree with the following statement? “Games are as important for
children as they are for adults.” Write an essay (about 150 to 200 words) to express your
personal point of view.(10 pts)
Marking scheme: The impression mark should be based on the following scheme:
1. Format: 2 pts ( coherence , cohesion , style )
The argument should have 3 parts
a. Introduction: should be put in one paragraph in which students’ points of view are expressed
clearly
b. Body: should consist of from one to three paragraphs. Each paragraph must have a topic
sentence, reasonings and examples.
c. Conclusion (summary of the main reasons, students’ comment)
2. Content : 5pts
3. Language : 3pts ( grammatical accuracy , wide range of vocabularies and structures )
Everyone likes to play games. Games are important at any age to keep your mind sharp,
learn new things, and maintain social skills.

When you play games, you exercise your mind. This becomes more important as you grow older.
By concentrating on the tactics of a game, memorizing moves, and following your opponent’s
strategies, you can keep your brain functioning and growing.

Playing games can teach you a lot. Games that ask questions, for example, show you what you
don’t know. You can learn about things like geography and history when you play certain games.
It’s a fun way to learn, and adults enjoy this as much as children do.

Games require the use of social skills. When you play games, you interact with other people. You
have to be considerate of them and you have to play fairly. Playing games allows you to maintain
personal contacts. This is important for people of all ages.Regardless of your age, playing games
can help you keep your mind alert, learn new things, and build friendships.
Đáp án tham khảo đề thi HSG Quốc gia
môn Tiếng Anh 2020-2021
LISTENING

PART 1

F 2. F 3. NG 4. T 5. NG

PART 2

6. tinder - dry 9. wineries and vineyards

7. annual acreage burned 10. further intensification

8. critical fire months

PART 3

11. B 12. C 13. C 14. B 15. A

PART 4

16. emblematic off Britishness 21. sweeter and squishier

17. a soupy stew 22. deemed fitting

18. no actual meat 23. garden more mythology

19. the English 24. much revelry

20. in the offing 25. a hefty pudding

II. LEXICO-GRAMMAR PART

PART 1

26. D 29. D 32. D 35. A 38. D

27. A 30. A 33. A 36. C 39. C

28. C 31. A 34. A 37. B 40. B

PART 2

41. after-sales 43. dignitaries 45. unencumbered

42. unconcernedly 44. inscrutably


READING

PART 1

46. latter 48. worshipped 50. due 52. corresponding 54. character

47. capable 49. victoy 51. in 53. about 55. circumstances

PART 2

56. TRUE 61. TRUE 66. sociology of education

57. FALSE 62. criticisms 67. ethnographic

58. TRUE 63. variables 68. quantitative data

59. NOT GIVEN 64. casual conversations

60. TRUE 65. open-ended questions

PART 3

69. F 70. B 71. G 72. D 73. H 74. A 75. E

PART 4

76. C 78. A 80. A 82. B 84. B

77. B 79. A 81. A 83. C 85. B

PART 5

86. D 88. C 90. A 92. B 94. B

87. D 89. C 91. B 93. A 95. D


TRƯỜNG THPT QUẾ VÕ SỐ 1 ĐỀ THI CHỌN ĐỘI TUYỂN HỌC SINH GIỎI T

NĂM HỌC 2020 - 2021


Môn: Tiếng Anh. Khối: 12
(Đề gồm có 08 trang)
Thời gian làm bài: 90 phút (Không kể thời gian phát đề)

Họ và tên: ……………………………………….......... SBD: …………………...................

Thí sinh không được sử dụng tài liệu. Cán bộ coi thi không giải thích gì thêm.
-----------------------------------------------------------------------
SECTION A. LISTENING
Part 1: You will hear two people speaking about their fondness for trains. For questions
1-5, choose the correct answer (A, B, C or D) which fits best according to what you hear.
1) What point does Philip make about the people who are involved in the locomotive side of trains?
A) Some avoid doing it because its too dirty. B) Some worry about the dangers involved.
C) Some find it to be an instinctive activity. D) Some only do it for the financial rewards itbrings.
2) When Mike discusses why he enjoys the world of trains, what contrast does he identify between the
different groups of people he interacts with?
A) A difference in social status. B) A difference in ages.
C) A difference in technical ability. D) A difference in motivation.
3) What view is stated by Mike about how the railway has helped him in his role as a parent?
A) It has helped him develop a closer relationship B) It has taught him patience. with his daughter.
C) It has helped him focus on his daughters D) It has given him stability in his life.
career possibilities.
4) When discussing different aspects of the railway, both speakers agree that ___________.
A) Age does not need to be a handicap. B) Both men and women are equally welcome.
C) It can be quite demanding. D) Cleanliness is not the first adjective that
springs to mind.
5) What final conclusion do both speakers reach about volunteering on the railway?
A) The time they have spent has been B) They have both learnt a lot.
difficult but worth it.
C) They are better people for having been D) It has helped them grow while at the same time
involved with it. giving them a direction for the future.

Part 2: Listen to the audio and choose the best answer to each question.
1. What does the lecturer provide for those who are interested in doing extra reading?
A. Personal consultation sessions.
B. Extra materials, such as a booklist.
C. Mid-term examination.
D. Free glasses.
2. In the past, time management meant you needed to
A. reduce your stress.
B. plan for every hour of the week.
C. own a good watch.
D. set goals and try to achieve these goals.
3. Today, wise time management means you need to
A. set goals and work in a systematic way.
B. work faster.
C. set an overview of your assignment.
D. make a list, plan for everything and try to stick to this plan.
4. In this college, students are assigned ____________ at the end of each semester.
A. team projects.
B. final term examinations.
C. essays.
D. time management courses.
5. One sign he lecturer mentions that students feel under pressure is
A. library books go missing.
B. students get angry for no reason.
C. lower class attendance rates.
D. trouble at the library.
6. What kind of suggestion does the lecturer give to the students?
A. Making a very detailed plan of their daily activities.
B. Not being so stressed just because there is an assignment.
C. A regular one-hour session in their personal timetables.
D. Wearing comfortable shoes.
7. According to the lecturer, there are three kinds of planners. They are:
A. one weekly planner, one daily planner and one hour planner.
B. one yearly planner, one weekly planner and one daily planner.
C. one term planner, one monthly planner and one weekly planner.
D. one term planner, one weekly and one daily planner.
8. If you want to set an overview of your time, you should need at least
A. one week
B. half a week.
C. one month.
D. one term.
9. The daily planner of time is mainly concerned with
A. the detailed planning.
B. how to plan all available time.
C. TV schedules.
D. an overview of everything you need to do for several days.

10. According to the lecturer, wise time management may have the following benefit:
A. having more time to spend on relaxation and other activities.
B. improving your performance in the final term assignment.
C. helping you write better essays.
D. improving your memory.

SECTION B. LEXICO-GRAMMAR
Mark the letter A, B, C or D to indicate the word whose underlined part differs from the other
three in pronunciation in each of the following questions.
Question 1: A. coverage B. beverage C. rouge D. gauge
Question 2: A. prepare B. preface C. preparation D. prejudice

Mark the letter A, B, C, or D to indicate the word that differs from the other three in
the position of primary stress in each of the following questions.
Question 3: A. fulltime B. farmhand C. bookshop D. tradesman
Question 4: A. interchange B. infamous C. contributory D. undercurrent

Mark the letter A, B, C, or D sheet to indicate the correct answer to each of the
following questions.
Question 5. __________ comes a time when you have to make a decision and stick to it.
A. It B. That C. Then D. There
Question 6. More and more trees in this area are found __________ from the effect of pollution.
A. died B. to be dead C. to be dying D. having died
Question 7. I don’t need any medicine. I’m as right as _______________.
A. rain B. clouds C. rays D. a haze
Question 8. Kathy __________ comes up with a solution when everyone else is at a loss.
A. absolutely B. invariably C. persistently D. universally
Question 9. I’d say let’s meet on Saturday but I’m none __________ sure what’s happening at the
weekend.
A. so B. very C. that D. too
Question 10. Every Christmas of my childhood was the same. My father __________ late for lunch,
weighed down with presents for the family.
A. would arrive B. could arrive C. was arriving D. got used to arriving
Question 11. The mayor failed to deliver key promises in his manifesto, but to give him his
improve the city's infrastructure.
A. account B. view C. owing D.due
Question 12. He was extremely protective _____________ his role as advisor.
A. of B. with C. about D. for
Question 13. “It’s really cold today”
“Yes, I didn’t want to get out of bed, _____________ go outside.”
A. even though B. much less C. aside from D. not only
Question 14. “Is John a good student?” “He is, _____________, an excellent student.”
A.in most parts B. for most parts
C. for the most part D. in most of the parts
Question 15. Our plan is completely _____________. Nothing can possibly go wrong.
A. foolproof B. airtight C. foolhardy D. waterproof
Question 16. He spent ten years in the army and for most of the time he was abroad.
A. Camped B. situated C. placed D. stationed
Question 17. Hardly had the van turned the corner when one of the back wheels
A. Broke away B. turned round C. came off D. rolled down

Question 18. I’d rather you ______________a noise last night, I couldn’t get to sleep.
A. Hadn’t made B. wouldn’t make C. didn’t make D. haven’t made
Question 19. If we have to pay a £1,000 fine, then ________. We’re not going to win a fight with the Tax Office.
A. so be it B. be it so C. thus be it D. be it thus
Question 20. He came into the room and sat down without _________ a word to
anyone.
A. as far as B. too much of C. very much of D. so much as
Question 21. We’re going to visit the Great Walls. Everybody says this is __________.
A. a too good chance for being lost B. too good a chance to be lost
C. too good a chance for being lost D. a too good chance to lose

Read the following passage and mark the letter A, B, C, or D to indicate the correct
answer to each of the question.
Archaeology has long been an accepted tool for studying prehistoric cultures. Relatively recently the same
techniques have been systematically applied to studies of the more immediate past. This has been called
"historical archaeology", a term that is used in the United States to refer to any archaeological investigation
into North American sites that postdate the arrival of Europeans.
Back in the 1930's and 1940's, when building restoration was popular, historical archaeology was
primarily a tool of architectural reconstruction. The role of archaeologists was to find the foundations of
historic buildings and then take a back seat to architects.
The mania for reconstruction had largely subsided by the 1950's and 1960's. Most people entering
historical archaeology during this period came out of university anthropology departments where they
had studied prehistoric cultures. They were, by training, social scientists, not historians, and their work
tended to reflect this bias. The questions they framed and the techniques they used were designed to
help them understand, as scientists, how people behaved. But because they were treading on historical
ground for which there was often extensive written documentation and because their own knowledge of
these periods was usually limited, their contributions to American history remained circumscribed. Their
reports, highly technical and sometimes poorly written, went unread.
More recently, professional archaeologists have taken over. These researchers have sought to demonstrate
that their work can be a valuable tool not only of science but also of history, providing fresh insights into the
daily lives of ordinary people whose existences might nt otherwise be so well documented. This newer
emphasis on archaeology as social history has shown great promise, and indeed work done in this area has
led to a reinterpretation of the United States' past.
In Kingston, New York, for example, evidence has been uncovered that indicates that English goods were
being smuggled into that city at a time when the Dutch supposedly controlled trading in the area. And in
Sacramento an excavation at the site of a fashionable nineteenth-century hotel revealed that garbage had
been stashed in the building' s basement despite sanitation laws to the contrary.
Question 22: What is the main topic of the passage?
A.How the purpose and the methods of historical archaeology have changed.

B.How archaeology has been applied to studies of prehistoric cultures.


C.The attitude professional archaeologists hold toward historical archaeology.
D.The contributions social scientists make to historical archaeology.
Question 23: According to the passage, what is a relatively new focus in archaeology?
A.studying prehistoric cultures

B.investigating ancient sites in what is now the United States


C.comparing the culture of North America to that of Europe
D.studying the recent past
Question 24: According to the passage, when had historical archaeologists
been trained as anthropologists?
A.before the 1930's B.during the 1930's and 1940's

C.during the 1950's and 1960's' D.after the 1960's


Question 25: In the third paragraph, the author implies that the questions and
techniques of history and those of social science are ______.
A.of equal value in studying prehistoric cultures

B.quite different from each other


C.all aiming to understand people's behaviour

D.all highly technical and poorly written


Question 26: The equivalent of the word "supposedly" in the last paragraph is ______.
A.rigidly B.barely C.seemingly D.ruthlessly

Read the following passage and mark the letter A, B, C, or D to indicate the correct
word or phrase that best fits each of the numbered blanks.
In the US, industries that generate hazardous wastes want to dispose of them as cheaply as
possible. Private companies hired to dispose of this waste compete with each other to offer the
lowest prices to these industries. The government does not get involved, beyond setting
minimum safety standards.
Unfortunately, the (27) _______ of companies that generate and dispose of waste is to save money, not
to guarantee safety. These companies usually send waste to landfills because this is cheaper than recycling
or incineration. Disposal firms who want to increase their business must cut corners to lower costs and (28)
________ customers. At the same time, relatively little is done to reduce the
volume of waste generated,because disposal cost (29) __________relatively modest.
Things are different in Denmark. There the government participates in the waste disposal process beginning
(30) ____________ the front end. Together with industry, the government formed a
corporation to establish and operate waste disposal facilities. This company, called Kommunichem, has a (31)
____________ on waste disposal. Generators of hazardous waste must ship their waste to one of
Kommunichem’s disposal facilities. In this system,there is no price competition in the waste
disposal business.

Question 27 A. solution B. license C. importance D. goal


Question 28: A. survive B. efficient C. gain D. prosper
Question 29: A. still B. have C. remain D. cheap
Question 30: A. to B. by C. of D. at
Question 31: A. power B. responsibility C. monopoly D. profit

Read the following passage and mark the letter A, B, C, or D to indicate the correct
answer to each of the question.
The spectacular aurora light displays that appear in Earth’s atmosphere around the north and
south magnetic poles were once mysterious phenomena. Now, scientists have data from
satellites and ground-based observations from which we know that the aurora brilliance is an
immense electrical discharge similar to that occurring in a neon sign.
To understand the cause of auroras, first picture the Earth enclosed by its magnetosphere, a huge
region created by the Earth’s magnetic field. Outside the magnetosphere, blasting toward the Earth is
the solar wind, a swiftly moving plasma of ionized gases with its own magnetic field.
Charged particles in this solar wind speed earthward along the solar wind’s magnetic lines of force
with a spiralling motion. The Earth’s magnetosphere is a barrier to the solar wind, and forces the
charged particles of the solar wind to flow around the magnetosphere itself. But in the polar regions, the
magnetic lines of force of the Earth and of the solar wind bunch together. Here many of the solar wind’s
charged particles breakthrough the magnetosphere and enter Earth’s magnetic field. They then spiral
back and forth between the Earth’s magnetic poles very rapidly. In the polar regions, electrons from the
solar wind ionize and excite the atoms and molecules of the upper atmosphere, causing them to emit
aurora radiations of visible light.
The colors of an aurora depend on the atoms emitting them. The dominant greenish white light
comes from low energy excitation of oxygen atoms. During huge magnetic storms oxygen atoms also
undergo high energy excitation and emit crimson light. Excited nitrogen atoms contribute bands of color
varying from blue to violet. Viewed from outer space, auroras can be seen as dimly glowing belts
wrapped around each of the Earth’s magnetic poles. Each aurora hangs like a curtain of light stretching
over the polar regions and into the higher latitudes. When the solar flares that result in magnetic storms
and aurora activity are very intense, aurora displays may extend as far as the southern regions of the
United States.
Studies of auroras have given physicists new information about the behavior of plasmas,
which has helped to explain the nature of outer space and is being applied in attempts to
harness energy from the fusion of atoms.
Question 32: What does the passage mainly discuss?
A. The methods used to observe auroras from outer space.
B. The formation and appearance of auroras around the Earth’s poles
C. The factors that cause the variety of colors in auroras.
D. The periodic variation in the display of auroras.
Question 33: The word “phenomena” in paragraph 1 is closest in meaning to __________.
A. ideas B. stars C. events D. colors
Question 34: The passage describes “the magnetosphere as a barrier” because __________.
A. its position makes it difficult to be observed from Earth
B. it prevents particles from the solar wind from easily entering Earth’s atmosphere
C. it increases the speed of particles from the solar wind
D. it is strongest in the polar regions
Question 35: According to the passage, which color appears most frequently in an aurora display?
A. greenish-white B. crimson C. blue D. violet
Question 36: The word “glowing” in paragraph 4 is closest in meaning to __________.
A. shining B. moving C. charging D. hanging
Question 37: Auroras may be seen in the southern regions of the United States when __________.
A. magnetic storms do not affect Earth B. solar flares are very intense
C. the speed of the solar wind is reduced D. the excitation of atoms is low
Question 38: The passage supports which of the following statement about scientists’
understanding of auroras?
A. Before advances in technology, including satellites, scientists knew little about auroras.
B. New knowledge about the fusions of atoms allowed scientists to learn more about auroras.
C. Scientists cannot explain the cause of the different colors in auroras.
D. Until scientists learn more about plasma physics, little knowledge about auroras will be
available.
Question 39: Which of the following terms is defined in the passage?
A. “magnetosphere” B. “electrons” C. “ionize” D. “fusion”
Mark the letter A, B, C, or D to indicate the word{s) CLOSEST in meaning to the underlined
word(s) in each of the following questions.
Question 40: A.I. techniques help medical doctors to discover subtle interactions between medications that
put patience in jeopardy because of their serious side effect.
A. at risk B. in reality C. under control D. under pressure
Question 41: The high mountain climate is cold and inhospitable.
A. rainy B. stormy C. uninviting D. intense
Mark the letter A, B, C, or D to indicate the word(s) OPPOSITE in meaning to the underlined
word(s) in each of the following questions.
Question 42: You'd think he would have blown his lid when he realized the chef was downstairs.

A. become tired B. stayed calm C. felt happy D. been furious Question


43: This is new washing machine is not a patch on our old one. These clothes are still dirty.

A. to be expensive B. to be broken C. to be strange D. to be better


Mark the letter A, B, C, or D to indicate the sentence that best completes each of the following
exchanges.
Question 44: Two students are talking about their friend Tim:
A: “Tim has a very easy job. He’s paid a lot ofmoney to do little.”
B:“ ”
A. It's the luck of the game B. It's an incredible piece ofluck

C. Wish him good luck D. He crossed his fingers for luck


Question 45: Two friends are talking about their lives:
X: “I feel really depressed at the moment. Nothing's going right in my life.”
Y:“ .”

A. I know. We really need some sunshine, don't we?


B. Cheer up! Things can't be that bad!
C. I'm sorry to hear that, but I'm sure he'll be all right.
D. That's good. I'm pleased to hear it.

Mark the letter A, B, C, or D to indicate the sentence that is closest in meaning to each of the following
questions.
Question 46: When we asked the Minister about the strike, he declined to comment.
A. On asking us about the strike, the Minister declined to comment.
B. When asked about the strike, the Minister declined to comment.
C. We declined to comment when the Minister asked us about the strike.
D. Declining to comment, the Minister asked us about the strike. Question 47: The
man with red hair may have caused it.
A. It may have been caused by the man whose his hair was red.
B. The man whose red hair may have caused it.
C. It may have caused by the man whose hair was red.
D. It may have been caused by the man whose hair was red.
Question 48. The likelihood of suffering a heart attack rises as one becomes increasingly obese.
A. Heart attacks are happening more and more often, and most of the sufferers are obese.
B. Obesity results in only a slight increase in the probability of having a heart attack.
C. The more obese one is, the higher the chances for a heart attack become.
D. Anyone who is obese is likely to experience a heart attack at any time.

Mark the letter A, B, C, or D to indicate the sentence that best combines each pair of
sentences in the following questions.
Question 49. New York is described as the world’s cultural centre. It situated on the
bank of Hudson river.
A. As long as New York is described as the world’s cultural centre, It situated on the bank of
Hudson river.
B. In spite of the fact that it situated on the bank of Hudson river, New York is described
as the world’s cultural centre.
C. Because it situated on the bank of Hudson river, New York is described as the world’s
cultural centre.
D. New York, which is described as the world’s cultural centre, situated on the bank of
Hudson river.
Question 50. He was overconfident. Therefore, he ruined our plan completely.
A. It was because his overconfidence that ruined our plan completely.
B. He was overconfident, which ruined our plan completely.
C. That he was overconfidence ruined our plan completely.
D. It was his overconfidence ruined our plan completely.

Mark the letter A, B, C, or D on your answer sheet to indicate the option that best
completes each of the following exchanges.

Question 51: Tom: It's the worst film we've ever seen.
Jenny : ________
A. I couldn't say that again. B. I couldn't agree more.
C. That’s right, you are. D. There you have it.
Question 52: Speaker 1: You look lost. ________?
Speaker 2: Yes, please. Can you tell us how to get to the railway station?
A. Can I be of any assistance? B. Is there anything to do?
C. Can I give a hand? D. Would you like a hand?
Mark the letter A, B, C, or D to indicate the underlined part that needs correction in
each of the following questions.
Question 53: He is a boy in seven (A) with white hair, like (B) snow, whose presence (C) is "like
being in the room with a feather" (D).
Question 54: Marie-Laure's father is also the creator of ingenious puzzles (A) and delightful
miniature (B) of (C) the streets and houses of Paris, for instance. (D)
Question 55: Although research has been ongoing since (A) 1930, the existence (B) of ESP-
perception and communication without the use (C) of the sight, hear (D), taste, touch or smell-is
still disputed.
------------------------------------- END OF THE TEST --------------------------
KỲ THI HỌC SINH GIỎI LỚP 12 THPT CẤP TỈNH
Năm học: 2020 – 2021
SỞ GIÁO DỤC VÀ ĐÀO TẠO
Môn thi : TIẾNG ANH
QUẢNG NAM
Thời gian : 90 phút (Không kể thời gian giao đề)
ĐỀ CHÍNH THỨC Ngày thi : 12/3/2021
Mã đề thi: 132
(Đề thi gồm có 10 trang)
(Thí sinh làm bài trên Phiếu trả lời trắc nghiệm )
Họ và tên thí sinh:……………………………………….Phòng thi ………Số báo danh: ……

SECTION I: LISTENING
HƯỚNG DẪN PHẦN THI NGHE HIỂU
2) Mỗi phần được nghe 2 lần, mở đầu và kết thúc mỗi phần có tín hiệu.
3) Mọi hướng dẫn cho thí sinh (bằng tiếng Anh) đã có trong bài nghe.

Part 1: You w ill hear people talking in eight different situations. For questions 1-
8, mark the letter A, B, or C on your answ er sheet to indicate the correct answ er
to each of the follow ing questions.

Question 1: You will hear someone talking about an activity. Why does she like the activity?
A. Because it's very demanding.
B. Because young people do it.
C. Because age is not a restriction.
Question 2: You will hear an architect talking about her work. What problem does
she describe?
A. Getting blocked on an idea.
B. Getting new ideas.
C. Applying the ideas she has.
Question 3: You hear someone talking about an event in nature. What is she doing?
A. Explaining why a problem exists.
B. Describing a situation.
C. Trying to persuade someone to do something.
Question 4: You will hear someone talking about a night out in a restaurant. What
is the speaker saying?
A. She will never go back.
B. She will tell her friends about it.
C. She is going to be nice to someone.
Question 5: You will hear someone having a conversation. How does the woman feel?
A. Irritated. B. Worried. C. Excited.
Question 6: You will hear someone talking about cooking. What is important to her?
A. Knowing what she has to cook.
B. Knowing where things are.
C. Knowing the quality of the ingredients.
Question 7: You will hear someone in business talking about expanding. What
problem is she describing?
A. Getting the right price.
B. Getting the right location.
C. Getting a good location at the right price.
Question 8: You will hear someone talking about a sporting activity. What is she
describing? A. Why she took up the activity.
B. Why she enjoys it.
C. Why other people would enjoy it.

Part 2: You hear a conversation w ith Andrea Thompson w ho is Britain's strongest


w oman. For questions 9-15 choose the best answ er A, B or C. You w ill hear the
recording tw ice.
Question 9: When asked how strong she is, Andrea explains ________.
A. that she could lift a big animal
B. that she could lift a vehicle
C. that she could pull a hippo
Question 10: How does Andrea describe her feeling towards sport as a kid?
A. She had no interest in sport at that time.
B. She enjoyed some events, but was not really convinced.
C. She was always passionate about sport.
Question 11: Andrea says her initial reason for getting involved in weight lifting was
________.
A. to motivate her younger sister
B. to compete with her older sister
C. to support her sister's attempt to lose weight
Question 12: How did Andrea feel when some men refused to train with
her? A. She didn't care because she was enjoying herself.
B. She felt humiliated.
C. She felt she had something to prove.
Question 13: Andrea explains that when she entered the strong woman
competition in 2015, ________.
A. she was well-informed because she had done her research
B. she had a lot of experience
C. she knew next to nothing about the details
Question 14: How did the coach she approached react to her request for help?
A. With disbelief. B. With admiration. C. With irritation.
Question 15: What drives her to push herself as much as she does?
A. The pressure from her family.
B. The pleasure of being the best.
C. The pride that her kids can see her win.
SECTION II: LEXICO-GRAMMAR
P art 1: Mark the letter A, B, C, or D on your answ er sheet to indicate the correct
answ er to each of the follow ing questions.
Question 16: “I’m ______ to listen to your pathetic excuses.” She said.
A. sick and tired B. in no mood
C. having enough D. sick to death
Question 17: “There’s no need to stand on ceremony. _______in,” said a disembodied voice
from the kitchen.
A. Plough B. Burrow C. Fork D. Dig
Question 18: About half of the rope _______ is used by the marine and fishing industries.
A. producing B. having produced C. produced D. to be producing Question 19: She’s a bit
down in the ______ at the moment – her husband has just lost his job.
A. world B. dumps C. heart D. bottom
Question 20: Manufacturer Johnson & Johnson released data on March 1st, 2021 showing
that its _______ version provided strong protection against COVID-19.
A. single-dose B. single-file C. single-harelip D. single-cell Question 21: Nowadays too
many people work too hard in offices and get too little exercise;
______ they tend to eat unhealthy food because they do not have time to shop or cook.
A. furthermore B. on the contrary C. for fear that D. the opposite Question 22:
Palaeontologists have come to no definite conclusions as to ______ there have
been periodic mass extinctions throughout pre-history.
A. what B. why C. which D. that
Question 23: I wish you would stop sitting on the _______ and decide whose side you’re on.
A. fence B. crossroads C. wall D. middle
Question 24: Tonight, I ______ a 1500-word essay for my literature class tomorrow, so I’d
better get started.
A. am supposed to write B. ought to be written
C. should have written D. would rather written
Question 25: I had______ got home ______ people began ringing to ask what was going on.
A. neither / nor B. no sooner / when C. either / or D. hardly / when
Question 26: Florida, _______ the Sunshine state, attracts many tourists per year.
A. known as B. is known C. is known as D. it is known as
Question 27: The candidate still expects to be re-elected ______ the results of the latest
opinion poll.
A. apart B. without C. nevertheless D. notwithstanding
Question 28: Not until Kentucky's Mammoth Cave had been completely explored in 1972
______.
A. did its full extent realize B. that its full extent was realized
C. was its full extent realized D. the realization of its full extent
Question 29: Of course you won't become more intelligent if you eat a lot of fish - that's just
an old ______ tale.
A. maids' B. ladies' C. mothers’ D. wives'
Question 30: Does Sheila work full time at the supermarket? - No, only ______ when they
need extra staff.
A. in and out B. on and off C. by and by D. up and around
Question 31: Those campers are really ______. They have no idea how to set up a tent.
A. white B. blue C. green D. black Question 32: Trade ______ from bad to
worse and staff ______ redundant now.
A. has gone - are being made B. have gone - is made
C. went - has made D. had gone - are made
Question 33: It is a ______.
A. blue polyester sleeping bag B. polyester sleeping blue bag
C. blue sleeping polyester bag D. sleeping blue polyester bag
Part 2: Mark the letter A, B, C, or D on your answ er sheet to indicate the w ord(s)
CLOSEST in meaning to the underlined w ord(s) in each of the follow ing questions.
Question 34: It is believed that the plane crash was caused by electrical malfunction of
its navigation system.
A. breaking through B. breaking in C. breaking down D. breaking up Question 35: Because
of the baby boom of the 1980s, preschools in the U.S. have proliferated.
A. changed in philosophy B. become more expensive
C. become more crowded D. increased in numbers

Part 3: Mark the letter A, B, C, or D on your answ er sheet to indicate the


underlined part that needs correction in each of the follow ing questions.
Question 36: The keynote speaker started with some complementary remarks about the
A B
organisers of the conference, and then proceeded with her speech.
C D
Question 37: Abraham Lincoln's boyhood home resembled that of many other mid-western
A B
pioneers with its dirt floor, sleeping loft, and crude fireplace.
C D
Question 38: A child of noble birth, his name was famous among the children in that school.
A B C D
Part 4: Mark the letter A, B, C, or D on your answ er sheet to indicate the option
that best completes each of the follow ing exchanges.
Question 39: Two students are talking about the upcoming examination.
Tim: “______? You look so concerned.”
Terry: “My final exam is coming.”
A. How is your well- B. How do you feel
being C. What’s up D. Where did it go
Question 40: Bill and Bob are in a coffee shop. Bill is asking Bob for his opinion about the
coffee.
Bill: "How's coffee here?"
Bob: "_______"
A. It's a little bitter, to tell the truth. B. No, I don't think so.
C. As a matter of fact, I'm not interested. D. It's a little better now that I've got a car.
Part 5: Mark the letter A, B, C, or D on your answ er sheet to indicate the w ord(s)
OPPOSITE in meaning to the underlined w ord(s) in each of the follow ing questions.
Question 41: Julia is on the horns of the dilemma; she just wonders whether to go for a
picnic with her friends or to stay at home with her family.
A. unwilling to make a decision B. able to make a choice
C. eager to make a plan D. unready to make up her mind
Question 42: Some people choose to assimilate into the new culture when migrating to
another country.
A. integrate into B. adapt to C. separate from D. prevent from

SECTION III: READING


Part 1: Read the follow ing passage and mark the letter A, B, C, or D on your answ
er sheet to indicate the correct w ord or phrase that best fits each of the numbered
blanks.

WE REALLY CAN TELL IF WE ARE BEING WATCHED


Stories about how people somehow know when they are being watched have been going
around for years. However, few attempts have been made to investigate the phenomenon
scientifically. Now, with the completion of the largest ever study of the so-called staring effect,
there is impressive evidence that this is a recognizable and (43) ______ sixth
sense. The study (44) _______ hundreds of children. For the experiments, they sat with their
eyes (45) _______ so they could not see, and with their backs to other children, who were
told to either stare at them or look away. Time and time again the results showed that the
children who could not see were able to (46) _______ when they were being stared at. In a
total of more than 18,000 trials (47) ________ worldwide, the children (48) ________
sensed when they were being watched almost 70% of the time. The experiment was repeated
with the (49) ________precaution of putting the children who were being watched outside
the room, separated from the starers by the windows. This was done just in case there was
some (50) ________ going on with the children telling each other whether they were looking
or not.
Question 43. A. genuine B. accepted C. received D. sure
Question 44. A. involved B. contained C. comprised D. enclosed
Question 45. A. shaded B. wrapped C. masked D. covered
Question 46. A. find B. notice C. tell D. reveal
Question 47. A. worked over B. worked through C. carried on D. carried out
Question 48. A. correctly B. exactly C. thoroughly D. perfectly
Question 49. A. attached B. added C. connected D. increased
Question 50. A. pretending B. lying C. cheating D. deceiving
Part 2: Read the follow ing passage and mark the letter A, B, C, or D on your answ
er sheet to indicate the correct answ er to each of the questions.
RIGHT-HAND DOMINANCE
Humans are disproportionately right-handed. Scientists have not been able to agree over the
exact percentages of right versus left-handers because there is no accepted standard for
identifying which hand is dominant. For example, some people who write or throw with their
right hands may perform other tasks with their left hands or may kick a ball with their left foot.
Absent an objective measure, therefore, the range of estimates is wide. Right-handers are
said to make up 85% to 95% of all people and left-handers 5%-15%, while the remaining tiny
percentage are ambidextrous, so they can use both hands with equal ability.
Perhaps the most unusual fact about right-hand dominance is how little we know about its
causes. Several theories have been proposed. Some evidence exists that the phenomenon is
genetic, but genetics cannot agree on the process by which handedness may be passed on by
inheritance. Social and cultural forces can also cause a preference for one hand, as when
teachers or parents force a naturally left-handed child to use their right hand. And it has been
observed by anthropologist that left-handedness tends to be less common in restrictive
societies and more common in permissive ones. But no consensus has been reached on how
that could occur.
The most credible explanations center on functions inside the brain. It has been shown that
the brain’s two hemispheres control the opposite side of the body. It has been suggested that
the nerves in the brain cross over at neck level to the other side of the body so that the right
half of the brain governs the left side of the body while the left half governs the right side.
Scientists believe that the left half of the brain evolved in such a way as to predominate over
the right half. As a result, the right side of the body is controlled by the more influential left
hemisphere, causing the right side to be more adept at physical tasks. But when a person is
born with a dominant right hemisphere, that person will be left-handed. Some researchers
have argued that some left-handedness may have a pathological origin, having been caused
by brain trauma during birth.
A theory grounded in evolution is the “warrior and his shield theory”. This theory explains
that right-handedness evolved over time to be dominant because a right-handed warrior
would hold his shield in his left hand to protect his heart and to leave his right hand to free to
hold a weapon. A left-handed warrior, in contrast, would hold his weapon in his left hand and
his shield in his right, leaving his heart exposed. Thus a right-handed warrior, with his heart
protected against enemy attacks, was more likely to survive. By the process of natural
selection, the trait for right-handedness became favored over that for left-handedness.
Another theory focuses on the naturally asymmetrical arrangement of the human body.
Such asymmetry is evidenced by the observable facts that the right side of the face is slightly
different from the left, that one leg is stronger or longer than the other, and that one foot is
larger than the other one. Right-handedness, the theory proposes, is just another example of
this natural asymmetry.
3) A consequence of right-hand dominance is that most common consumer products
are geared to right-handers only, leaving left-handers to struggle to adapt to designs not
made with them in mind. (B) Some of these include scissors, doorknobs, locks, screwdrivers,
automobile fixtures, refrigerators, can openers, clothes buttons, and fasteners, and musical
instruments. (C) The result of this design bias can be more than mere inconvenience. (D)
Some left-handed soldiers shooting rifles designed for right-handers have sustained eye and
head injuries from ejected shell casings.
Hand dominance does not seem to occur in non-human animal species. While some individual
animals can be seen developing a preference for one hand or the other, there is no evidence
that this preference is common to the species as a whole, as it is in humans. Some scientists
claim to have observed such dominance in animals but only in controlled settings, such as a
zoo or laboratory, and only when the animals are performing manual tasks that do not mirror
how they use their hands in the wild.
(Source: An Insight into IELTS)
Question 51. The word “absent” in the passage is closest in meaning to ________.
A. using B. resisting C. lacking D. substituting
Question 52. According to paragraph 3, which of the following is a possible cause of left-
hand dominance?
A. the dominance of the brain’s left hemisphere
B. the natural weakness of a human’s right side
C. a child’s choice upon reaching school age
D. brain trauma in birth
Question 53. The word “adept” in the passage is closest in meaning to ________.
A. speedy B. skillful C. careful D. accustomed
Question 54. According to paragraph 4, which of the following are true about the “warrior
and his shield theory”?
A. a left-handed warrior is favored by natural selection.
B. a right-handed warrior holds his weapon in his left hand.
C. a left-handed warrior holds his weapon in his right hand.
D. a left-handed warrior leaves his heart unprotected.
Question 55.The word “that” in the passage refers to ________.
A. warrior B. heart
C. process of natural selection D. trait
Question 56. According to paragraph 6, left-handers would have trouble handling all of the
following EXCEPT ________.
A. refrigerators B. violins C. pencils D. shirt buttons
Question 57. The word “asymmetrical” in the passage is closest in meaning to _______.
A. deformed B. imbalanced C. geometrical D. variable
Question 58. Look at the four letters (A), (B), (C), (D) that indicate where the following
sentence could be added to the passage.
Left-handers often search for custom-made versions of these products.
Where would the sentence best fit?
A. (A) B. (B) C. (C) D. (D)
Question 59. Why does the author mention “eye and head injuries” suffered by some left-
handed soldiers shooting their rifles?
A. to illustrate the “warrior and his shield theory”
B. to give an example of the problems faced by left-handers
C. to argue that soldiers should wear head protection
D. to contrast rifle design with the design of common consumer products
Question 60. According to the last paragraph, which of the following is true about hand
dominance in animals?
A. it is the same as in humans.
B. it is observed only in the wild.
C. animals in controlled settings adopt the hand dominance of their handlers.
D. it has been observed only with manual tasks.
Part 3: Read the follow ing passage and mark the letter A, B, C, or D on your answ
er sheet to indicate the correct answ er to each of the questions.
Legend has it that sometime toward the end of the Civil War (1861-1865) a government
train carrying oxen traveling through the northern plains of eastern Wyoming was caught in a
snowstorm and had to be abandoned. The driver returned the next spring to see what had
become of his cargo. Instead of the skeletons he had expected to find, he saw his oxen, living,
fat, and healthy. How had they survived?
The answer lay in a resource that unknowing Americans lands trampled underfoot in their
haste to cross the "Great American Desert" to reach lands that sometimes proved barren.
In the eastern parts of the United States, the preferred grass for forage was a cultivated
plant. It grew well with enough rain, then when cut and stored it would cure and become
nourishing hay for winter feed. But in the dry grazing lands of the West that familiar bluejoint
grass was often killed by drought. To raise cattle out there seemed risky or even hopeless.
Who could imagine a fairy-tale grass that required no rain and somehow made it possible for
cattle to feed themselves all winter? But the surprising western wild grasses did just that. They
had wonderfully convenient features that made them superior to the cultivated eastern
grasses. Variously known as buffalo grass, grama grass, or mesquite grass, not only were they
immune to drought; but they were actually preserved by the lack of summer and autumn
rains. They were not juicy like the cultivated eastern grasses, but had short, hard stems. And
they did not need to be cured in a barn, but dried right where they grew on the ground. When
they dried in this way, they remained naturally sweet and nourishing through the winter.
Cattle left outdoors to fend for themselves thrived on this hay. And the cattle themselves
helped plant the fresh grass year after year for they trampled the natural seeds firmly into the
soil to be watered by the melting snows of winter and the occasional rains of spring. The dry
summer air cured them much as storing in a bam cured the cultivated grasses.
(Source: TOEFL Reading Comprehension 5)
Question 61. What does the passage mainly discuss?
A. A type of wild vegetation B. Western migration after Civil War
C. The raising of cattle D. The climate of the western United States
Question 62. What could be inferred by the phrase "Legend has it" in line 1?
A. Most history books include the story of the train.
B. The story of the train is similar to other ones from that time period.
C. The driver of the train invented the story.
D. The story of the train may not be completely factual.
Question 63. The word "they" in paragraph 1 refers to _______.
A. plains B. skeletons C. oxen D. Americans
Question 64. What could be inferred about the "Great American Desert" mentioned in
paragraph 2?
A. Many had settled there by the 1860's.
B. It was not originally assumed to be a fertile area.
C. It was a popular place to raise cattle before the Civil War.
D. It was not discovered until the late 1800's.
Question 65. The word "barren" in paragraph 2 is closed in meaning to _______.
A. lonely B. uncomfortable C. infertile D. dangerous Question 66. The word
"preferred" in paragraph 2 is closed in meaning to_______.
A. favored B. available C. ordinary D. required
Question 67. Which of the following could be inferred about the cultivated plant
mentioned in the second paragraph?
A. Cattle raised in the western United States refused to eat it.
B. It had to be imported into the United States.
C. It would probably not grow in the western United States.
D. It was difficult for cattle to digest.
Question 68. Which of the following was NOT one of the names given to the western grasses?
A. Mesquite grass B. Bluejoint grass C. Buffalo grass D. Grama grass
Question 69. Which of the following was NOT mentioned as a characteristic of western grasses?
A. They contain little moisture B. They have tough stems
C. They can be grown indoors D. They are not affected by dry weather
Question 70. According to the passage, the cattle helped promote the growth of the wild
grasses by ______.
A. eating only small quantities of grass.
B. continually moving from one grazing area to another.
C. naturally fertilizing the soil.
D. stepping on and pressing the seeds into the ground.

SECTION IV: WRITING


Part 1: Mark the letter A, B, C, or D on your answ er sheet to indicate the sentence
that best combines each pair of sentences in the follow ing questions.
Question 71: Alice registered for the course. She then received the scholarship.
A. Hardly had Alice registered for the course when she received the scholarship.
B. Only after Alice registered for the course has she received the scholarship.
C. Having received the scholarship, Alice registered for the course.
D. Registering for the course helped Alice receive the scholarship.
Question 72: Ann always keeps up with the latest fashions. She works for a famous
fashion house.
A. Despite working for a famous fashion house, Ann hardly keeps up with the latest fashions.
B. Ann always keeps up with the latest fashions so as not to work for a famous fashion house.
C. Worked for a famous fashion house, Ann always keeps up with the latest fashions.
D. Ann works for a famous fashion house so she always catches the fashion trends.
Question 73: She helped us a lot with our project. We couldn't continue without her.
A. Unless we had her contribution, we couldn’t continue with the project.
B. But for her contribution, we couldn’t have continued with the project.
C. If she hadn't contributed positively, we couldn't have continued with the project.
D. Provided her contribution wouldn't come, we couldn't continue with the project.
Question 74: They are in a dilemma. They can’t decide on how to tackle the financial crisis.
A. They can’t decide to tackle the financial crisis on the spur of the moment.
B. They are caught between the devil and the deep blue sea about how to tackle the
financial crisis.
C. They don’t have a leg to stand on when it comes to the way of tackling the financial crisis.
D. They hit the nail on the head as far as the way of tackling the financial crisis is concerned.
Question 75: “I do wish you’d stop biting your nails, Brian. It really annoys me!” Brian’s
mother said to him.

A. Brian’s mother told him to stop to bite his nails because it really annoyed her.
B. Brian’s mother didn’t like to stop biting his nails because it really annoys her.
C. Brian’s mother didn’t like him to bite his nails because it got on her nerves.
D. Brian’s mother didn’t like him to bite his nails because it was annoyed.
Part 2: Mark the letter A, B, C, or D on your answ er sheet to indicate the sentence
that is closest in meaning to the sentence given in each of the follow ing questions.
Question 76: Calling Jim is pointless because his phone is out of order. A. It’s hopeless
calling Jim because his phone is out of order.
B. It’s no use to call Jim because his phone is out of order.
C. There’s no point in calling Jim because his phone is out of order.
D. It’s worth not calling Jim because his phone is out of order.
Question 77: Many people nowadays find it increasingly difficult to exist on the money they earn.
A. Many people are getting less and less money from work, so they don’t have enough to live.
B. The prices are getting higher and people can’t live with the money they earn.
C. Many people nowadays do not live on salary.
D. Many people nowadays find it difficult to make both ends meet.
Question 78: Farmers are confused and yields are affected by unpredictable rainfall in terms
of quantity and timing.
A. Unpredictable rainfall in terms of quantity and timing keeps farmers on track and affects yields.
B. Unpredictable rainfall in terms of quantity and timing lets farmers off the hook and affects yields.
C. Unpredictable rainfall in terms of quantity and timing brings farmers down to earth and affects yields.
D. Unpredictable rainfall in terms of quantity and timing throws farmers off balance and affects yields.
Question 79: Jenny is so creative that we all can rely on her for original ideas.
A. Creative as Jenny is, we can rely on her for original ideas.
B. Being creative, we can all rely on Jenny for original ideas.
C. So creative is Jenny that we all can rely on her for original ideas.
D. Jenny is such creative girl that we all can rely on her for original ideas.
Question 80: As far as I am concerned, you can do whatever you like.
A. What you like is not as much as what I am concerned about.
B. You can do what you like providing that it is not too far.
C. I am concerned about what you like.
D. In my opinion, you can do what you like.

***** END OF TEST – BEST OF LUCK *****


STT 1 2 Câu 23 A D
MaDe 132 246 Câu 24 A C
Câu 1 C C Câu 25 D D
Câu 2 A A Câu 26 A B
Câu 3 B B Câu 27 D D
Câu 4 C C Câu 28 C C
Câu 5 A A Câu 29 D B
Câu 6 B B Câu 30 B A
Câu 7 C C Câu 31 C A
Câu 8 A A Câu 32 A B
Câu 9 A A Câu 33 A A
Câu 10 B B Câu 34 C A
Câu 11 C C Câu 35 D D
Câu 12 A A Câu 36 B A
Câu 13 C C Câu 37 C D
Câu 14 A A Câu 38 B C
Câu 15 B B Câu 39 C D
Câu 16 B A Câu 40 A B
Câu 17 D D Câu 41 B C
Câu 18 C B Câu 42 C A
Câu 19 B B Câu 43 A A
Câu 20 A C Câu 44 A C
Câu 21 A C Câu 45 D D
Câu 22 B D Câu 46 C B
Câu 47 D C Câu 64 D D
Câu 48 A B Câu 65 C D
Câu 49 B C Câu 66 A C
Câu 50 C A Câu 67 C B
Câu 51 C B Câu 68 B C
Câu 52 D C Câu 69 C B
Câu 53 B A Câu 70 D D
Câu 54 D A Câu 71 A A
Câu 55 D D Câu 72 D D
Câu 56 C C Câu 73 B C
Câu 57 B D Câu 74 B D
Câu 58 C A Câu 75 C C
Câu 59 B B Câu 76 C A
Câu 60 D C Câu 77 D C
Câu 61 A C Câu 78 D B
Câu 62 D D Câu 79 C C
Câu 63 C B Câu 80 D D

ĐỀ THI HỌC SINH GIỎI MÔN TIÊNG ANH 12


Nam 2019_2020

SECTION A: GRAMMAR & VOCABULARY (40 POINTS)

I. Choose the word or phrase (A, B, C or D) which best completes each sentence. (20points)

1. He_________ his son of the dangers of driving too fast in his new car

A. warned B. remembered C. threatened D. concerned

2. The child was_________ by a lorry on the safety crossing in the main street.

A. knocked out B. run across C. run out D. knocked down

3. The independent arbitrator managed to_________ the confrontation between the union and
the employers.

A. refuse B. confuse C. refute D. defuse

4. When I heard the footsteps behind me I was_________ that I would be attacked.

A. horrified B. terror-struck C. terrorized D. terrified

5. His illness made him_________ of concentration.

A. incompetent B. unable C. incapable D. powerless

6. Medieval travelers’ tales of fantastic creatures were often fascinating but not always________.

A. credible B. creditable C. credulous D. imaginable

7. An almost________ line of traffic was moving at a snail’s pace through the town.

A. continuous B. constant C. continual D. stopping

8. Somebody ran in front of the car as I was driving. Fortunately I________ just in time.

A. could stop B. could have stopped C. managed to stop D. must be able to stop

9. You are being thoroughly________ in refusing to allow this ceremony to take place.

A. unrequited B. unrepresentative C. unreliable D. unreasonable

10 The sudden resignation of the financial director put the company in a very_________position.

A. weak B. unsteady C. vulnerable D. collapsed


11. David: Would you like fish or meat? Mary: I_________ fish, please.

A. would rather B. would prefer C. suppose D. believe

12. Many teenagers show signs of anxiety and_________ when being asked about their future.

A. depress B. depression C. depressed D. depressing

13. . A part – time job gives me the freedom to_________ my own interests.

A. pursue B. chase C. seek D. catch

14. The new road currently under_________ will solve the traffic problems in the town.

A. design B. progress C. construction D. work

15. - Daisy: “What a lovely house you have!” - Mary: “_______________.”

A. Lovely, I think so B. Thank you. Hope you will drop in

C. Of course not, it’s not costly D. No problem

16. We were shocked to hear the news of your ________.

A. having fired B. being fired C. having been fired D. to have been fired

17. _______ as taste is really a composite sense made up of both taste and smell.

A. That we refer to B. What we refer to

C. To which we refer D. What do we refer to

18. They are happily married although, of course, they argue _______.

A. most times B. from day to day

C. every now and then D. on the occasion

19. I don’t know French, but I’ll ________.

A. get Tom to translate it B. have it translate

C. have Tom to translate it D. make it translate

20. Doctors advise people who are deficient __________ vitamin C to eat more fruit and
vegetables.
A. from B. of C. in D. for

II. Use the word in capitals at the end of these sentences to form a word that fits in

the blank space. (10 points)

1. The main goals of the Association of Southeast Asian Nations are to promote peace
and________ in the region.
STABLE

2. The security of the earth can be threatened by________ groups.


TERROR

3. I don’t care if you had had too much to drink. Your behaviour last night was______.
DEFEND

4. Her son is always mischievous and________ which annoys her very much.
OBEY

5. The Americans are much more concerned than the Indians and the Chinese with
physical________ when choosing a wife or a husband.
ATTRACT

6. You can never be sure what my sister is going to do. She is so________.
PREDICT

7. He is completely________. Not only is he lazy but he is dishonest too.


EMPLOY

8. His boss told him off because he had behaved________


RESPONSIBLE

9. He won the discus event at the Olympic Games but was later________ when a medical

check proved that he had been taking drugs.


QUALIFY

10. The trouble with Mr. Brown is that he’s so________. One minute he goes mad when you
come late; the next he says nothing. You never know where you are!
CONSIST

III. Find one mistake in each sentence below by choosing the letter A, B, C or D. (10 pts)?
1. Jill mustn’t have arrived yet, otherwise she would have phoned me
A B C D
2. Not many people realize that apples have been cultivating for over 3,000 years
A B C D
3. The building manager is having all the windows and doors replace on the second and third
A B
floor as well as in the restaurant
C D

3. Having live here for seven years, my friend is used to speaking English with all her
A B C D

classmates.

5. If only we knew all this information about the market many weeks ago
A B C D
6. Peter apologized me for not working hard for the final exam.

A B C D

7. John had so interesting and creative plans that everyone wanted to work with him.

A B C D

8. Species become extinct or endangered for the number of reasons, but the primary cause

A B C

is the destruction of habitat by human activities .

9. Were she be invite to their wedding nniversary, she would be very happy .
A B C D

10. Not until the end of prehistoric times that did the first wheeled vehicles appear.
A B C D

SECTION B: READING (40 POINTS)

I. Read the text below and fill in each blank with ONE suitable word. (10 pts)

TSUNAMI IN JAPAN
Japan's most powerful earthquake since records began has struck the north-east coast,triggering a
massive tsunami. Cars, ships and buildings were (1)________ away by a wall ofwater after the 8.9
- magnitude tremor, which struck about 400 kms (250 miles) north-east ofTokyo. A state of
emergency has been declared at a nuclear power plant, (2)________pressure has exceeded normal
levels. Officials say more than 10,000 people are dead and about 5,000 (3)________, but it is
feared the final death toll will be (4)________ higher. Inone ward alone in Sendai, a port city in
Miyagi prefecture, 200 to 300 bodies were found.“The quake has been the fifth-largest in the
world (5)________ 1900 and nearly 8,000(6)________ stronger than the one which devastated
Christchurch, New Zealand, last month”, said scientists. Thousands of people (7)________ near
the Fukushima nuclear power plant have been ordered to evacuate. Japanese nuclear officials said
that pressureinside a boiling water reactor at the plant was running much higher than normal after
the cooling system failed. Officials said they might need to deliberately (8)________ some
radioactive steam to relieve pressure, but that there would be no health risk. US Secretary of State
Hillary Clinton had earlier said the US Air Force had flown emergency coolant to the site. But US
officials later said (9)________ coolant had been handed over because the Japanese had decided
to handle the situation (10)________.The UN's nuclear agency said four nuclear power plants had
been shut down safely.

1._____________ 2._____________ 3._____________ 4._____________

5.____________ 6._____________ 7._____________ 8._____________

9._____________ 10.____________

II. Read the passage and choose the best option for each of the following blanks. (10 pts)

Women nowadays have more (1)________ than those in the past. For example, ourgreat
grandmothers were not allowed to go to school or to work to earn their own
living.(2)________, they had to depend on their husbands financially. Modern women, on
thecontrary, can get good education, have their own careers, and (3)________ their
interests.They can even take good positions in politics if they are competent (4)________ it.
However,women living in our modern society have their (5)________ too. Today women work
harderthan their great grandmothers so that they can gain the (6)________ between working
lifeand family life. Many people predict that by 2032, most (7)________ positions at work
willbe taken by women. Then, it is possible that women will have more (8)________ life
because, (9)________ in a very modern society, the women can’t (10)________ their role inthe
family.

1. A. advances B. advantages C. benefits D. conveniences


2. A. Therefore B. However C. As a result D. Although

3. A. pursue B. support C. promote D. stimulate

4. A. to B. at C. with D. of

5. A. obstacles B. disputes C. profits D. problems

6. A. equality B. stables C. balance D. steadiness

7. A. senior B. junior C. inferior D. superior

8. A. sheltered B. healthy C. strenuous D. active

9. A. though B. even C. ever D. never

10. A. perform B. adopt C. fulfill D. neglect

III. Read the following passage and choose the option that indicates the correct answer to
each of the following questions.(10 pts)

Over the past 600 years, English has grown from a language of few speakers to become the
dominant language of international communication. English as we know it today emerged
around 1350, after having incorporated many elements of French that were introduced
following the Norman invasion of 1066. Until the 1600s, English was, for the most part,
spoken only in England and had not extended even as far as Wales, Scotland, or Ireland.
However, during the course of the next two centuries, English began to spread around the
globe as a result of exploration, trade (including slave trade), colonization, and missionary
work. That small enslaves of English speakers became established and grew in various parts of
the world. As these communities proliferated, English gradually became the primary
language of international business, banking, and diplomacy. Currently, more than 80 percent of
the information stored on computer systems worldwide is in English. Two thirds of the world‟s
science writing is in English, and English is the main language of technology, advertising,
media, international airports, and air traffic controllers. Today there are 700 million English
users in the world, and over half of these are nonnative speakers, constituting the largest
number of nonnative users of any language in the world.

1. What is the main topic of the passage?

A. The number of non-native users of English. C. The expansion of English as an international


language.
B. The French influence on the English language.
D. The use of English for science and technology

2. English began to be used beyond England approximately.............................

A. in 1066 B. around 1350 C. before 1600 D. after 1600

3. According to the passage, all of the following contributed to the spread of English around
the world EXCEPT .....................................

A. the slave trade B. the Norman invasion C. missionaries. D. colonization

4. Which of the following statements is NOT true?

A. Most of the information stored on computer systems is in English.

B. Only one thirds of the world‟s science writing is in languages other than English.

C. English is the only language used in technology, and advertising.

D. International airports and air controllers use mostly English.

5. According to the passage, approximately how many non-native users of English are there in
the world today?

A. A quarter million B. Half a million C. 350 million D. 700 million.

Sustainable architecture - lessons from the ant

Termite mounds were the inspiration for an innovative design in sustainable living

Africa owes its termite mounds a lot. Trees and shrubs take root in them. Prospectors mine
them, looking for specks of gold carried up by termites from hundreds of metres below. And of
course, they are a special treat to aardvarks and other insectivores.

Now, Africa is paying an offbeat tribute to these towers of mud. The extraordinary Eastgate
Building in Harare, Zimbabwe's capital city, is said to be the only one in the world to use the
same cooling and heating principles as the termite mound.

Termites in Zimbabwe build gigantic mounds inside which they farm a fungus that is their
primary food source. This must be kept at exactly 30.5°C, while the temperatures on the
African veld outside can range from 1.5°C at night- only just above freezing - to a baking hot
40°C during the day. The termites achieve this remarkable feat by building a system of vents in
the mound. Those at the base lead down into chambers cooled by wet mud carried up from
water tables far below, and others lead up through a flue to the peak of the mound. By
constantly opening and closing these heating and cooling vents over the course of the day the
termites succeed in keeping the temperature constant in spite of the wide fluctuations outside.

Architect Mick Pearce used precisely the same strategy when designing the Eastgate Building,
which has no air conditioning and virtually no heating. The building - the country's largest
commercial and shopping complex - uses less than 10% of the energy of a conventional
building its size. These efficiencies translated directly to the bottom line: the Eastgate's owners
saved $3.5 million on a $36 million building because an air-conditioning plant didn't have to be
imported. These savings were also passed on to tenants: rents are 20% lower than in a new
building next door.

The complex is actually two buildings linked by bridges across a shady, glass-roofed atrium
open to the breezes. Fans suck fresh air in from the atrium, blow it upstairs through hollow
spaces under the floors and from there into each office through baseboard vents. As it rises and
warms, it is drawn out via ceiling vents and finally exits through forty-eight brick chimneys.

To keep the harsh, high veld sun from heating the interior, no more than 25% of the outside is
glass, and all the windows are screened by cement arches that jut out more than a metre.

During summer's cool nights, big fans flush air through the building seven times an hour to
chill the hollow floors. By day, smaller fans blow two changes of air an hour through the
building, to circulate the air which has been in contact with the cool floors. For winter days,
there are small heaters in the vents.

This is all possible only because Harare is 1600 feet above sea level, has cloudless skies, little
humidity and rapid temperature swings - days as warm as 31°C commonly drop to 14°C at
night. 'You couldn't do this in New York, with its fantastically hot summers and fantastically
cold winters,' Pearce said. But then his eyes lit up at the challenge. 'Perhaps you could store the
summer's heat in water somehow .... '

The engineering firm of Ove Arup & Partners, which worked with him on the design, monitors
daily temperatures outside, under the floors and at knee, desk and ceiling level. Ove Arup's
graphs show that the temperature of the building has generally stayed between 23°C and 25°C,
with the exception of the annual hot spell just before the summer rains in October, and three
days in November, when a janitor accidentally switched off the fans at night. The atrium,
which funnels the winds through, can be much cooler. And the air is fresh - far more so than in
air-conditioned buildings, where up to 30% of the air is recycled.

Pearce, disdaining smooth glass skins as 'igloos in the Sahara', calls his building, with its
exposed girders and pipes, 'spiky'. The design of the entrances is based on the porcupine-quill
headdresses of the local Shona tribe. Elevators are designed to look like the mineshaft cages
used in Zimbabwe's diamond mines. The shape of the fan covers, and the stone used in their
construction, are echoes of Great Zimbabwe, the ruins that give the country its name.

Standing on a roof catwalk, peering down inside at people as small as termites below, Pearce
said he hoped plants would grow wild in the atrium and pigeons and bats would move into it,
like that termite fungus, further extending the whole 'organic machine' metaphor. The
architecture, he says, is a regionalised style that responds to the biosphere, to the ancient
traditional stone architecture of Zimbabwe's past, and to local human resources.

Choose the correct answer, A, B, C or D.

1 Why do termite mounds have a system of vents?

A to allow the termites to escape from predators C to allow the termites to work efficiently

B to enable the termites to produce food D to enable the termites to survive at night

2 Why was Eastgate cheaper to build than a conventional building?

A Very few materials were imported. C Its tenants contributed to the costs.

B Its energy consumption was so low. D No air conditioners were needed.

3 Why would a building like Eastgate not work efficiently in New York?

A Temperature change occurs seasonally rather than daily.

B Pollution affects the storage of heat in the atmosphere.

C Summer and winter temperatures are too extreme.

D Levels of humidity affect cloud coverage.

4 What does Ove Arup's data suggest about Eastgate's temperature control system?

A It allows a relatively wide range of C It functions well for most of the year.
temperatures.
D The temperature in the atrium may fall too
B The only problems are due to human error. low.

5 Pearce believes that his building would be improved by

A becoming more of a habitat for wildlife. B even closer links with the history of
Zimbabwe.
C giving people more space to interact D better protection from harmful
with nature. organisms.

The accidental rainforest

According to ecological theory, rainforests are supposed to develop slowly over millions of
years. But now ecologists are being forced to reconsider their ideas

When Peter Osbeck, a Swedish priest, stopped off at the mid-Atlantic island of Ascension in
1752 on his way home from China, he wrote of 'a heap of ruinous rocks' with a bare, white
mountain in the middle. All it boasted was a couple of dozen species of plant, most of them
ferns and some of them unique to the island.

And so it might have remained. But in 1843 British plant collector Joseph Hooker made a brief
call on his return from Antarctica. Surveying the bare earth, he concluded that the island had
suffered some natural calamity that had denuded it of vegetation and triggered a decline in
rainfall that was turning the place into a desert. The British Navy, which by then maintained a
garrison on the island, was keen to improve the place and asked Hooker's advice. He suggested
an ambitious scheme for planting trees and shrubs that would revive rainfall and stimulate a
wider ecological recovery. And, perhaps lacking anything else to do, the sailors set to with a
will.

In 1845, a naval transport ship from Argentina delivered a batch of seedlings. In the following
years, more than 200 species of plant arrived from South Africa. From England came 700
packets of seeds, including those of two species that especially liked the place: bamboo and
prickly pear. With sailors planting several thousand trees a year, the bare white mountain was
soon cloaked in green and renamed Green Mountain, and by the early twentieth century the
mountain's slopes were covered with a variety of trees and shrubs from all over the world.

Modern ecologists throw up their hands in horror at what they see as Hooker's environmental
anarchy. The exotic species wrecked the indigenous ecosystem, squeezing out the island's
endemic plants. In fact, Hooker knew well enough what might happen. However, he saw
greater benefit in improving rainfall and encouraging more prolific vegetation on the island.

But there is a much deeper issue here than the relative benefits of sparse endemic species
versus luxuriant imported ones. And as botanist David Wilkinson of Liverpool John Moores
University in the UK pointed out after a recent visit to the island, it goes to the heart of some of
the most dearly held tenets of ecology. Conservationists' understandable concern for the fate of
Ascension's handful of unique species has, he says, blinded them to something quite
astonishing - the fact that the introduced species have been a roaring success.
Today's Green Mountain, says Wilkinson, is 'a fully functioning man-made tropical cloud
forest' that has grown from scratch from a ragbag of species collected more or less at random
from all over the planet. But how could it have happened? Conventional ecological theory says
that complex ecosystems such as cloud forests can emerge only through evolutionary processes
in which each organism develops in concert with others to fill particular niches. Plants co-
evolve with their pollinators and seed dispersers, while microbes in the soil evolve to deal with
the leaf litter.

But that's not what happened on Green Mountain. And the experience suggests that perhaps
natural rainforests are constructed far more by chance than by evolution. Species, say some
ecologists, don't so much evolve to create ecosystems as make the best of what they have. 'The
Green Mountain system is a man-made system that has produced a tropical rainforest without
any co-evolution between its constituent species,' says Wilkinson.

Not everyone agrees. Alan Gray, an ecologist at the University of Edinburgh in the UK, argues
that the surviving endemic species on Green Mountain, though small in number, may still form
the framework of the new ecosystem. The new arrivals may just be an adornment, with little
structural importance for the ecosystem.

But to Wilkinson this sounds like clutching at straws. And the idea of the instant formation of
rainforests sounds increasingly plausible as research reveals that supposedly pristine tropical
rainforests from the Amazon to south-east Asia may in places be little more than the
overgrown gardens of past rainforest civilisations.

The most surprising thing of all is that no ecologists have thought to conduct proper research
into this human-made rainforest ecosystem. A survey of the island's flora conducted six years
ago by the University of Edinburgh was concerned only with endemic species. They
characterised everything else as a threat. And the Ascension authorities are currently turning
Green Mountain into a national park where introduced species, at least the invasive ones, are
earmarked for culling rather than conservation.

Conservationists have understandable concerns, Wilkinson says. At least four endemic species
have gone extinct on Ascension since the exotics started arriving. But in their urgency to
protect endemics, ecologists are missing out on the study of a great enigma.

'As you walk through the forest, you see lots of leaves that have had chunks taken out of them
by various insects. There are caterpillars and beetles around,' says Wilkinson. 'But where did
they come from? Are they endemic or alien? If alien, did they come with the plant on which
they feed or discover it on arrival?' Such questions go to the heart of how rainforests happen.
The Green Mountain forest holds many secrets. And the irony is that the most artificial
rainforest in the world could tell us more about rainforest ecology than any number of natural
forests.

Do the following statements agree with the information given in Reading Passage 3? In
boxes 1-6 on your answer sheet write

TRUE if the statement agrees with the information

FALSE if the statement contradicts the information

NOT GIVEN if there is no information on this

1 When Peter Osbeck visited Ascension, he found no inhabitants on the island.

2 The natural vegetation on the island contained some species which were found nowhere else.

3 Joseph Hooker assumed that human activity had caused the decline in the island's plant
life.

4 British sailors on the island took part in a major tree planting project.

5 Hooker sent details of his planting scheme to a number of different countries.

6 The bamboo and prickly pear seeds sent from England were unsuitable for Ascension.

Complete each sentence with the correct ending A-G from the box below.

Write the correct letter A-G in boxes 7-10 on your answer sheet.

7 The reason for modern conservationists' concern over Hooker's tree planting programme is that

8 David Wilkinson says the creation of the rainforest in Ascension is important because it
shows that

9 Wilkinson says the existence of Ascension's rainforest challenges the theory that

10 Alan Gray questions Wilkinson's theory, claiming that

A other rainforests may have originally been planted by man.

B many of the island's original species were threatened with destruction.


SECTION
C the species in the original C:were
rainforest WRITING (20 POINTS)
more successful than the newer arrivals.

D rainforests can only develop through a process of slow and complex evolution.

E steps should be taken to prevent the destruction of the original


I. Finish the second sentence in such a way that it means exactly the same as the sentence
printed before it. (10 pts)

1. I am sure it wasn’t Mrs. Brown you saw yesterday because she had gone abroad.
It can’t have been Mrs. Brown you saw yesterday because she had gone abroad.
2. Some scientists report that dolphins have a brain capacity larger than human beings’

Dolphins are reported to have a brain capacity larger than human beings.
3. They think that someone started the fire on purpose

The fire is thought to have started been started on purpose.


4. After Louie had written his composition, he handed it to his teacher.
Having written his composition, Louie handed it to his teacher.
5. If only I had studied hard enough to pass the final exam.
I regret not studying hard enough to pass the final exam.

6. John speaks Chinese fluently because he used to live in China for ten years.
Had John not lived in China for ten years, he could not speak Chinese fluently.

7. "How beautiful is the dress you have just bought!" Peter said to Mary.
Peter complimented Mary on her beautiful dress.
8. "You’re always making terrible mistakes," said the teacher.
The teacher complained about his student‟s making terrible mistakes

9. Sue is too slow to understand what you might say.

So slow is Sue that she can’ t understand what you might say.

10. Although it was expected that he would stand for election, he didn’t.

Contrary to what people expected, he didn’t stand for election.


II. Write a new sentence similar in meaning to the given one, using the word given in the
brackets. Do not alter the word in any way. (10 pts)

1. You looked tired. Why don’t you go to bed early tonight? (better)

You had better go to bed early tonight as you looked tired.

2. Zoe has a job which makes her feel very stressful. (less)

Zoe wishes she had a less stressful job.

3. Simon wants to be left alone because he’s upset. (rather)

Simon would rather be left alone because he’s upset.

4. He’d rather eat with friends than eat alone.. ( prefers)

He prefers eating with friends to eating alone.

5. He is too irresponsible to run the department. (charge)

He is not responsible enough to be in charge.

6. His arrival was completely unexpected. (took)

His arrival took us by surprise.

7. If I help you now, don’t assume I’ll help you next time.(count)

If I help you now, don’t count on me to help you next time.

8. He owes his life to that surgeon. (indebted)

He is indebted to that surgeon for saving his life

9. Don’t pay any attention when she complains. (notice)

Don’t take any notice of her when she complains.

10. A rejection of their offer would have been unwise. (accepted)

Not to have accepted their offer would have been unwise


ANSWER KEYS

SECTION A

I.

1.A 5.C 9.D 13.A 17.B

2.D 6.A 10.C 14.C 18.C

3.D 7.A 11.B 15.B 19.A

4.D 8.C 12.B 16.C 20.C

II.

11.STABILITY 16.UNPREDICTABLE
12.TERRORIST 17.UNEMPLOYED
13.INDEFENSIBLE 18.IRRESPONSIBLY
14.DISOBEDIENT 19.DISQUALIFIED
15.ATTRACTIVENESS 20.INCONSISTENT

III.
1.A 3.B 5.B 7.A 9.A
2.C 4.A 6.A 8.C 10.B

SECTION B

I.
1. SWEPT 5. SINCE 9. NO
2. WHERE 6.TIMES 10. THEMSELVES
3. MISSING 7.LIVING
4. MUCH 8. RELEASE
II.
11.B 13.A 15.D 17.A 19.B
12.C 14.B 16.C 18.C 20.D

III. READING COMPREHENSION


PASSAGE1
6. C 7. D 8. B 9. C 10.C
PASSAGE 2

6. B 7. D 8. A 9. C 10.A
PASSAGE 3

11.NG
12.T
13.F
14.T
15.NG
16.F
17.B
18.F
19.D
20.G
TRƯỜNG THPT NÔNG CỐNG I ĐỀ THI THỬ KHẢO SÁT CHẤT LƯỢNG
TỔ TIẾNG ANH ĐỘI TUYỂN HỌC SINH GIỎI CẤP TỈNH LẦN 4
MÔN THI: TIẾNG ANH 12 Thời gian làm bài: 180
ĐỀ CHÍNH THỨC phút, không kể thời gian phát đề. (Đề thi có 12 câu, gồm
4 trang)

Part A : PHONETICS ( 5pts )


I. Choose the word whose underlined part is pronounced differently from the rest in the same line. (2pts)
1.A. teenage B. dosage C. voyage D. carriage
2.A. monkey B. hunger C. hunter D. banker
4) Identify the word whose stressed pattern is different from that of the others. (3pts)
3.A. instrument B. indispensable C. stimulate D. symphony
4.A. preserves B. tropical C. climate D. temperature
5.A. satisfaction B. mathematics C. opportunity D. mathematician

Part B : LEXICAL AND GRAMMAR ( 45 pts )


I. Choose the option A, B, C or D to indicate the word or phrase which best completes each sentence.(15pts)
6.The criminal was sentenced to death because of _______ of his crime.
A. the severity B. the complexity C. a punishment D. the importance
7.“ So how are things at school, Tim? “________________________”
A. Oh, pretty good, actually . B. Well, I can’t agree with you.
C. It’s my pleasure. D. I was not very good at it.
8.Laura is very thin, _______her young sister, who is quite heavy.
A. dissimilar to B. unlikely C. unlike D. dislike
4) Please_______and see us some time. You’re always welcome.
A. come to B. come about C. come away D. come around
10. The proposal has not met with_______.agreement.
A. voluntary B. universal C. informal D. effective
11. A child's vocabulary_______.through reading.
A. expands B. expends C. expels D. exposes
12. We_______friends even after we grew up and left home.
A. became B. made C. struck up D. remained
13. This picture book, the few pages _____ are missing, is my favorite.
A. for which B. of that C. to which D. of which
14. I am_______ tired to think about that problem now.
A. much more B. far too C. nearly D. very
15. My neighbour _______ me a cup of coffee.
A. begged B. suggested C. offered D. invited
16. Why don’t the police take _______ measures against crime?
A. effective B. affective C. ineffective D. effecient
17. What happened ___ their car broke down on the motorway so they didn’t get to Jo’s wedding on time.
A. to be that B. being that C. was that D. to that
18. War stole his youth and his home, _______.
A. as a result B. moreover C. however D. furthermore
19.- “Do you like frog’s legs?” - “_______ them, I don’t really know.”
A. Never try B. Not to have try C. Never having tried D. Never had tried
20. My friend has _______ for a bargain.
A. a sharp ear B. a keen eye C. a strong head D. a keen ear
2 The following passage contains 10 errors. Find and correct them (10pts)
Who were the people responsible for collection and sending plants from one country to another? And why did they
do it? Initially they were travelers with other purposes: traders, colonists, pilgrims and missionaries have all been
important in providing new plants for English gardens. They sent back indigenous wild plants, or sometimes, as in
the cases of visitors to China and Japan, plants which have been cultivated and improved for hundreds of years. This
worked, of course, in both directions: English gardens were making in the most unlikely places. Travelers did not
always recognize an interesting plant on seen it – interesting, that is, to the collector at home.
So in the 16th and 17th century, attempts were made to collect on a most professional basis, either 26
by patrons sending collections into the field, or by subscriptions to finance local enthusiasts in the
27
most promised areas. By 1611 John Tradescant was traveling and collecting in France and other
28
parts of Europe. Lately, Peter Collinson, a London merchant, who had seen the richness of the
29
plant material sending back by Tradescant, organized a syndicate to finance the amateur botanist
30
John Bartram. Before long, special collectors were being dispatched to all parts of the world by
institutions such as the Chelsea Physic Garden.

4. Put each verb in brackets into an appropriate form. (10pt )


31. A - Stop! You (not see) _______ the notice?
B.- I see it but I can’t read it. What it (say) _____?
32. That man was a little mad. He always (try) _______ to improve that the earth was flat.
33. Something tells me that you (not listen)______ to a single word I have said in the past ten minutes.
34. Hi, Paul! What a nice surprise! I (not think) _______ I (run) _______ into you today.
35. I’m so confused that I don’t know what (do) _______
36. Over the last few months, garages (put) ________ up the price of petrol four times.
37. Smith had a lucky escape. He (kill) _______.
38. The two men never (introduce) ________ formally stared at each other across the table.

IV. Put each word in brackets into an appropriate form. (10pt)

39. The policy is __________ and so will cost the government a lot of money. ECONOMY
40. Some parents feel dismayed because of their children’s_________ BEHAVE
41. Whatever we most dislike or fear in others is sure to be a(n) ______ aspect of ourselves. KNOWLEDGE
42. “Have you got any ___ about the corporation?” - “Oh, no, I’m sure it will be successful.” GIVE
43. In every presentation, it is crucial that there is ________ between the presenter and the ACT
audience.
44. Not sick, Mai guessed, but probably ____ now that she drank a lot at the party last night. HUNG
45. Vietnamese people are happy to talk about their past and show an amazing resilience and FORGIVE
________.
46. This chemical removes unsightly _____ in order to restore wood to its natural colour. COLOR
47. ________ beautiful, Hue, the former capital of Vietnam, is steeped in history. BREATHE
48. The reckless driver was imprisioned due to his______ of the traffic law. OBSERVE

Part C : READING ( 30 pts )

I. Read the following passage and then choose the most suitable word or phrase for each space. (10 pt)

THE LANGUAGE OF TEARS

The ability to weep is a uniquely human form of emotional response. Some scientists have suggested that human
tears are evidence of an aquatic past - but this does not seem very likely. We cry from the moment we enter this
(49) _______ for a number of reasons. Helpless babies cry to persuade their parents that they are ill, hungry or

uncomfortable. As they (50) _______ they will also cry just to attract parental attention and will often stop when
they get it.

The idea that having a good cry can do you (51) _______ is a very old one and now it has scientific (52)_______
since recent research into tears has shown that they contain a natural painkiller called enkaphalin. By (53)
_______ sorrow and pain this chemical helps you to feel better. Weeping can increase the quantities of enkaphalin
you (54) _______.
Unfortunately, in our society we impose restrictions upon this naturally (55) _______ activity. Because some

people still regard it as a (56) _______ of weakness in men, boys in particular are admonished when they cry. This
kind of repression can only increase stress, both emotionally and physically.

Tears of emotion also help the body (57) _______ itself of toxic chemical waste, for there is more protein in

them than in tears resulting from cold winds or other irritants. Crying comforts, calms and can be very enjoyable -
consider the popularity of the highly emotional films which are commonly (58) ______ "weepies". It seems that
people enjoy crying together almost as much as laughing together.

49. A. world B. place C. earth D. space


50. A. evolve B. change C. develop D. alter
51. A. better B. fine C. good D. well
52. A. validity B. truth C. reality D. reason
53. A. struggling B. fighting C. opposing D. striking
54. A. construct B. achieve C. provide D. produce
55. A. curing B. treating C. healing D. improving
56.A. hint B. symbol C. feature D. sign
57.A. release B. rid C. loosen D. expel
58.A. named B. entitled C. subtitled D. called

11.Supply the most suitable word for each blank. (10 pts)

True relaxation is most certainly not a matter of flopping down in front of the television with a welcome drink. Nor
is it about drifting (59) ___ an exhausted sleep. Useful though these responses to tension and over-tiredness

E. ___ be, we should distinguish between them and conscious relaxation in (61) ___ of quality and effect. (62)

___ of the level of tiredness, real relaxation is a state of alert yet at the same time passive awareness, in which our
bodies are (63) ___ rest while our minds are awake.

Moreover, it is as natural for a healthy person to be relaxed when moving as resting. (64) ___ relaxed in action
means we bring the appropriate energy to everything we do, so as to have a feeling of healthy tiredness by the end
of the day, (65) ___ than one of exhaustion. Unfortunately, as a result of living in today’s competitive world, we
are under constant strain and have difficulty in coping, (66) ___ alone nurturing our body’s abilities. What needs to
be rediscovered is conscious relaxation. With (67) ___ in mind we must apply ourselves to understanding stress
and the nature of its causes (68) ___ deep-seated.

III. Read the following passage, and then choose the best answer from A, B, C or D.(10pts)

We find that bright children are rarely held back by mixed-ability teaching. On the contrary, both their knowledge
and experience are enriched. We feel that there are many disadvantages in streaming pupils. It does not take into
account the fact that children develop at different rates. It can have a bad effect on both the bright and the not-so-
bright child. After all, it can be quite discouraging to be at the bottom of the top grade!
Besides, it is rather unreal to grade people just according to their intellectual ability. This is only one aspect of their
total personality. We are concerned to develop the abilities of all our pupils to the full, not just their academic
ability. We also value personal qualities and social skills, and we find that mixed-ability teaching contributes to all
these aspects of learning.

In our classrooms, we work in various ways. The pupils often work in groups: this gives them the opportunity to
learn to co-operate, to share, and to develop leadership skills. They also learn how to cope with personal problems
as well as learning how to think, to make decisions, to analyze and evaluate, and to communicate effectively. The
pupils learn from each other as well as from the teachers.

Sometimes the pupils work in pairs; sometimes they work on individual tasks and assignments, and they can do this
at their own speed. They also have some formal class teaching when this is appropriate. We encourage our pupils to
use the library, and we teach them the skills they need in order to do this effectively. An advanced pupil can do
advanced work; it does not matter what age the child is. We expect our pupils to do their best, not their least, and
we give them every encouragement to attain this goal.

69. In the passage, the author’s attitude towards “mixed-ability teaching” is ______.
A. objective B. questioning C. critical D. approving
70. The words “held back” in 1st paragraph means “______”.
A. made to lag behind in study B. forced to study in lower classes

C. prevented from advancing D. made to remain in the same classes

71. The author argues that a teacher’s chief concern should be the development of the pupils’ ______.

A. total personality B. learning ability and communicative skills

C. intellectual abilities D. personal and social skills


E. Which of the following is NOT mentioned in the passage?

Group work provides the pupils with the opportunity to learn to be capable organizers.
Pupils also learn to develop their reasoning ability.
Group work gives pupils the opportunity to learn to work together with others.
Pupils also learn how to participate in teaching activities.
F. The author’s purpose of writing this passage is to ______.
recommend pair work and group work classroom activities
argue for teaching bright and not-so-bright pupils in the same class
emphasize the importance of appropriate formal classroom teaching
offer advice on the proper use of the school library

G. According to the passage, which of the following is NOT TRUE?


There is no fixed method in teaching pupils to develop themselves to the full.
Development of pupils as individuals is not the aim of group work.
It’s not good for a bright child to find out that he performs worst in a mixed-ability class.
Pupils cannot develop in the best way if they are streamed into classes of different intellectual abilities.

H. According to the passage, which of the following is an advantage of mixed-ability teaching?


Pupils can be hindered from an all-round development.

Formal class teaching is the important way to give the pupils essential skills such as those to be used in
the library.

A pupil can be at the bottom of a class.

Pupils as individuals always have the opportunities to work on their own.

Part D : WRITING ( 20 pts )


I. Finish each of the following sentences in such a way that it is as similar as possible in meaning to the
sentence printed before it.(5pts)

79. David said that the accident was his fault. TOOK
→……………………………………………………………………………………

80. The stranded climber would never have been rescued if his brother hadn’t had an ingenious plan.
INGENUITY
→……………………………………………………………………………………

81. We didn’t think that he would win the price DOUBTFUL


→……………………………………………………………………………………

82. She didn’t shed a tear when the story ended in tragedy. NOT
→……………………………………………………………………………………

83. They pretended to be enjoying themselves, but they weren’t really. AS


→……………………………………………………………………………………

II. Complete each of the following sentences in such a way that it means exactly the same as the sentence
printed before it. (5pts)

84. Children learn a lot about how to behave in a situation like this.
→ Only …………………………………………………..…………………………
85. Such a ridiculous proposal isn't worth serious consideration.
→ There is ……………………………………………………………………………

86. I certainly won’t change my mind about resigning.


→ My changing …………………………………………………………..……………

87. Harriet was upset because she saw Peter with another woman.
→ It was ……………………………………………………………….………………
88. When Marry heard the results, he began to feel more confident.
→ Since …………………………………………………..……………………………

III. Do you agree or disagree with the following statement? “Games are as important for children as they are
for adults.” Write an essay (about 150 to 200 words) to express your personal point of view.(10 pts)

_________________ The End __________________________


ĐÁP ÁN
Part A : PHONETICS ( 5pts )
I. Choose the word whose underlined part is pronounced differently from the rest in the same line.
( 2pts )
1A 2C
II. Pick out the word whose main stressed syllable is different from the rest in the same line. (3pts )
3B 4A 5D
Part B : LEXICAL AND GRAMMAR ( 45 pts )
I. Choose the option A, B, C or D to indicate the word or phrase which best completes each
sentence.(15pts)
6A 7A 8C 9D 10 B
11 A 12 D 13 D 14 B 15 C
16 A 17 C 18 A 19 C 20 B
II. The following passage contains 10 errors. Find and correct them (10pts)
Who were the people responsible for collection and sending plants from one country to another? And why did
they do it? Initially they were travelers with other purposes: traders, colonists, pilgrims and missionaries have
all been important in providing new plants for English gardens. They sent back indigenous wild plants, or
sometimes, as in the cases of visitors to China and Japan, plants which have been cultivated and improved for
hundreds of years. This worked, of course, in both directions: English gardens were making in the most
unlikely places. Travelers did not always recognize an interesting plant on seen it – interesting, that is, to the
collector at home. So in the 16th and 17th century, attempts were made to collect on a most professional basis,
either by patrons sending collections into the field, or by subscriptions to finance local enthusiasts in the most
promised areas. By 1611 John Tradescant was traveling and collecting in France and other parts of Europe.
Lately, Peter Collinson, a London merchant, who had seen the richness of the plant material sending back by
Tradescant, organized a syndicate to finance the amateur botanist John Bartram. Before long, special collectors
were being dispatched to all parts of the world by institutions such as the Chelsea Physic Garden.

Mistake Correction Mistake Correction


21 collection collecting 26 most more
22 cases case 27 collections collectors
23 have had 28 promised promising
24 making made 29 Lately Later
25 seen seeing 30 sending sent
III. Put each verb in brackets into an appropriate form. ( 10pts )

31. Don’t you see/ does it say


33. haven’t been listening
35. should be done
37. could have been killed
32. was always trying
34. didn’t think/ would run
36. have put
38. having never been introduced
IV. Put each word in brackets into an appropriate form. ( 10pts )
39. UNECONOMICAL 40. MISBEHAVIORS/ MISBEHAVIOURS
41. UNACKNOWLEDGED: không được thừa nhận 42. MISGIVINGS
43. INTERACTION 44. HUNG-OVER choáng váng, buồn nôn

45. FORGIVENESS: sự khoan dung. 46. DISCOLORATION sự đổi màu;


sự bẩn màu; sự bạc màu
47. BREATHTAKINGLY. 48. NON-OBSERVATION
Part C : READING ( 30 pts )
I. Read the following passage and then choose the most suitable word or phrase for each space. (10 pt )

49.A 50. C 51. C52.A 53. B 54.D 55. C 56. D57. B 58. D
II. Supply the most suitable word for each blank. (10 pts)
59. into 60. might/ can/ may 61. terms 62. regardless/ 63. at
irrespective
64. Being 65. rather 66. let 67. that/this 68. how

69. D 70. C 71. A 72. D 73.B


74. B 75. D 76. D 77. A 78.D
Part D : WRITING ( 20 pts )
I. Finish each of the following sentences in such a way that it is as similar as possible in meaning to the
sentence printed before it.(5pts)
79. David took the blame/responsibility for the accident.

80. But for the ingenuity of his brother’s plan, the stranded climber would never have been rescued.
81. We were doubtful about his change of winning the price.
82. Not a tear did she shed when the story ended in tragedy.
83. They acted as if they were enjoying themselves, but they weren’t really.

II. Complete each of the following sentences in such a way that it means exactly the same as the sentence
printed before it. ( 5pts )

84. in a situation like this can children learn a lot about how to behave.
85. no point in considering such a ridiculous proposal seriously.
86. my mind about resigning is out of question.
87. because she saw Peter with another woman that Harriet was upset / that upset Harriet.
88. hearing the results, Marry has felt more confident.

III. Do you agree or disagree with the following statement? “Games are as important for children as they are
for adults.” Write an essay (about 150 to 200 words) to express your personal point of view.(10 pts)

Marking scheme: The impression mark should be based on the following scheme:

1.Format: 2 pts ( coherence , cohesion , style )


The argument should have 3 parts
a. Introduction: should be put in one paragraph in which students’ points of view are expressed clearly

b. Body: should consist of from one to three paragraphs. Each paragraph must have a topic sentence,
reasonings and examples.
c. Conclusion (summary of the main reasons, students’ comment)
2. Content : 5pts
3. Language : 3pts ( grammatical accuracy , wide range of vocabularies and structures )

Everyone likes to play games. Games are important at any age to keep your mind sharp, learn new things, and
maintain social skills.

When you play games, you exercise your mind. This becomes more important as you grow older. By
concentrating on the tactics of a game, memorizing moves, and following your opponent’s strategies, you can
keep your brain functioning and growing.

Playing games can teach you a lot. Games that ask questions, for example, show you what you don’t know.
You can learn about things like geography and history when you play certain games. It’s a fun way to learn,
and adults enjoy this as much as children do.

Games require the use of social skills. When you play games, you interact with other people. You have to be
considerate of them and you have to play fairly. Playing games allows you to maintain personal contacts. This
is important for people of all ages.

Regardless of your age, playing games can help you keep your mind alert, learn new things, and build
friendships.

HƯỚNG DẪN CHẤM:

Bài thi chấm theo thang điểm: 20

Điểm bài thi làm tròn đến 0,25


Tổng số point thí sinh làm đúng
Điểm bài thi = ---------------------------------------
5
SỞ GD&ĐT VĨNH PHÚC ĐỀ THI CHỌN HỌC SINH GIỎI CẤP TRƯỜNG LẦN 1
TRƯỜNG THPT ĐỒNG ĐẬU MÔN: TIẾNG ANH 12
NĂM HỌC 2019 - 2020
(Đề thi gồm có 05 trang)
(Thời gian làm bài: 180 phút - không kể thời gian giao đề)

A. PHONETICS (5 POINTS)
Find the word with the stress pattern different from that of the other three words in each question.
1. A. community B. continue C. condition D. interested
2. A. conclude B. advise C. depend D. suffer
3. A. flourish B. season C. today D. product
4. A. attitude B. infamously C. biology D. socialism
5. A. magazine B. possession C. behavior D. experience

B. GRAMMAR – VOCABULARY (35 points)


I. Choose the word, phrase or expression which best completes each sentence (15 points)
1. - Jack: “I think married men should share the housework with their wives.”
- Peter: “_________”
A. It’s my pleasure. B. No problem. C. Well done! D. You can say that again!
2. It’s becoming _________ difficult to find a good job nowadays.
A. more and more B. the more C. the better D. better and better
3. Jasmine, _______ , grows only in warm places.
A. is a vine plant with fragrant flowers B. a vine plant with fragrant flowers
C. that is a vine plant with fragrant flowers D. a vine plant with fragrant flowers it is
4. He did not share his secrets with many people, but he _________ in her.
A. confessed B. concealed C. consented D. confided
5. You've been studying hard for your exam. I think you should call it _______
A. a week B. a day C. a night D. a minute
6. Your drink cost $40, you gave me a $50 note and here is your _______.
A. change B. supply C. cash D. cost
7. Mary bought ________ yesterday.
A. a red big plastic hat B. a big red plastic hat
C. a plastic big red hat D. a bit plastic red hat
8. _________ of birds over a city usually predict cold weather.
A. Herds B. Packs C. Flocks D. Groups
9. If you had gone to bed early last night, you _________ late for school now.
A. wouldn’t have been B. wouldn’t be C. weren’t D. hadn’t been
10. Never before _________ such a wonderful child.
A. I saw B. have I seen C. I had seen D. I have seen
11, A(n) _________is a story long enough to fill a complete book, in which the
characters and events are usually imaginary.
12. This is ...................... the most difficult job I have ever had to do.
A. by far B. by chance C. by heart D. by myself
13. In Vietnam, two or more ________ may live in a home.
A. generations B. generous C. generation D. generators
14. People can become very _______ when they are stuck in traffic for a long time.
A. single-minded B. bad-tempered C. even-tempered D. even-handed
15. You __________ those tomatoes. I have already washed them.
A. must wash B. mustn’t wash C. need washing D. needn’t wash

5) Write the correct FORM of each bracketed word in the numbered space provided in the column
on the right. (0) has been done as an example. (10 points).
According to some (0) _____ (SCIENCE), high-risk sports can be 0. scientists
particularly (1) _____ (VALUE) for certain types of people. Such 1. _____________________
activities help them to learn that being (2) _____ (FRIGHT) doesn’t 2. _____________________
mean that they have to lose control. The recent fashion for jumping
from bridges attached to a (3) _____ (LONG) of elastic rope, known 3. _____________________
as “bungee jumping”, has now been tried by over one million people
(4) _____ (WORLD) and interest in it is continuing to grow. 4. _____________________
Before the special elastic rope (5) _____ (TIGHT) around them, 5. _____________________
jumpers reach speeds of nearly 160kph. First-timers are usually too (6) 6. _____________________
_____ (TERROR) to open their mouths, and when they are finally (7) 7. _____________________
_____ (LOW) safely to the ground, they walk around with broad smiles
on their faces, saying (8) _____ (REPEAT) how amazing it was. 8. _____________________
However, for some people, it is only the (9) _____ (EMBARRASS) of 9. _____________________
refusing to jump at the last minute that finally persuades them to
conquer their fear of (10) _____ (HIGH) and push themselves off into 10. _____________________
space.

III. Find out one mistake in each sentence and correct it. (5 points)
3 I watched the train to leave until it was out of sight.
………………………………………………………………………………………………
4 They didn't seem to take any notice of that the teacher said.
………………………………………………………………………………………………
5 Tom's very good at science when his brother is absolutely hopeless.
………………………………………………………………………………………………
6 Nobody watched, so the little boy took the packet of sweets from the shelf and putting it in his
pocket.
………………………………………………………………………………………………
7 Maria asked Sandra whether she knew the post office had been at strike for the past week.
………………………………………………………………………………………………

IV) Put each verb in brackets into an appropriate form. (5 pts)


5. I feel as if I (fly)………… now.
6. On the first day of next month, she (be)…………a teacher for 3 years.
7. Hardly I (arrive)…………when a quarrel broke out.
8. It is vitally important that the order (cancel)…………… immediately.
9. The cake (make) ……….. by my mom tastes really delicious.
C. READING (30 points)
I. Read the text and fill in each blank with ONE suitable word. (10 points)
A small rain cloud has saved the lives of three sailors on a sinking boat. Water began to pour into their
boat about 300 kilometers from the coast of Portugal, and the frightened sailors called …(1)… help on
their radio. The waves …(2)… almost 20 meters high and the …(3)… blew the tiny boat from side
…(4)… side. Soon even the radar broke …(5)….
Then suddenly they heard the calm …(6)… of a French sea captain over …(7)… radio. “Is it
raining?” he asked. …(8)… thought it was rather a silly …(9)… but they answered. “It is raining very
…(10)…,” they called back over the radio.
“I’ve seen rain cloud on my radar screen,” the French captain said. “If you are under it, I can
easily find you.” He changed course immediately and went straight to the small boat.

12.Read the following passages carefully, then choose the best answer to each question. (10
points)
Langston Hughes was one of the greatest American writers of the twentieth century. He was born
in Joplin, Missouri, and moved to Cleveland at the age of 14. Several years later he spent one year in
Mexico before attending Columbia University in New York. For a few years after that, he roamed the
world as a seaman, visiting ports around the world and writing some poetry. He returned to the United
States and attended Lincoln University, where he won the Witter Bynner Prize for undergraduate
poetry. After graduating in 1928, he traveled to Spain and to Russia with the help of a Guggenheim
fellowship. His novels include Not without Laughter (1930) and The Big Sea (1940). He wrote an
autobiography in 1956 and also published several collections of poetry. The collections include The
Weary Blues (1926), The Dream Keeper (1932), Shakespeare in Harlem (1942), Fields of Wonder
(1947), One Way Ticket (1947), and Selected Poems (1959) A man of many talents, Hughes was also
a lyricist, librettist, and a journalist. As an older man in the 1960s, he spent much of his time collecting
poems from Africa and from African-Americans to popularize black writers. Hughes is one of the most
accomplished writers in American literary history, and he is seen as one of the artistic leaders of the
Harlem Renaissance, the period when a neighborhood that was predominantly black produced a flood
of great literature, music, and other art forms depicting daily city life for African-Americans.

1.. What is the main topic of this passage?


A. The life of Langston Hughes B. The Harlem Renaissance
C. African-American writers D. American twentieth-century writers
2 .Where was Langston Hughes born?
A. Spain B. New York C. Missouri D. North Carolina
3, As used in the passage, which of the following words could best replace the word "ports"?
.
A. Islands B. Ships C. Friends. D. Harbors
4.. To which of the following movements might “Shakespeare in Harlem” refer?
A. The Civil War B. The Harlem Riots
C. The Harlem Renaissance D. The Civil Rights Movement
5.. What provided Hughes with assistance for his travel to Spain and Russia?
A. His job as a reporter B. His career as a soldier
C. A literary fellowship D. A college study program
6. The word "talents" in the passage could be replaced by which of the following?
A Desires B. Abilities C. Strategies D. Careers
7. According to the author, what did Hughes do during the later years of his life?
A. Write short stories B. Popularize African-American writers
C. Advocate racial equality D. Write about life in Harlem
8. Which of the following could best replace the word "accomplished" as used in the passage?
A Successful B. Prolific C. Brilliant D. Imaginative
9. Which of the following can best substitute for the word "depicting" in the passage?
A. Congratulating B. Blessing C. Screening D. Portraying
10. According to the passage, Langston Hughes was all of the following EXCEPT:
A. A novelist B. A poet C. A historian D. A journalism

F. Read the passage and then decide which word (A, B, C, or D) best fits each space. (10 points)
My Dutch holiday
It was the first time I had ever been to Holland before and when I set off to the ferry in early
September, I never (1) ….. what adventures lay ahead of me.
In order to make the most of my time, I had carefully planned a route to (2) ….. a number of small
islands as well as some pretty countryside. I reached Wijkam See, a (3) ….. starting point for cyclists
where a local tourist office gave out information. I headed off along one of Holland’s many cycle tracks
which (4) ….. out of town and made my way to Holtheim, where I had planned to spend my first night.
From personal (5) ….., when you are travelling on your own unexpected and often pleasant surprises
tend to crop up and this trip was no exception. I popped into a local bar in the outskirts of Amsterdam
for a coffee and ended up chatting with a really friendly man who turned (6)….. to be the owner of the
only restaurant in the village. He insisted on (7)….. me for lunch. Not only that, but he (8)….. two
enormous glasses of beer to celebrate!
After a delicious lunch, Johann (9) ….. me luck on the rest of my cycling trip and gave me the address
of a friend in Amsterdam who would (10) ….. me up for the night. Not a bad start to my first day in the
Netherlands.
1 A wondered B doubted C imagined D realised
2 A consist B include C contain D conclude
3. A appropriate B adequate C convenient D capable
4. A lead B reach C ride D join
5. A knowledge B belief C viewpoint D experience
6 A into B in C up D out
7 A arrange B inviting C paying D give
8. A asked B ordered C demanded D fixed
9. A hoped B sent C wished D expected
10. A put B pass C stay D keep

D. WRITING.
I. Finish each of the following sentences in such a way that it means exactly the same as the sentence
printed before it. (10 points)
1. Tear gas was released as soon as the thieves touched the safe.
-> No sooner ………………………………………………………………………………………
2. Although the team played well, they lost.
-> Despite …………………………………………………………………………………………
3. She didn’t say a word as she left the room.
-> She left the room ……………………………………………………………………………….
4. If I were you, I’d look for another job.
-> I suggest ……………………………………………………………………………………….
5. He didn’t talk to me until I gave my official apology.
I. It was not ………………………………………………….
6. He would prefer me to concentrate more on my study.
J. He’d rather …………………………………………………
7. I’m sure he took the money on purpose.
K. He couldn’t possibly ……………………………………....
8. He overslept so he came late for the meeting.
L. If he …………………………………………………………
9. “I am happy you have passed the final exams. Congratulations!” Tom said to me.
 Tom congratulated ……………………………………………………………………………….
10. This matter is so complicated that we don't know how to deal with it.
→ So ...............................................................................................................................

JJ.Finish each of the following sentences in such a way that it is as similar as possible in meaning
to the sentence printed before it. Do not change the form of the given word. (5 pts)
1. They paid no attention to the man’s sudden disappearance. ( NOTICE)
…………………………………………………………………………………………….
2. The manager should think about experience when hiring new staff. (CONSIDERATION)
……………………………………………………………………………………………….
3. I tried to talk to Jack about the problem but he was too busy. (WORD)
……………………………………………………………………………………………….
4. Fred tried hard to start the car, but without success. (MATTER)
………………………………………………………………………………………………..
5. He likes to be addressed as “Professor”. (CALL)
…………………………………………………………………………………………………

84. Living in a traditional family of three or four generations has both advantages and disadvantages.
What do you think of that matter? Write an essay of about 180- 220 words to support your points ( 15
points)
________ The end ________

( Thí sinh không được sử dụng tài liệu. Cán bộ coi thi không giải thích gì thêm.)
ĐÁP ÁN ĐỀ THI HỌC SINH GIỎI CẤP TRƯỜNG LẦN 1
Môn : Tiếng Anh 12
A: PHONETICS. (5pts)
1.D 2.D 3.C 4.C 5.A

B: GRAMMAR – VOCABULARY.
I. (15pts).
1. D 2. A 3. B 4. D 5. B
6. A 7. B 8. C 9. B 10. B
11. D 12. A 13. A 14. B 15. D

II. (10pts).
31. value → valuable 36. terror → terrified
32. fright → frightened 37. low → lowered
33. long → length 38. repeat → repeatedly
34. world – worldwide 39. embarrass→ embarrassment
35. tight → tightened 40. high → heights
III. ( 5pts)
JJ. to leave => leave LL. When => while/whereas
KK. That => what MM. putting => put
NN. the past week => the previous week

IV. (5pts)
1. were flying 4. ( should) be cancelled
2. will have been 5. made
3. had I arrived

C. READING.
I. (10pts)
1. for 2. were 3. wind 4. to 5. down
6. voice 7. the 8. They 9. question 10. hard/heavily

II. (10pts).
1. A 2. C 3. D 4. C 5. C
6. B 7. B 8. A 9. D 10. C
III. (10P)
1. C 2. B 3. C 4. A 5. D
6. D 7. B 8. B 9. C 10. A

D. WRITING.
I. 10 (pts).
1. No sooner had the thieves touched the safe than tear gas was released.
2. Despite their good game/ the fact they played well, the team lost.
3. She left the room without saying a word.
4. I suggest (that) you look for another job.
5. It was not until I gave my official apology that he talked to me.
6. He’d rather I concentrated more on my study.
7. He couldn’t possibly have taken the money by accident/chance/ accidentally.
8. If he hadn’t overslept, he wouldn’t have come late for the meeting.
9. “I am happy you have passed the final exam. Congratulations!” Tom said to me.
 Tom congratulated me on having passed the final exam.
10. So complicated is this matter that we don't know how to deal with it.

II. (5pts).
I. They took no/ didn’t take notice of the man’s sudden disappearance.
II.The manager should take experience into consideration when hiring new staff.
III. I tried to have a word with Jack about the problem but he was too busy.
IV. No matter how hard Fred tried, he couldn’t start the car.
V. He wants/likes you/ people / students to call him “professor”

III. (15pts).
1. Content:
- 50% of total mark: a provision of all main ideas and details as appropriate
2. Language:
- 30% of total mark: a variety of vocabulary and structures appropriate to the level of English
language gifted upper-secondary school students
3. Presentation:
- 20% of total mark: coherence, cohesion, and style appropriate to the level of English language
gifted upper-secondary school students.

________ The end ________


SỞ GD&ĐT VĨNH PHÚC ĐỀ THI THỬ HSG LỚP 12 CẤP TRƯỜNG LẦN 2
TRƯỜNG THPT ĐỒNG ĐẬU NĂM HỌC 2019-2020
MÔN: Tiếng Anh
Thời gian làm bài: 180 phút, không kể thời gian giao đề

( Học sinh không được sử dụng tài liệu, cán bộ coi thi không giải thích gì thêm)
( Học sinh làm bài trực tiếp trên tờ đề thi, không cần làm ra tờ giấy thi)

Full name:…………………………………………………..Class…………….

A. PHONETICS.
Pick out the word whose stress pattern is different from that of the other words. Identify your
answer by circling the corresponding letter A, B, C or D. (5pts)
1. A. introduce B. volunteer C. understand D. mechanize
2. A. magnifier B. majority C. machine D. malignant
3. A. permanent B. management C. conference D. abundant
4. A. celebrity B. charitable C. anonymous D. respectable
5. A. populate B. industrialise C. environment D. inhabitants

B. LEXICO – AND GRAMMAR.


Part 1. Complete the following sentences by choosing the correct answer A, B, C or D.(15pts)
1. It is possible ______ may assist some trees in saving water in the winter.
A. to lose leaves B. when leaves have lost
C. that the loss of leaves D. the leaves are lost
2. My father gave me ______ as a birthday present on my birthday.
A. a digital useful alarm clock B. an alarm useful digital clock
C. a useful alarm digital clock D. a useful digital alarm clock
3. She .......... modern art. She visits all the local exhibitions.
A. looks down on B. goes in for C. fixes up with D. comes up against
4. It is often a good idea to start with small, easily ________ goals.
A. achievable B. achieve C. achievement D. achiever
5. Fast food is very popular. .............., a diet of burgers, pizzas and fried chicken is not very healthy.
A. Consequently B. Moreover C. Unfortunately D. In contrast
6. Julie was upset because her job application was ..............twice.
A. turned off B. turned down C. turned out D. turned over
7. We’ve bought some …………. chairs for the garden so that they are easy to bring out.
A. adapting B. adjusting C. bending D. folding
8. He talked as if he …………. where she was.
A. knew B. would know C. had known D. were knowing
9. Tom: “ I’m sorry. I won’t be able to come”.
Mary: “…………”
A. Great B. Oh, that’s annoying C. Well, never mind D. Sounds like fun
10. By the end of next month Annabel and I…………. out with each other for three years.
A. will have been going B. will go C. have been going D. will be going
11. I really can’t ______ the twins apart. They look so alike.
A. say B. talk C. take D. tell
12. The thief was _____ to six months’ imprisonment.
A. given B. allowed C. sent D. sentenced
13. Never before _________ such a wonderful child.
A. I saw B. have I seen C. I had seen D. I have seen
14. It is a ……………...
A. blue polyester sleeping bag B. polyester sleeping blue bag
C. blue sleeping polyester bag D. sleeping blue polyester bag
15. Max is not a(n) _________ drinker but he likes a glass of wine occasionally.
A. addict B. heavy C. drunk D. obsessed
Part 2 . Put each verb in brackets into an appropriate form. ( 5 points)
1. It is essential that Jane (inform) ________________ the plan to everyone in advance.
2. The cake (make) _______________ by my mom tastes really delicious.
3. I don’t like that guy. He says as if he (be) ____________ my father.
4. By the time we got home, the children (go) ____________ to bed.
5. The old lady always ( complain) ______________ about anything she doesn’t like.

Part 3: The passage below contains 10 mistakes. Underline the mistakes and correct them. (0. from
– of ) has been done as an example (10 points)

Most people are afraid from sharks, but they usually do not know ……of …...
very much about them. For example, there are about 350 kinds of sharks, ……………...…
and all of them are eaters meat. Some sharks are very big. The whale shark …………..…….
is 50 and 60 feet long. But some sharks are very small. The dwarf shark is …………..…….
only 6 inches long. Sharks are 100 million year old. In fact, they lived at ………..……….
the same time with dinosaurs. Today, sharks live in every ocean in the …………..…….
world, but most sharks live in warm water. They keep the oceans cleaning ………..……….
because they eat sick fish and animals. Most sharks have four to six lines of …………..…….
teeth. When a shark’s tooth falls out another tooth moves in from before. ………..……….
Sharks do not have ears. However, they “hear” sounds and movements in ………..……….
the water. Any sound or movement causes the water vibrate. Sharks can ………..……….
feel these vibrations, and they help the sharks find food. Sharks use their ……..………….
large eyes to find food, too. Most sharks see wellest in low light. They ……..………….
often hunt for food at dawn, in the evening, or in the middle of the night. …………..…….
Scientists want to learn more about sharks for several reasons. For ………………...
example, cancer is common in many animals, including people. However, …………………
it is rare in sharks. Scientists want to find out what sharks almost never get …………………
cancer. Maybe this information can help people prevent cancer too. …………………

Part 4. Give the correct form of the word in blanket. (10pts)


MOVING HOUSE
Moving house is said to be the third most stressful experience you can have (coming after the
(1. DIE) _________________ of a close relative, or a divorce). The reason for this is partly the (2-
ORGANISE) ________________________ involved, but also the feeling of (3. SECURITY)
__________________ caused by completely changing your environment. Of course, a (4. SYSTEM)
__________________ approach can help ease the difficulties, especially on the day of (5. REMOVE)
________________. Plan your packing carefully or, better, employ a (6. RELY) ________________
company to pack and move your things. This will (7. CERTAIN) __________________ lessen the
amount of damage to your (8. POSSESS) ___________________. It’s also a good idea to take out (9.
INSURE) _________________. Some worry is, of course, (10. AVOID) ___________________ but try
to keep calm and look forward to life in your new home.

C. READING.
Part 1. Read the passage and fill in each numbered gap with ONE suitable word.( 10 Points)
DREAMS
Dreams have always fascinated human beings. The idea that dreams provide us with useful
information about our lives goes (1)……………. thousands of years. For the greater part of human history
it was taken for granted that the sleeping mind was in touch with the supernatural world and dreams were
to be interpreted as messages with prophetic or healing functions. In the nineteenth century,
(2)………..…. was a widespread reaction (3)……………..…. this way of thinking and dreams were
widely dismissed as being very little more than jumbles of fantasy (4)……………... about by memories of
the previous day.
It was not (5)…………..…. the end of the nineteenth century that an Austrian neurologist,
Sigmund Freud, pointed out that people who have similar experiences during the day, and who are
then subjected (6) ……………..….the same stimuli when they are asleep, produce different dreams.
Freud (7)……………..… on to develop a theory of the dream process which could enable him to interpret
dreams as clues to the conflicts taking place within the personality. It is by no (8)…………….… an
exaggeration to say that (9)………………… any other theories have had (10)………………….… great
an influence on subsequent thought.

Part 2. Choose the best answer from the four options marked A, B, C or D to complete each numbered
gap in the passage below. ( 10 pts)
School exams are, generally speaking, the first kind of tests we take. They find out how much
knowledge we have gained. But do they really show how intelligent we are? After all, isn’t it a (1)______
that some people who are very academically successful don’t have any common sense?
Intelligence is the speed (2)______ which we can understand and react to new situation and it is
usually tested by logic puzzles. Although scientists are now preparing (3) ______ computer technology
that will be able to ‘read’ our brains, (4) ______ tests are still the most popular ways of measuring
intelligence.
A person’s IQ is their intelligence (5) ______ it is measured by a special test. The most common
IQ tests are (6) ______ by Mensa, an organization that was founded in England in 1946. By 1976, it (7)
______ 1,300 members in Britain. Today there are 44,000 in Britain and 100,000 worldwide, largely in
the US.
People taking the tests are judged in (8) ______ to an average score of 100, and those who score
over 148 are entitled to join Mensa. This works out at percent of the population. Anyone from the age of
six can take the tests. All the questions are straightforward and most people can answer them if (9)
______ enough time. But that’s the problem, the whole (10) ______ of the tests is that they’re against the
clock.

1. A. case B. fact C. circumstance D. truth


2. A. on B. to C. in D. at
3. A. advanced B. forward C. ahead D. upper
4. A. at this age B. for the present C. at the time D. now and then
5. A. how B. that C. as D. so
6. A. appointed B. commanded C. run D. steered
7. A. held B. had C. kept D. belonged
8. A. concerned B. relation C. regard D. association
9. A. allowed B. spared C. let D. provided
10. A. reason B. point C. matter D. question

Part 3 . Read the following passage and mark the letter A, B, C or D on your answer sheet to
indicate the correct answer to each of the following questions.(10pt)
The National Automobile Show in New York has been one of the top auto shows in the United
States since 1900. On November 3 of that year, about 8,000 people looked over the “horseless
carriages." It was the opening day and the first opportunity for the automobile industry to show off
its wares to a large crowd; however, the black-tie audience treated the occasion more as a social affair
than as a sales extravaganza. It was also on the first day of this show that William McKinley became
the first U.S. President to ride in a car.
The automobile was not invented in the United States. That distinction belongs to Germany.
Nicolaus Otto built the first practical internal-combustion engine there in 1876. Then, German
engineer Karl Benz built what are regarded as the first modern automobiles in the mid-1880s. But
the United States pioneered the merchandising of the automobile. The auto show proved to be an effective
means of getting the public excited about automotive products.
By happenstance, the number of people at the first New York show equaled the entire car
population of the United States at that time. In 1900, 10 million bicycles and an unknown number of
horse-drawn carriages provided the prime means of personal transportation. Only about 4,000 cars
were assembled in the United States in 1900, and only a quarter of those were gasoline powered. The
rest ran on steam or electricity.
After viewing the cars made by forty car makers, the show’s audience favored electric cars
because they were quiet. The risk of a boiler explosion turned people away from steamers, and the
gasoline-powered cars produced smelly fumes. The Duryea Motor Wagon Company, which launched
the American auto industry in 1895, offered a fragrant additive designed to mask the smells of the
naphtha that it burned. Many of the 1900 models were cumbersome—the Gas mobile, the Franklin,
and the Orient, for example, steered with a tiller like a boat instead of with a steering wheel. None of
them was equipped with an automatic starter.
These early model cars were practically handmade and were not very dependable. They were
basically toys of the well-to-do. In fact, Woodrow Wilson, then a professor at Princeton University
and later President of the United States, predicted that automobiles would cause conflict between
the wealthy and the poor. However, among the exhibitors at the 1900 show was a young engineer
named Henry Ford. But before the end of the decade, he would revolutionize the automobile industry
with his Model T Ford. The Model T, first produced in 1909, featured a standardized design and a
streamlined method of production — the assembly line. Its lower costs made it available to the mass
market.
Cars at the 1900 show ranged in price from $1,000 to $1,500, or roughly $14,000 to $21,000 in
today’s prices. By 1913, the Model T was selling for less than $300, and soon the price would drop
even further. “I will build cars for the multitudes,” Ford said, and he kept his promise.
1. The passage implies that the audience viewed the 1900 National Automobile Show
primarily as a(n)___________
A. chance to buy automobiles at low prices. B. formal social occasion.
C. chance to invest in one of thirty-two automobile manufacturers.
D. opportunity to learn how to drive.
2. According to the passage, who developed the first modern car?
A. William McKinley B. Nicolaus Otto C. Henry Ford D. Karl Benz
3. Approximately how many cars were there in the United States in 1900?
A. 4,000 B. 8,000 C. An unknown number D. 10 million
4. The phrase “By happenstance” in paragraph 3 is closest in meaning to________.
A. By design B. Coincidentally C. For example D. Generally
5. Approximately how many of the cars assembled in the year 1900 were gasoline powered?
A. 1,000 B. 2,000 C. 4,000 D. 32
6. Which of the following is closest in meaning to the word “launched” in paragraph 4 __________.
A. designed B. initiated C. joined D. anticipated
7. The purpose of the “additive” mentioned in paragraph 4 was to___________
A. hide strong smells. B. make engines run more efficiently.
C. increase the speed of cars. D. make cars look better.
8. Which of the following is NOT mentioned in the passage as steering with a tiller rather than with
a steering wheel?
A. A Gas mobile B. A Duryea C. A Franklin D. An Orient
9. It is clear from the passage that the early cars___________
A. were more formal. B. involved less expensive cars.
C. involved fewer manufacturers. D. were more spectacular.
10. What was the highest price asked for a car at the 1900 National Automobile Show in the
dollars of that time?
A. $300 B. $21,000 C. $14,000 D. $1,500
D. WRITING.
Part 1. Rewrite each of the following sentences in such a way that it has a similar meaning to the
original one.(10pts)
1. I only recognised him when he came into the light.
 Not until ………………………………………………....…
2. His love increases with his admiration for her.
 The more …………………………………………….…..…
3. “I’m sorry I didn’t ring you to say I’d be late.”
 He apologised ……………………………………………….
4. You are in this mess right now because you didn’t listen to me in the first place.
 If you had ……………………………………………..…..…
5. Press photographers are banned from taking photographs backstage.
 On no ………………………………………………….…..…
6. I don’t intend to apologize to either of them.
 I have ………………………………………………………………….

7. The result of the match was never in doubt.


 At no time ………………………………………………………………
8. Under no circumstances should you phone the police.
 The last ……………………………………………………………………….
9. I didn’t know you were coming, so I didn’t wait for you.
 If……………………………………………………………………….
10. When I picked up my pen I found that the nib had broken
 On………………………………………………………………………
Part 2. Write an essay within 250 words on the following topic: (15pts)

Your first impressions when you meet someone are always right.?
Do you agree or disagree with this statement?
Give reasons for your answer and include any relevant examples from your own knowledge or experience.
………………………………………………………………………………………………………………
………………………………………………………………………………………………………………
………………………………………………………………………………………………………………
………………………………………………………………………………………………………………
………………………………………………………………………………………………………………
………………………………………………………………………………………………………………
………………………………………………………………………………………………………………
………………………………………………………………………………………………………………
………………………………………………………………………………………………………………
………………………………………………………………………………………………………………
………………………………………………………………………………………………………………
………………………………………………………………………………………………………………
………………………………………………………………………………………………………………
………………………………………………………………………………………………………………
………………………………………………………………………………………………………………
………………………………………………………………………………………………………………
………………………………………………………………………………………………………………
………………………………………………………………………………………………………………
………………………………………………………………………………………………………………
………………………………………………………………………………………………………………
………………………………………………………………………………………………………………
………………………………………………………………………………………………………………
………………………………………………………………………………………………………………
………………………………………………………………………………………………………………
………………………………………………………………………………………………………………
…………………………………………………………………………………………………….……….
………………………………………………………………………………………………………………
………………………………………………………………………………………………………………
………………………………………………………………………………………………………………
………………………………………………………………………………………………………………
………………………………………………………………………………………………………………
………………………………………………………………………………………………………………
………………………………………………………………………………………………………………
………………………………………………………………………………………………………………
………………………………………………………………………………………………………………
………………………………………………………………………………………………………………
………………………………………………………………………………………………………………
………………………………………………………………………………………………………………
………………………………………………………………………………………………………………
…………………………………………………………………………………………………….……….
………………………………………………………………………………………………………………
………………………………………………………………………………………………………………
………………………………………………………………………………………………………………
………………………………………………………………………………………………………………

The end.
ĐÁP ÁN ĐỀ THI HSG LỚP 12 CẤP TRƯỜNG LẦN 2

A. PHONETICS .
Pick out the word whose stress pattern is different from that of the other words. Identify your
answer by circling the corresponding letter A, B, C or D. (5 pts: 1p/each correct answer)

1 D. 2 A. 3. D. 4. B. 5. A.

B. LEXICO- AND GRAMMAR.

Part 1. Complete the following sentences by choosing the correct answer A, B, C or D. (1p/each
correct answer)

1 C. 2 D. 3 B. 4A. 5 C. 6 B. 7 D. 8 C. 9 C. 10 A.
11. D 12. D 13. B 14. A 15. B

Part 2. . Put each verb in brackets into an appropriate form. ( 5 points)

1. (should) inform 2. made 3. were 4. had gone 5. is …always... complaining


Part 3: The passage below contains 10 mistakes. Pick them out and correct them. (0. from – of ) has
been done as an example (1p/each correct answer)

1. eaters meat - meat eaters 6. lines - rows


2. and – to 7. before - behind
3. year – years 8. causes – makes
4. with – as 9. wellest - best
5. cleaning – clean 10. what - why

Part 4. Give the correct form of the word in blanket. (1p/each correct answer)

2. organisation/
1. death organization 3. insecurity 4. systematic 5. removal

6. reliable 7. certainly 8. possessions 9. insurance 10. unavoidable

C. READING.
Part 1. Read the passage and fill in each numbered gap with ONE suitable word.( 10 Points)

1. back 2. there 3. against 4. brought 5. until/ till


6. to 7. went 8. means 9. hardly/ scarcely 10. so/ as/ that

Part 2. Choose the best answer from the four options marked A, B, C or D to complete each numbered
gap in the passage below ( 10 pts)

1. B 2. D 3. A 4. B 5. C
6. C 7. B 8. B 9. A 10. B

Part 3. Read the following passage and mark the letter A, B, C or D on your answer sheet to
indicate the correct answer to each of the following questions.(10pt)
1. B 2. D 3. B 4. B 5. A
6. B 7. A 8. B 9. D 10. D
D. WRITING.
Part 1. (10 pts: 1p/each correct answer)
1. Not until …… he came into the light did I recognise him.
2. The more …… he admires her, the more he loves her.
3. He apologised ……for not ringing/ having rung to say he’d be late.
4. If you had …. listened to my advice in the first place, you wouldn’t be in this mess right now.
5. On no........ account are press photographers allowed to take photographs bachstages
6. I have… no intention of apologising to either of them/
I have no intention to apologise to either of them.
7. At no… time was the result of the match in doubt.
8. The last …thing you should/ ought to/ must do is (to) phone the police.
9. If …I had known you were coming, I would have waited for you.

10. On ….picking up my pen, I found that the nib had broken.

Part 2. Essay Writing (15 points)

Correct form of - Contents: a provision of all main reasons and appropriate supporting ideas and
essay writing relevant examples (8pts)
(15 points) - Language: a variety of vocabulary and appropriate structures (4pts)
- Presentation: coherence, cohesion, and appropriate style (3pts)

Incorrect form of - Contents: a provision of all main reasons and appropriate supporting ideas and
essay writing relevant examples (4pts)
(7 points) - Language: a variety of vocabulary and appropriate structures (2pts)
- Presentation: coherence, cohesion, and appropriate style (1p)

- The end-

SỞ GIÁO DỤC VÀ ĐÀO TẠO KỲ THI HỌC SINH GIỎI LỚP 12 THPT CẤP TỈNH
QUẢNG NAM Năm học: 2019 – 2020
Môn thi : TIẾNG ANH
ĐỀ CHÍNH THỨC Thời gian : 90 phút (Không kể thời gian giao đề)
(Đề thi gồm có 12 trang) Ngày thi : 10/6/2020
Mã đề thi: 132

(Thí sinh làm bài trên Phiếu trả lời trắc nghiệm)
Họ và tên thí sinh:……………………………………….Phòng thi ………Số báo danh: ……

SECTION I: LISTENING
HƯỚNG DẪN PHẦN THI NGHE HIỂU
 Mỗi phần được nghe 2 lần, mở đầu và kết thúc mỗi phần có tín hiệu.
 Mọi hướng dẫn cho thí sinh (bằng tiếng Anh) đã có trong bài nghe.
Part 1: You will hear people talking in eight different situations. For questions 1- 8, mark the
letter A, B, or C on your answer sheet to indicate the correct answer to each of the following
questions.
Question 1: You hear a mother talking about her baby. What has the baby just learnt to do?
A. count B. walk C. talk
Question 2: You hear a man buying some flowers. What colour does he buy?
A. red B. yellow C. white
Question 3: You hear a girl talking to her friend on the phone. What has happened?
A. She didn't remember to do her homework.
B. She left her homework at home.
C. She got bad marks for her homework.
Question 4: You hear a man talking to a vet. What is the problem with his dog?
A. She has become very aggressive.
B. She keeps biting her paw.
C. She doesn't walk properly.
Question 5: You hear a woman talking to a plumber. Where is the problem in her house?
A. the veranda B. the kitchen C. the bathroom
Question 6: You hear a man talking about a train journey. When did he travel?
A. Tuesday B. Wednesday C. Thursday
Question 7: You hear a boy telling his mother about a football match. How many goals did
his team score?
A. two B. three C. one
Question 8: You hear a woman talking about going shopping. What did she buy?
A. a pair of shoes B. a pair of jeans C. a pair of glasses

Part 2: You will hear an interview with someone who has started a news service called
“Children’s Express”. For questions 9-15 choose the best answer: A, B or C. You will hear the
recording twice.
Question 9: The purpose of Children’s Express is to encourage children to:
A. Think in a more adult way.
B. Consider important matters.
C. Train as journalists.
Question 10: Bob says that the children who work on Children’s Express:
A. Are carefully chosen.
B. Learn from each other.
C. Get on well together.
Question 11: What success has Children’s Express had?
A. TV programmes have been made about it.
B. Adults read some of the articles it produces.
C. It has affected the opinions of some adults.
Question 12: What did the survey in the Indianapolis Star show about the page they write?
A. It is read by a lot of adults.
B. It is the most popular page in the newspaper.
C. It interests adults more than children.
Question 13: Important public figures agree to be interviewed by the children because:
A. Children’s Express has a good reputation.
B. They like the questions children ask.
C. They want children to like them.
Question 14: When an article is being prepared, the editors:
A. Help the reporters in the interviews.
B. Change what the reporters have written.
C. Talk to the reporters about the interview.
Question 15: What is unique about their type of journalism?
A. Nothing in their articles is invented.
B. Everything that is recorded appears in the articles.
C. It is particularly suitable for children.
SECTION II: LEXICO-GRAMMAR
Part 1: Mark the letter A, B, C, or D on your answer sheet to indicate the correct answer to
each of the following questions from 16 to 32.
Question 16: TikTok, also known as Douyin in China, is a social media app downloaded more than 2 billion times
globally with a (an) ______ to creating and sharing videos.
A. aim B. view C. purpose D. plan
Question 17: ______ the invention of the steam engine, most forms of transport were horse-
drawn.
A. With reference B. Akin C. Prior to D. In addition to
Question 18: ______ before, his first performance for the amateur dramatic group was a success.
A. Though having never acted B. Despite he had never acted
C. As he had never acted D. In spite of his never having acted
Question 19: Come by to my place so that we can have a chat about that, ______?
A. can’t we B. aren’t you C. shall we D. will you
Question 20: The coronavirus COVID-19 is affecting 213 countries and territories around the
world and 2 international ______.
A. conveyors B. conveyancers C. conveyances D. coveyancing
Question 21: I am afraid a rise in salary is ______ just now.
A. out of sight B. out of control C. out of date D. out of question
Question 22: She is ______ her sister is.
A. nothing near as ambitious B. nowhere like so ambitious
C. nothing as ambitious than D. nowhere near as ambitious as
Question 23: The matter comes when a manager strays outside his area of _____.
A. knack B. skill C. expertise D. speciality
Question 24: It is estimated that she earned ______ her brother last year.
A. twice as much as B. twice more than
C. twice as many as D. twice as more as
Question 25: The man ______ of the murder hasn’t been brought to trial yet.
A. suspect B. having suspected
C. suspecting D. suspected
Question 26: I don’t think you have been watering the plants near the gate. The soil is ______.
A. as dry as rice B. as dry as a tile
C. as dry as a bone D. as dry as wood
Question 27: Because my little son couldn’t have his dream bike on his birthday, he was ______
disappointed.
A. bitterly B. desperately C. strictly D. severely
Question 28: ______ he was kidnapped by the Iraqi Guerrillas yesterday has been confirmed.
A. What B. If C. That D. Ø
Question 29: I utterly _______ your argument. In my opinion, you have distorted the facts.
A. confound B. refute C. dispute D. declined
Question 30: Turn off this machine, please. The harsh sound really ______ me crazy.
A. takes B. worries C. drives D. bothers
Question 31: People in financial difficulties sometimes fall _______to unscrupulous money
lenders.
A. prey B. fool C. scapegoat D. sacrifice
Question 32: The police ______ a good deal of criticism over their handling of the
demonstration.
A. came in for B. brought about C. back out D. back up

Part 2: Mark the letter A, B, C, or D on your answer sheet to indicate the word(s) CLOSEST in
meaning to the underlined word(s) in each of the following questions from 33 to 34.

Question 33: Important features of dehydrated foods are their lightness in weight and their
compactness.
A. organic B. dried C. frozen D. healthy
Question 34: We went away on holiday last week, but in rained day in day out.
A. every other day B. every single day
C. every second day D. every two days

Part 3: Mark the letter A, B, C, or D on your answer sheet to indicate the underlined part that
needs correction in each of the following questions from 35 to 36.

Question 35: Digital clocks, however precise, they cannot be perfectly accurate
A B

because the earth’s rotation changes slightly over years.


C D
Question 36: Genetic engineering is helping researchers unravel the mysteries of
A
previously incurable diseases so that they get to its root causes and find cures.
B C D
Part 4: Mark the letter A, B, C, or D on your answer sheet to indicate the option that best
completes each of the following exchanges from 37 to 38.
Question 37: Jack failed the driving test again.
Jack: “Failed again, I’m afraid.”
You: “______”
A. Well, good luck. B. Oh, hard luck. C. Lucky you. D. I’m not sure.
Question 38: Two friends, Peter and Linda, are talking about pets.
Peter: “Well, cats are very good at catching mice around the house”
Linda: “_______”
A. Nothing more to say. B. You can say that again.
C. Yes, I hope so. D. No, dogs are good, too.

Part 5: Mark the letter A, B, C, or D on your answer sheet to indicate the word(s) OPPOSITE
in meaning to the underlined word(s) in each of the following questions from 39 to 40.
Question 39: Hazardous waste is waste that has substantial or potential threats to public health
or the environment.
A. vivid B. risky C. dangerous D. secure
Question 40: “What I’ve got to say to you now is strictly off the record and most certainly not
for publication”, said the government officer to the reporter.
A. already official B. beside the point C. not popular D. not recorded

SECTION III: READING


Part 1: Read the following passage and mark the letter A, B, C, or D on your answer sheet to
indicate the correct word or phrase that best fits each of the numbered blanks from 41 to 48.
ANIMAL WATCH: VOICE FROM THE WILD
Are you in favor of taking part in the battle to save the world's wildlife? Animal Watch is
the book which will (41) ______ you in the fight for survival that faces many of our endangered
animals and show how they struggle on the edge of extinction. As you enjoy the book's 250 pages
and over 150 colour photographs, you will have the (42) ______ of knowing that part of your
purchase money is being used to help animals (43) ______. From the comfort of your armchair,
you will be able to observe the world's animals close-up and explore their habitats. You will also
discover the terrible results of human (44) ______ for land, flesh and skins.
Animal Watch is packed with fascinating facts. Did you know that polar bears cover their
black noses with their (45) ______ so they can hunt their prey in the snow without being seen, for
example? Or that for each orangutan which is captured, one has to die?
This superb (46) ______ has so impressed Britain's leading wildlife charity that it has been
chosen as Book of the Year, a title awarded to books which are considered to have made a major
contribution to wildlife conservation. You will find Animal Watch at a special low (47) ______
price at all good bookshops, but hurry while (48) ______ last.
Question 41. A. combine B. involve C. bring D. lead
Question 42. A. satisfaction B. enjoyment C. virtue D. value
Question 43. A. preserve B. conserve C. revive D. survive
Question 44. A. greed B. interest C. care D. concern
Question 45. A. feet B. claws C. paws D. toes
Question 46. A. publicity B. periodical C. publication D. reference
Question 47. A. beginning B. preparatory C. original D. introductory
Question 48. A. stores B. stocks C. goods D. funds

Part 2: Read the following passage and mark the letter A, B, C, or D on your answer sheet to
indicate the correct answer to each of the questions from 49 to 56.
The oceans are so vast and deep that until fairly recently, it was widely assumed that no
matter how much trash and chemicals humans dumped into them, the effects would be negligible.
Proponents of dumping in the oceans even had a catchphrase: "The solution to pollution is
dilution”.
Today, we need look no further than the New Jersey-size dead zone that forms each
summer in the Mississippi River Delta, or the thousand-mile-wide swath of decomposing plastic
in the northern Pacific Ocean to see that this "dilution" policy has helped place a once flourishing
ocean ecosystem on the brink of collapse.
There is evidence that the oceans have suffered at the hands of mankind for millennia. But
recent studies show that degradation, particularly of shoreline areas, has accelerated dramatically
in the past three centuries as industrial discharge and run-off from farms and coastal cities have
increased.
Pollution is the introduction of harmful contaminants that are outside the norm for a given
ecosystem. Common man-made pollutants reaching the oceans include pesticides, herbicides,
chemical fertilizers, detergents, oil, sewage, plastics, and other solids. Many of these pollutants
collect at the ocean's depths, where they are consumed by small marine organisms and introduced
into the global food chain.
Many ocean pollutants are released into the environment far upstream from coastlines.
Nitrogen-rich fertilizers applied by farmers inland, for example, end up in local streams, rivers,
and groundwater and are eventually deposited in estuaries, bays, and deltas. These excess
nutrients can spawn massive blooms of algae that rob the water of oxygen, leaving areas where
little or no marine life can exist.
Solid wastes like bags, foam, and other items dumped into the oceans from land or by ships
at sea are frequently consumed, with often fatal effects, by marine mammals, fish, and birds that
mistake them for food. Discarded fishing nets drift for many years, ensnaring fish and mammals.
In certain regions, ocean currents corral trillions of decomposing plastic items and other trash into
gigantic, swirling garbage patches. One in the North Pacific, known as the Pacific Trash Vortex,
is estimated to be the size of Texas.
Pollution is not always physical. In large bodies of water, sound waves can carry
undiminished for miles. The increased presence of loud or persistent sounds from ships, sonar
devices, oil rigs, and even from natural sources like earthquakes can disrupt the migration,
communication, and reproduction patterns of many marine animals, particularly aquatic mammals
like whales and dolphins.
(Source: http://www.oceannationalgeographic.com)
Question 49. What does the passage mainly discuss?
A. Marine pollution and its many forms.
B. Noise and its disruptive effects on marine life.
C. Various kinds of harmful pollutants.
D. The end of the "dilution" era.
Question 50. The word "negligible" in paragraph 1 is closest in meaning to ____.
A. serious B. insignificant C. unpredictable D. positive
Question 51. It can be inferred from paragraph 2 that the "dilution" policy is related to _______.
A. dealing with the problems of water pollution
B. helping the ecosystem of the oceans flourish
C. neglecting the effects of dumping trash into the oceans
D. treating harmful materials in the oceans properly
Question 52. The word "they" in paragraph 4 refers to _______.
A. ocean's depths B. man-made pollutants
C. marine organisms D. the oceans
Question 53. The word "spawn" in paragraph 5 can be best replaced by _______.
A. appear B. prevent C. produce D. limit
Question 54. According to the passage, nitrogen-rich fertilizers _______.
A. are created by massive blooms of algae
B. cause a shortage of oxygen in the ocean water
C. do not relate to the disappearance of marine life
D. cannot be found inland
Question 55. Which of the following statements is NOT supported in the passage?
A. Many pollutants deposited in the oceans finally become part of the global food chain.
B. It is apparent that the oceans have been polluted for a long time.
C. Industrial wastes and agricultural run-off are blamed for the degradation of the oceans.
D. The oceans in the past were more contaminated than they are now.
Question 56. Whales and dolphins are mentioned in the final paragraph as an example of
marine creatures that _______.
A. can communicate with each other via sound waves
B. can survive earthquakes because of their large bodies
C. suffer from loud or persistent sounds at sea
D. are forced to migrate because of water pollution
Part 3: Read the following passage and mark the letter A, B, C, or D on your answer sheet to
indicate the correct answer to each of the questions from 57 to 64.
Salinity is the saltiness or amount of salt dissolved in a body of water. If the salinity of
ocean waters is analyzed, it is found to vary only slightly from place to place. Nevertheless, some
of these small changes are important. There are three basic processes that cause a change in
oceanic salinity. One of these is the subtraction of water from the ocean by means of evaporation-
conversion of liquid water to water vapor. In this manner, the salinity is increased, since the salts
stay behind. If this is carried to the extreme, of course, white crystals of salt would be left behind.
This, by the way, is how much of the table salt we use is actually obtained.
The opposite of evaporation is precipitation, such as rain, by which water is added to the
ocean. Here the ocean is being diluted so that the salinity is decreased. This may occur in areas of
high rainfall or in coastal regions where rivers flow into the ocean. Thus, salinity may be
increased by the subtraction of water by evaporation, or decreased by the addition of fresh water
by precipitation or run off.
Normally in tropical regions where the Sun is very strong, the ocean salinity is somewhat
higher than it is in other parts of the world where there is not as much evaporation. Similarly, in
coastal regions where rivers dilute the sea salinity is somewhat lower than in other oceanic areas.
A third process by which salinity may be altered is associated with the formation and
melting of sea ice. When seawater is frozen, the dissolved materials are left behind. In this
manner, seawater directly beneath freshly formed sea ice has a higher salinity than it did before
the ice appeared. Of course, when this ice melts, it will tend to decrease the salinity of the
surrounding water.
In the Weddell Sea, off Antarctica, the densest water in the oceans is formed as a result of
this freezing process, which increases the salinity of cold water. This heavy water sinks and is
found in the deeper portions of the oceans of the world.
(Source: Toefl Reading Comprehension 4)
Question 57. What does the passage mainly discuss?
A. The bodies of water of the world. B. The elements of salt.
C. The many forms of ocean life. D. The salinity of ocean water.
Question 58. According to the passage, the ocean generally has more salt in _____ .
A. coastal areas B. tropical areas C. rainy areas D. turbulent areas
Question 59. All of the following are processes that decrease salinity EXCEPT
A. evaporation B. precipitation C. run off D. melting
Question 60. Which of the following statements about the salinity of a body of water can best be
inferred from the passage?
A. The temperature of the water is the most important factor.
B. How quickly the water moves is directly related to the amount of alt.
C. Ocean salinity has little effect on sea life.
D. Various factors combine to cause variations in the salt content of water.
Question 61. The word "it" in paragraph 4 refers to which of the following?
A. Sea ice B. Salinity C. Seawater D. Manner
Question 62. Why does the author mention the Weddell Sea?
A. To show that this body of water has salinity variations
B. To compare Antarctic waters with Arctic waters
C. To give an example of cold-water salinity
D. To point out the location of deep waters
Question 63. Which of the following is NOT a result of the formation of ocean ice?
A. The salt remains in the water B. The surrounding water sinks
C. Water salinity decreases D. The water becomes denser
Question 64. What can be inferred about the water near the bottom of oceans?
A. It is relatively warm. B. Its salinity is relatively high.
C. It does not move. D. It is formed by melting sea ice.
SECTION IV: WRITING
Part 1: Mark the letter A, B, C, or D on your answer sheet to indicate the sentence that best
combines each pair of sentences in the following questions from 65 to 72.
Question 65: Jimmy works so hard. I’m sure he’ll be promoted soon.
A. Only when I am sure he’ll be promoted soon does Jimmy work hard.
B. Were I not sure of his being promoted soon, Jimmy wouldn’t work so hard.
C. So hard is his work that I’m sure he’ll be promoted soon.
D. So hard does Jimmy work that I’m sure he’ll be promoted soon.
Question 66: The Prime Minister failed to explain the cause of the economic crisis. Furthermore, he did
not offer any solutions.
A. Although the Prime Minister explained the cause of the economic crisis, he failed to offer any solutions.
B. Not only did the Prime Minister explain the cause of the economic crisis, but he also offered solutions.
C.The Prime Minister offered some solutions based on the explanation of the cause of the economic crisis.
D. The Prime Minister didn’t explain the cause of the economic crisis, nor did he offer any solutions.
Question 67 : You can eat the candy. Make sure you leave some for me.
A. You can eat the candy provided you leave some for me.
B. You can eat the candy unless you leave some for me.
C. You can’t eat the candy if you leave some for me.
D. You can’t eat the candy in case you leave some for me.
Question 68: The soccer team knew they lost the match. They soon started to blame each other.
A. Not only did the soccer team lose the match but they blamed each other as well.
B. No sooner had the soccer team started to blame each other than they knew they lost the match.
C. As soon as they blamed each other, the soccer team knew they lost the match.
D. Hardly had the soccer team known they lost the match when they started to blame each other.
Question 69: The plan is not excellent. Its early reveal spoils it.
A. The plan will be excellent provided it were revealed so early.
B. But for its early reveal, the plan wouldn’t be excellent.
C. The plan would be excellent unless it were revealed so early.
D. If the plan were revealed so early, it would be excellent.
Question 70: I did not read his book. I did not understand what the lecturer was saying.
A. What the lecturer wrote and said was too difficult for me to understand.
B. I found it very difficult to understand what the lecturer said in his book.
C. I would have understood what the lecturer was saying if I had read his book.
D. The lecturer's book which I had not read was difficult to understand.
Question 71: Hartford is the capital of Connecticut. It is the second largest city in the state.
A. The capital of Connecticut, Hartford which is the second largest city in the state.
B. Hartford, the capital of Connecticut, is the second largest city in the state.
C. Hartford, the second largest city in the state, which is the capital of Connecticut.
D. Hartford, the capital of Connecticut, that is the second largest city in the state.
Question 72: She tries to learn very hard. She will never beat Sue at tennis.
A. However hard she tries to learn, she will never beat Sue at tennis.
B. No matter how hardly she tries to learn, she will never beat Sue at tennis.
C. It doesn't matter how she tries hard, she will never beat Sue at tennis.
D. In spite of trying to learn very hard, she will beat Sue at tennis.

Part 2: Mark the letter A, B, C, or D on your answer sheet to indicate the sentence that is closest in
meaning to the sentence given in each of the following questions from 73 to 80.
Question 73: The number of accidents has gone down steadily since the speed limit was imposed.
A. There has been a steady decline in the number of accidents since the speed limit was imposed.
B. It is the speed limit that reduce steadily the number of accidents.
C. The imposing of speed limit has resulted from the number of accidents.
D. There have been few accidents than before since they used the speed limit.
Question 74: The government does not know what to do with household rubbish in large cities.
A. Little does the government know what to do with household rubbish in large cities.
B. It is unknown what to do with household rubbish in large cities by the government
C. Rarely the government knows what to do with household rubbish in large cities.
D. Hardly any government knows what to do with household rubbish in large cities.
Question 75: At no time did the two sides look likely to reach an agreement.
A. The two sides had no time to reach an agreement.
B. The two sides never looked likely to reach an agreement.
C. If the two sides had had time, they would have reached an agreement.
D. The two sides never looked like each other.
Question 76: It’s compulsory for me to finish my work this evening.
A. I can finish my work this evening. C. I needn’t finish my work this evening.
B. I must finish my work this evening. D. I shouldn’t finish my work this evening.
Question 77: Rather than interrupt the conference, she left without saying goodbye.
A. She interrupted the conference because she said goodbye.
B. The conference was interrupted as she left saying goodbye.
C. She would rather interrupt the conference than leave without saying goodbye.
D. She left without saying goodbye as she didn't want to interrupt the conference.
Question 78: By being absent so often John failed the examination.
A. John’s frequent absences cost him his chance of passing the examination.
B. Being absent so often caused John fail his examination.
C. John failed his examination although he was absent quite often.
D. John’s failure in his examination suffered from his frequent absences.
Question 79: He found himself at a loss to understand his closest friend’s words.
A. He found his closest friend’s words quite incomprehensible.
B. He misunderstood his closest friend’s words completely.
C. He lost heart and didn’t understand his closest friend’s words.
D. He found his closest friend’s words easy to understand.
Question 80: It is a basic requirement in the modern world to be able to deal with figures.
A. The world requires us to have a basic understanding of figures.
B. Being able to deal with figures is a basic requirement in the modern world.
C. Dealing with the modern world requires a basic knowledge of figures.
D. Dealing with the figures requires a basic knowledge of the modern world.

***** THE END *****

ĐÁP ÁN MÃ ĐỀ 132

Câu Đáp án Câu Đáp án Câu Đáp án Câu Đáp án


1 B 21 D 41 B 61 C
2 A 22 D 42 A 62 C
3 A 23 C 43 D 63 C
4 C 24 A 44 A 64 B
5 B 25 D 45 C 65 D
6 B 26 C 46 C 66 D
7 C 27 A 47 D 67 A
8 C 28 C 48 B 68 D
9 B 29 B 49 A 69 C
10 B 30 C 50 B 70 C
11 B 31 A 51 C 71 B
12 A 32 A 52 B 72 A
13 A 33 B 53 C 73 A
14 C 34 B 54 B 74 A
15 A 35 A 55 D 75 B
16 B 36 D 56 C 76 B
17 C 37 B 57 D 77 D
18 D 38 B 58 B 78 A
19 D 39 D 59 A 79 A
20 C 40 A 60 D 80 B
SỞ GIÁO DỤC VÀ ĐÀO TẠO KỲ THI HỌC SINH GIỎI THPT CHUYÊN VÀ CHỌN ĐỘI
QUẢNG NAM TUYỂN DỰ THI HỌC SINH GIỎI QUỐC GIA
NĂM HỌC 2019-2020

HƯỚNG DẪN CHẤM Môn thi : TIẾNG ANH


(Gồm 3 trang + 3 trang audioscript) Thời gian: 180 phút (không kể thời gian giao đề)
Ngày thi : 9/10/2019

Điểm phần A,B,C Họ tên và chữ ký của giám khảo Mã phách


Ghi số Ghi chữ Giám khảo 1 Giám khảo 2

SECTION A. LISTENING (50pts)

HƯỚNG DẪN PHẦN THI NGHE HIỂU


- Bài nghe gồm 4 phần. Mỗi phần thí sinh được nghe 2 lần.
- Mọi hướng dẫn cho thí sinh đã có trong bài nghe.

Part 1. For questions 1 – 5, listen to a radio news report about ‘Google’, a popular Internet search engine
and answer the questions. Write NO MORE THAN FIVE WORDS taken from the recording for each
answer.
1. What way did Google rely on to market its product?
_________________________________________________________________________
2. What position did Google achieve last week as the Internet search engine for America Online?
_________________________________________________________________________
3. What group of people was mentioned to favour Google as a search engine?
_________________________________________________________________________
4. What verb is the word ‘google’ said to be replacing?
_________________________________________________________________________
5. Who invented the original term ‘googol’?
_________________________________________________________________________

Part 2. For questions 6 – 15, listen to a piece of news about future technology and complete the following
sentences. Write NO MORE THAN THREE WORDS AND/OR A NUMBER taken from the recording in
each space.

Strange-looking as they are, the robots can help to explore collapsed buildings for
6._________________________ efforts.
Flying robots, going airborne, help engineers inspect 7.___________________________ safely.
The robot goes and collects data; it is acting as 8._________________________ to the inspector.
In the game Eyewire, players could help researchers by tracing brain neurons to create
9._________________________.
The professor says the human brain has 85 billion neurons and that with 10._________________________,
they may trace one to two neurons a day.
They take the players’ input and use it to train 11.__________________________ to speed up the process.
New technologies are also exploring our 12._________________________, as our limitations aren’t just
physical.
The technology they’re building doesn’t allow us to have 13._________________________ of something
really subjective like emotions.
Multi-sense tracks facial expressions and 14.__________________________ to help clinicians diagnose
mental illnesses such as depression or PTSD.
What clinicians need is more of a technology to be 15.___________________________ in the real-time.
Part 3. For questions 16 – 20, listen to two nutritionists, Fay Wells and George Fisher, discussing methods
of food production and choose the correct answer A, B, C or D which fits best according to what you hear.
Write your answers in the corresponding numbered boxes provided.

16. Looking at reports on the subject of GM foods, Fay feels ____________.


A. pleased to read that the problem of food shortages is being addressed
B. surprised that the fears of the public are not allayed by them
C. frustrated by contradictory conclusions
D. critical of the scientists' methodology

17. What does George suggest about organic foods?


A. Consumers remain surprisingly poorly informed about them.
B. People need to check out the claims made about them.
C. They need to be made more attractive to meat-eaters.
D. They may become more widely affordable in future.

18. What is George's opinion of 'vertical farming'?


A. It could provide a realistic alternative to existing methods.
B. It's a highly impractical scheme dreamt up by architects.
C. It's unlikely to go much beyond the experimental stage.
D. It has the potential to reduce consumption of energy.

19. George and Fay agree that the use of nanotechnology in food production will ____________.
A. reduce the need for dietary supplements
B. simplify the process of food-labelling
C. complicate things for the consumer
D. introduce potential health risks

20. In Fay's view, returning to self-sufficiency is only an option for people who ____________.
A. have no need to get a return on their investment
B. are willing to accept a high level of regulation
C. reject the values of a consumer society
D. already have sufficient set-up funds

Your answers:
16. 17. 18. 19. 20.

Part 4. For questions 21 – 25, listen to a radio discussion on technology in sport and decide whether the
statements are true (T) or false (F). Write your answers in the corresponding numbered boxes provided.

21. Geoff thinks the use of cameras for refereeing decisions will add to the excitement of sport.
22. Sally enjoys the speed at which tennis is played nowadays.
23. Geoff suggests that if everyone has access to doping, then it should be acceptable.
24. Geoff says that certain banned practices should be made legal.
25. He feels that there are adequate restrictions on the use of technology in sport.

Your answers:
21. 22. 23. 24. 25.
SECTION B. LEXICO-GRAMMAR (30 points)

Part 1. For questions 1-20, choose the correct answer A, B, C, or D to each of the following questions.
Write your answers in the corresponding numbered boxes provided.

1. Successful athletes cannot afford to be _________; they need to stay cool and focused.
A. highly-paid B. highly-motivated C. highly-trained D. highly-strung
2. Her excellent grades in college led _________ a high-paying job after graduation.
A. to get her B. in getting her C. to her getting D. her getting
3. - “Are you working late again tonight?”
- “Yes, I’ll be here _________ the report.”
A. until I finish B. since I’ve finished C. when I’ve finished D. by the time I’ve finished
4. The kind of exercises you can do depends on your _________ of fitness.
A. stage B. level C. step D. phase
5. Aware that his pension will be small, he _________ a part of his salary for his old age.
A. sets apart B. sets aside C. puts up D. puts apart
6. I have no appetite and I am lethargic. I've been feeling under _________ for ages.
A pair B par C stress D threat
7. I think having a beer during a meeting with your boss is clearly _________ the mark.
A. overlooking B. overreaching C. overstepping D. overseeing
8. The new soap opera on Channel 3 _________ the depths in terms of tastelessness.
A plumbs B reaches C fills D achieves
9. We’ll keep you _________ on any further changes in the examination specifications.
A noticed B announced C mailed D posted
10. We must be sure to make the right decision because there is a lot at _________.
A. range B. stake C. chance D. expectation
11. His past behaviour had a definite _________ on what the judges decided.
A. bearing B. weight C. decision D. conclusion
12. If you are a student on a low budget you are probably _________ from paying tax.
A. except B. excluded C. exempt D. apart
13. The mailing list has done much to _________ the numbers of people attending.
A. lift B. encourage C. heighten D. boost
14. As was _________ predicted, the company has announced hundreds of job losses.
A. considerably B. widely C. substantially D. amply
15. She felt that travelling had greatly _________ her life.
A. moved B. enriched C. expanded D. increased
16. A study that’s just been published _________ our theory completely.
A. puts up B. holds up C. backs up D. takes up
17. I recommend reading the books _________, starting with the very first.
A. by accident B. at random C. in sequence D. on impact
18. If we have to pay a £1,000 fine, then ________. We’re not going to win a fight with the Tax Office.
A. so be it B. be it so C. thus be it D. be it thus
19. He came into the room and sat down without _________ a word to anyone.
A. as far as B. too much of C. very much of D. so much as
20. The restaurant has _________ recently, and the food is much better now.
A. had its hands full B. lived hand to mouth C. changed hands D. gained the upper hand

Your answers:
1. 2. 3. 4. 5. 6. 7. 8. 9. 10.
11. 12. 13. 14. 15. 16. 17. 18. 19. 20.
Part 2. For questions 21-30, write the correct form of each bracketed word in the corresponding numbered
boxes provided. 0 has been done as an example.

FUSSY EATERS
If there is one thing that is likely to be (0. WORRY) ____________ for first-time parents, it is a young
child’s eating problems. Most of these parents’ worries, however, are (21. FOUND)____________
since the incidence of children who do not enjoy their food is far more (22. SPREAD)____________
than the majority imagine and the retention beyond (23. CHILD)____________ of such problems to
adolescence is (24. COMPARE)____________ rare.

There are, of course, cases which have perished into adulthood and those which appear to be more
than just a (25. PASS)____________ phase. In these cases, professional (26. GUIDE)____________
has to be sought.

Up to now, psychiatrists have (27. CATEGORY)____________ nine distinct types of eating


(28.ORDER)____________, each with its own particular treatment. The least serious of these is
selective eating, when the child displays his/her (29. WILL)____________ to try anything but a
narrow range of foods. This affects about 12% of three-year-olds but it rarely persists. The most
serious is persuasive refusal syndrome, which affects only a (30. HAND)____________ of people and
requires psychiatric supervision and treatment.

Your answers:
0. worrying

21. 22. 23. 24. 25.


26. 27. 28. 29. 30.

SECTION C. READING (60 points)


Part 1. For questions 1 – 10, fill each of the following numbered blanks with ONE suitable word. Write
your answers in the corresponding numbered boxes provided.

HYPERINFLATION
Inflation may be defined as either a rise in the general level of prices of goods and services in an economy
over a period of time, or a fall in the value of money over time. 'Hyperinflation' refers to extremely rapid or
(1) ____________ of control inflation. Perhaps the most famous example of hyperinflation in recent history is
that which took (2) ____________ in Germany after World War I. Between 1922 and 1923, prices in
Germany increased (3) ____________ a factor of 20 billion. Inflation was so out of control that prices rose
not just by the day, but by the hour and even minute. A loaf of bread cost just 463 marks in Germany in
March 1923, but by November that (4) ____________ year cost over 200,000,000,000 marks. The effect on
society was devastating. Because wages received in the morning would (5) ____________ worthless by the
afternoon, people spent their money as quickly as possible, buying any physical goods they could get their
hands (6) ____________ (whether they needed it or not) in a desperate attempt to get rid of currency units (7)
____________ they lost value. This only had the effect of stoking the fires of inflation further. Savings were
wiped out overnight. People lived in constant fear. Bartering and crime became the order (8) ____________
the day. Interestingly, hyperinflation is not a rare event. Since Weimar Germany, there have been 29
additional hyperinflations around the world, including those in Austria, Argentina, Greece and Brazil, to (9)
____________ but a few. On average, that's one every three years (10) ____________ so.

Your answers:
1. 2. 3. 4. 5.
6. 7. 8. 9. 10.
Part 2. For questions 11 – 20, read the text below and decide which answer A, B, C or D best fits each gap.
Write your answers in the corresponding numbered boxes provided.

THE BYGONE ERA

We live in an era (11) ________ by and increasingly dependent on technological innovations. It is for this
reason that younger generations find it (12) _________ impossible to envision a future devoid of the
convenience and comfort they provide us with. Small wonder then that when asked to (13) _________ what
life will be like in the future, they come up with something that sounds as if it has been taken out of a science-
fiction book. But this description is actually not the product of an (14) _________ imagination. Based on the
present speed at which breakthroughs are being (15) _________, it is actually a fairly accurate prediction. It
looks as if technology will have the (16) _________ hand and that fully automated systems will (17)
________ for people in all areas. People will take the back seat and instead of (18) ________ away at work
we will be able to take advantage of the time made available to us to engage in more recreational activities.
(19) ________ this time constructively will be a feat in itself. Perhaps, a case of too much of a good thing.
Things might just come to the point where, (20) ________ time to time, we will reminisce about the good old
days.

11. A. rivalled B. dominated C. surpassed D. overtaken


12. A. virtually B. fully C. potentially D. greatly
13. A. perceive B. forecast C. divine D. enact
14. A. intrepid B. ultimate C. inherent D. unbridled
15. A. done B. happened C. made D. occurred
16. A. upper B. back C. first D. high
17. A. emulate B. devise C. substitute D. duplicate
18. A. beavering B. badgering C. hounding D. monkeying
19. A. Utilising B. Manipulating C. Operating D. Manoeuvring
20. A. at B. in C. for D. from

Your answers:

11. 12. 13. 14. 15. 16. 17. 18. 19. 20.

Part 3. Read the following passage and answer the questions 21 – 30.

THE HISTORY OF A COOL IMAGE


A The history of sunglasses can be traced back to ancient Rome around the year AD 60, where the
Emperor Nero is said to have watched gladiator fights whilst holding up polished emerald-green gems to his
eyes, thus reducing the effect of the sun's glare. The very first actual recorded evidence of the use of
sunglasses can be found from a painting by Tommaso da Modena in Italy, 1352, showing a person wearing
sunglasses.
Earlier, around the twelfth century in China, sunglasses were worn by court judges, not to protect their eyes
from the sun, but in order to conceal any expressions in their eyes as it was important to keep their thoughts
and opinions secret until the end of each trial. These were flat panes of quartz that had been polished smooth
and then smoked to give their tint. It was not until 1430 that prescription glasses were first developed in Italy
to correct vision, and these early rudimentary spectacles soon found their way to China, where they were
again tinted by smoke to be used by the judges. The frames were carved out of either ivory or tortoiseshell,
and some were quite ornate. During the 17th century, prescription glasses were being used in England to help
elderly long-sighted people to see better. The Spectacle Makers Company was founded in England, which
started manufacturing prescription glasses for the public and whose motto was "A Blessing to the Aged".
B The development of sunglasses, however, remained static until the work of James Ayscough, who was
known for his work on microscopes in London around 1750. He experimented with blue and green tinted
lenses, believing they could help with certain vision problems. These were not sunglasses, however, as he was
not concerned with protecting the eyes from the sun's rays.
Prescription spectacles continued to be developed over the next few decades, especially regarding the design
of the spectacle frames and how to get them to sit comfortably on the nose. The frames were made from
leather, bone, ivory, tortoiseshell and metal, and were simply propped or balanced on the nose. The early
arms or sidepieces of the frames first appeared as strips of ribbon that looped around the backs of the ears.
Rather than loops, the Chinese added ceramic weights to the ends of the ribbons which dangled down behind
the tops of the ears. Solid sidepieces finally arrived in 1730, invented by Edward Scarlett.
C Sunglasses, as we know them today, were first introduced by Sam Foster in America, 1929. These were
the first sunglasses designed specifically to protect people's eyes from the harmful sun's rays. He founded the
Foster Grant Company, and sold the first pair of Foster Grant sunglasses on the boardwalk by the beaches in
Atlantic City, New Jersey. These were the first mass-produced sunglasses, and from this year onwards,
sunglasses really began to take off.
D In 1936, Edwin H Land patented the Polaroid filter for making polarized sunglasses. This type of tint
reduces glare reflected from surfaces, such as water. Later in that same year, Ray-Ban took the design of
pilots' sunglasses further by producing the aviator style sunglasses that we know today, using this recently
invented polarized lens technology. The edge of the frame characteristically drooped away at the edges by the
cheeks in a sort of tear drop shape, to give a full all-round protection to the pilots' eyes, who regularly had to
glance down towards the aircraft's instrument panel. The polarized lens reduced the glare from light reflected
off the instrument panel. Pilots were given these sunglasses free of charge, but in 1937 the general public
were allowed to purchase this aviator-style model that "banned" the sun's rays as Ray-Ban sunglasses.
E In 1960, Foster Grant started a big advertising campaign to promote sunglasses, and pretty soon famous
film stars and pop stars started wearing sunglasses as part of their image. The public began to adopt this new
fashion of wearing sunglasses, not just to protect their eyes from bright light, but also as a way of looking
good. Today, sunglasses are continuing to be improved with efficient UV blocking tints, cutting out all the
harmful ultra-violet light. Various coloured tints are now available and, of course, the frame styles are very
varied and exciting. Now you can really make a statement with your fashion sunglasses, transforming your
image or creating a new one. Designer sunglasses have certainly come a long way in just a few years, and
now not only protect our eyes from the harmful sun's rays, but are also an important fashion accessory - and it
all started nearly 2,000 years ago with the Roman Emperor Nero!

For questions 21- 25, choose the correct heading for sections A-E. There are THREE extra headings that
you do not need to use. Write your answers in the spaces provided.
List of Headings
i New developments in sunglasses lenses
ii The use of sunglasses in early courts
iii How the physical shape of early sunglasses developed
iv The introduction of sidepieces on sunglasses
v The origins and early history of sunglasses
vi Ways in which sunglasses have become trendy
vii The arrival of modern sunglasses
viii Advertising campaigns for sunglasses

Your answers:
21. Section A ______ 23. Section C ______ 25. Section E ______
22. Section B ______ 24. Section D ______
For questions 26-30, decide whether the following statements agree with the information given in the
reading passage. Write in the corresponding numbered boxes provided
YES if the statement agrees with the claims of the writer.
NO if the statement contradicts the claims of the writer.
NOT GIVEN if it is impossible to say what the writer thinks about this.
26. The earliest reference to sunglasses can be found in early Roman times.
27. Early Chinese sunglasses were worn to correct the wearer's eyesight.
28. The work of James Ayscough had a profound effect on the development of modern lenses.
29. Prior to 1730, sidepieces on glasses were made of many different materials.
30. Sam Foster's sunglasses were the first to be made for a mass market.
Your answers:
26. 27. 28. 29. 30.

Part 4. For questions 31 – 40, read an extract from an article on advertising and choose the answer A, B, C
or D which you think fits best according to the text. Write your answers in the corresponding numbered
boxes provided.

ADVERTISING SHIFTS FOCUS


The average citizen is bombarded with TV commercials, posters and newspaper advertisements wherever he
goes. Not only this, but promotional material is constantly on view, with every available public space from
shop to petrol station covered with advertising of some kind. People who are foolish enough to drive with
their windows open are likely to have leaflets advertising everything and anything thrust in at them. The
amount of advertising to which we are exposed is phenomenal, yet advertisers are being hurt by their
industry's worst recession in a decade and a conviction that is in many respects more frightening than the
booms and busts of capitalism: the belief that advertising can go no further. Despite the ingenuity of the
advertisers, who, in their need to make their advertisements as visually attractive as possible, often totally
obscure the message, the consumer has become increasingly cynical and simply blanks out all but the subtlest
messages. The advertising industry has therefore turned to a more vulnerable target: the young.

The messages specifically aimed at children are for toys and games - whose promotional budgets increased
fivefold in the 1990s - and fast food, which dominates the children's advertising market. Advertisers
acknowledge that the commercial pressures of the 1990s had an extraordinary effect on childhood: it is now
generally believed that the cut-off point for buying toys has been falling by one year every five years.
Research, suggests that while not so many years ago children were happy with Lego or similar construction
games at ten or eleven, most of today's children abandon them at six or seven. In effect, the result is the
premature ageing of children.

There is nowhere where the advertising industry's latest preoccupation with the young is so evident as in
schools. Increasingly low budgets have left schools vulnerable to corporate funding and sponsorship schemes
in order to provide much needed equipment, such as computers, or to enable them to run literacy schemes.
While on the face of it this would seem to be a purely philanthropic gesture on the part of the companies
concerned, the other side of the coin is a pervasive commercial presence in the classroom, where textbooks
and resource books are increasingly likely to bear a company logo.

This marked shift in advertising perceptions also means that a great deal of supposedly adult advertising has
an infantile appeal, inasmuch as adult products can be presented within an anecdote or narrative, thus making
the message more accessible to young teenagers and smaller children. Children obviously cannot buy these
things for themselves; what is behind these advertisements is more subtle. Advertisers have come to
recognize that if children can successfully pester their parents to buy them the latest line in trainers, then they
can also influence their parent's choice of car or credit card, and so children become an advertising tool in
themselves.

There are many, on all sides of the ideological spectrum, who would argue that advertising has little influence
on children, who are exposed to such a huge variety of visual images that advertisements simply become lost
in the crowd. Rather, they would argue that it is the indulgent parents, who do not wish their children to lack
for anything, who boost sales figures. While there may be a great deal of truth in this, it would seem that to
deny that advertising influences at all because there is so much of it, while accepting that other aspects of life
do have an effect, is a little disingenuous. In fact, the advertising industry itself admits that since peer
pressure plays such an important role in children's lives, they are not difficult to persuade. And of course,
their minds are not yet subject to the advertising overload their parents suffer from. The question that arises is
whether indeed, we as a society can accept that children, far from being in some sense protected from the
myriad of pressures, decisions and choices which impinge on an adult's life, should now be exposed to this
influence in all aspects of their lives, in ways that we as adults have no control over. Or do we take the
attitude that, as with everything else from crossing city streets to the intense competition of the modern world,
children will have to learn to cope, so the sooner they are exposed the better?

31. What does the writer say about advertising in the first paragraph?
A. Capitalism has led to the demise of advertising.
B. We should have a cynical view of advertisers.
C. Advertising is facing new challenges these days.
D. The industry has run out of new ideas.
32. The bombardment of advertisements has led to ________.
A. children taking more notice of them
B. greater difficulty in attracting consumers' attention
C. more appealing advertisements
D. people being less likely to spend money
33. How have children changed during the past decade?
A. They have become consumers.
B. They are growing up more quickly.
C. They are becoming cleverer.
D. They are not playing as much.
34. Which of the following square brackets [A], [B], [C], or [D] best indicates where in the paragraph the
sentence “However, the main thrust of advertising in this area is no longer towards traditional children's
products.” can be inserted?
[A] The messages specifically aimed at children are for toys and games - whose promotional budgets
increased fivefold in the 1990s - and fast food, which dominates the children's advertising market. [B]
Advertisers acknowledge that the commercial pressures of the 1990s had an extraordinary effect on
childhood: it is now generally believed that the cut-off point for buying toys has been falling by one year
every five years. [C] Research, suggests that while not so many years ago children were happy with Lego or
similar construction games at ten or eleven, most of today's children abandon them at six or seven. In effect,
the result is the premature ageing of children. [D]
A. [A] B. [B] C. [C] D. [D]
35. Which of the following sentences best expresses the meaning of the underlined sentence in paragraph 3?
A. The advertising industry's latest obsession with young people is rather obvious in schools.
B. Nowhere else can we see the advertising industry's latest products for the young as in schools.
C. Schools are places where the advertising industry's latest concern with youngsters is the least obvious.
D. It is in schools that the advertising industry's latest concern with youngsters is the most clearly seen.
36. What does the writer imply in the third paragraph?
A. Advertising agencies need to preserve their reputations.
B. Schools welcome aid from big business.
C. There are restrictions on how financial aid may be used.
D. Companies expect nothing in return for their help.
37. How have children changed the face of advertising?
A. Children are influencing the purchases of adult products.
B. They are now the advertising industry's sole market.
C. More products have to be sold to children.
D. Children have become more selective in their choices.
38. The word “who” in the last paragraph refers to ________.
A. many people B. the crowd C. parents D. children
39. What does the writer suggest in the last paragraph?
A. Adults feel increasingly threatened by advertising.
B. Children are unlikely to be influenced by their friends.
C. Parents avoid spending too much money on their children.
D. Children have a less sheltered existence than they used to.
40. In the text as a whole, the writer's purpose is to ________.
A. explain the inspiration for advertisements
B. expose the exploitation of children
C. deter parents from giving in to advertisers
D. prevent advertisers from infiltrating schools

Your answers:
31. 32. 33. 34. 35. 36. 37. 38. 39. 40.

Part 5. The passage below consists of five paragraphs marked A, B, C, D and E. For questions 41-50, read
the passage and do the task that follows. Write your answers in the corresponding numbered boxes
provided. Each letter may be used more than once.

SEEKING SOCRATES

It may be more than 2,400 years since his death, but the Greek philosopher can still teach us a thing or two
about leading ‘the good life’. Bettany Hughes digs deeper.

A Sharing breakfast with an award-winning author in an Edinburgh hotel a few years back, the
conversation came round to what I was writing next. 'A book on Socrates,' I mumbled through my muesli.
'Socrates!' he exclaimed. 'What a brilliant doughnut subject. Really rich and succulent with a great hole in the
middle where the central character should be.' I felt my smile fade because, of course, he was right. Socrates,
the Greek philosopher, might be one of the most famous thinkers of all time, but, as far as we know, he wrote
not a single word down. Born in Athens in 469BC, condemned to death by a democratic Athenian court in
399BC, Socrates philosophized freely for close on half a century. Then he was found guilty of corrupting the
young and of disrespecting the city's traditional gods. His punishment? Lethal hemlock poison in a small
prison cell. We don't have Socrates' personal archive; and we don't even know where he was buried. So, for
many, he has come to seem aloof and nebulous – a daunting intellectual figure – always just out of reach.
B But that is a crying shame. Put simply, we think the way we do because Socrates thought the way he did.
His famous aphorism, 'the unexamined life is not worth living', is a central tenet for modern times. His
philosophies 24 centuries old - are also remarkably relevant today. Socrates was acutely aware of the dangers
of excess and overindulgence. He berated his peers for a selfish pursuit of material gain. He questioned the
value of going to fight under an ideological banner of 'democracy'. What is the point of city walls, warships
and glittering statues, he asked, if we are not happy? The pursuit of happiness is one of the political pillars of
the West. We are entering what has been described as 'an age of empathy'. So Socrates' forensic, practical
investigation of how to lead 'the good life' is more illuminating, more necessary than ever.
C Rather than being some kind of remote, tunic-clad beardy who wandered around classical columns,
Socrates was a man of the streets. The philosopher tore through Athens like a tornado, drinking, partying,
sweating in the gym as hard as, if not harder than the next man. For him, philosophy was essential to human
life. His mission: to find the best way to live on earth. As Cicero, the Roman author, perceptively put it:
'Socrates brought philosophy down from the skies.' And so to try to put him back on to the streets he loved
and where his philosophy belonged, I have spent 10 years investigating the eastern Mediterranean landscape
to find clues of his life and the 'Golden Age of Athens'. Using the latest archaeology, newly discovered
historical sources, and the accounts of his key followers, Plato and Xenophon, I have endeavoured to create a
Socrates shaped space, in the glittering city of 500BC Athens – ready for the philosopher to inhabit.
D The street jargon used to describe the Athens of Socrates' day gives us a sense of its character. His
hometown was known as 'sleek', 'oily', 'violet crowned', 'busybody' Athens. Lead curse tablets left in drains,
scribbled down by those in the world's first true democracy, show that however progressive fifth-century
Athenians were, their radical political experiment - allowing the demos (the people) to have kratos (power)
did not do away with personal rivalries and grudges. Far from it. In fact, in the city where every full citizen
was a potent politician, backbiting and cliquery came to take on epic proportions. By the time of his death,
Socrates was caught up in this crossfire.
E His life story is a reminder that the word 'democracy' is not a magic wand. It does not automatically
vaporize all ills. This was Socrates' beef, too – a society can only be good not because of the powerful words
it bandies around, but thanks to the moral backbone of each and every individual within it. But Athenians
became greedy, they overreached themselves, and lived to see their city walls torn down by their Spartan
enemies, and their radical democracy democratically voted out of existence. The city state needed someone to
blame. High-profile, maddening, eccentric, freethinking, free-speaking Socrates was a good target. Socrates
seems to me to be democracy's scapegoat. He was condemned because, in fragile times, anxious political
masses want certainties – not the eternal questions that Socrates asked of the world around him.

In which paragraph is each of the following mentioned? Your answers:

relationships between people in Socrates' time 41.______


the continuing importance of Socrates' beliefs 42.______
the writer's theory concerning what happened to Socrates 43.______
why little is known about Socrates as a man 44.______
how the writer set about getting information relevant to Socrates 45.______
the difference between common perceptions of Socrates and what he was really like 46.______
an aim that Socrates was critical of 47.______
the realization that finding out about Socrates was a difficult task 48.______
how well known Socrates was during his time 49.______
an issue that Socrates considered in great detail 50.______
Điểm phần D1, D2 Họ tên và chữ ký của giám khảo Mã phách
Ghi số Ghi chữ Giám khảo 1 Giám khảo 2

SECTION D. WRITING (60 points)


Part 1. Read the following passage and use your own words to summarise it. Your summary should be
between 100 and 120 words long.
Today, the majority of the world's population may not be vegetarians, but vegetarianism is rapidly gaining
popularity. People who decide to become vegetarians generally have very strong feelings about the issue and
may choose a vegetarian diet for different reasons. Health issues, awareness of environmental problems and
moral issues are three common arguments in favour of vegetarianism that are quite convincing.
Many non-vegetarians claim that a vegetarian diet does not give a person the necessary vitamins and proteins
that their body needs. However, doctors and medical associations say that a vegetarian diet is able to satisfy
the nutritional needs of people of all ages. All the nutrients and proteins one's body needs can be found in
vegetables, nuts and grains, as well as in dairy products. Eating meat may be an easy way to get the protein
one needs, but it is not the only way.
Vegetarians also argue that the meat industry is the source of many environmental problems that could be
eliminated if people ate less meat or even stopped eating it altogether. Raising livestock for the meat industry
takes a huge toll on the world's natural resources; for example forests are cut down to clear land for crops to
feed livestock or for pastureland. This in turn leads to an increase in global warming, loss of topsoil and loss
of plant and animal life.
Finally, many people refrain from eating meat for ethical reasons. They object to taking the life of another
living creature in order to satisfy their hunger. Moreover, they argue that we inflict great pain and suffering
on animals that are raised for meat. Poultry and livestock raised on factory farms are kept under abominable
conditions, confined in areas that hardly allow them to move, fed with antibiotics and, in the end, they are
cruelly slaughtered.
Becoming a vegetarian might not appeal to everyone, but it is a choice that is gaining popularity as our
awareness of health and environmental issues as well as our concern for animal welfare is growing. It is also
becoming more feasible as restaurants and supermarkets increasingly cater for the vegetarian market.

……………………………………………………………………………………………………………...
……………………………………………………………………………………………………………...
……………………………………………………………………………………………………………...
……………………………………………………………………………………………………………...
……………………………………………………………………………………………………………...
……………………………………………………………………………………………………………...
……………………………………………………………………………………………………………...
……………………………………………………………………………………………………………...
……………………………………………………………………………………………………………...
……………………………………………………………………………………………………………...
……………………………………………………………………………………………………………...
……………………………………………………………………………………………………………...
……………………………………………………………………………………………………………...
……………………………………………………………………………………………………………...
……………………………………………………………………………………………………………...
Part 2. The pie charts below show the expenditure of two technology companies of similar size in the UK
in 2012.
Summarise the information by selecting and reporting the main features, and make comparisons where
relevant. You should write about 150 words.

……………………………………………………………………………………………………………...
…………………………………………………………………………………………………………...…
……………………………………………………………………………………………………………...
……………………………………………………………………………………………………………...
……………………………………………………………………………………………………………...
……………………………………………………………………………………………………………...
……………………………………………………………………………………………………………...
……………………………………………………………………………………………………………...
……………………………………………………………………………………………………………...
……………………………………………………………………………………………………………...
……………………………………………………………………………………………………………...
……………………………………………………………………………………………………………...
……………………………………………………………………………………………………………...
……………………………………………………………………………………………………………...
……………………………………………………………………………………………………………...
……………………………………………………………………………………………………………...
……………………………………………………………………………………………………………...
……………………………………………………………………………………………………………...
……………………………………………………………………………………………………………...
……………………………………………………………………………………………………………...
……………………………………………………………………………………………………………...
……………………………………………………………………………………………………………...
……………………………………………………………………………………………………………...
……………………………………………………………………………………………………………...

Điểm phần D3 Họ tên và chữ ký của giám khảo Mã phách


Ghi số Ghi chữ Giám khảo 1 Giám khảo 2
Part 3. Write an essay of 350 words on the following topic.

Drug addiction is becoming an increasing problem. In order to reduce this problem, anyone caught using
drugs should be automatically sentenced to time in prison.
Do you agree or disagree?

Give reasons for your answer and include any relevant examples from your own knowledge and
experience.

……………………………………………………………………………………………………………...
……………………………………………………………………………………………………………...
……………………………………………………………………………………………………………...
……………………………………………………………………………………………………………...
……………………………………………………………………………………………………………...
……………………………………………………………………………………………………………...
……………………………………………………………………………………………………………...
……………………………………………………………………………………………………………...
……………………………………………………………………………………………………………...
……………………………………………………………………………………………………………...
……………………………………………………………………………………………………………...
……………………………………………………………………………………………………………...
……………………………………………………………………………………………………………...
……………………………………………………………………………………………………………...
……………………………………………………………………………………………………………...
……………………………………………………………………………………………………………...
……………………………………………………………………………………………………………...
……………………………………………………………………………………………………………...
……………………………………………………………………………………………………………...
……………………………………………………………………………………………………………...
……………………………………………………………………………………………………………...
……………………………………………………………………………………………………………...
……………………………………………………………………………………………………………...
……………………………………………………………………………………………………………...
……………………………………………………………………………………………………………...
……………………………………………………………………………………………………………...
……………………………………………………………………………………………………………...
……………………………………………………………………………………………………………...
……………………………………………………………………………………………………………...
……………………………………………………………………………………………………………...
……………………………………………………………………………………………………………...
……………………………………………………………………………………………………………...
……………………………………………………………………………………………………………...
……………………………………………………………………………………………………………...
……………………………………………………………………………………………………………...
……………………………………………………………………………………………………………...
……………………………………………………………………………………………………………...
……………………………………………………………………………………………………………...
……………………………………………………………………………………………………………...
……………………………………………………………………………………………………………...
……………………………………………………………………………………………………………...
……………………………………………………………………………………………………………...
……………………………………………………………………………………………………………...
……………………………………………………………………………………………………………...
……………………………………………………………………………………………………………...
……………………………………………………………………………………………………………...
……………………………………………………………………………………………………………...
……………………………………………………………………………………………………………...
……………………………………………………………………………………………………………...
……………………………………………………………………………………………………………...
……………………………………………………………………………………………………………...
……………………………………………………………………………………………………………...
……………………………………………………………………………………………………………...
……………………………………………………………………………………………………………...
……………………………………………………………………………………………………………...
……………………………………………………………………………………………………………...
……………………………………………………………………………………………………………...
……………………………………………………………………………………………………………...
……………………………………………………………………………………………………………...

----THE END----
SỞ GIÁO DỤC VÀ ĐÀO TẠO KỲ THI HỌC SINH GIỎI THPT CHUYÊN VÀ CHỌN ĐỘI
QUẢNG NAM TUYỂN DỰ THI HỌC SINH GIỎI QUỐC GIA
NĂM HỌC 2019-2020

HƯỚNG DẪN CHẤM Môn thi : TIẾNG ANH


(Gồm 3 trang + 3 trang audioscript) Thời gian: 180 phút (không kể thời gian giao đề)
Ngày thi : 9/10/2019

SECTION A: LISTENING ( 50pts)


Part 1: (2 x 5= 10 points)
1. (by) word of mouth
2. world largest service provider / the world's largest service provider
3. academics
4. (to) browse
5. A mathematician’s nephew / An American mathematician’s nephew / A nine-year-old boy /
A nephew of a mathematician

Part 2: (2 x 10= 20 points)


6. search(-)and(-)rescue 7. bridges and dams
8. (the) apprentice 9. (a) giant roadmap
10. 10,000 active users 11. (an) artificial intelligence
12. psyches 13. objective measures
14. nonverbal cues 15. multi-sense / multisense

Part 3: (2 x 5= 10 points)
Answers:
16. C 17. D 18. A 19. C 20. B

Part 4: (2 x 5= 10 points)
Answers:
21. T 22. F 23. F 24. F 25. T

SECTION B: LEXICO-GRAMMAR (30 points)


Part 1: (1 x 20= 20 points)
Answers:
1. D 2. C 3. A 4. B 5. B 6. B 7. C 8. A 9. D 10. B
11. A 12. C 13. D 14. B 15. B 16. C 17. C 18. A 19. D 20. C

Part 2: (1 x 10= 10 points)


Answers
21. unfounded 22. widespread 23. childhood 24. comparatively 25. passing
26. guidance 27. categorized / 28. disorders 29. unwillingness 30. handful
categorised

SECTION C: READING (60 points)


Part 1: (1.5 x 10= 15 points)
Answers:
1. out 2. place 3. by 4. same 5. be / become
6. on 7. before 8. of 9. name 10. or
Part 2: (1 x 10= 10 points)
Answers:

11. B 12. A 13. B 14. D 15. C 16. A 17. C 18. A 19. A 20. D

Part 3: (1 x 10= 10 points)


Answers:
21. v 22. iii 23. vii 24. i 25. vi 26. Yes 27. No 28. Not Given 29. No 30. Yes

Part 4: (1 x 10= 10 points)


Answers:

31. C 32. B 33. B 34. B 35. D 36. B 37. A 38. C 39. D 40. B

Part 5: (1.5 x 10= 15 points)


Answers:

41. D 42. B 43. E 44. A 45. C 46. C 47. B 48. A 49. E 50. B

SECTION D: WRITING (60 points)


Part 1: Read the following passage and use your own words to summarise it. Your summary should be between 100 and 120
words long.

Contents (10 points)


- The summary MUST cover the following points:
 The increasing popularity of vegetarianism today
 The three reasons why people choose a vegetarian diet (health issues (claim by non-vegetarians vs
doctors and medical experts’ ideas); awareness of environmental problems (less or no meat = fewer
environmental problems); and moral issues (It’s unfair to raise animals in bad conditions and then
kill them for meat to feed humans.)
- The summary MUST NOT contain personal opinions.
Language use (5 points)
The summary should:
- show attempts to convey the main ideas of the original text by means of paraphrasing (structural and
lexical use),
- demonstrate correct use of grammatical structures, vocabulary, and mechanics (spelling,
punctuations,...),
- maintain coherence, cohesion, and unity throughout (by means of linkers and transitional devices).
Penalties
- A penalty of 1 point to 2 points will be given to personal opinions found in the summary.
- A penalty of 1 point to 2 points will be given to any summary with more than 30% of words copied
from the original.
- A penalty of 1 point will be given to any summary longer than 130 words or shorter than 90 words.

Part 2: The pie charts below show the expenditure of two technology companies of similar size in the UK in 2012.
Summarise the information by selecting and reporting the main features, and make comparisons where relevant.

Contents (10 points)


- The report MUST cover the following points:
 Introduce the pie charts (2 points) and state the striking features (2 points)
 Describe main features with relevant data from the charts and make relevant comparisons
(6 points)
- The report MUST NOT contain personal opinions. (A penalty of 1 point to 2 points will be given to
personal opinions found in the answer.)
Language use (5 points)
The report should:
- demonstrate a wide variety of lexical and grammatical structures, have correct use of words (verb
tenses, word forms, voice,…); and mechanics (spelling, punctuations,...).

Part 3: Write a composition of about 350 words on the following topic:


Drug addiction is becoming an increasing problem. In order to reduce this problem, anyone caught using
drugs should be automatically sentenced to time in prison.
Do you agree or disagree?
Give reasons for your answer and include any relevant examples from your own knowledge and
experience.

The mark given to part 3 is based on the following criteria:


1. Task achievement/ fulfillment (10 points)
a. All requirements of the task are sufficiently addressed.
b. Ideas are adequately supported and elaborated with relevant and reliable explanations,
examples, evidence, personal experience, etc.
2. Organization (10 points)
a. Ideas are well organized and presented with coherence, cohesion, and unity.
b. The essay is well-structured:
1. Introduction is presented with a clear thesis statement introducing the points to be
developed.
2. Body paragraphs develop the points introduced with unity, coherence, and cohesion. Each
body paragraph must have a topic sentence and supporting details and examples when
necessary.
3. Conclusion summarises the main points and states personal opinions (prediction,
recommendation, consideration,…) on the issue.
3. Language use (5 points)
a. Demonstration of a variety of topic-related vocabulary
b. Excellent use and control of grammatical structures
4. Punctuation, spelling, and handwriting (5 points)
a. Correct punctuation and no spelling mistakes
b. Legible handwriting

Markers should discuss the suggested answers and the marking scale thoroughly before marking the papers.

-----

LISTENING TRANSCRIPTS
Part 1: You will hear a radio news report about ‘Google’, a popular Internet search engine. For questions
1 – 5, answer the questions. Use NO MORE THAN FIVE WORDS for each answer.

Presenter: Internet browsing is not conceivable without search engines – the various web pages which help
us find our way around the stupendous amount of cyber-locations in the World- Wide Web. And, since the
early nineties, hundreds of search engines have come and gone. One, however, has achieved a kind of success
that even New-Tech giants Microsoft are envious of: its name has become synonymous with the verb
“search”. Anna Mills has the report.

Woman: He may seem the most powerful man on the planet, but Bill Gates has not yet managed the ultimate
achievement in the New Technology industry: turning a product into a common word. The first such honour
is falling to Google, the Internet search engine devised by two Stanford PhD nerds, Larry Page and Sergey
Brin. The success of Google has come about through the most timeless form of marketing: word of mouth.
The site has for some time been the default tool for millions of people looking for anything they want to find
online, from obscure quotations to brass lamps. And there are increasing signs that the business is growing a
commercial sharpness to match the blade it uses to cut through Internet junk. Last week, Google secured a
place as the Internet search engine for America Online, the world's largest service provider, capping its
stealthy rise to the top.

But its success stretches far beyond the world of the Internet. In these dog days of the long university summer
break, I was up in the nearly deserted university library when I heard one professor say to another, "Me, I'm
just googling around". I knew what he meant. It wasn't that he was totally idle, but he wasn't really engaged in
sharply focused research, either. He was following leads from one source to another, happily wandering
through the archive, not knowing quite what he would find next.

Google – the search engine favoured by most academics – seems destined to be one of those proprietory
labels that becomes a word, a brand (like Hoover) that loses its initial capital letter. And the word itself is,
slowly but surely, replacing the verb "to browse", the paper-based metaphor that electronic catalogues use, as
if you were fingering the spines at some antiquarian bookstall. "Googling" is a different kind of sampling,
coming across relevant findings amongst an impossibly huge amount of information.

The company name is a corruption of "googol", spelt g- double o-g-o-l, the word apparently coined by the
nine-year-old nephew of American mathematician Edward Kasner to refer to the number represented by one
followed by 100 zeros, back in the 1940s. Little did he know that in the early 21st century, the use of the term
would become so commonplace amongst academics and laymen alike.

Part 2: You will hear a piece of news on future technology. Listen and complete the sentences. Write NO
MORE THAN THREE WORDS AND/OR A NUMBER in each space.

P = Presenter, M = Man, W = Woman

Future Combines Human and Machine intelligence ….

P: Believe it or not these strange-looking robots are really you. Well, more like an extension of you.
M: They can get into areas where humans really can't fit or be dangerous for humans to go.
P: That means the ability to explore collapsed buildings for search-and-rescue efforts or going airborne
with flying robots like this one which help engineers inspect bridges and dams safely.
W: We see that the robot acting as the apprentice to the inspector and the inspector tells the robot to go and
collect data.
P: For scientists at the recent frontiers conference in Pittsburgh, Pennsylvania, the future combines human
intelligence with machine intelligence. Eyewire is a game where players trace brain neurons to create a giant
roadmap that could help researchers.
M: The human brain has 85 billion neurons and with 10,000 active users we trace maybe one to two neurons
a day. We're gonna be here a long time which is why we take the players’ input and we use it to train an
artificial intelligence so that we can speed up that process.
P: And since human limitations aren't just physical, new technologies are also exploring our psyches.
M: We're not building technology that allows us to have objective measures of something that can be really
subjective: emotions.
P: Multi-sense tracks facial expressions and nonverbal cues to help clinicians diagnosed mental illnesses
such as depression or PTSD.
M: They already have the knowledge of medical knowledge; what they need is more of a technology to be
multi-sense in the real-time, these behaviors.
P: Technologies that have your back, physically and mentally, it's the next frontier.
Tina Terran VOA news Pittsburgh

Part 3: You will hear two nutritionists, Fay Wells and George Fisher, discussing methods of food
production. For questions 16 – 20, choose the answer (A, B, C or D) which fits best according to what you
hear.
M1 = Presenter, M2 = George, F = Fay

M1: Food, we might say, is always on our minds! Here today in the studio we have nutritionists Fay Wells
and George Fisher, who’ll be talking about methods of food cultivation and related issues that concern us all.
Fay, let’s kick off with the ‘hot potato’ of the day, genetically-modified foods.
F: Yes, you do hear a lot of hype from certain quarters on this topic. But let’s face it, public concern isn’t
helped by the fact that the various scientific reports available seem to leave you none the wiser. On the one
hand, you’ve got a group that’s finding GM foods to be quite safe and actually applaud them as a way of
dealing with food shortages in certain countries. Then, there’s another view that condemns them as
potentially dangerous to health and insufficiently trialled; whilst at the same time pointing out, quite
reasonably by the way, that the use of GM crops hasn’t actually made a dramatic difference to levels of food
production worldwide. In my view, it’s high time that science spoke with one voice on this issue.
M1: So, George, are organic foods the safest option then?
M2: Well, people are horrified to hear the level of herbicide and pesticide residues that remains in fruit and
vegetables, even after they’ve been carefully washed, because they go straight into our system. Organic foods
are one way round that. They don’t come cheap though, so it’s not currently an option for low income groups,
although that could come if mass production brings economies of scale. And the residues retained in fruit and
vegetables do vary, so some are safer to buy non-organically than others. It’s not an area that many
consumers are clued up about, but there’s no excuse for that ‘cos there’s plenty of factual information
available online. The other aspect of eating organically, by the way, that people often forget about is meat-
eating. Many people prefer to buy organic meat since it doesn’t contain the amount of antibiotics and growth
hormones that normal meat does.
M1: So what else is new on the food production front?
M2: Well, there’s a movement afoot in the direction of something called ‘vertical farming.’ It’s an attempt,
as the name suggests, to make use of vertical, mainly citycentre, space. Architects have designed skyscrapers
filled with orchards and fields that have the potential to produce crops all the year round! The only drawback,
as things stand at the moment, would be the prohibitive cost of the artificial lighting required! I know the idea
sounds a bit farfetched – pie in the sky, we might say! – but it’s not impossible that this vision might become
a reality one day. Already, urban rooftop farming is being developed, plus some special greenhouses
containing multiple racks of vegetables are in use in various parts of the world.
F: Another development, which sounds a bit spooky to me, is that of using nanotechnology in this area.
Basically we’re talking here about ‘atomically-modified’ foods containing invisibly small additives. Some
nano-scale additives and pesticides are already on the market and this looks as if it might change the face of
the large-scale food industry. To me, it seems like it will confuse the picture even more as regards what we’re
putting in our stomachs! What will be classified as ingredients?
M2: Yes, you’re not wrong there. I believe some dietary supplements are also being manufactured using
nanotechnology. Personally, I think that if people follow a reasonably healthy, well-balanced diet, they don’t
really need to take extra vitamins – certainly not on a long-term basis, anyway.
M1: So where would you stand on all this, Fay?
F: Actually, I’m just wondering whether people might not just react against all these developments and, to
be on the safe side, go back to more traditional forms of selfsufficiency! I’m feeling quite tempted, myself, to
go and buy a goat and a few chickens and start planting lots of vegetables! In fact, I’ve been doing some
personal research into this area and I’ve picked up a few valuable ideas. Like, if you set up a self-sufficient
smallholding or farm, you’ve got to be prepared to deal with an incredible barrage of rules and red tape, and
you’ve no choice but to toe the line. Secondly, organic horticulture on a large scale needs quite a bit of
investment. Though, of course, you can simply concentrate your energies on cultivating enough crops for
your own use.

Part 4: You will hear a radio discussion on technology in sport. Decide which of the following statements
are true and which are false. For questions 6 – 10, write T for true and F for false in the space provided.

P = Presenter GW = Geoff Winning SP = Sally Plumtree


P: Now, let’s pick up on one or two of those points you made there. Interestingly, you both mentioned the
role of technology in decision-making during football and tennis matches. Sally said it reduces the spectator’s
enjoyment. Would you agree with that, Geoff?
GW: Not at all, no. I’m absolutely certain that the cameras will only serve to heighten interest, to intensify
the drama and the tension. And we’ll still see the same displays of passion and anger from competitors – but
they’ll be directed more at themselves rather than at the referee or the umpire.
SP: Hmm, they’ll certainly be good for match officials, but I really cannot see that they’ll make a game
more exciting. Anyway, I think there are other reasons why tennis in particular no longer captivates spectators
like it used to. And it’s all down to technology.
P: In what way?
SP: Well, it was always such an exciting sport before, with long rallies that had everyone on the edge of their
seats. Then in the late 1970s, early 80s, players began using the oversize racket – the one with the very large
head. And sure, tennis became a much faster sport, but there aren’t so many of those rallies now, and there’s a
lot more dead time without any action, time when nothing’s happening. And as a spectator, I find that dull.
Gw: Maybe, but the oversize racket makes it easier to hit the ball, and that can only be a good thing for
amateur players – let’s not forget them.
P: OK. Some interesting points there. Geoff, let’s go back to what you said about technology helping us to
push out the boundaries of human achievement. Are all forms of technology acceptable in your book?
GW: Yes, I think if everyone has access to the same equipment, then virtually any technological innovation
is acceptable.
SP: Oh right. So presumably then, doping is acceptable, as long as everyone has access to it. That’s
brilliant.
GW: I’m not suggesting that at all. That should be obvious. There is no way we can justify the use of
performance-enhancing drugs or any kind of interference with our blood or genetic make-up. These practices
are not permitted by sporting authorities, and for good reason. There are moral issues involved here, quite
apart from the legal aspects and the potential dangers to our health.
P: Yes, you did use the word equipment, perhaps we should emphasise that.
GW: That’s right. And its use has to be standardized by the relevant sporting authority. A good example of
this is in the sport of cycling. Radical new designs in the 1980s and 90s meant that previous records were
being smashed beyond recognition. The International Cycling Federation felt these so-called ‘superbikes’
were having too much influence on the sport and their huge cost gave richer countries an unfair advantage. So
consequently they were banned from certain competitions such as the Olympics or the world hour record. It
just shows that there are controls on the use of technology and we should feel safe in that knowledge.
P: Anything to say on that, Sally?
SP: Well, I’m pleased the Federation saw sense in the end, but I just think the whole episode highlights the
uneasy relationship that exists ...
SỞ GD&ĐT VĨNH PHÚC KÌ THI CHỌN HSG CẤP TRƯỜNG NĂM HỌC 2019-
TRƯỜNG THPT LIỄN SƠN 2020
ĐỀ THI MÔN: TIẾNG ANH 12
ĐỀ CHÍNH THỨC
Thời gian thi: 150 phút, không kể thời gian giao đề
(Đề thi gồm 05 trang)
A. LISTENING
SECTION I. You will hear a new student, Tom, talking to a student representative called Rachel about
university clubs. You will listen to the talk TWICE.
Question 01 – 04. Complete the table below. Write NO MORE THAN THREE WORDS and/or A NUMBER
for each answer.
University Clubs Monday Tuesday Wednesday
Names Film Climbing Chess
Extra activities Discussions (01) (02)__________
__________
Current number of members (03)___________ 40 55
Contact Event’s organizer (04)__________ Math’s tutor
Question 05 - 10. Complete the notes below. Write NO MORE THAN THREE WORDS for each answer.
Details of climbing club: Benefits:
- meetings: (05)_____________ per month . - discounts on
- excursion to France in the (08)______________.
(06)______________. - annual (09)_______________.
- subscriptions paid (07)_________________. - tickets to the (10)______________ in
Cardiff.
SECTION II. You will hear a lecture about climate. Choose the correct letter A, B or C to answer the
questions. You will listen to the talk TWICE.
11. How many people died because of the severe heat wave in Europe in 2003?
A. 20,300 B. 70,000 C. 17,000
12. In 2009, bush fires killed a thousand people in _________.
A. Greece B. Australia C. Pakistan
13. Russian drought and heat led to _________.
A. rising grain cost B. rising unemployment C. drop in economy
14. Which country is NOT suffered from flash floods in 2010?
A. China B. Germany C. Greece
15. It is probably that _____ can make many kinds of extreme weather more frequent and severe.
A. fossil fuels B. natural gas C. global warming
16. What is the main suspect that leads to unprecedented extreme weather events?
A. human emission B. satellite measurement C. animal extinction
17. According to a simple fact of physics, warm air can hold more__________.
A. oxygen B. moisture C. dust
18. What pattern can change atmospheric circulation?
A. carbon-dioxide effects B. extreme regions C. planet changes
19. What crisis does human have to face beside weather extremes?
A. unemployment B. rising sea levels C. economic downturn
20. According to the lecturer, by the end of this century, the earth could heat up by_________.
A. 20 Celsius B. 3-40 Celsius C. 5-70 Celsius
B. GRAMMAR AND VOCABULARY
I. Mark the letter A, B, C, or D on your answer sheet to indicate the correct answer to each of the
following questions.
21. Sarah and I ________ reserved the room in the same hotel. She was very surprised to see me there.
A. practically B. intentionally C. deliberately D. coincidentally
22.________ interested in that subject, I would try to learn more about it.
A. Were I B. If I am C. Should I D. I was
23.It was a fair result because we played the game ________ the rules.
A. on account of B. due to C. according to D. apart from
24.The kind-hearted woman _________ all her life to helping the disabled and the poor.
A. spent B. dedicated C. lived D. wasted
25.Everyone in this office ________ busily planning the dance for a week.
A. is B. has been C. have been D. are
26.I need to finish a bench, so I’ll go and look for _______ in the garage.
A. one more wood B. much woods C. many woods D. a piece of wood
27.Helen: “Mr. Adams is going to retire next month.”Tom: “ ___________________”
A. I have no idea. B. You don’t say!
Right, you’d probably be the next. D. Congratulations!
28. So little ________ about mathematics that the lecture was completely beyond me.
A. did I know B. I have known C. do I know D. I knew
29.Let’s go to the library, ________?
A. would we B. will we C. shall we D. should we
30.The ________ horse began to run as fast as he could.
A. frightening B. frighten C. frightened D. frightful
31.It is a biological fact that children _________ their parents.
A. take after B. take to C. look up D. care for
32._______ I’d like to help you out, I’m afraid I just haven’t got any spare money at the moment.
A. Even B. Despite C. Much as D. Try as
33.By the time you arrive tomorrow, we _________ the work.
A. have completed B. had completed
C. will have completed D. would have completed
34.I doubt ________ the company will make any profit at all this year.
A. when B. since C. so that D. whether
35.We were angry about the situation and insisted _________ by the president.
A. to see B. on seeing C. on being seen D. to be seen
36._________ in the atmosphere is the temperature falling below the freezing point.
A. What produces frost B. Frost is produced
C. Frost produces D. What is frost produced
37.________ that’s happened is a waste of time.
A. Regret something B. Something to regret
C. Something regretted D. Regretting something
38.John: “Would you take this along to the office for me?”Peter: “_________________.”
A. Never mind B. Yes, with pleasure C. Yes, that’s right D. Not at all
39.We are fed up________ your perpetual moaning.
A. to B. with C. of D. at
40.No matter ________, Mozart was an accomplished composer while he was still a child.
A. how remarkable it seems B. how it seems remarkable
C. how it remarkable seems D. how seems it remarkable
8 Supply the correct form of the word in capital letter. Write your answers on your answer sheet.
It is important that you stand (41)________ with both feet rooted on the floor but COMFORT
slightly apart. Without realizing it, there will be a (42)________ to “retreat” from TEND
the audience or you may sway from side to side. Therefore, think of ways to
(43)________ this when rehearsing (e.g. imagine you are wearing heavy lead ACT
shoes on your feet which are keeping your heels down). Try not to lean on the
table or get your arms too close to your body as you will restrict your lungs and
impair your (44)________ to project your voice. Although it may help you feel ABLE
more (45)________, try not to walk about when delivering your (46)________ as RELAXATION
this will distract your audience. SPEAK
Like any skills, practice is vital. The more you practice, the more confident you
will be. Try not to (47)________ read out what you have prepared, and picture SIMPLE
yourself talking confidently and (48)________ about your subject. After you have KNOWLEDGE
been presenting in front of the mirror for a while, you will begin to see which
gestures and (49)________ movements work and which ones look wrong. FACE
Remember that nobody wants you to do badly and try to feel positive that you are
going to deliver an interesting and (50)________ talk. INFORM
10. Find the mistake in each of the following sentences by choosing A, B, C or D. Write your answers
on your answer sheet.
51. He was very pride of the work he had done.
A. very B. pride C. the work D. done
52. An important factor should be considered is Mr. Lopez’s ability to keep the new restaurant going
for several months with limited revenue.
A. should be considered B. ability
C. for several D. with limited revenue
53, A conductor uses signals and gestures to let the musicians to know when to play different parts of a
composition.

54, The threat of being dismissed do not worry me anymore because I have started my own business.
A. The B. being dismissed C. do not worry D. my own
business
55. The lion has long been a symbol of strength, power, and cruel.
A. The B. long C. a symbol D. cruel
C. READING
I. Read the text below and think of the word which best fits each space. Use only ONE word in each
space. Write your answers on your answer sheet.
“Nobody plans to fail, they simply fail to plan.” Nowhere is this truer (56)________ in the arena
of public speaking. The classic format for any talk (57)________ a beginning, a middle and an end.
Think about how long you want the talk to (58)________ and plan the content of the talk to fit the right
amount of time. Also, think about your audience: make sure your approach to the subject of your talk is
(59)________ too complex nor too simple for them. If you are going to use prompts, small cards are quite
good (60)________ they can be discreet and relatively unnoticeable compared (61)________ large sheets
of paper.
Wherever possible, make sure you know the venue of your talk and (62)________ that people can
see you and any slides or diagrams you might be using to support your points. When (63)________ to a
large group, try to keep very (64)________ to the front row as this creates a feeling of togetherness.
Some of (65)________ best comedians use this closeness to encourage a feeling of secrecy, telling you
things as a stage whisper in order to build this intimacy.
F. Read the following passage and choose the options among A, B, C or D that best complete the
blanks. Write your answers on your answer sheet.
Quite apart from the economic similarity between present – day automation and the mechanization, which
has been proceeding for centuries, it must also be stressed that even in the United States, automation is by
no means the only factor (66)________ people from existing jobs. The increasing number of unneeded
workers in recent years (67)________ the result of much more simple and old-fashioned influences: farm
laborers have been (68)________ out of work by bigger tractors, miners by the cheapness of oil, and
railway - men by better roads. It is quite wrong, (69)________, to think of automation as some new
monsters (70)________ arrival threatens the existence of employment in the same way that the arrival of
myxomatosis threatened the existence of rabbits. Automation is one aspect of (71)________ changes
(changes in tastes, changes in social patterns, changes in organization) which (72)________ in certain
jobs disappearing and certain skills ceasing to be required. And even in America, which has a level of
technology and output per (73)________ much in advance of Britain’s, there is no evidence that the
(74)________ of change is actually speeding up. Nevertheless, changes in the amount of labor needed to
produce a certain output are proceeding fairly rapidly in America – and in (75)________ countries, and
may proceed more rapidly in future. Indeed, it is one of the main objects of economic policy.
66.A. riding B. displacing C. passing D. missing
67.A. has been B. have been C. was D. were
68.A. put B. fit C. set D. dismissed
69.A. however B. therefore C. although D. but
70.A. which B. what C. that D. whose
71.A. technology B. technical C. technological D. technologist
72.A. resides B. ends C. results D. prospects
73.A. human B. head C. unit D. piece
74.A. step B. rush C. pace D. leap
75.A. another B. others C. each D. other
KK. Read the passage and mark the letter A, B, C or D on your answer sheet to indicate the correct
answer to each of the following questions.
The advent of the Internet may be one of the most important technological developments in recent years.
Never before have so many people had access to so many different sources of information. For all of the
Internet’s advantages, however, people are currently becoming aware of some of its drawbacks and are
looking for creative solutions. Among the current problems, which include a general lack of reliability
and numerous security concerns, the most crucial is speed.
First of all, the Internet has grown very quickly. In 1990, only a few academics had ever heard of the
Internet. In 1996, over 50 million people used it. Every year, the number of people with access to the
Internet doubles. The rapid growth has been a problem. The computer systems which run the Internet
have not been able to keep up with the demand. Also, sometimes, a request for information must pass
through many routing computers before the information can be obtained.
A request for information made in Paris might have to go through computers in New York, Los Angeles
and Tokyo in order to obtain the required information. Consequently, the service is often slow and
unpredictable. Service also tends to be worse when the Internet is busiest – during the business day of the
Western Hemisphere – which is also when companies need its service the most.
Some people are trying to harness the power of networked computers in such a way as to avoid this
problem. In 1995, a group of American universities banded together to form what has come to be known
as Internet II. Internet II is a smaller, more specialized system intended for academic use. Since it is more
specialized, fewer users are allowed to access. Consequently, the time required to receive information has
decreased.
Businesses are beginning to explore a possible analogue to Internet II. Many businesses are creating their
own “Intranets”. These are systems that can only be used by the members of the same company. In
theory, fewer users should translate into a faster system. Intranets are very useful for larger national and
international companies whose branches need to share information. Another benefit of an Intranet is an
increased amount of security. Since only company employees have access to the information on the
Intranet, their information is protected from competitors. While there is little doubt that the Internet will
eventually be a fast and reliable service, industry and the academic community has taken their own steps
toward making more practical global network.
76. According to the passage, which of the following statements about the Internet is NOT true?
A. It tends to be unreliable. B. It has created a sense of security.
C. It is sometimes slow to access. D. It has become increasingly less popular.
76. According to the passage, which of the following was TRUE in 1990?
A. The Internet was a secure means to gain information.
B. The Internet experienced enormous growth rates.
C. Internet data proved to be impractical.
D. Few people were using the Internet.
78. According to the author, what is the reason why the Internet is sometimes slow?
a. Phone lines are often too busy with phone calls and fax transmissions to handle the Internet.
b. Most people do not have computers that are fast enough to take advantages of the Internet.
c. Often a request must travel through many computers before it reaches its final destination.
d. Scientists take up too much time on the Internet; thus, they slow it down for everyone else.
79. The word “obtained” in paragraph 2 is closest in meaning to _________.
A. understood B. acquired C. purchased D. distributed
80. The word “harness” in paragraph 4 is closest in meaning to _________.
A. utilize B. disguise C. steal D. block
81. According to the passage, what benefits does Internet II have over the Internet?
a. There is no government intervention regulating Internet II.
b. Small businesses pay higher premiums to access to the Internet.
c. Internet II contains more information than the Internet.
d. Internet II has fewer users; therefore, it is faster to access.
82. The word “analogue” in paragraph 5 most nearly means___________.
A. similarity B. alternative C. use D. solution
83. The word “their” in paragraph 5 refers to __________.
A. competitors B. company employees
C. Intranets D. companies
84, With which of the following conclusions would the author probably agree?
a. Fewer academic communities need to create their own internet systems.
b. Businesses who develop their own Intranets are limiting their information data base.
c. The technology used by the internet creators is too complex for computer owners to understand.
d. An internet system with fewer users would be quicker.
85, All of the following are advantages of “Intranets” mentioned in the passage EXCEPT.
e. They provide a higher level of security.
f. They share information with other company branches.
g. They are cheaper than other alternatives.
h. They move data faster.
D. WRITING
I. Rewrite each of the following sentences so that it has a similar meaning to the original one.
2 His parents made him join the English speaking club.
 He _________________________________________________.
3 “Who ate all the cakes last night?” Mary asked her daughter.
 Mary asked_______________________________________.
88. We couldn’t have finished our project without your help.
A If it ________________________________________________.
10. “You broke my vase of flowers!” Susan said to Jimmy.
A Susan accused ___________________________________.
11. She knows more about it than I do.
A I don’t ____________________________________________.

 Write from 250 – 300 words about the following topic:


As technology advances, traditional cultures will be lost. It seems that we cannot have these two
things together.
To what extent do you agree or disagree?
Give reasons for your answer and include any relevant examples from your own knowledge or
experience.

__________HẾT_________

Cán bộ coi thi không giải thích gì thêm.

Họ và tên thí sinh:……………………………………….………..…….; Số báo danh………………

ĐÁP ÁN

Total: 100/100 points = 10/10 points


A. LISTENING
SECTION I. You will hear a new student, Tom, talking to a student representative called Rachel about
university clubs. You will listen to the talk TWICE. (10pts: 1,0pt/ 1 item)
01. weekend trip(s) 02. competition(s) 03. 125 04. club secretary 05. twice
06. spring 07. weekly 08. equipment 09. magazine 10. Exhibition
SECTION II. You will hear a lecture about climate. Choose the correct letter A, B or C to answer the
questions. You will listen to the talk TWICE. (10pts: 1pts/ 1 item)
11. B 12. B 13. A 14. C 15. C 16. A 17. B 18. A 19. B 20. C

B. GRAMMAR AND VOCABULARY

I. Mark the letter A, B, C, or D on your answer sheet to indicate the correct answer to each of the
following questions. (10pts: 0,5pt/ 1 item)

21. D 22. A 23. C 24. B 25. B 26. D 27. B 28. A 29. C 30. C
31. A 32. C 33. C 34. D 35. C 36. A 37. D 38. B 39. B 40. A

2. Supply the correct form of the word in capital letter. Write your answers on your answer sheet.
(10pts:1,0pt/ 1 item)

41. comfortably 42. tendency 43. counteract 44. ability 45. relaxed

46. speech 47. simply 48. knowledgeably 49. facial 50. informative

3, Find the mistake in each of the following sentences by choosing A, B, C or D. Write your
answers on your answer sheet. (5pts: 1,0pt/ 1 item)

51. B 52. A 53. B 54. C 55. D

C. READING
I. Read the text below and think of the word which best fits each space. Use only ONE word in
each

space. Write your answers on your answer sheet. (10pts: 1,0 pt/ 1 item)

56. than 57. has/ includes 58. last 59. neither 60. as/ because/cause/ since

61. to 62. check 63. talking 64. close 65. the

II. Read the following passage and choose the options among A, B, C or D that best complete the
blanks. Write your answers on your answer sheet. (10pts: 1,0 pt/ 1 item

66. B 67. A 68. A 69. B 70. D 71. C 72. C 73. B 74. C 75. D

III. Read the passage and mark the letter A, B, C or D on your answer sheet to indicate the
correct answer to each of the following questions. (10pts: 1,0 pt/ 1 item)

76. D 77. D 78. C 79. B 80. A 81. D 82. A 83. B 84. D 85. C
D. WRITING
I. Rewrite each of the following sentences so that it has a similar meaning to the original one.
(10pts: 2,0pt/ 1 item)

86. He was made to join the English speaking club.

87. Mary asked her daughter who had eaten all the cakes the previous night/the night before.

88. If it hadn’t been for your help, we couldn’t have finished our project.
89. Susan accused Jimmy of having broken/ breaking her vase of flowers.

90. I don’t know about it as much as she does.

II. Write from 250 – 300 words about the following topic (15 pts)

As technology advances, traditional cultures will be lost. It seems that we cannot have
these two things together.

To what extent do you agree or disagree?

Give reasons for your answer and include any relevant examples from your own
knowledge or experience.
Marking scheme

The impression mark given is based on the following scheme:

Content: 50% of total mark: a provision of all main ideas and details as appropriate

Language: 30% of total mark: a variety of vocabulary and structures appropriate to the level of
English language gifted upper-secondary school students

Presentation: 20% of total mark: coherence, cohesion, and style appropriate to the level of
English language gifted lower-secondary school students.

__________HẾT_________
SỞ GIÁO DỤC VÀ ĐÀO TẠO KỲ THI CHỌN HỌC SINH GIỎI THÀNH PHỐ LỚP 12
HẢI PHÒNG Năm học 2019-2020

ĐỀ CHÍNH THỨC MÔN: TIẾNG ANH - BẢNG KHÔNG CHUYÊN


(Đề thi gồm 04 trang, tổng điểm 100)
Thời gian: 180 phút (không kể thời gian giao đề)
Ngày thi: 19/9/2019
Chú ý: - Thí sinh làm bài vào Phiếu trả lời.
1. Thí sinh không sử dụng bất kỳ tài liệu gì, kể cả từ điển.
2. Giám thị không giải thích gì thêm.

SECTION ONE: LISTENING


Part I. You will hear an interview with Trina Trevose, a pop singer who is only fifteen.
Choose the best answers A, B or C. You will hear the conversation TWICE. Write your answers on the answer sheet. (5 pts)
1, When Trina went to the USA, she ______.

a. thought the records she made would be unsuccessful


b. knew her friends would be jealous of her
c. didn’t tell many people why she was going
2, Trina told ______ what she was doing in the USA.

d. her close friends


e. her relations
f. her classmates
3, When Trina was in the USA, she wrote songs about ______.
g. her home
h. the weather
i. people she met
4. Why did Trina sing with David Pearson?
a. He needed some help.
b. She wrote a song for him.
c. The record company asked her to.
5. How does Trina see her future?
a. She will continue making records in the USA.
b. She may make singing her career eventually.
c. She wants to study music at college.
Part II. You will hear a photographer called Ian Gerrard talking about his career. Complete the sentences with a word or
short phrase. You will hear the recording TWICE. Write your answers on the answer sheet. (5 pts)
Ian - Photographer
The subject that Ian studied at university was (6) …………………. .
Ian did a presentation on (7) …………………. as part of his final year.
Ian worked for a (8) …………………. in the USA after leaving university.
Ian says that (9) …………………. is the season when he takes the best photographs.
When Ian came back to Britain, he travelled around by (10) …………………. taking photographs.

SECTION TWO: PHONOLOGY


I. Choose the word whose underlined part is pronounced differently from that of the others.
Write your answers (A, B, C or D) on the answer sheet. (2.5 pts)
1. A. closure B. measure C. pleasure D. mature
2. A. exhale B. exalt C. exert D. exact
3. A. ghost B. both C. cloth D. sold
4. A. homemaker B. heritage C. honesty D. hurricane
5. A. chauffeur B. chlorine C. moustache D. chivalry
9 Choose the word with a different stress pattern.
Write your answers (A, B, C or D) on the answer sheet. (2.5 pts)
1. A. immunize B. infamous C. astonish D. reference
2. A. rescue B. concern C. creature D. colleague
3. A. compulsory B. scientific C. generation D. residential
4. A. underdone B. undergo C. underpass D. underpay
5. A. climate B. counsel C. govern D. career
SECTION THREE: VOCABULARY AND GRAMMAR
I. Choose the correct word or phrase to complete each sentence. Write your answers (A, B, C or D) on the answer sheet. (5 pts)
1. Hardly ______ with his brother, which makes their parents very happy.

2. Having ______ the table, the girl called her parents and sister for dinner.

3. For centuries, Rome was the ______ power in the Mediterranean.

4. Fish are the most ancient form of vertebrate life, and ______ all other vertebrates.

5. Rudy was ______ with emotion at seeing his team win the championship.

6. - John: “______?”- Margaret: “Oh, It’s great.”


A. How is the English competition B. What do you think of the English competition
C. Would you like the English competition D. What do you like about the English competition
7. Some products are spread by word of ______ and not through the television or radio.

8. After many years as a doctor, Simon had become ______ to scenes of human distress.

9. It’s my job to ______ the budget.


A. weigh B. balance C. add D. raise
10. The soya plant is an annual, _____ new seeds must be planted every year.
A. so B. and since C. while D. such
4 Supply the correct form of the words given in brackets to complete the sentences.
Write your answers on the answer sheet. (5 pts)
1. She always listens ……………………. to what she is told. (ATTEND)
2, Despite his ……………………. lack of concern, I think Gabriel really does care what other people think of him. (APPEAR)
3. John gave all his Christmas presents to charity, which was so ……………………. . (PRAISE)
4. We’d like to do volunteer work in ……………………. remote areas. (GEOGRAPHY)
5. James got into trouble for ……………………. his boss. (PERSON)
6. Astronomers at the ……………………. have announced that they have discovered a new planet. (OBSERVE)
7. They are ……………………. by a desire to help people. (MOTIVE)
8. The very ……………………. of free speech is unknown to them. (CONCEIVE)
9. Confidence is the key to ……………………. in life. (SUCCEED)
10. You must learn to ……………………. between politeness and real kindness. (DIFFERENT)
 Complete each gap in the sentences below with a correct preposition or adverb
particle. Write your answers on the answer sheet. (10 pts)
VI. There are plenty of exceptions …………….. this view in this country, but I myself take it …………….. granted.
VII. I am writing this letter …………….. behalf …………….. the manager.
VIII. Could you put me …………….. for a few days until I find a place …………….. my own?
IX. Colin was convicted …………….. using the firm’s money to pay …………….. his gambling debts.
X. The child has been deprived …………….. affection for so long that he hardly knows how to respond …………….. love.
XI. Damage …………….. the building resulted …………….. unusually high wind.
XII. She feels it is …………….. her to socialise …………….. uneducated people.
XIII. He called …………….. my house …………….. tea last Sunday morning.
XIV. …………….. his way out of the room, he shut the door …………….. him silently so as not to wake up the sleeping child.
XV. Your explanation …………….. how it works is quite …………….. me.
IV. Fill in each gap with a suitable phrasal verb in the correct form to complete the sentences.
Write your answers on the answer sheet. (10 pts)
back down set off stick up for get over hold with
look back on take apart run down come across make for
4. Constant daydreaming doesn’t ……………….…….. a successful career in music.
5. Little children like to …………….. their toys ………… .
6. When we get old, we’ll all ……………….…….. this holiday with great pleasure.
7. While trying to find her book, my mother ……………….…….. some old photographs in a drawer.
8. The fireworks must be stored safely in order to avoid …………….. them ………… accidentally.
9. He is disappointed at not getting the job, but I’m sure he’ll ……………….…….. it soon.
10. When the critics attacked her work, Jane ……………….…….. it vigorously.
11. I am afraid I don’t ……………….…….. this kind of thing.
12. Sheila was right, so Paul had to ……………….…….. .
13. It is highly recommended that you replace the batteries before they completely ……………….…….. .

V. Identify one of the four underlined parts in each sentence that is not correct, then get it right.
Write your choices (A, B, C or D) and your corrections on the answer sheet. (5 pts)
1. If he had laid quietly under the tree as he had been instructed to do, they wouldn’t have found him.
A B C D
2. The crises in the countries of the Middle East has been discussed at our weekly forums.
A B C D
3. Energy exists in different forms, such as light, heat, and chemical, mechanic, and electrical energy.
A B C D
4. So far most a million different species of insects have been identified and named.
A B C
5. The giant panda eats chiefly mainly bamboo shoots, though it also eats some other plants and occasionally feeds on fish.
A B C D
SECTION FOUR: READING COMPREHENSION
I. Read the following passage and choose the correct answer to each question.
Write your answers on the answer sheet. (10 pts)
Two major avenues are being pursued by biomedical scientists in the fight against HIV infection and AIDS. One strategy is to
develop a vaccine that can induce neutralizing antibodies against HIV and protect uninfected individuals exposed to virus. The
second approach involves the discovery and development of therapeutic agents against HIV infection and AIDS. At present no
vaccine exists to protect against infection but the testing of a number of candidate vaccines in human volunteers has been taking
place since the early 1990s. Progress is also being made in the treatment of HIV infection. The focus has been on two major
areas: anti-viral drugs with a direct effect against the causative agent and immunomodulators - substances that act to reconstitute
or enhance immune-system function. Efforts to develop and improve treatments of specific opportunistic infections and tumors
continue, and more new drugs have been approved. However, unlike most known disease-producing micro-organisms, HIV
infects the very cells that are intended to lead the immune system’s attack against invaders. This makes it technically very hard to
come up with an anti-HIV vaccine that will kill the virus without destroying the already threatened immune system. In the final
analysis, in the absence of an effective vaccine or therapy, risk reduction remains the most powerful tool in the fight against AIDS.
I. What is the main topic of the passage?
A. The victims of AIDS B. HIV prevention and therapy C. The work of
biomedical scientists D. The immune system
2. What are the objectives of biomedical scientists combating HIV and AIDS?
A. To visit two places B. Strategies to research AIDS
C. Prevention and cure D. The inclusion of anti-bodies and development of therapies
3. What could best replace “induce” in line 2?
4. How are prospective vaccines tested?
A. With candidate mutations B. In early phases
C. On human guinea pigs D. On volunteer viruses
5. What could best replace “candidate” in line 4?
6. What are the main research directions into the treatment of HIV?
A. Anti-viral drugs and immunomodulators B. Improved treatment and varying substances
C. Specific opportunistic infections D. Infections and tumors
7. What is the function of immunomodulators?
A. To prevent the causative agents of AIDS B. To retain and enclose the immune system
C. To prepare anti-viral drugs D. To repair and strengthen the immune system
8. In what way is HIV different from other disease-producing micro-organisms?
A. It infects many cells. B. It attacks the body’s means of resistance.
C. It presents as a scientific challenge. D. It leads the attack against invading micro-organisms.
9. What is hindering the creation of an anti-HIV vaccine?
A. The number of candidate vaccines B. Neutralizing antibodies
C. A lack of vaccine D. The vulnerability of the immune system
10. According to the passage, what is the best means to combat AIDS?
A. Taking risks B. Anti-viral and immune-enhancement therapies
C. Reducing potential risks D. Vaccines that can induce neutralizing antibodies

II. Read the following passage and choose the correct word to fill each gap.
Write your answers (A, B, C or D) on the answer sheet. (10 pts)

HOW TO HAVE PERFECT POSTURE

Next time you take a look in the mirror, try standing sideways on. Do you see a long elegant back or are your shoulders as rounded
as the hump of a camel? Bad posture normally starts in our teenage years when we slouch around with our mates trying to look
cool. But, hands up those who now while away their days sitting in front of a computer where it's (1) ______ too easy to

neglect something as essential as good posture.

As we grow older these poor (2) ______ are exacerbated, but by taking (3) ______ immediately we can not only look and feel

better but also protect our body for the future. Make sure you sleep on a firm mattress at night. (4) ______ lying flat on your back;
instead try sleeping in a curled position on your side. The (5) ______ revival in platform shoes doesn't help and wearing shoes
with heels more than (6) ______ centimetres high is just asking for (7) ______. Don't load everything into a bag that you wear on

one shoulder or you'll end up lopsided.

(8) ______ the weight evenly by wearing a rucksack or even a bag around your waist. When you get home from a day's studying or
work, it's all too (9) ______ to slump on the sofa in front of the telly. Sitting in this way (10) ______ feel comfortable but it prevents
you from breathing properly. Try to sit upright with the stomach pulled firmly in.

1. A. so B. quite C. all D. rather


2. A. customs B. habits C. rituals D. practices
3. A. course B. something C. moves D. action
4. A. Evade B. Prevent C. Avoid D. Restrain
5. A. existing B. topical C. contemporary D. current
6. A. various B. several C. considerable D. numerous
7. A. concern B. problem C. trouble D. fuss
8. A. Scatter B. Circulate C. Disseminate D. Distribute
9. A. enticing B. attractive C. irresistible D. tempting
10. A. may B. could C. will D. should

III. Fill each gap in the following passage with ONE suitable word. Write your answers on the answer sheet. (10 pts)

TEENAGERS AND TELEVISION

Until very recently, teenagers (1) ______ been hooked on television. Parents have worried that their children are becoming fat,

lazy (2) ______ potatoes, and teenagers seem to have preferred watching TV (3) ______ almost any other activity in the home,
except perhaps sleeping. But no more! According to the latest statistics, teenagers have (4) ______ off TV and are turning off in
droves. Given the choice (5) ______ TV and the Internet, it’s clear what most teens prefer. The Internet (6) ______ an interactive,
social need that TV doesn’t. Teenagers at a loose (7) ______ in their bedrooms can hang (8) ______ with their mates in
cyberspace. As websites such as MySpace have (9) ______ off, teenagers have been only (10) ______ eager to join in their
millions and spend hours a day – and night – online. We’re witnessing the birth of the generation of the “keyboard potato”, for want
of a better expression.

SECTION FIVE: WRITING

I. Finish each of the sentences below in such a way that it means the same as the original one.
Write your answers on the answer sheet. (5 pts)

1. Andrew had difficulty in watching the film on a small tablet screen.


Andrew found ....................................................................................................................................................................

2. We were lucky to find a very good restaurant to have lunch.

By a stroke ........................................................................................................................................................................

3. “Have you been feeling less stressed?” Kim’s boss asked him.
Kim’s boss asked him .......................................................................................................................................................

4. She herself admitted to being rather selfish.


On her own .......................................................................................................................................................................

5. The police believe that the thief is still in the house.


→ The thief ............................................................................................................................................................................

II. Rewrite each of the sentences below in such a way that its meaning stays the same, using the word given in
capitals. Do NOT change this word in any way. Write your answers on the answer sheet. (5 pts)

1. The last time I saw my family was four years ago. (SEEN)
................................................................................................................................................................................................
2. This problem is certainly not so difficult as it appears. (MEANS)
................................................................................................................................................................................................
3. A famous architect designed Tom’s house. (HAD)
................................................................................................................................................................................................
4. Harry’s job is to supervise all the employees of the company. (RESPONSIBLE)
................................................................................................................................................................................................
5. The warmth of her welcome surprised me. (ABACK)
................................................................................................................................................................................................

III. In about 150 words, write a paragraph about the effects of Facebook on high school students’ lives. (10 pts)
T
H
E

E
N
D
SỞ GD - ĐT HẢI DƯƠNG KÌ THI CHỌN HỌC SINH GIỎI TỈNH LỚP 12 THPT
NĂM HỌC 2019 - 2020
ĐỀ CHÍNH THỨC MÔN THI: TIẾNG ANH
Thời gian làm bài: 180 phút (Không tính thời gian giao đề)
(Đề thi có 08 trang)

Chú ý: Thí sinh làm bài vào tờ giấy thi


Phần trắc nghiệm: Chỉ cần viết đáp án A, hoặc B, C, D
Phần tự luận: Viết đầy đủ theo yêu cầu của bài
(Thí sinh không được sử dụng bất cứ tài liệu gì)

A. LISTENING:
Hướng dẫn làm bài Nghe:
Bài Nghe gồm có 04 phần, mỗi phần thí sinh được nghe 02 lần.
Mở đầu và kết thúc phần nghe có tín hiệu nhạc.
Mọi hướng dẫn cho thí sinh (bằng tiếng Anh) có trong bài.

I. Listen to five short conversations and choose the correct answer to each question (5.0 points).
1. How will they book their flights?
A. on the Internet B. at the travel agent C. on the phone
2. What has the daughter forgotten to bring on holiday?
A. toothpaste B. hairbrush C. shampoo
3. What will the man and the woman do on Sunday?
A. do the housework B. go on a picnic C. watch DVDs and read newspapers
4. Which blouse does the girl decide to buy?
A. a collar, long sleeves B. no collar, short sleeves C. a collar, without sleeves
5. When is the girl having a party?
A. 15th B. 14th C. 13th

6. Listen to the recording and choose the correct answer to each question (6.0
points). 6. What made Holly start writing when she was young?
a. keeping a diary each day
b. wanting to remember her dreams
c. completing a homework project
7. Holly tells the story about the starfish to show ______.
a. how to enjoy wildlife without hurting it
b. how to keep wild animals at home
c. how to find the most interesting animals in the wild`
8. What does Holly think is the most interesting thing about nature?
a. that animals live so close to us.
b. that nature is so beautiful.
c. that there is so much to learn.
9. What does Holly say about trees?
a. They have lots of uses.
b. They are often unnoticed.
c. They are everywhere.
10. Holly hopes that her young readers ______.
a. go on to study nature at college or university
b. improve their genneral reading skills
c. learn how to discover nature for themselves
11. What kind of books does Holly enjoy reading in her spare time?
A. classic novels B. cookery books C. field guides

III. Listen to the recording and fill in each blank with NO MORE THAN 3 WORDS / NUMBERS (5.0 points).

SUNNINGTON SPORTS CAMP

Choice of afternoon activity:


Sign list outside the (12) ______
Clothes:
Wear track suit, but also bring shorts and a T-shirt
(13) ______ sports shoes
Food:
Lunch served in canteen every half-hour bewteen 12.15 and (14) ______
Snack bar sells drinks, chocolate and biscuits
Certificate:
Marks given for:
12. Attitude: (15) ______ and team-work
13. Performance: strength, speeds and skill
14. Level six certificate to anyone who gets (16) ______ or more.

IV. Listen to the recording and state whether the statements are true (T) or false (F) (4.0 points).
T F
17. Maria would like to travel a lot as part of her job.
18. Maria is confident she will be able to work for an airline.
19. Rolf intends to do a job connected with his degree.
20. Maria and Rolf agree it is important to have a good salary

B. GRAMMAR – VOCABULARY – LANGUAGE FUNCTIONS


I. Choose the word, phrase or expression which best completes each sentence (8.0 points).
21. The answer Henry gave was so confusing that his lecturer could hardly make ______ of it at all.

22. Considered America’s first great architect, ______.


a. many of the buildings at Harvard University were designed by Henry Hobson Richardson
b. it was Henry Hobson Richardson who designed many of the buildings at Harvard University
c. Henry Hobson Richardson designed many of the buildings at Harvard University
d. Harvard University has many buildings that were designed by Henry Hobson Richardson.
23. “Jack, can you help me push the piano to the corner of the hall to ______ our party?” said the teacher
A. give place to B. make place for C. take room for D. make room for
24. The surrealistic movement in art in the 1920s and 1930s placed ______ is pictured in the unconscious and
often incorporated dreamlike images.
A. to emphasize it B. an emphasis on what
C. an emphasis on it D. emphasize what
25. “Frankly, Ms. Adamson works ______ that her figures never need ______,” said the General Manager.
A. such efficiently that / to check B. so efficient that / checking
C. such an efficient that / to be checked D. so efficiently that / to be checked
26. It is highly recommended that language learners in general and English learners ______ should learn the
language in the native community so as to master it.
A. in particular B. on the whole C. in short D. on the other hand
27. Lama: “What’s your greatest phobia, Carolina?
Carolina: “______”
A. I’m afraid not. B. I haven’t made up my mind.
C. What an absurd idea! D. Worms, definitely.
28. Waiter: “How would you like your steak, sir?
Man: “______”
A. Not too bad B. Very good C. Rare, please D. Yes, of course I like it

II. Give the correct form of the words in brackets (5.0 points).
29. The (PREDICT) ______ weather in the mountains can make climbing extremely hazardous.
30. Both sides have shown a distinct lack of (ENTHUSIAST) ______ for discussion.
31. The woman tried to break into a security file for which she was not (AUTHORITY) ______.
32. Life (EXPECT) ______ for both men and women has improved greatly in the past 50 years.
33. Julie Johnson and Mark Thomas, aged 19 and 20 (RESPECT) ______, are taking part in the competition.

III. Choose the underlined word or phrase in each sentence that needs correcting (3.0 points).
34. The American architect Frank Lloyd developed a theory of architecture stressed the needs of the people who
A B
used it.

35. Children subjected to violence, exploration, abuse, and neglect are in risk of death, poor physical and
A B C
Mental health, HIV/AIDS infection, and educational problems.
D
36. The works of early American woodcarvers had many artistic qualities, but these craftsmen probably did
A B C
Not think of them as artists.
D
IV. Choose the word that is CLOSEST in meaning to the underlined word in the following sentences
(2.0 points).
37. Sarah stood there, looking at his boyfriend without showing any reaction when he told her the
news.

38. Martha decided to remain celibate and devoted her life to helping the homeless and orphans.
A. married B. separated C. single D. divorced

V. Choose the word that is OPPOSITE in meaning to the underlined word in the following sentences
(2.0 points).
39. Written sources are considered absolutely indispensable for today’s history teaching.

40. The court concluded from the evidence that Mr. Smith was innocent and released him from prison.
A. guilty B. benevolent C. innovative D. naive
C. READING:
I. There are FIVE blanks in the passage below. From the phrases / sentences in the box, choose the
most suitable for each blank. There is ONE EXTRA option that you DO NOT need to use (5.0 points)

A. by attracting as many as several hundred thousand international students to their countries


B. so are not classed as international students
C. as defined by the Immigration Control and Refugee Recognition Act
D. in accordance to its own national education system
E. preferred by international students are the United States, United Kingdom, Germany, France, Canada and Australia
F. by sending hundreds of thousand students to study abroad

The definition of "international student" varies in each country (41) ______. In Australia, international
students are defined as those studying onshore only with visa subclasses 570 to 575, excluding students on Australian-
funded scholarships or sponsorship or students undertaking study while in possession of other temporary visas. New
Zealand citizens do not require a visa to study in Australia, (42) ______. In Japan, international students
are defined as foreign nationals who study at any Japanese university, graduate school, junior college, college of
technology, professional training college or university preparatory course on a “college student” visa, (43) ______.
According to the United Nations Educational, Scientific and Cultural Organization (UNESCO) in their 2009 World
Conference on Higher Education report, over 2.5 million students were studying outside their home country.
UNESCO also predicted that the number of international students might rise to approximately 7 million by the year
2020. The main destinations (44) ______. Overall, the number of international students more than doubled to over
2 million between 2000 and 2007.
However, the sharpest percentage increases of international students have occurred in New Zealand, Korea, the
Netherlands, Greece, Spain, Italy and Ireland. In recent years, some Asian and Middle East countries have started
to attract more international students. These regions have entered the market with declared ambitions to become
regional education centers (45) ______.

Choose the most suitable heading from the box for each paragraph below. There is ONE EXTRA
heading that you DO NOT need to use (5.0 points).

HEADINGS
A. The amount that American eat
B. Americans even getting fatter
C. Changing meal times everywhere
D. Why Americans love fast food
E. Unhealthy American schools
F. How American schools deal with obesity among teenagers

46. ______
In the past year, the adult obesity rate rose in 48 of America’s 52 states. Despite the campaigns to make people
aware of the dangers of over-eating, about 119 million, or 64% of US adults are either overweight or obese.
Worryingly, this figure is predicted to rise to 73% by the end of this year. What does this mean for America?
Already 300,000 deaths each year are caused by obesity (only smoke kills more people), and the annual cost to the
country is around $100 billion.
47. ______
Many people blame for the fast food industry, along with sedentary life styles, for the worrying increase. The
average American now consumes about three hamburgers and four portions of French fries every week. That’s 90
grams of fat and 2.250 calories (the average person needs about 2.000 calories a day). A recent survey found that
every month 90% of America children eat at least one meal in fast food restaurant.

48. ______
Most Americans know that fast food isn’t good for them. It is high in calories, fat, salt and sugar. Yet fast-food
restaurants are part of American life. After World War II, higher salaries and productivity, together with
technological developments and increased consumerism, made fast food restaurants popular. It presented a
modern lifestyle, and is still a popular place for teenagers to get together with friends. The food is cheap, and many
say they love the taste. The advertising is often aimed at children, with offers of entertainment and free gifts.

49. ______
Another in fast food sales – not only in the USA, but all over the world – is our increasingly busy lifestyles. In the
past, families ate a home-cooked meal together at dinner time. These days, meals can be eaten at any hour of the
day, and people rely on easy, instant food – not always nutritious – that is available “24/7”. America invented the
“TV dinner”, and it is something most people around the world now enjoy.

50. ______
Finding nutritious food is also difficult in schools, which is often have contracts with fast food supplies and drinks
companies. Teenage boys in the US each drink an average of 868 cans of fizzy drinks per year. Exercise in school
has become less important too. A recent report showed that half of all US teens aged 12 – 21 got only America’s
problem – Europe is also getting faster. It is estimated that over one million children in the UK are obese and this
number has trebled over the past 20 years.

III. Read the passage below and decide which option A, B, C or D best fits each space (10 points).
Childhood is the time when there are (51) ______ responsibilities to make life difficult. If a child has good
parents, he is fed, looked after and loved, (52) ______ he may do. It is improbable that he will ever again in his life
be given so much without having to do anything in return. In (53) ______, life is always presenting new things to
the child - things that have lost their interest for older people because they are too well-known. A child finds
pleasure playing in the rain, or in the snow. His first visit to the seaside is a marvellous adventure. (54) ______, a
child has his pain. He is not so free to do as he wishes as he thinks older people are: he is continually told not to do
things, or being punished (55) ______ what he has done wrong. His life is therefore not perfectly happy.
When a young man starts to (56) ______ his own living, he becomes free from the discipline of school and
parents, but at the same time he is forced to accept responsibilities. He can no longer expect (57) ______ to pay
for his food, his clothes and his room, but he has to work if he wants to live comfortably. If he spends most time
playing about in a day he used to as a child, he will be hungry. And if he breaks the laws of society as he used to
break the laws of his parents, he may go to prison. If, however, he works hard, keeps out of troubles and has good
health, he can have the greatest happiness of seeing himself make steady (58) ______ in his job and of building up
for himself his own position in society.
Old age has always been thought of as the worst age to be, but it is not necessary for the old to be unhappy. With
old age (59) ______ wisdom and the ability to help others with advice wisely given. The old can
have the joy of seeing their children making progress in life: they can watch their grandchildren growing up around
them, and perhaps best of all, they can, if their life has been a useful one, feel the happiness of having come (60)
______ the battle of life safely and of having reached a time when they can lie back and rest, leaving others to
continue the fight.
51. A. little B. few C. a little D. a few
52. A. whenever B. wherever C. whatever D. whosoever
53. A. short B. term C. reality D. addition
54. A. However B. But C. Therefore D. Moreover
55. A. because B. for C. at D. by
56. A. take B. have C. create D. earn
57. A. other B. another C. others D. someone
58. A. progress B. achievement C. improvements D. accomplishments
59. A. coming B. come C. came D. had come
60. A. out B. across C. through D. back

IV. Read the following passage and choose the best answer to each question (10 points).

Paul Bunyan is perhaps America’s best-known folk hero. A fictional logger of incredible strength, he was most
likely based on an actual nineteenth-century logger from the northern United States or Canada. As a folk hero, he
struck a chord with Americans on some level, perhaps because
Line he was incredibly strong but also because he was hard-working and capable, ingenious in solving
10. problems, and fun-loving.
Though there is evidence that Paul Bunyan tales were part of oral tradition in the nineteen century, Paul Bunyan
stories did not appear in written form until the early twentieth century. Journalist James McGillivray included
descriptions of Bunyan in a series of essay entitled “The Round River Drive” which appeared in a number of
Midwestern newspapers between 1906 and 1910.
JJ. However, it was through an extensive advertising campaign that Paul Bunyan moved solidly into print.
Recognizing the appeal of Paul Bunyan as a figure for his company’s advertising, William
Laughead, an advertising executive for the Red River Lumber Company, initiated a campaign that consisted of a
series of publications featuring Paul Bunyan. For several decades, the company distributed these publications free
of charge and made no attempt to obtain a copyright on them. In
(9) fact, the company vigorously encouraged other writers to make use of Paul Bunyan because it felt
that the use of this character enhanced the name recognition of the Red River Lumber Company
inasmuch as the name of the folk hero and the name of the company had become interwoven.
The Bunyan stories published by Red River and further circulated by others were tall tales of gigantic proportions.
In these tales, Banyan is depicted as a man of superhuman proportions, who is
JJJ. strong, hard-working, entrepreneurial, and innovative. In one story, for example, Paul is credited with
digging the Great Lakes in order to create a watering hole for his giant ox, Babe. In another of these
tales, Paul caused an entire winter of blue snow to fall by swearing a blue streak after he injured
himself by smashing his thumb with a large hammer. A third story in the series describes Paul’s role in
establishing the Mississippi River.
(25) Fascination with Paul Bunyan has continued to grow, and today he is a standard of American folklores.
The prevalence of Bunyan as a figure of folklore today is evidenced by references to him in countless stories,
cartoons, poems, and songs as well as the numerous community festivals and logging competitions featuring Paul
Bunyan that can be found throughout the sections of the country where logging has strong tradition.
61. The purpose of this passage is to ______.
A. present the actual feats of a real-life logger
B. provide an overview of American folktales
C. describe logging in North America
D. discuss a “larger than life” folk hero
62. It is NOT stated in the passage that Paul Bunyan is known for his ______.

A. unusual strength B. dedication to work C. ingenuity in difficult situations D. serious nature


63. The passage states that Paul Bunyan tales first appeared ______.
A. in oral stories B. in advertising C. in newspapers D. in a series of essays
64. Which of the following CANNOT be inferred about the Red River Lumber Company’s advertising campaign
featuring Paul Bunyan?
A. It endured for quite a time.
B. The company did not protect its ownership of the stories.
C. The campaign did little to enhance the company’s profitability.
D. The company wanted the name Paul Bunyan to be known as widely as possible.
65. The pronoun “them” in line 14 refers to ______.

66. The word “interwoven” in line 17 could be best replaced by ______.

67. Where in the passage does the author discuss a weather phenomenon that Paul Bunyan supposedly caused?

68. The word “countless” in line 27 could be best replaced by the expression ______.

A. an overestimated number of B. an insubstantial number of C. a large number of D. a specified number of

69. Which paragraph describes the plots of some of the tales of Paul Bunyan?
A. The second paragraph B. The third paragraph C. The fourth paragraph D. The fifth paragraph
70. The author’s tone in this passage is ______.
A. humorous B. neutral C. sarcastic D. pessimistic

D. WRITING:
I. Finish each of the following sentences in such a way that it means exactly the same as the
sentence printed before (5.0 points).
71. Something must be done to help the victims of the earthquake in Mexico.
 Urgent …………………………………………………………………………………………………………………..……………………………………………………………………………………………
72. It was careless of you to allow your 16-year-old son to drive your car.
 You should ………………………………………………………………………………………………………………….……………………………………………………………………………………
73. I shall never lend John any money, no matter what happens.
 Under no ………………………………………………….…………………………………………………………………….…………………………………………………………………………………
74. Someone has suggested abolishing income tax.
 It …………………………………………………………………………………………….………………………….………………………………………….……………………………………………….……
75. Thank you for reminding me about the timetable, otherwise I would have missed the last train.
 If……………………..……………………………………………………………………….…………………………………………………………………….……………………………………………….……

II. This is a part of a letter that you have received from David, a pen friend from New Zealand. In
about 100 - 120 words, write a letter to answer his questions (10 points).

I cannot decide whether I should go to Vietnam to work after my graduation.


Can you give me some advice?
You are required to begin and finish your letter as followed:
Dear David,
……………………………………………………………………………………………………………………………………………..…………………………………………………………………………

……………………………………………………………………………………………………………………………………………..…………………………………………………………………………

……………………………………………………………………………………………………………………………………………..…………………………………………………………………………

Yours,
Tran Thuy Hanh

III. Vietnamese society has changed significantly in the past decades. However, many traditional
values and practices have been lost, which is a pity. To what extent do you agree or disagree with
this statement? In about 230 - 250 words, write an essay to support your opinion (15 points).

=== THE END ===


ĐÁP ÁN VÀ BIỂU ĐIỂM
A. LISTENING:
I. (5.0 points): 1.0 point for each correct answer:
1. B 2. C 3. B 4. A 5. C
II. (6.0 points): 1.0 point for each correct answer:
6. A 7. A 8. C 9. B 10. C 11. C
III. (5.0 points): 1.0 point for each correct answer:
12. changing room 13. Two / 2 pairs of
14. 1.45 15. effort 16. 180 marks
IV. (4.0 points): 1.0 point for each correct answer:
17. T 18. F 19. F 20. T
B. GRAMMAR – VOCABULARY – LANGUAGE FUNCTIONS
I. (8.0 points): 1.0 point for each correct answer:
21. A 22. C 23. D 24. B
25. D 26. A 27. D 28. C
II. (5.0 points): 1.0 point for each correct answer:
29. UNPREDICTABLE 30. ENTHUSIASM
31. AUTHORIZED / SED 32. EXPECTANCY 33. RESPECTIVELY
III. (3.0 points): 1.0 point for each correct answer:
34. C 35. C 36. D
IV. (2.0 points): 1.0 point for each correct answer:
37. B 38. C
V. (2.0 points): 1.0 point for each correct answer:
39. B 40. A
C. READING:
I. (5 points): 1.0 point for each correct answer:
41. D 42. B 43. C 44. E 45. A
II. (5 points): 1.0 point for each correct answer:
46. B 47. A 48. D 49. C 50. E
III. (10 points): 1.0 point for each correct answer:
51. B 52. C 53. D 54. A 55. B
56. D 57. C 58. A 59. B 60. C
IV. (10 points): 1.0 point for each correct answer:
61. D 62. D 63. A 64. C 65. A
66. B 67. C 68. C 69. C 70. B
D. WRITING:
I. (5.0 points): 1.0 point for each correct answer:
71. Urgent actions must be taken to help the victims of the earthquake in Mexico.
72. You should not have allowed your 16-year-old son to drive your car.
73. Under no circumstances will / shall I lend John any money.
74. It has been suggested that income tax (should) be abolished.
75. If you had not reminded me about the timetable, I would have missed the last train.
II. (10 points):
1. Form:
- Informal letter (1.0 point)
2. Task fulfillment:
- Task completed with relevant information (2.0 points)
- Well-organized (2.0 point)
3. Language:
- Accurate grammar (2.0 points)
- Appropriate vocabulary (2.0 points)
- Correct spelling and punctuation (1.0 point)
III. (15 points):
Score of 14-15: An essay at this level:
 shows very effective writing skills
 is very well organized and well developed
 uses details clearly and properly to support a thesis or illustrate ideas
 displays consistent ability in the use of language
 demonstrates variety in sentence structure and proper word choice
Score of 11 - 13: An essay at this level:
 shows effective writing skills
 is well organized and well developed
 uses details clearly and properly to support a thesis or illustrate ideas
 displays consistent ability in the use of language
 demonstrates variety in sentence structure and proper word choice
Score of 9 - 10: An essay at this level:
 may address some parts of the task more effectively than others
 is generally well organized and developed
 uses details to support a thesis or illustrate an idea
 displays ability in the use of the language
 shows some variety in sentence structure and range of vocabulary
Score of 7 - 8: An essay at this level:
 addresses the writing topic adequately but does not meet all of the goals of the task
 is adequately organized and developed
 uses some details to support a thesis or illustrate an idea
 shows adequate but possibly inconsistent ability with sentence structure
 may contain some usage errors that make the meaning unclear
Score of 5 - 6: An essay at this level may reveal one or more of the following weaknesses:
 inadequate organization or development
 poor choice of details or does not provide enough details to support or illustrate generalizations
 a noticeably improper choice of words or word forms
 numerous errors in sentence structure and/or usage
Score of 3 - 4: An essay at this level is seriously flawed by one or more of the following weaknesses:
 serious disorganization or underdevelopment
 little or no detail, or irrelevant specifics
 serious and frequent errors in sentence structure or usage
 serious problems with focus

 may be incoherent
 may be undeveloped
 may contain severe and persistent writing errors
Score of 0: An essay will be rated 0 if it:
 contains no response
 merely copies the topic
 is off-topic, written in a foreign language or consists only of keystroke characters
SỞ GD&ĐT VĨNH PHÚC KÌ THI KSCL ĐỘI TUYỂN HỌC SINH GIỎI KHỐI 12
TRƯỜNG THPT YÊN LẠC 2 ĐỀ THI MÔN TIẾNG ANH
—————— NĂM HỌC 2019 - 2020
Thời gian làm bài 180 phút, không kể thời gian giao đề.
Đề thi gồm: 07 trang.
———————
Giám thị coi thi không giải thích gì thêm.
Học sinh làm bài vào giấy thi.
A. LISTENING

Part 1: Listen and complete the passage below. Write NO MORE THAN THREE WORDS OR A
NUMBER for each answer.
Korean parents love to send their children abroad to study. They believe the benefit to
the child is worth all the (1)__________. Critics say it is not. They say the

Korean is good enough. It is a system that already provides English language to study.
There are also many English (2)__________in Korea. They are a good way for children

to learn English. Native speakers teach students English games and songs. These camps
are set up to be (3)___________.
Despite the fact that most students are staying in Korea, studying abroad is a

(4)___________. There were about 94,000 Korean students in America, according to the
2006 Asian Migration Atlas. Though it is popular, there are some risks for students
(5)__________. Parents are concerned about the (6)__________of their children.
Another worry is that families will be split up. Children and their mothers often move to
English speaking countries together. The father is then (7)__________. He is called

"goose dad." He must migrate like a goose to see his family. Also, Korean students often
meet other Koreans while overseas. They fall into the habit of only speaking Korean, and let
their chance to learn English slip through their fingers.

Critics also say overseas education costs too much money. This may be true for many
families. For example, a home stay in Canada in 2010 cost (8)_________Canadian

dollars. However, a home stay is a popular choice for Korean parents. Their children get the
added benefit of living with a local family. They take part in (9)__________. The
children also brush up on their language skills. This experience is valuable because they also learn
the culture.

Yes, there are problems with studying overseas. However, education in Korea is very important. In
spite of the costs and concerns, parents will always make (10)_________ for their children's
education.
Part 2: Listen to a conversation between Marcus and Catherine about their homework. Decide
the statements are True (T) or False (F). Write your answers in the corresponding numbered
boxes .

Statements True False

1. Catherine finds it hard to understand why Marcus has so much homework.

2. Catherine thinks visiting the museum was a good experience for Marcus.

3. Catherine offers to show Marcus the math homework she has already done.

4. Marcus worries that the teacher might be angry if Catherine helps him.

5. After talking to Catherine, Marcus feels more confident about his


homework.

Part 3: You will hear a radio interview with a man called Robin Marshall, who has written a
book about Argentina. Listen and answer questions.

1. What was Robin’s job in Argentina?


A. translator B. tour guide C. travelling salesman
2. On Robin’s last trip to Argentina, the weather was ______.
a. colder than he expected
b. suitable for what he planned
c. different from the forecasts he heard
3. How did Robin feel during the dance performance he saw?
a. He wanted to get up and dance.
b. He wished he had continued his dance classes.
c. He was sad he didn’t dance well.
4. What did Robin do while he stayed in the village?
A: He went on a bus tour. B. He went into the forest. C. He went on a river trip.
5. What did Robin like about his favourite place?
A. the wildlife B. the views C. the peace
B. PHONETICS
Pick out the word whose stress pattern is different from that of the other words. Identify your
answer by circling the corresponding letter A, B, C or D.
1.A. important B. accident C. candidate D. institute
2.A. diversity B. biography C. economy D. fundamental
3.A. generous B. suspicious C. constancy D. sympathy
4.A. understand B. appearance C. remember D. composer
5.A. communicate B. independent C. university D. entertainment
C. LEXICO-GRAMMAR

Part 1. Complete the following sentences by choosing the correct answer A, B, C or D.


1.I remember ___________ my mother said the grass in the garden needed _________.
A. to hear/cutting B. hear/cut C. heard/to cut D. hearing/ cutting
2.You gave me a lift yesterday; otherwise I__________ late for school.
A. would be B. wouldn't be C. would have been D. wouldn't have been
3.Under the table_____________ that has lived with him for 20 years.
A. his dog sits B. does his dog sit C. sit his dog D. sits his dog
4. She is a kind of woman who does not care much of work but generally _____________ only
with colleagues for meals, movies or late nights at a club.
A. supposes B. socializes C. attends D. discusses
5. I read the contract again and again __________ avoiding making spelling mistakes.
A. in view of B. in terms of C. with a view to D. by means of
6, ________ as a masterpiece, a work of art must transcend the ideals of the period in which it was
created.
A. Ranking B. To be ranked C. Being ranked D. In order to be ranking
7.These ___________ boots belong to Jim.
A. leather red riding B. red leather riding C. leather riding red D. red riding leather
8.A: “Excuse me, can you tell me the way to the post office?”
B: “Sorry, I’m new here.”
A: “________”
A. You’re welcome B. Bad luck C. That’s true D. Thank you all the same
9. Teacher: “Self-studying is an inseparable part of effective learning.”
Student: “____________”
A. I’ll say B. Good job! C. By all means D. I got a clue.
10. It ___________without saying that winners never quit and quitters never win.
A. comes B. means C. is D. goes

Part 2. Supply the correct form of the word provided to the right of each blank.
Education is more important today than ever before. It helps people acquire the skills they need for
such everyday (1. act)_______ as reading a newspaper or managing their money.
It also gives them the (2. special)_______ training they may need to prepare for a job or career.
For example, a person must meet certain (3. educate) _______ requirements and obtain a license
or certificate before he can practice law or medicine. Many fields, like computer operation or police
work, require (4. satisfy) _______ completion of special training courses.

Education is also important because it helps people get more out of life. It increases their (5. know)
_______ and understanding of the world. It helps them acquire the skills that make
life more interesting and (6. enjoy)_______ such as the skills needed to participate in a sport,
paint a picture, or play a (7. music) _______ instrument. Such education becomes (8.
increase)_______ important as people gain more and more leisure time.
Education also helps people adjust to change. This habit has become necessary because social
changes today take place with increasing speed and (9. effect)_______ the lives of more
and more people. Education can help a person understand these changes and provide him with the
skills for (10. adjust) _______ to them.

Part 3: There are 10 errors in the following passage. Underline the mistakes and correct them
in the space provided.
Example: 0. was  is
It was the human factor that contributes to the absolute majority of road accidents which involves
the tremendous toll of fatalities each year. Other, less decisive, causes are vehicle malfunctions or
road shortcomings.
Speeding motorists are notorious for failing to give way at junctions, misjudging the situation on the
road or being unable to accurate estimate the distance while overtaking the “snail pacers” ahead.
Drinkers whom settle behind the wheel after one glass or two may be running the risk of causing a
tragedy through their impaired perception, which is not such rare a case again.
Unfortunately, it is much simpler to introduce the necessary alterations in the traffic system that
change the behavioral patterns of drivers. There are voices that more severe disciplinary resolutions
ought to be put into practice if the vehicle users are to benefit with the greater security on the road.
The idea of producing safe road users through pre-school parental instructions or through
incorporating the safety regulations into school curriculum have been widely acclaimed in many
communities and is expected yielding the required results as the first step in bettering the
qualifications of the future drivers and acquainting them with the potential hazards that may raise on
the road.
1.
2.

D. READING COMPREHENSION

Part 1. Read the following passage and use ONE word to fill in each gap.
THE BLOOD MOON
A lunar eclipse occurs (1) ________the Earth’s shadow falls directly onto the moon.
5. ________ happens as a result is that the moon stops being a silvery white colour and
turns coppery red instead; the sort of colour usually only seen in the sky at dawn or
sunset. An astronaut on the moon, looking towards Earth (3) ________ a lunar eclipse,
would see a black
disc, surrounded (4) ________ a bright red ring. It’s the light from this red ring which is
reflected back to the dark Earth from the moon’s surface. In ancient times, long (5) ________
any of this was understood, the lunar eclipse was known (6) ________a blood moon and was
thought to be an omen of disaster.
Total eclipses can only occur when there is a full moon, and then only if it is lined up with the Earth
in a particular way. (7) ________easily obscured by cloud cover, blood moons are fairly common,
and it is relatively easy to calculate where and when you might be (8)
________ to see one.
For example, in 1504 Christopher Columbus was stranded in Jamaica and in dire need of
provisions, but the local inhabitants were reluctant to help. However, Columbus knew that a lunar
eclipse would occur on 29 February. Therefore, the day before, he warned local leaders that the
moon would disappear (9) ________ they helped him. They remained skeptical. But
when the moon slowly started to change colour, they became (10)__________frightened that
they started to bring food.
Part 2. Read the passage and choose a suitable word to fill in each blank.
“Just imagine a day without paper,” reads one advertisement for a Finnish paper company. It adds,
“You almost (1)________ see our products every day.” And they’re right.
But in most industrial countries, people are so (2)________ to paper – whether it’s for holding
their groceries, for drying their hands or for (3)________ them with the daily news- that its
(4)________ in their daily lives passes largely unnoticed.
At one (5)________ paper was in short supply and was used mainly for important
documents, but more recently, growing economies and new technologies have (6)________ a
dramatic increase in the (7)________ of paper used. Today, there are more than 450 different
grades of paper, all designed for a different (8)________.
Decades ago, some people predicted a “paperless office”. (9)________, the widespread
use of new technologies has gone hand-in-hand with an increased use of paper. Research into the
relationship between paper use and the use of computers has shown that the general (10)________ is
likely to be one of growth and interdependence. However, the costs involved in
paper production, in terms of the world's land, water and air resources, are high. This raises some
important questions. How much paper do we really need and how much is wasted?
1. A. positively B. obviously C. certainly D. absolutely
2. A. conscious B. acquainted C. familiar D. accustomed
3. A. providing B. delivering C. contributing D. giving
4. A. task B. operation C. service D. role
5. A. time B. instance C. date D. occasion
6. A. called on B. come around C. brought about D. drawn up
7. A. total B. portion C. number D. amount
8. A. point B. goal C. purpose D. result
9. A. Instead B. Besides C. Otherwise D. Alternatively
10.A. method B. order C. trend D. system
Part 3: Read the following passage and choose the best answers.
We live in a world of tired, sleep deprived people. In his book Counting Sheep, Paul Martin
– a behavioral biologist – describes a society which is just too busy to sleep and which does not
give sleeping the importance it deserves.
Modern society has invented reasons not to sleep. We are now a 24/7 society where shops and
services must be available all hours. We spend longer hours at work than we used to, and more time
getting to work. Mobile phones and email allow us to stay in touch round the clock and late-night
TV and the Internet tempt us away from our beds. When we need more time for work or pleasure,
the easy solution is to sleep less. The average adult sleeps only 6.2 hours a
night during the week, whereas research shows that most people need eight or even eight and a half
hours’ sleep to feel at their best. Nowadays, many people have got used to sleeping less than they
need and they live in an almost permanent state of ‘sleep debt’.
Until the invention of the electric light in 1879 our daily cycle of sleep used to depend on the hours
of daylight. People would get up with the sun and go to bed at nightfall. But nowadays our hours of
sleep are mainly determined by our working hours (or our social life) and most people are woken up
artificially by an alarm clock. During the day caffeine, the world’s most popular drug, helps to keep
us awake. 75% of the world’s population habitually consume caffeine, which up to a point masks
the symptoms of sleep deprivation.
What does a chronic lack of sleep do to us? As well as making us irritable and unhappy as humans,
it also reduces our motivation and ability to work. This has serious implications for society in
general. Doctors, for example, are often chronically sleep deprived, especially when they are on
‘night call’, and may get less than three hours’ sleep. Lack of sleep can seriously impair their mood,
judgment, and ability to take decisions. Tired engineers, in the early hours of the morning, made a
series of mistakes with catastrophic results. On our roads and motorways lack of sleep kills
thousands of people every year. Tests show that a tired driver can be just as dangerous as a drunken
driver. However, driving when drunk is against the law but driving when exhausted isn’t. As Paul
Martin says, it is very ironic that we admire people who function on very little sleep instead of
criticizing them for being irresponsible. Our world would be a much safer, happier place if
everyone, whatever their job, slept eight hours a night.
1. According to the passage, which of the following statements is TRUE about Paul Martin?
a. He shows his concern for sleep deprivation in modern society.
b. He gives an interesting account of a sleepless society.
c. He is a scientist who is chronically deprived of sleep.
d. He describes the modern world as a place without insomnia.
2. The phrase “round the clock” in the second paragraph is similar in meaning to______.
A. all day and night B. surrounded with clocks C. during the daytime D. having a round clock
3. The writer mentions the Internet in the passage as______.
A. a temptation that prevents us from sleeping
B. an easy solution to sleep deprivation
C. an ineffective means of communication
D. a factor that is not related to sleep deprivation
4, According to the third paragraph, which of the following statements is NOT TRUE?
a. Our social life has no influence on our hours of sleep.
b. The sun obviously determined our daily routines.
c. The electric light was invented in the 19th century.
d. The electric light has changed our daily cycle of sleep.
5, The word “which” in the third paragraph refers to______.
A. reaching a point B. masking the symptom C. the world’s population D.caffeine consumption
6. Which of the following is TRUE, according to the last paragraph?
a. Doctors ‘on night call’ do not need more than three hours of sleep a day.
b. Our motivation decreases with the bigger number of hours we sleep.
c. Sleep deprivation has negative effects on both individuals and society.
D. Thousands of people are killed every day by drunken drivers.
7. The word “catastrophic” in the last paragraph probably means______.
A. bound to bring satisfaction B. becoming more noticeable
C. causing serious damage or loss D. likely to become worthless
8, Which of the following would the writer of the passage approve of?
a. Our world would be a much safer place without drinkers.
b. Both drunken drivers and sleep-deprived people should be criticized.
c. There is no point in criticizing irresponsible people in our society.
d. We certainly can function well even when we hardly sleep.
9, All of the following are mentioned as those whose performance is affected by ‘sleep debt’
EXCEPT_____.
A. doctors B. drivers C. biologists D. engineers
10. Which of the following could best serve as the title of the passage?
A. Accident Prevention: Urgent! B. Sleep Deprivation: Causes and Effects
C. A Society of Sleepless People D. A Well-known Biologist
E. WRITING
Part 1. Rewrite each of the following sentences in such a way that it has a similar meaning
to the original one. You must not change the word given.
1.My manager didn’t agree with the idea. (favor)
My manager________________________________________________________
2.I’d prefer him not to phone me at home. (rather)
I’d________________________________________________________________
3.I did not intend to tell you what I knew. (intention)
I_________________________________________________________________
4.Linda’s plan for a picnic has been spoilt by the weather (fallen)
Linda’s plan ________________________________________________________
5.His handwriting is illegible. (sense)
I__________________________________________________________________
Part 2. Rewrite the sentences beginning with the given words or phrases so that their
meanings stay the same.
7. “ You have done very well to pass your driving test so quickly, John” said Jane
Jane congratulated____________________________________________________
8. Although Mary was exhausted, she agreed to take part in the activity.
Exhausted___________________________________________________________
9. Because he didn’t study hard, he failed the exam.
Had ________________________________________________________________
9, My brother works better when he is pressed for time.
The less______________________________________________________________
10. My decision to get up and dance coincided with the band’s decision to stop playing.
The moment___________________________________________________________
Part 3: Essay writing
In the recent years, there has been a dramatic rise in the number of unemployed people. What
should youngsters do in order not to be out of work?
Write an essay of at least 250 words to express your viewpoint. Support your opinion by using
relevant examples and explanations.
The end
SỞ GD&ĐT VĨNH PHÚC HƯỚNG DẪN CHẤM
TRƯỜNG THPT YÊN LẠC 2 KÌ THI KSCL ĐỘI TUYỂN HSG KHỐI 12
—————— ĐỀ THI MÔN TIẾNG ANH
NĂM HỌC 2019 - 2020
Đáp án gồm:07 trang.
———————
Total: 100pts
A. LISTENING(20pts)
Part 1. (10pts:1 pt/each correct answer)
1. trouble and 2. camps and 3. fun and 4. growing trend 5. studying
expense academies educational. overseas
6. health and safety 7. left behind 8. about 14,000 9. family events 10. sacrifices
Part 2. (5 pts:1 pt/each correct answer)
1. T 2. T 3. F 4. T 5. F
Part 3. (5 pts:1 pt/each correct answer)
1. C 2. B 3. A 4. C 5. C
B. PHONETICS. (5 pts: 1p/each correct answer)
1. A 2. D 3. B 4. A 5. A

C. LEXICO -GRAMMAR. (25pts)

Part 1.(10pts: 1pt/each correct answer)


1. D 2. C 3. D 4. B 5. C
6. B 7. B 8. D 9. A 10. D

Part 2.(10pts: 1pt/each correct answer)


1. activities 2. specialized 3. educational 4. satisfactory 5. knowledge
6. enjoyable 7. musical 8. increasingly 9. affect 10. adjusting

Part 3.( 5 pts: 0,5 pt / each correct answer)

stt line Mistake Correction stt lineMistake Correction


1 2 involves involve 6 10 that than
2 4 misjudging judging 7 11 with from
3 5 accurate accurately 8 13 have has
4 6 whom who 9 14 yielding to yield
5 7 such so 10 16 raise arise

D. READING(30 pts)
Part 1.(10 pts: 1p/each correct answe
1. when 2. what 3. during 4. by 5. before
6. as 7. Although 8. able/likely 9. unless 10. so
Part 2.(10 pts: 1p/each correct answer)
1. C 2. D 3. A 4. D 5. A
6. C 7. D 8. C 9. A 10. C

Part 3 (10 pts: 1p/each correct answer)


1. A 2. A 3. A 4. A 5. D
6. C 7. C 8. B 9. C 10. B
E. WRITING. (20 pts )
Part 1.Rewrite each of the following sentences in such a way that it has a similar
meaning to the original one. You must not change the word given.
(2,5pts: 0,5p /each correct sentence)
11. Jane congratulated John on passing his driving test so quickly.
12. Exhausted as/though Mary was, she agreed to take part in the activity.
13. Had he studied hard, he wouldn’t have failed the exam.
14. The less time my brother has, the better he works.
15. The moment I decided to get up and dance, the band decided to stop playing.
Part 2. Rewrite the sentences beginning with the given words or phrases so that their
meanings
stay the same. (2,5pts: 0,5p /each correct sentence)
16. My manager was not in favor of the idea.
17. I’d rather he didn’t phone me at home.
18. I had no intention of telling you what I knew.
19. Linda’s plan for a picnic has fallen through because of the weather.
20. I make no sense of his handwriting.
Part 3: Essay writing (15pts)
In the recent years, there has been a dramatic rise in the number of unemployed people.
What should youngsters do in order not to be out of work?
Write an essay of at least 250 words to express your viewpoint. Support your opinion by using
relevant examples and explanations.

Length: (2ps) : 250 words.

Organization & style: (3ps) ideas are organized and


presented with coherence, style, and clarity appropriate to
the level of English language with high school students.

Ideas (5ps): a provision of main ideas and appropriate details and


examples.

21. Grammar and Vocabulary (5ps) a variety of vocabulary and structures


appropriate to the level of English language with high school students.
SỞ GIÁO DỤC VÀ ĐÀO TẠO TỈNH
KỲ THI HỌC SINH GIỎI LỚP 12
QUẢNG NAM Năm học 2018 - 2019
ĐỀ CHÍNH THỨC Môn thi: TIẾNG ANH
Thời gian : 90 phút
(Không kể thời gian giao đề)
Mã đề thi: 179 (Đề thi gồm có 10 trang)
(Thí sinh làm bài trên Phiếu trả lời trắc nghiệm)
Họ và tên thí sinh: ……………………………………… Phòng thi ………Số báo danh: ……

SECTION I: LISTENING (20 points)


HƯỚNG DẪN PHẦN THI NGHE HIỂU
1. Mỗi phần được nghe 2 lần, mở đầu và kết thúc mỗi phần có tín hiệu.
2. Mọi hướng dẫn cho thí sinh (bằng tiếng Anh) đã có trong bài nghe.

Part 1: You will hear people talking in eight different situations. For questions 1-8,
mark the letter A, B, C or D on your answer sheet to indicate the correct answer to
each of the following questions.
1. You hear two friends talking about a book. What does the boy like about it?
A. It’s short. B. It’s educational. C. It’s amusing. D. It’s informative.
2. You hear a man talking about family holidays as a teenager.
What part did he find most enjoyable?
A. the journey B. being outdoors C. the places D. watching films
3. You hear two friends talking about a hotel they stayed in. What do they agree
about?
A. Its location was convenient. B. The staff were friendly.
C. The room was comfortable. D. The hotel was well- equipped.
4. You hear a woman talking about a new film. How did she feel after seeing it?
A. confused B. frustrated C. sad D. embarrassed
5. You hear a woman telling a friend about her job in a supermarket. What was
she doing there this morning?
A. working on the checkout B. filling the shelves
C. serving at the fish counter D. meeting other people
6. You hear a man talking about a band he saw at a concert. What is he critical of ?
A. the image of the band B. the quality of the music
C. the length of the show D. the band members
7. You hear a sports instructor giving advice. Which sport is he talking about?
A. Sailing B. Swimming C. Skating D. Scuba diving

8. You hear a woman talking about being at university. What surprised her about it?
A. How much money she spent. B. How many friends she made.
C. How well people got on with each other. D. How quickly time passed.
Part 2: You will hear a man, David, being interviewed about his life as a professional
footballer. For questions 9- 16, mark the letter A, B, C or D on your answer sheet to
indicate the correct answer to each of the following questions.
9. How long has David been a professional football player?
A. one match B. two years C. four years D. two matches
10.When was first he included in the club?
A. When he was seventeen. B. When he was twelve.
C. When he was fifteen. D. When he was fourteen.
11.What is in a normal day for David?
A. fitness training and tactics B. fitness training and a full match
C. fitness training only D. fitness training, tactics and a full match
12.What do the team not watch videos about?
A. the opposition B. warming up
C. their own performance. D. playing at home
13. What do they do at weekends?
A. enjoy a different atmosphere B. remember the game
C. do a warm-up D. have a game
14. What does David say about the diet of a footballer?
A. It is often unpleasant and bad.
B. It has lots of rice, meat and pasta.
C. Footballers have to be careful about what they eat.
D. They drink alcohol regularly.
15. What is true about David’s free time?
A. He spends most of his free time with his friends.
B. He has very little free time, except in the summer.
C. He does not usually manage to see his family.
D. He gets home on time.
16. What does David say about his future ambitions?
A. He firstly wants to secure a regular place in the team.
B. He wants to play for a European team in the next two years.
C. He never thinks about playing in the World Cup.
D. He thinks of the World Cup.
SECTION II: LEXICO-GRAMMAR (30 points)

Part 1: Mark the letter A, B, C, or D on your answer sheet to indicate the correct
answer to each of the following questions from 17 to 32.

Question 17: They always keep on good ______ with their next- door neighbors for the

children’s sake.
A. terms B. friendship C. relationship D. words
Question 18: He bought two white shirts; one for him and ______ for his son.

A. others B. the other C. another D. the others

Question 19: Charles was wearing ______ at the party.

A. a silk funny pink tie B. a funny pink silk tie

C. a funny silk pink tie D. a pink silk funny tie


Question 20: Good restaurants serving traditional English food are very hard to ______.

A. come into B. come by C. come to D. come at


Question 21: On being told about her sack, ______.

A. her boss felt sorry for Mary B. Mary was shocked

C. Mary’s face turned pale D. her boss changed his attitude

Question 22: Travellers will need some cash in ______ currency but they can use their

credit cards.

A. area B. local C. real D. current

Question 23: She is reported to be ______ a spectator.

A. more of an athlete than B. more an athlete than

C. an athlete of more than D. an athlete more than

Question 24: The children ______ by social networks are likely to suffer from

depression and other health problems.

A. are obsessed B. obsessing C. obsessed D. who obsessed

Question 25: They were at the stadium with us last night, so they ______ at the theatre then.

A. needn’t have been B. might have been


C. should have been D. can’t have been
Question 26: I feel terrible, I didn’t sleep ______ last night.
A. a jot B. a wink C. an inch D. an eye
Question 27: Sarah and I reserved the rooms in the same hotel. She was really
surprised to see me there.
A. coincidentally B. deliberately C. intentionally D. practically

Question 28: ______ we work with her, we get confused because of her fast speaking pace.

A. Although B. So that C. Whenever D. Since


Question 29: The sign warns people the dangers of swimming in this river.
A. about B. against C. to D. from

Question 30: ______ eighty percent of the students in our school are eager to work with

foreign teachers, aren’t they?


A. Most of B. Mostly C. Most D. Almost
Question 31: Many people hate Mondays, saying that they really ______.
A. get over them B. get down them C. get them over D. get them down
Question 32: Jane would never forget the first prize in such a prestigious
competition.
A. to be awarded B. having awarded C. to have awarded. D. being awarded
Part 2: Mark the letter A, B, C, or D on your answer sheet to indicate the word(s)
CLOSEST in meaning to the underlined word(s) in each of the following questions
from 33 to 34.

Question 33: The fighter plane gave pivotal assistance to the ground forces that were

surrounded by the enemy.

A. necessary B. significant C. reliable D. compulsory

Question 34: He didn’t bat an eye when he realized he failed the exam again.
A. didn’t want to see B. didn’t show surprise
C. wasn’t happy D. didn’t care

Part 3: Mark the letter A, B, C, or D on your answer sheet to indicate the underlined
part that needs correction in each of the following questions from 35 to 36.

Question 35: Stayed strong, family members have to be engaged in each other’s lives.
A B C D
Question 36: Quinine, cinnamon and other useful substances are all derived of the
A B C
bark of trees.
D

Part 4: Mark the letter A, B, C, or D on your answer sheet to indicate the option that
best completes each of the following exchanges from 37 to 38.

Question 37: Jordan and Susan are talking about Ted’s accident last week.

Jordan: “A motorbike knocked Ted down.”

Susan: “______!”

A. What a motorbike B. How terrific C. Poor Ted D. Well- done

Question 38: Viva and Bob have been reading about the wonders of the world.

Ralph: “Would you say The Great Wall is among the seven man- made wonders of

the world?”

Anna: “______”

A. That’s the least I could do. B. Do you say so?

C. There’s no doubt about that. D. I’m glad you like it.

Part 5: Mark the letter A, B, C, or D on your answer sheet to indicate the word(s)
OPPOSITE in meaning to the underlined word(s) in each of the following questions
from 39 to 40.
Question 39: It is believed that people are now far more materialistic than their

predecessor years ago.

A. monetary B. greedy C. object- oriented D. spiritual

Question 40: If you are at a loose end this weekend, I will show you around the city.

A . confident B. occupied C. decisive D. flexible

SECTION III: READING (30 points)

Part 1: Read the following passage and mark the letter A, B, C, or D on your answer
sheet to indicate the correct word or phrase that best fits each of the numbered blanks
from 41 to 48.
DREAMS AND DREAMING

Over the past many dream theories have emerged. The most famous expert (41)______
dreaming was Sigmund Freud, an Austrian doctor. At the end of the 19th century, he
found out that a part of our brain is subconscious. Our mind keeps everything

that we might want but cannot have them. It is a place where we keep our desires and
wishes. In dreams we (42)______ these wishes.

Dreaming is an activity of the brain. It produces electrical waves which can be measured
with an electroencephalograph. Dreams occur (43)______ brain waves are especially
fast.

Everyone dreams. If you think that you never dream, you are wrong. Most of the time,
we cannot remember our dreams. Theorists say that we dream (44)______ about

the thoughts and wishes that we repress. Others say that if dreaming is not so
(45)______, it is difficult to remember them anyway. Another reason may be that

dreams are never the same, they have to be repeated in order for us to remember them.
Most people cannot control their dreams, they just happen. (46)______

people experience lucid dreaming. They can control what happens in their dreaming
while they are asleep.

Even blind people dream. They may see images of events that they have experienced
before they (47)______ blind. On the other hand, a person who was

born blind does not see pictures in dreams, but they can hear, smell and taste in them.
People often have the same dreams over and over again. They may be events in your life
that have caused problems which you have not solved yet. Nightmares are dreams that
are so terrible that you usually wake up. There is medical treatment for nightmares. You
can go to a therapist who tries to find out why you have nightmares and how to
(48)______ them.
Question 41. A. A. give out Question 43. A. since
on Question 42. Question 44. A. almost Question 45. A.
intensify Question 46. A.
Very few Question 47. A. B. about C. of D. down
made Question 48. A. get on B. put out
with B. when B. C. make out D. act out
most of B.
C. because D. although
intensity
B. A few C. mostly D. most
B. became
C. intense D. intensely
B. keep up with
C. Very little D. A little

C. took D. held

C. go with D. deal with


Part 2: Read the following passage and mark the letter A, B, C, or D on your answer sheet to
indicate the correct answer to each of the questions from 49 to 56.
A number of factors related to the voice reveal the personality of the speaker. The first is the broad area
of communication, which includes imparting information by use of language, communicating with a
group or an individual and specialized communication through performance. A person conveys
thoughts and ideas through choice of words, by a tone of voice that is pleasant or unpleasant, gentle or
harsh, by the rhythm that is inherent within the language itself, and by speech rhythms that are flowing
and regular or uneven and hesitant, and finally, by the pitch and melody of the utterance. When
speaking before a group, a person's tone may indicate uncertainty or fright, confidence or calm. At
interpersonal levels, the tone may reflect ideas and feelings over and above the words chosen, or may
believe them. Here the participant’s tone can consciously or unconsciously reflect intuitive sympathy
or antipathy, lack of concern or interest, fatigue, anxiety, enthusiasm or excitement, all of which are
usually discernible by the acute listener. Public performance is a manner of communication that is
highly specialized
with its own techniques for obtaining effects by voice and or gesture. The motivation derived
from the text, and in the case of singing, the music, in combination with the performer's skills,
personality, and ability to create empathy will determine the success of artistic, political, or
pedagogic communication.
Second, the voice gives psychological clues to a person's self-image, perception of others, and
emotional health. Self-image can be indicated by a tone of voice that is confident, pretentious,
shy, aggressive, outgoing, or exuberant, to name only a few personality traits. Also the sound may
give a clue to the facade or mask of that person, for example, a shy person hiding behind an
overconfident front. How a speaker perceives the listener's receptiveness, interest or sympathy in
any given conversation can drastically alter the tone of presentation, by encouraging or
discouraging the speaker. Emotional health is evidenced in the voice by free and melodic sounds
of the happy, by constricted and harsh sound of the angry, and by dull and lethargic qualities of
the depressed.
(Source: psychologytoday.com)
Question 49: What does the passage mainly discuss?
A. the function of the voice in performance
B. communication styles
C. the connection between voice and personality
D. the production of speech
Question 50: What does the author mean by stating that, "At interpersonal levels, tone may
reflect ideas and feelings over and above the words chosen"?
A. Feelings are expressed with different words than ideas are.
B. The tone of voice can carry information beyond the meaning of words.
C. A high tone of voice reflects an emotional communication.
D. Feelings are more difficult to express than ideas.
Question 51: The word "Here" in line 10 refers to ______.
A. with the tone B. with chosen words C. by ideas and feelings D. at interpersonal levels
Question 52: Why does the author mention "artistic, political, or pedagogic communication"?
A. as examples of public performance
B. as examples of basic styles of communication
C. to contrast them to singing
D. to introduce the idea of self-image
Question 53: According to the passage, an exuberant tone of voice may be an indication
of a person's
______.
A. personality B. general physical health
C. ability to communicate D. vocal quality
Question 54: According to the passage, an overconfident front may hide ______.
A. hostility B. shyness C. friendliness D. strength
Question 55: The word "drastically" in line 24 is closest in meaning to ______.
A. frequently B. exactly C. severely D. easily

Question 56: According to the passage, what does a constricted and harsh voice indicate?
A. lethargy B. depression C. boredom D. anger

Part 3: Read the following passage and mark the letter A, B, C, or D on your answer sheet to
indicate the correct answer to each of the questions from 57 to 64.
Composers today use a wider variety of sounds than ever before, including many that were once
considered undesirable noises. Composer Edgard Varese (1883-1965) called this the “liberation of
sound…the right to make music with any and all sounds.” Electronic music, for example, made
with the aid of computers, synthesizers, and electronic instruments, may include sounds that in the
past would not have been considered musical.
Environmental sounds, such as thunder, and electronically generated hisses and blips can be
recorded, manipulated, and then incorporated into a musical composition. But composers also draw
novel sounds from voices and non-electronic instruments. Singers may be asked to scream, laugh,
groan, sneeze, or to sing phonetic sounds rather than words. Wind and string players may lap or
scrape their instruments. A brass or woodwind player may hum while playing, to produce two
pitches at once; a pianist may reach inside the piano to pluck a string and then run a metal blade
along it. In the music of the Western world, the greatest expansion and experimentation have
involved percussion instruments, which outnumber strings and winds in many recent compositions.
Traditional percussion instruments are struck with new types of beaters; and instruments that used
to be considered unconventional in Western music, tom-toms, bongos, slapsticks, maracas, are
widely used.
In the search for novel sounds, increased use has been made in Western music of Microtones. Non-
Western music typically divides and intervals between two pitches more finely than Western music
does, thereby producing a greater number of distinct tones or micro tones, within the same interval.
Composers such as Krzysztof Penderecki create sound that borders on electronic noise through tone
clusters, closely spaced tones played together and heard as a mass, block, or band of sound. The
directional aspect of sound has taken on new importance as well. Loud speakers or groups of
instruments may be placed at opposite ends of the stage, in the balcony, or at the back and sides of
the auditorium. Because standard music notation makes no provision for many of these innovations,
recent music scores may contain graph like diagrams, new note shapes and symbols, and novel
ways of arranging notation on the page.

Question 57: The word “wider” in line 1 is closest in meaning to ______.


A. more impressive B. more distinctive C. more controversial D. more extensive
Question 58: The passage suggests that Edgard Varese is an example of a composer who
______.
A. criticized electronic music as too noise like.
B. modified sonic of the electronic instruments he used in his music.
C. believed that any sound could be used in music.
D. wrote music with environmental themes.
Question 59: The word “it” in line 13 refers to ______.
A. piano B. string C. blade D. music
Question 60: According to the passage, which of the following types of instruments has played a
role in much of the innovation in Western music?
A. string B. percussion C. woodwind D. brass
Question 61: The word “thereby” in line 21 is closest in meaning to ______.
A. in return for B. in spite of C. by the way D. by this means
Question 62: According to the passage, Krzysztof Pederecki is known for which of the following
practices?
A. using tones that are clumped together
B. combining traditional and nontraditional instruments
C. seating musicians in unusual areas of an auditorium
D. playing Western music for non-Western audiences
Question 63: According to the passage, which of the following would be considered traditional
elements of Western music?
A. microtones B. tom-toms and bongos
C. pianos D. hisses
Question 64: In paragraph 3, the author mentions diagrams as an example of a new way to ______.
A. chart the history of innovation in musical notation
B. explain the logic of standard musical notation
C. design and develop electronic instruments
D. indicate how particular sounds should be produced

Part 1: Mark the letter A, B, C, or D on your answer sheet to indicate the sentence that is closest
in meaning to the sentence given in each of the following questions from 65 to 72.
Question 65: When I arrived, they were having dinner.
A. I came in the middle of their dinner.
B. They ate their dinner as soon as I arrived.
C. When they started having their dinner, I arrived.
D. I came to their invitation to dinner.
Question 66: The woman was too weak to lift the suitcase.
A. The woman wasn’t able to lift the suitcase, so she was very weak.
B. Though the woman was weak, she could lift the suitcase.
C. So weak was the woman that she couldn’t lift the suitcase.
D. The woman shouldn’t have lifted the suitcase as she was weak.
Question 67: You should have persuaded him to change his mind.
a. It was essential to persuade him to change his mind but you didn’t.
b. You didn't persuade him to change because of his mind.
c. You should persuade him to change his mind.
d. You persuaded him to change his mind but he didn't listen.

Question 68: “Would you like some more beer?” he asked.


A. He wanted to invite me for a glass of beer.
B. He asked me would I like some more beer.
C. He offered me some more beer.
D. He asked me if I wanted some beer.
Question 69: He cannot practice scuba diving because he has a weak heart.
A. The fact that he has a weak heart cannot stop him practicing scuba diving.
B. Scuba diving makes him suffer from having a weak heart.
C. The reason why he cannot practice scuba diving is that he has a weak heart.
D. He has a weak heart but he continues to practice scuba diving.
Question 70: People believe that neither side wanted war.
A. Neither side is believed to have wanted war.
B. Neither side is responsible for the outbreak of war.
C. War is believed to be wanted by either side.
D. It is believed that war broke out from both sides.
Question 71: Soil erosion is a result of forests being cut down carelessly.
A.That forests are being cut down carelessly results from soil erosion.
B. Soil erosion contributes to forests being cut down carelessly.
C.Soil erosion results in forests being cut down carelessly
D. That forests are being cut down carelessly leads to soil erosion.
Question 72: No matter how hard Fred tried to lose weight, he did not succeed.
A. It did not matter whether Fred could lose weight.
B. However hard Fred tried, he could not lose weight.
C. Fred tried very hard to lose weight and succeeded.
D. It was hard for Fred to lose weight because he never succeeded.
Part 2: Mark the letter A, B, C, or D on your answer sheet to indicate the sentence that best
combines each pair of sentences in the following questions from 73 to 80.
Question 73: She wasn’t wearing a seat belt. She was injured.
a. If she hadn’t been wearing a seat belt, she wouldn’t have been injured.
b. If she had been wearing a seat belt, she would have been injured.
c. If she had been wearing a seat belt, she wouldn’t be injured.
d. If she had been wearing a seat belt, she wouldn’t have been injured.
Question 74: The agreement ended six-month negotiation. It was signed yesterday.
A. The negotiation which lasted six months was signed yesterday.
B. The agreement which signed yesterday ended six-month negotiation.
C. The agreement signed yesterday lasted six months.
D: The agreement signed yesterday ended six-month negotiation.
Question 75 : You’d better take the keys. It’s possible I’ll come home late.
A. If I come home late, you’d better take the keys.
B. You’d better take the keys in case I come home late.
C. You'd better take the keys as long as I possibly come home late.
D. I'll probably come home late so that you’d better take the keys.
Question 76: Ann always keeps up with the latest fashions. She works for a famous fashion
house.
A. Despite working for a famous fashion house, Ann hardly keeps up with the latest fashions.
B. Ann always keeps up with the latest fashions so as not to work for a famous fashion house.
C. Not working for a famous fashion house, Ann always keeps up with the latest fashions.
D. Ann works for a famous fashion house, so she always keeps up with the latest fashions.
Question 77: We didn’t want to spend a lot of money. We stayed in a cheap hotel.
A. Rather than spending a lot of money, we stayed in a cheap hotel.
B. In spite of spending a lot of money, we stayed in a cheap hotel.
C. We stayed in a cheap hotel, but we had to spend a lot of money.
D. We didn’t stay in a cheap hotel as we had a lot of money to spend.
Question 78: Overeating is a cause of several deadly diseases. Physical inactivity is another cause
of several deadly diseases.
A. Both overeating and physical inactivity result from several deadly diseases.
B. Apart from physical activities, eating too much also contributes to several deadly diseases.
C. Not only overeating but also physical inactivity may lead to several deadly diseases.
D. Overeating and physical inactivity are caused by several deadly diseases.
Question 79: Jack has won a jackpot prize. 10% of it was donated to flooded areas.
A. Jack has won a jackpot prize, 10% which was donated to flooded areas.
B. Jack has won a jackpot prize, 10% of which was donated to flooded areas.
C. Jack has won a jackpot prize, which of 10% was donated to flooded areas.
D. Jack has won a jackpot prize, which was donated 10% to flooded areas.
Question 80: Home schooling is apparently very successful. Many people still believe in the
benefits of traditional schooling.

A. Because home schooling is apparently very successful, many people still believe in the
benefits of traditional schooling.
B. As long as home schooling is apparently very successful, many people still believe in the
benefits of traditional schooling.
C. Even though home schooling is apparently very successful, many people still believe in the
benefits of traditional schooling.
D. In spite of home schooling is apparently very successful, many people still believe in the
benefits of traditional schooling.
***** THE END *****

ĐÁP ÁN MÃ ĐỀ 179

Câu Đáp án Câu Đáp án Câu Đáp án Câu Đáp án


1 A 21 B 41 A 61 D
2 D 22 B 42 D 62 A
3 A 23 A 43 B 63 C
4 B 24 C 44 C 64 D
5 C 25 D 45 C 65 A
6 C 26 B 46 A 66 C
7 B 27 A 47 B 67 A
8 D 28 C 48 D 68 C
9 B 29 A 49 C 69 C
10 C 30 D 50 B 70 A
11 A 31 D 51 D 71 D
12 B 32 D 52 B 72 B
13 D 33 B 53 A 73 D
14 C 34 B 54 B 74 D
15 B 35 A 55 C 75 B
16 A 36 C 56 D 76 D
17 A 37 C 57 D 77 A
18 B 38 C 58 C 78 C
19 B 39 D 59 B 79 B
20 B 40 B 60 B 80 C

SỞ GIÁO DỤC VÀ ĐÀO TẠO KỲ THI CHỌN HỌC SINH GIỎI CẤP TỈNH THPT
NĂM HỌC 2018 – 2019
ĐỀ CHÍNH THỨC Môn thi: TIẾNG ANH 12
THÚC
(Đề thi gồm có 10 trang)

A. LISTENING (2.0 points) (The recording will be played TWICE)


Listen and complete the form below. Write NO MORE THAN ONE WORD OR A NUMBER for each
answer.
Question 1-5.
PAN ASIAN AIRWAYS - LOST PROPERTY REPORT FORM
First name Kirsty
Surname Allen
Address 1. ___________________ Windham Road, Richmond
Postcode 2.___________________
Home. Tel 020 8927 7651
Mobile. Tel 3.___________________
Flight number 4.___________________
Seat number 5.___________________
From: New York To: London Heathrow
Question 6-8. Choose THREE letters from A to F.
What items did Kirsty’s bag contain?
A. 17 pounds B. Pens C. Her passport
D. A book E. 200 dollars F. Her house keys
Question 9-10. Choose a letter A, B, C, or D that correctly answer questions 9 and 10.
9. What has Kirsty done regarding to the loss of her credit card?
A. informed the police but not the credit card company.
B. informed the credit card company but not the police.
C. informed both the police and the credit card company.
D. informed neither the police nor the credit card company.
10. What must Kirsty do after the call regarding to her lost handbag?
A. Call back after one hour and a half.
B. Just wait for a call back.
C. Call back after one hour and a half if she has heard nothing.
D. Call back the next day if she has heard nothing.
Your answers:
1. 2. 3. 4. 5.
6. 7. 8. 9. 10.

B. PHONETICS (2.0 points)


Identify the word whose underlined part is pronounced differently from that of the others.
(1.0 point)
Question 1. A. architect B. parachute C. chemistry D. psychology
Question 2. A. advent B. adverb C. advertise D. advance
Question 3. A. jumped B. grabbed C. remarked D. laughed
Question 4. A. ecosystem B. knowledge C. technology D. commodity
Question 5. A. treachery B. treasure C. feature D. meadow

Identify the word whose stressed pattern is different from that of the others. (1.0 point)
Question 6. A. method B. invent C. intense D. effect
Question 7. A. writer B. careful C. visit D. compete
Question 8. A. director B. machinery C. excellent D. encourage
Question 9. A. intelligent B. population C. opportunity D. economics
Question 10. A. advisable B. admirable C. reliable D. desirable
Your answers:
1. 2. 3. 4. 5.
6. 7. 8. 9. 10.

C. VOCABULARY AND GRAMMAR (6.0 points)


I. Choose the best word or phrase to complete the following sentences. (2.0 points)
Question 1. When the teacher came in, they pretended __________.
A. to read B. reading C. being reading D. to be reading
Question 2. I called Jeny yesterday with a view____________asking her about the project.
A. to B. of C. in D. about
Question 3. It is believed that there is a _________ between the two crimes.
A. join B. chain C. link D. connector
Question 4. We were so late that we __________ had time to catch the train.
A. nearly B. almost C. hardly D. simply
Question 5. They turned _________at the party despite the awful weather.
A. round B. in C. back D. up
Question 6. There wasn't __________ news in his letter.
A. a great many B. hardly any C. a great deal of D. large number of
Question 7. Don’t be late for the interview; _________ people will think you are a disorganized person.
A. otherwise B. unless C. if not D. or so
Question 8. Peter is very tall. He really _________ from the rest of his class.
A. stands out B. sits out C. sits up D. stands up
Question 9. I'm sure it's not my fault that Peter found out what we were planning. I don't remember
__________ anyone about it.
A. to tell B. being told C. having told D. to be told
Question 10. becoming extinct is of great concern of zoologists.
A. When giant pandas are B. Giant pandas are
C. Are giant pandas D. That giant pandas are
Question 11. Tim looks so frightened and upset. He something terrible.
A. must experience B. can have experienced
C. should have experienced D. must have experienced
Question 12. My friend has just bought from a shop on Tran Phu street.
A. a digital useful alarm clock B. an alarm useful digital clock
C. a useful alarm digital clock D. a useful digital alarm clock
Question 13. we invested in telecommunication industry, we would be rich now.
A. Should B. Were C. Would D. Had
Question 14. we have finished the course, we should start doing more revision work.
A. For now B. Now that C. Ever since D. By now
Question 15. There are two small rooms in the beach house, served as a kitchen.
A. the smaller of which B. the smallest of which
C. the smaller of them D. smallest of that
Question 16. The car burst into but the driver managed to escape.
A. fire B. burning C. heat D. flames
Question 17. It is the recommendation of many psychologists to associate words and remember
names.
A. that a learner uses mental images B. that a learner use mental images
C. that a learner must use mental images D. mental images are used

Mark the letter A, B, C, or D in the space provided to indicate the word(s) CLOSEST in meaning to the
underlined word(s) in each of the following questions.
Question 18. The reason why Aurora is dressed up to the nines is because she's got a date tonight.
A. dressed too casually for the occasion B. dressed in a size nine
C. dressed up and looking great D. dressed in a plain-looking suit

Mark the letter A, B, C, or D in the space provided to indicate the word(s) OPPOSITE in meaning to the
underlined word(s) in each of the following questions.
Question 19. Sorry, I can’t come to your party. I am snowed under with work at the moment.
A. busy with B. free from C. relaxed about D. interested in

Mark the letter A, B, C, or D in the space provided to indicate the most suitable response to complete
each of the following exchanges.
Question 20. Mary is answering the telephone at the office.
- Davis’ friend: “May I leave a message for Ms Davis?”
- Mary: “____________________________”
A. I’m afraid she is not here at the moment. B. No, she’s not here now.
B. She’s leaving a message for you now. D. Yes, I’ll make sure she gets it.
Your answers:
1. 2. 3. 4. 5.
6. 7. 8. 9. 10.
11. 12. 13. 14. 15.
16. 17. 18. 19. 20.

II. Put the verbs in bracket into the correct form. (2.0 points.)
Question 1. It’s about time the government (do)___________ something about the sewage system
which hasn’t been upgraded for years.
Question 2. Critics say the mayor is too busy (campaign)___________ to do his job properly.
Question 3. It’s essential that every student (know)___________ how to use a computer.
Question 4. Hurry up, or all the tickets (sell)___________ out by the time we arrive at the theatre.
Question 5. Do you feel like (go)_______ to the theatre tonight or would you rather stay at home?
Question 6. The weapon (use)___________ in the murder has just been found.
Question 7. No sooner Peter (come)___________ into his office than he noticed the document on
the table.
Question 8. (Bite)___________ twice, the postman refused to deliver our letters unless we chained
our dog up.
Question 9. His roommate (always/ enter)___________ the room without knocking first, which
annoys him a lot.
Question 10. Working as a doctor (be)___________ rewarding but stressful.
Your answers:
1. 2. 3. 4. 5.

6. 7. 8. 9. 10.

III. Write the correct form of the bold-typed words in the following passage. (2.0 points)
The origins of Halloween
Halloween is celebrated in many parts of the (1)________ world, and is the time 1. WEST
when people dress up as witches or ghosts, and go "trick-or treating". (2)________, it 2. DOUBT
is one of the most popular traditions in the United States and Britain.
The celebration (3)________ about two thousand years ago with the Celts. 3. ORIGIN
These people were the (4)________ of an area that includes Britain and Ireland. 4. INHABIT
They relied on the land for their (5)________ and this meant that they were at the 5. LIVE
mercy of (6)________ weather conditions, especially during the winter. 6. PREDICT
The Celtic new year began on 1st November, which also marked the beginning
of winter, a period (7)________ associated with death. On the eve of the new year, it 7. TRADITION
was believed that the barriers between the worlds of the living and the dead were
(8)________ withdrawn, and it was possible to communicate with spirits. The Celts 8.TEMPORARY
believed that the spirits offered them (9)________ and protection, and the Druids 9. GUIDE
(Celtic priests) were (10)________able to predict the future on this point. 10. REPUTE

Your answers:
1. 2. 3. 4. 5.
6. 7. 8. 9. 10.

D. READING (5.0 points)


I. Read the following passage and choose the correct answer A, B, C or D to complete the passage.
(2.0 points)
Save money on the book that aims to save animals
Have you ever raised such a question “Do you want to take part in the battle to save the world’s
wildlife?” Animal Watch is a book which will (1) you in the fight for survival that faces many
of our endangered animals and show how they struggle on the (2) of extinction.
As you enjoy the book’s 250 pages and over 150 color photographs, you will have the
(3)________ of knowing that part of your purchase money is being used to help animals (4) .
From the comfort of your armchair, you will be able to observe the world’s animals close-up and explore
their habitats. You will also discover the terrible results of human (5) for land, flesh and skins.
Animal Watch is packed with fascinating facts. Did you know that polar bears cover their black
noses with their paws so they can hunt their (6) in the snow without being seen.
This superb publication has ( 7) impressed Britain’s leading wildlife charity that it has
been chosen as Book of the Year, a title awarded to books (8) are considered to have made a
major contribution to wildlife conservation. You will find Animal Watch at a special low (9)
price at all good bookshops, but hurry while (10) last.
Question 1. A. combine B. involve C. bring D. lead
Question 2. A. edge B. start C. limit D. end
Question 3. A. value B. enjoyment C. virtue D. satisfaction
Question 4. A. survive B. raise C. observe D. explore
Question 5. A. interest B. greed C. care D. concern
Question 6. A. fruit B. plant C. prey D. food
Question 7. A. too B. enough C. so D. such
Question 8. A. which B. in which C. where D. at which
Question 9. A. beginning B. preparatory C. original D. introductory
Question 10. A. market B. stocks C. goods D. fund

Your answers:
1. 2. 3. 4. 5.
6. 7. 8. 9. 10.

II. Complete the following passage by filling in each blank with ONE suitable word. (1.0 point)
I was reading an article last week in which the writer described (1)_______ her children has
changed as they grow up. When they were small, she had to (2)_______ up with noisy games in the house
or join in interminable games of football in the garden which wore her out. If the house went quiet, she
wondered what the monsters were getting up to, or what crisis she would have to (3)_______ with next.
She dreaded the fact that they might (4)_______ after her husband, who admitted having (5)_______ an
uncontrollable child who (6)_______ most of time showing off to his friends by breaking things or getting
into fights. What was worse was that everyone else thought he was (7)_______ sweet child, and he got
away with the most terrible things. However, she had experienced an even greater shocking with her
children. They had (8)_______ out of all their naughty behavior, and took up serious hobbies such as
playing chess and the piano. They never did anything (9)_______ talking it over first, and coming to a
serious decision. She had to face up to the fact that they made her feel rather childish as they got
(10)_______, and that in some ways she preferred them when they were young and noisy.
Your answers:
1. 2. 3. 4. 5.
6. 7. 8. 9. 10.

III. Read the following passage and choose the correct answer A, B, C or D (2.0 points)
What I love about my job is the variety. I get enquiries from people all over the world asking me
how they should go about setting up their own website. I’ve been asked about so many subjects -
anything from someone wanting to teach people how to throw boomerangs to other people selling paper
flowers which they make at home in their spare time.
Obviously with all the thousands of websites available at the click of a button, you want to
create an impression with your website so that it becomes a must-see destination. Not everyone is
prepared, however, for the way in which a website can become so popular that it actually has to be closed
down.
When people first set up their websites they probably pay their web advertiser a monthly fee based
on the number of hits or page impressions their site receives. If they can pay their monthly fee without it
costing them too much, that is the best thing most people hope for. One guy, Pete Bennett, whom I
helped, wanted to set up a one-stop shop to provide decent images of the world’s flags. He’d been
fascinated by flags since his boyhood and had no idea that thousands of other people share his passion.
Anyway, in one month his web page had over 1.5 million hits. As a result his internet provider trebled the
fee that he was being charged. He wasn’t a rich person and he couldn’t afford to spend that amount of
money on a hobby without any benefit to him, so he decided to carry advertising on his site. He found a
company which specializes in smaller sites and adverts were added to the pages on his website. So,
although he doesn’t make a huge profit, at least his hobby provides him with a small income.
If you have specialist skills or expertise, it can pay you to sell the products that people want. I
helped one woman design a page to advertise the fact that she tells fortunes, based on the information that
her clients supply her with. If you want her to tell your fortune, you fill in a questionnaire online - your
age, date of birth, hobbies, interests and so on and for a small fee she e-mails you back your fortune. You
can print it out and it looks really good, decorated with moons and stars, your zodiac sign and
your birthstone. I tried it myself and although I’m not sure I believe it, my future according to her is
positive and exciting. I also found out that for someone born in August, like me, the birthstone is a period,
a pale green stone which I’d never even heard of!
I also get a fair number of complaints from people e-mailing me to say that they can’t access the
website. When they click on the site a message appears on their screen saying “An error has occurred
in the script on this page”. This usually happens when someone has tried to achieve fancy effects on their
website by using programming techniques based on a scripting language. This means that unless they
really know what they are doing, whoever designed the site has probably made a mistake in their
programming. This is where people like me t o come in. Most computer instruction guides make
things appear quite straightforward, but unless you’re very skilled, you’re likely to run into problems. It’s
generally worth getting a professional to help you set up your site in the first place - otherwise people like
me would be out of work. And let’s face it, this is big business.
Question 1. What does the author enjoy most about his job?
A. Dealing with different people. C. His worldwide contacts.
B. The range of topics. D. Teaching design skills.
Question 2. What does the author mean by “a must-see destination” in paragraph 2?
A. A website that can no longer be seen. C. A website that everyone wants to visit.
B. A website that does not make a charge. D. A website that has been well prepared.
Question 3. Why did Pete Bennett set up a website on flags?
A. He knew lots of people shared his interest. C. He hoped to make a lot of money.
B. A web advertiser wanted to sell flags. D. He’d been interested in flags for years.
Question 4. What word can best replace the word “hits” in paragraph 3?
A. bad effects B. beats C. attacks D. visits
Question 5. Why did Pete Bennett accept advertising on his website?
A. To attract more hits. B. To add more interest.
C. To repay the huge fee. D. To help him earn some money.
Question 6. Who are the “clients” in paragraph 4 referred to?
A. Interested people B. Web page designers
C. Internet providers D. Product advertisers
Question 7. Why do error messages sometimes appear?
A. People make a mistake in their e-mail address.
B. People try to put too much on the web page.
C. People have used a program incorrectly.
D. People have clicked on the wrong button.
Question 8. What comment does the author make about setting up a website?
A. It is usually fairly easy to do. B. You must use a good instruction guide.
C. It can be quite complicated. D. You should rely on your own skills.
Question 9. What word can best replace the word “script” in paragraph 5?
A. instruction B. picture C. handwriting D. play
Question 10. What does the author’s final sentence suggest about his work?
A. There’s a lot of money to be made in designing websites.
B. There are far too many websites on the Internet.
C. There’s a big chance of becoming unemployed.
D. There are more web page designers than necessary.
Your answers:
1. 2. 3. 4. 5.
6. 7. 8. 9. 10.
E. WRITING (5.0 points)
I. There is one mistake in each of the following sentences. Find out and correct them. (1.0 point)
Question 1. All of us have to study their lessons carefully if we expect to pass this examination.
Question 2. They didn’t seem to take any notice of that the teacher said.
Question 3. Maria asked Sandra whether she knew the post office had been at strike for the past week.
Question 4. It’s hardly to imagine how tedious the work of accountants and clerks used to be in the past
without a computer.
Question 5. Mr Smith, as well as his wife, often drink coffee in the morning, doesn’t he?
Question 6. Despite the metric system is used throughout the world, it is still not commonly used in the
United States.
Question 7. Many places of history, scientific, cultural, or scenic importance have been designated national
monuments.
Question 8. In 1837, Victoria, an eighteen-year-old woman, named Queen of England.
Question 9. Families who are fortunately enough to own a historic home may be able to get restoration
funds from the government.
Question 10. Cool temperatures, shade, moist, and the presence of dead organic material provide the ideal
living conditions for mushrooms.

Your answers:
Mistake Correction Mistake Correction
1. 6.
2. 7.
3. 8.
4. 9.
5. 10.

II. Finish each of the following sentences in such a way that it means exactly the same as the
sentence printed before it. (1.0 point)
Question 1. I should like someone to take me out to dinner.
 What …………………………………………………………………………
Question 2. The storm completely wiped out all my crops.
 I got .................................................................................................................
Question 3. He said he had won as a result of good luck.
 He attributed ………………………………………………………...............
Question 4. The truth only came out on the publication of the general’s personal diaries.
 Only when ……………………………………………………………...
Question 5. This problem cannot be solved instantly.
 There is ………………………………………………………………………
III. Rewrite each of the following sentences in such a way that it is as similar as possible in meaning to
the original sentence. Use the word given and other words as necessary. Do not change the form of the
given word. (1.0 point)

Question 1. There is no point in asking Peter to the party as he’s so busy. WORTH
…………………………………………………………………………………………..
Question 2. “It was my fault to break your vase yesterday” said Jane to her ADMITTED
brother.
…………………………………………………………………………………………..
Question 3. She was so beautiful that I couldn't stop looking at her. EYES
…………………………………………………………………………………………..
Question 4. We are looking forward to watching the program. WAIT
…………………………………………………………………………………………..
Question 5. Although we were impressed by the new restaurant, we found it AS
rather expensive.
…………………………………………………………………………………………..
IV. Some parents think that they can take advantage of modern technology to educate their
children at home. Others argue that the learning process can only be facilitated with the appearance of
teachers. Which opinion are you in favor of and Why?
Write an essay at least 250 words to express your opinion. (2.0 points)
………………………………………………………………………………………………………………………………
………………………………………………………………………………………………………………………………
…………………………………………………………………………………………………… …

THE END

ĐÁP ÁN

A. LISTENING (2.0 points) ( 0.2p for each correct answer)


Listen and complete the form below.
1. 48 2. R16 GH7 3. 07754 897 432 4. PA 365 5. E6
6.B 7. C 8.E 9. C 10. C

B. PHONETICS (2.0 points) ( 0.2p for each correct answer)


Identify the word whose underlined part is pronounced differently from that of the others. (1.0 point)
1. B 2. D 3. B 4. A 5. C
Identify the word whose stressed pattern is different from that of the others. (1.0 point)
6. A 7. D 8. C 9. A 10. B

C. VOCABULARY AND GRAMMAR (6.0 points)

I. Choose the best word or phrase to complete the following sentences.(2.0 points)
( 0.1p for each correct answer)
1. D 2. A 3. C 4. C 5. D
6. C 7. A 8. A 9. C 10. D
11. D 12. D 13. D 14. B 15. A
16. D 17. B 18. C 19. B 20. D
II. Put the verbs in blanket into the correct form. (2.0 points.) ( 0.2p for each correct answer)
1. did 2. campaigning 3. know 4.will have been sold 5. going

6. used 7. had ----come 8. Having been 9. is always entering 10. is


bitten

III. Write the correct form of the bold-typed words in the following passage. (2.0 points)
( 0.2p for each correct answer)
1. western 2. undoubtedly 3. originated 4. inhabitants 5. livelihood

6. unpredictable 7. traditionally 8. temporarily 9. guidance 10. reputedly


D. READING (5.0 points)

I. Read the following passage and choose the correct answer (A, B, C or D) to complete the passage.
(2.0 points) ( 0.2p for each correct answer)
1. B 2. A 3. D 4. A 5. B
6. C 7. C 8. A 9. D 10. B

II.Complete the following passage by filling in each blank with ONE suitable word. (1.0 points)
( 0.1p for each correct answer)
1. how 2. put 3. deal/face/cope 4. take 5. been
6. spent 7. a 8. grown 9. without 10. older

III.Read the following passage and choose the correct answer A, B, C or D (2.0 points)
( 0.2p for each correct answer)
1. B 2. C 3. D 4. D 5. D
6. A 7. C 8. C 9. A 10. B

E. WRITING (5.0 points)

I. There is one mistake in each of the following sentences. Find out and correct them .(1 point)
( 0.1p for each correct answer)
Mistake Correction Mistake Correction
1. their our 6. Despite Although/ Eventhough
2. that what 7. history historical
3. past previous 8. named was named
4. hardly hard 9. fortunately fortunate
5. drink drinks 10. moist moisture

II. Finish each of the following sentences in such a way that it means exactly the same as the sentence
printed before it. (1.0 point) ( 0.2p for each correct answer)
I. What I should like is being/to be taken out to dinner
II. I got all my crops completely wiped out by the storm.
III. He attributed his victory/ success to good luck.
IV. Only when the general’s personal diaries were published did the truth come out.
V. There is no/ not any instant solution to this problem.
III. Rewrite each of the following sentences in such a way that it is as similar as possible in meaning to
the original sentence. Use the word given and other words as necessary. Do not change the form of the
given word. (1.0 point) ( 0.2p for each correct answer)
1. It’s not worth asking Peter to the party as he’s so busy.
2. Jane admitted (to) having broken/breaking her brother’s vase the previous day.
3. She was so beautiful that I couldn't take my eyes off her.
4. We can't wait to watch the program.
5. Much as we were impressed by the restaurant, we found/ did find it rather expensive.
OR /Impressed as we were by the restaurant, we found/ did find it rather expensive.

IV. Some parents think that they can take advantage of modern technology to educate their
children at home. Others argue that the learning process can only be facilitated with the
appearance of teachers. Which opinion are you in favor of and Why? Write an essay at least 250
words to express your opinion. (2.0 points)

- Contents: a provision of all main reasons and appropriate supporting


ideas and relevant examples (1.0 pts)
Correct form of essay
- Language: a variety of vocabulary and appropriate structures (0.6pts)
writing (2.0 points)
- Presentation: coherence, cohesion, and appropriate style (0.4pts)

- Contents: a provision of all main reasons and appropriate supporting


ideas and relevant examples (0.8pts)
Incorrect form of essay - Language: a variety of vocabulary and appropriate structures (0.4pts)
writing(1.5 points) - Presentation: coherence, cohesion, and appropriate style (0.3p)

_______HẾT___________
ĐỀ THI CHỌN HỌC SINH GIỎI CẤP TỈNH
MÔN: TIẾNG ANH 12
PART ONE: PHONETICS
I. Choose the word whose underlined part is pronounced differently from that of the others )
1. A. supposedly B. markedly C. allegedly D. determinedly
2. A. endure B. feature C. procedure D. measure
3. A. complete B. command C. common D. community
4. A. comb B. climb C. debt D. cable
5. A. neighbour B. height C. sleigh D. weight
II. Choose the word whose stress pattern is different from that of the others
1. A. possibility B. disappointed C. manufacture D. instrument
2. A. environment B. mystery C. contribute D. terrific
3. A. deficiency B. psychology C. ecological D. competitor
4. A. recommend B. difficulty C. admirable D. document
5. A. encouragement B. interviewer C. acknowledge D. miraculously

1. Only after the atomic bomb ________ and development in the air travel _______, ______ science
fiction really become popular.
A. had created/ had taken off/ was B. had been created/ had been taken off/ has
C. had been created/ had taken off/ did D. had been created/ / had taken off/ had
2. We’ve bought some ________ chairs for the garden so that they are easy to store away.

3. I don’t think she can get her message _______ to the students. She seems too nervous.

4. _______, it is obvious that the whole thing was a waste of time and effort.
a. None of us wanted to go in the first place
b. Staff meetings are often boring and have no apparent point to them
c. Since the results were far more satisfactory than anyone had expected
d. Seeing that we couldn’t solve anything in the end

5. There are ______ words in English having more than one meaning. Pay close attention to this fact.
A. a large many B. quite many C. a great many D. quite a lot
VI. This car has many features including _________.
a. stereo, safety devices, air condition, and it saves gas
b. good music, safety devices, air conditioning, and gas
c. stereo, safety devices, air conditioned, and good gas
d. stereo, safety devices, air conditioning, and low gas mileage
VII. Round and round ___________.
A. the wheels of the engine went B. did the wheels of the engine go
C. went the wheels of the engine D. going the wheels of the engine

VIII. The replacement of shops such as the groceries and chemists’ by the café _______ the
housewives with insufficient facilities for shopping.
A. leave B. have left C. has left D. to have left
9. Your argument _______ that Britain is still a great power, but this is no longer the case.
A. outlines B. presupposes C. concerns D. presents
10. They are happily married although, of course, they argue _______.
A. most times B. from day to day C. every now and then D. on the occasion
11. He promised to mend the broken wheel soon without ___________ .
A. fail B. failure C. trouble D. mistake
12. One of the first exercises in math class is ______ measure the radius of a circle.
A. to learn and B. to learn how to C. learning to D. learn to
13. We were shocked to hear the news of your ________.
A. having fired B. being fired C. having been fired D. to have been fired
14. I don’t know French, but I’ll ________.
A. get Tom to translate it B. have it translate
C. have Tom to translate it D. make it translate

15. _______ as taste is really a composite sense made up of both taste and smell.
A. That we refer to B. What we refer to
C. To which we refer D. What do we refer to
16. _______ have settled, one of their first concerns has been to locate an adequate water supply.
A. Wherever people B. There are people who
C. Where people D. People
17. Politicians should never lose ______ of the needs of the people they represent.
A. view B. sight C. regard D. prospect
18. _______ team sports require cooperation.
A. Of all B. They are all C. Why all are D. All
19. Studies indicate _________ collecting art today than ever before.
A. there are that more people B. more people that are
C. that there are more people D. people there are more
20. Doctors advise people who are deficient __________ vitamin C to eat more fruit and vegetables.
A. from B. of C. in D. for
21. Dick put ____ ball in ____ net in _____ second half but ____ goal was disallowed.
A. a - a - a - a B. the - the - the - the
C. the - the - a - a D. a - a - the - the
22. There is a real possibility that these animals could be frightened _______ a sudden loud noise.
A. being there B. should there be C. there was D. there have been
23. The computer has dramatically affected ______ photographic lenses are constructed.
A. is the way B. that the way C. which way do D. the way
24. Six novels a year, you say? He’s certainly a __________ writer.
A. fruitful B. fertile C. virile D. prolific
25. The handwriting is completely ______ . This note must have been written a long time ago.
A. inedible B. indelible C. illegible D. unfeasible
II. Give the correct form or tense of the verbs in brackets.
IX. The children were frightened because the lights suddenly (1.go) ________ out and they
(2.sit) ________ in the dark.
X. What tune (3. play) ________ when we (4. come) ________ in?
XI. She was badly hurt when her car hit another car. If she (5. wear) ________ her seat
belt, she (6. not hurt) ________ so badly.
XII. It is vital that no one else (7. know) _______ about the secret government
operation.
XIII. It seems strange to be standing here, (8. look) _______ out at SydneyHarbor.
XIV. Tom had a lucky escape. He (9. kill) ________ when a car crashed into the front of
his house.
XV. _____ (10. Rank) as a masterpiece, a work of art must transcend the ideals of
the period in which it was created.
III. There are ten mistakes in the following passage. Find and correct them. (1.5 pt)
It seems that the mystery of why the Pyramids were built may have solved. Until quite recently
people got used to think that they were just tombs for pharaohs. Instead, the connection with
astronomy seems much more important. Egyptologists have often asked them how long it spent to
build them and why people built them in first place. Experts came up with a suggestion that the
Egyptians may have believed in the River Nile was the earthly equivalent of the Milky Way.
Many agree that the sizes of the three Giza Pyramids are in promotion to the three stars of Orion.
Nothing, then, was by the chance. Rather, the souls of dead pharaohs were deliberately being
project through shafts to reach at their goal of the Orion constellation.
IV. Give the correct form of the word in each of the following brackets.(1.5 pt)
It was not so long ago that we dealt with colleagues through face-to-face (1. INTERACT)
______
and with counterparts and customers by phone or letter. But the world of communication has
undergone a dramatic transformation, not for all the good. Email, while (2. DOUBT) _____ a
swift means of communication providing your server is fully (3.FUNCTION) ______ and that
the address you have contains no (4. ACCURATE) _____ has had a (5. SIGNIFY) _____ effect
on certain people’s behaviour, both at home and business. For those people, the use of email has
become irresistibly (6. ADDICT) _____ to the extent that it is (7. THREAT) _____ their mental
and physical health. Addicts spend their day (8. COMPULSION) _____ checking for email and
have a (9. TEND) ______ to panic if their server goes down. It is estimated that one in six people
spend four hours a day sending and receiving messages, the equivalent to more than two working
days a week. The negative effect on (10. PRODUCE) ________ is something employers are well
aware of.
PART THREE: READING (6.0 POINTS)
I. Read the following passage and choose the option that indicates the correct answer to each of
the following questions.(2.0 pts)
In the United States in the early 1800's, individual state governments had more effect on the
economy than did the federal government. States chartered manufacturing, banking, mining, and
transportation firms and participated in the construction of various internal improvements such as
canals, turnpikes, and railroads. The states encouraged internal improvements in two distinct
ways; first, by actually establishing state companies to build such improvements; second, by
providing part of the capital for mixed public-private companies setting out to make a profit.
In the early nineteenth century, state governments also engaged in a surprisingly large amount of
direct regulatory activity, including extensive licensing and inspection programs. Licensing
targets reflected both similarities and differences between the economy of the nineteenth century
and that of today: in the nineteenth century, state regulation through licensing fell especially on
peddlers, innkeepers, and retail merchants of various kinds. The perishable commodities of trade
generally came understate inspection, and such important frontier staples as lumber and
gunpowder were also subject to state control. Finally, state governments experimented with direct
labor and business regulation designed to help the individual laborer or consumer, including
setting maximum limits on hours of work and restrictions on price-fixing by businesses.
Although the states dominated economic activity during this period, the federal government was
not inactive. Its goals were the facilitation of western settlement and the development of native
industries. Toward these ends, the federal government pursued several courses of action. It
established a national bank to stabilize banking activities in the country and, in part, to provide a
supply of relatively easy money to the frontier, where it was greatly needed for settlement. It
permitted access to public western lands on increasingly easy terms, culminating in the
Homestead Act of 1862, by which title to land could be claimed on the basis of residence alone.
Finally, it set up a system of tariffs that was basically protectionist in effect, although
maneuvering for position by various regional interests produced frequent changes in tariff rates
throughout the nineteenth century.
XVI. What does the passage mainly discuss?
a. States's rights versus federal rights.
b. The participation of state governments in railroad, canal, and turnpike
construction.
c. The roles of state and federal governments in the economy of the
nineteenthcentury.
d. Regulatory activity by state governments.
XVII. The word “effect” in bold in paragraph 1 is closest in meaning to __________.
A. value B. argument C. influence D. restraint
XVIII. All of the following are mentioned in the passage as areas that involved
statevernments in the nineteenth century EXCEPT _________.
A. mining B. banking C. manufacturing D. higher education
XIX. The word “distinct” in bold in paragraph 1 is closest in meaning to_________.
A. separate B. innovative C. alarming D. provocative
XX. It can be inferred from the first paragraph that in the nineteenth century canals
and railroads were _________.
a. built with money that came from the federal government
b. much more expensive to build than they had been previously
c. built predominantly in the western part of the country
d. sometimes built in part by state companies
XXI. The regulatory activities of state governments included all of the following
EXCEPT_______.
A. licensing of retail merchants B. inspecting materials used in turnpike
maintenance
C. imposing limits on price-fixing D. control of lumber
XXII. The word “setting” in bold in paragraph 2 is closest in meaning to _________.

XXIII. The word “ends” in bold in paragraph 3 is closest in meaning to ________.

XXIV. According to the passage, which of the following is true of the Homestead Act
of 1862?
A. It made it increasingly possible for settlers to obtain land in the West.
B. It was a law first passed by state governments in the West.
C. It increased the money supply in the West.
D. It established tariffs in a number of regions.

XXV. Which of the following activities was the responsibility of the federal government in
the nineteenth century?
A. Control of the manufacture of gunpowder.
B. Determining the conditions under which individuals worked.
C. Regulation of the supply of money.
D. Inspection of new homes built on western lands.
II. Read the text below and fill in each blank with ONE suitable word. (2.0 pts)
The British are widely (1) _____ to be a very polite nation, and in (2)______ respects this is true.
An
Italian journalist once commented of the British that they need (3) _____ fewer than four “thank
you”
merely to buy a bus ticket. The first, from the bus conductor means, “I’m here.” The second
accompanies the handing over of the money. The third, again from the conductor, (4) ____ “Here
is your ticket.”, and then the passenger utters a final one as he accepts the tickets. Such
transactions in most (5) ____ parts of the world are usually conducted in total silence. In sharp
contrast to this excessive politeness with strangers, the British are strangely lacking (6) _____
ritual phrases for social interaction. The exhortation “Good appetite”, uttered in so (7) ______
other languages to fellow-diners before a meal, does not exist in English. The nearest equivalent –
Enjoy your dinner! – is said only by people who will not be partaking of the meal in question.
What’s more, the British (8) ____ happiness to their friends or acquaintances only at the start of a
new year and at (9)_____ such as birthdays, (10) _____ the Greeks routinely wish all and sundry
a “good week” or a “good month”.
Your answers:
1._____________ 2._____________ 3._____________ 4._____________
5.____________
6._____________ 7._____________ 8._____________ 9._____________
10.____________
XXVI. Read the passage and choose the best option for each of the following blanks.
(2.0 pts) Media and advertising
After more than fifty years of television, it might seem only obvious to conclude that it is here to
(1)
______. There have been many objections to it during this time, of course, and (2) ______ a
variety of
grounds. Did it cause eye-strain? Was the (3) ______ bombarding us with
radioactivity? Did the
advertisements contain subliminal messages, persuading us to buy more? Did children turn to
violence through watching it, either because so (4) ______ programmes taught them how to
shoot, rob, and kill, or because they had to do something to counteract the hours they had spent
glued to the tiny screen? Or did it simply create a vast passive (5) ______ drugged by glamorous
serials and inane situation (6) ______ ? On the other hand did it increase anxiety by
sensationalizing the news [or the news which was (7) ______ by suitable pictures] and
filling our living rooms with war, famine and political unrest? (8) ______ in all, television proved
to be the all-purpose scapegoat for the second half of the century, blamed for everything, but
above all, eagerly watched. For no (9) ______ how much we despised it, feared it, were bored by
it, or felt that it took us away from the old paradise of family conversation and hobbies such as
collecting stamps, we never turned it off. We kept staring at the screen, aware that our own tiny
(10) ______ was in if we
looked carefully.
1. A. be B. stay C. exist D. prolong
2. A. with B. over C. by D. on
3. A. screen B. danger C. machine D. reason
4. A. that B. far C. many D. what
5. A. programme B. personality C. audience D. tense
6. A. comedies B. programmes C. perhaps D. consequently
7. A. taken B. presented C. capable D. accompanied
8. A. Taken B. All C. Somewhat D. Thus
9. A. one B. matter C. difference D. reason
10. A. fault B. reflection C. situation D. consciousness
PART FOUR: WRITING (6.0 POINTS)
I. Finish the second sentence in such a way that it means exactly the same as the sentence
printed before it. (2.0 pts)
1. If the work is finished by lunchtime, you can go home.
→Get_____________________________________________________________________
XXVII. You haven’t done your work, have you?
→It’s about___________________________________________________________________
3. The fourth time he asked her to marry him, she accepted.
→Only on his __________________________________________________________________
4. He said that he had won as a result of good luck.
→He attributed_________________________________________________________________
5. That reminds me of the time I climbed to the top of Mount Fuji.
→That takes me_________________________________________________________________
6. People rumour that he is rich but stingy.
→What_______________________________________________________________________
XXVIII. Such a ridiculous proposal isn’t worth serious consideration.
→There is_______________________________________________________________________
8. Just thinking about his face at that moment makes me laugh.
→The very______________________________________________________________________
9.We cannot see animals in a vast area after the forest fire.
→There is an_____________________________________________________________________
10.The staff hated Frank’s new policies so intensely that they went on strike.
→So intense
______________________________________________________________________
XXIX. Write a new sentence similar in meaning to the given one, using the word given
in the brackets. Do not alter the word in any way. (2.0 pts)
1. I suddenly realized the meaning of a “freebie”.(dawned)
→_______________________________________________________________________

________
2. After the scandal, he was asked to resign. (HAND)
→_______________________________________________________________________

________
3. Bruce said that the situation at work was like a family argument. (likened)
→_______________________________________________________________________

________
4. My father is not feeling well these days. (weather)
→_______________________________________________________________________

________
5. I don’t think this record will ever be popular. (catch)
→_______________________________________________________________________

________
6. His arrival was completely unexpected. (took)
→_______________________________________________________________________

________
7. The success of our local theater has made our city famous. (map)
→_______________________________________________________________________

________
8. He is certainly not a reliable witness. (means)
→_______________________________________________________________________

________
9. Our company is the only company allowed to import these chemicals. (monopoly)
→_______________________________________________________________________

________
10. It’s uncertain whether the band’s tour will take place. (BALANCE)
→_______________________________________________________________________

________
III. Essay writing (2.0 pts)
Some high schools require all students to wear school uniforms. Other high schools permit
students to decide what to wear to school. Which of these two school policies do you think is
better?
In about 250 words, write an essay to expess your opinion, usingspecific reasons and examples
to support your opinion.
(You may continue your writing on the back page if you need more space.)
………………………………………………………………………………………………………
….
………………………………………………………………………………………………………
….
………………………………………………………………………………………………………
….
………………………………………………………………………………………………………
….
………………………………………………………………………………………………………
….
………………………………………………………………………………………………………
….
………………………………………………………………………………………………………
….

ĐÁP ÁN
PART ONE: PHONETICS (1.0 POINT)

I. Choose the word whose underlined part is pronounced differently fromthat of the others. (0.5
pt)
1.D 2.A 3.C 4.D 5.B
II. Choose the word whose stress pattern is different from that of the others.(0.5 pt)
1. D 2. B 3. C 4. A 5. B

PART TWO: LEXICO-GRAMMAR (7.0 POINTS)


I. Choose the best option to complete each of the following sentences.(2.5pts)
1. C 2. D 3. A 4. D 5.C 6. D 7. C
8. C 9. B 10. C 11. A 12. B 13. C 14. A
15. B 16. A 17. B 18.D 19. C 20. C 21. B
22. B 23. D 24. D 25. C
II. Give the correct form or tense of the verbs in brackets. (1.5 pt)
1. had gone 2. were sitting
3. was being played 4. came
5. had been wearing 6. wouldn’t have been hurt
7. (should) know 8. looking
9. could have been killed 10. To be ranked
III. There are ten mistakes in the following passage. Find and correct them. (1.5 pt)

It seems that the mystery of why the Pyramids were built may (1. have solved have been solved). Until quite

recently people got used (2. to think


→to thinking) that they were just tombs for pharaohs. Instead, the connection

with astronomy (3.seem


→ seems) much more important. Egyptologists have often asked (4. them → themselves)

how long it (5. spent


→ took) to build them and why people built them in (6. first place → the first place).

Experts came up with a suggestion that the Egyptians may have (7. believed in
→ believed that) the River Nile was
the earthly equivalent of the Milky Way. Many agree that the sizes of the three Giza Pyramids are in propotion to the

three stars of Orion. Nothing, then, was (8. By the chance


→ by chance). Rather, the souls of dead pharaohs were

deliberatedly being (9. project


→ projected) through shafts to (10. reach at → reach) their goal of the Orion
constellation.

No Line Mistake Correction


1 1 have solved have been solved
2 2 to think to thinking
3 3 seem seems
4 3 them themselves
5 3 spent took
6 4 first place the first place
7 5 believed in believed that
8 7 by the chance by chance
9 7 project projected
10 8 reach at reach

IV. Give the correct form of the word in each of the following brackets. (1.5 pt)
1. INTERACTION 2. UNDOUBTEDLY 3. FUNCTIONAL
4. INACCURACIES 5. SIGNIFICANT 6. ADDICTIVE
7. THREATENING 8. COMPULSIVELY 9. TENDENCY
10. PRODUCTION

PART THREE: READING (6.0 POINTS)


I. Read the following passage and choose the option that indicates the correct answer to each of
the
following questions.(2.0 pts)
1. C 2. C 3. D 4. A 5. D
6. B 7. C 8. D 9. A 10. C
II. Read the text below and fill in each blank with ONE suitable word. (2.0 pts)
1. considered 2. some 3. no 4. means 5. other
6. in 7. many 8. wish 9. celebrations 10. while
III. Read the passage and choose the best option for each of the following blanks. (2.0 pts)
1.B 2.D 3.A 4.C 5.C
6.B 7.D 8.B 9.B 10.B

PART FOUR: WRITING (6 POINTS)


I. Rewrite the following sentences beginning with the given words. (2.0 pts)

XXX. Get the work finished by lunchtime and you can go home.
XXXI. It’s about time you did your homework.
XXXII. Only on his fourth proposal did she accept to marry him.
XXXIII. He attributed his win/ victory/ success/ achievement/ triumph to good luck.
XXXIV. That takes me back to the time I climbed to the top of Mount Fuji.
XXXV. What people rumour is that he is rich but stingy.
XXXVI. There is no point in considering such a ridiculous proposal seriously.
XXXVII. The very thought of his face at that moment makes me laugh.
9.There is an absence of animals in a vast area after the forest fire.
XXXVIII. So intense was the hatred for Frank’s new policies that the staff went on
strike.

XXXIX. Write a new sentence similar in meaning to the given one, using the word given
in the brackets. Do not alter the word in any way. (2.0 pts)

a. It suddenly dawned on me what the meaning of a “freebie” was.


b. After the scandal, he was asked to hand in his resignation.
c. Bruce likened the situation at work to a family argument
d. My father is feeling (a bit) under the weather these days.
e. I don’t think this record will ever catch on.
f. His arrival took us (completely) by surprise.
g. The success of our local theater has put our city on the map.
h. He is by no means a reliable witness.
i. Our company has got the monopoly of/on/in importing these chemicals.
j. The band’s tour is in the balance.

IV. Essay writing (2.0 pts) Marking scheme

Markers should discuss the suggested answers and the marking scale thoroughly before
marking the papers.

The mark given to part III is based on the following scheme:


Mô tả tiêu chíđánh giá Điểm tối đa
1. Bố cục 0.40
• Câu đề dẫn phải thể hiện được rõ ý kiến của người viết 0.10
• Bố cục hợp lí, rõ ràng phù hợp với yêu cầu của đề bài và đầy đủ 3 phần: mở 0.15
bài, thân bài, kết luận.
• Bố cục uyển chuyển từ mở bài đến kết luận 0.15
2. Phát triển ý 0.40
• Phát triển ý có trình tự logic và mạch lạc 0.20
• Có giải thích, dẫn chứng, ví dụ xác thực đủđể bảo vệý kiến của người viết 0.20
3. Sử dụng ngôn ngữ 0.40
• Sử dụng ngôn từ phù hợp với nội dung của bài viết 0.10
• Sử dụng ngôn từđúng văn phong/ thể loại, đa dạng về từ vựng và cấu trúc. 0.15
• Sử dụng từ nối các ý cho bài viết uyển chuyển 0.15
4. Nội dung 0.40
•Đủ thuyết phục người đọc 0.15
•Đủ dẫn chứng, ví dụ, lập luận 0.15
•Độ dài: Số từ không nhiều hơn hoặc ít hơn so với quy định 10 % 0.10
5. Ngữ pháp, dấu câu và chính tả 0.40
• Sử dụng đúng dấu câu 0.10
• Chính tả: Viết đúng chính tả 0.10
- Lỗi chính tả gây hiểu nhầm/ sai lệch ý sẽ bị tính một lỗi (trừ 1% điểm của
bài viết)
- Cùng một lỗi chính tả lặp lại chỉ tính là một lỗi
• Sử dụng đúng thời, thể, cấu trúc câu đúng ngữ pháp. (Lỗi ngữ pháp gây hiểu 0.20
nhầm/ sai lệch ý sẽ bị trừ 1% điểm bài viết.)
Tổng 2.00
KÌ THI CHỌN HSG TỈNH MÔN TIẾNG ANH 12

Thời gian : 150 phút (không kể thời gian giao đề)


SECTION A – PHONETICS

I. Identify the word whose stressed pattern is different from that of the others.

1. A. already B. complain C. arrangement D. temperature


2. A. inspire B. wealthy C. protect D. instinct
3. A. advance B. ancient C. cancer D. annual
4. A. mathematics B. statistics C. academy D. mechanic
5. A. gravitation B. behaviour C. manufacture D. recommend

SECTION B – VOCABULARY AND GRAMMAR

I. Choose the best answer from A, B, C or D.


1. In his student days he was as poor as a church ............................... .
A. beggar B. miser C. mouse D. pauper
2. She may have been poor, but she was ............................... honest.
A. finally B. in the end C. at least D. at last
3. The manager was very ....................... with me about my prospects of promotion.
A. sincere B. friendly C. just D. frank
4. The unmarried ladies regard him as a very ............................ young man.
A. ineligible B. illegible C. illicit D. eligible
5. Mr Lazybones ............................. to work harder in future.
A. excepted B. agreed C. accorded D. accepted
6. He believed that promotion should be awarded on ........, not on length of service.
A. equality B. merit C. characteristics D. purposes
7. It is a criminal offence to ................................... the facts.
A. oppress B. suppress C. repress D. express
8. He ....................... the cart before the horse by buying the ring before he had proposed to her.
A. fastened B. tied C. put D. coupled
9. Every delicacy Miss Cook produces is done ................................... .
A. there and then B. at will C. sooner or later D. to a turn
10. She tells her small boy everyday not to be rude, but it’s like water off a duck’s .................... .
A. wings B. beak C. back D. feathers
11. Announcing that he was totally done ......................... , Grandfather retired to bed.
A. out B. with C. in D. down
12. Oliver Twist had already had his fair ............................... of food.
A. ratio B. help C. ration D. division
13. Some great men have had an ............................. school record.
A. indistinct B. indistinguishable C. extinguished D. undistinguished
14. Buyers and sellers were ............................... over prices.
A. hacking B. hugging C. heckling D. haggling
15. Within a few weeks all this present trouble will have blown ......................... .
A. along B. over C. out D. away
16. The six (and last) volume in the series is ....................... with its predecessors.
A. uniform B. similar C. like D. identical
17. Politicians often promise to solve all a country’s problems ............................. .
A. thick and fast B. on the whole C. of set purpose D. at a stroke
18.When the detectives finally trapped him, he had ........................ to lying.
A. resource B. retort C. resort D. recourse
19.My late grandmother ........................... me this silver teapot.
A. bequested B. willed C. bequeathed D. inherited
20.It was getting ............................. midnight when he left.
A. on B. on to C. to D. past
2, The following paragragh has 10 mistakes. Identify the mistakes in the lines and correct
them. Write your answer in the space provided.
COMMUNICATION
Managers spend most of their free time communicating - reading,writing, talking or listening. Yet
the evidence is they do not always do this successfully. One reason that has been suggested for this
is that, in the past, communication was regarded like a natural process, not taught in any formal
sense. This theory has been changing, and the concept in communication as an “art” now appears
regularly in management courses and seminars. Communication is probably one of the least
appreciated aspect of management, and more and more organisations are realising that effective
communication involves to tell staff why things are happening. This not only helps day - to - day
working but allows changes to be introducing more smoothly, and sometimes leads to improvements
being mentioned by staff. Both the morale and efficiency of an organisation depends to a great
extent on the abilities of its staff to communicate effectively. Communication is not something
which should be undertaken only when trouble occurs. I should be a daily habit if the organisation is
to run smoothly and avoid difficulties and, of course, it should be the two - way process, involving
listening as well as talking. Regular exchanges of ideas between managers and staff will help to
create a good teamwork.

Mistake Correction Mistake Correction


1. 6.
2. 7.
3. 8.
4. 9.
5. 10.
3. Use the correct form of each of the words given in parentheses to fill in the blank in
each sentence.
WEDDING OF STRANGERS
A wedding between two strangers who met for the first time when they exchanged marital vows
during a peak time radio broadcast has come in for widespread (1)............................ (critic).
Carla Germaine and Greg Cordell were married after winning each other in a ‘lonely heart’
competition organized by BRMB radio station. The service, perhaps (2)............................. (surprise),
attracted the highest ratings figures of the year.
The model and salesman were (3)....................................... (dismiss) of their critics and say
they have made a serious (4)................................... (commit) to make their marriage work.
‘Everyone seems to have the (5)................................... (expect) that we will split up, but we’re
going to prove them wrong,’ Cordell said (6)............................ (defy).
The couple were selected from 200 (7)...................................... (hope) candidates by a panel
including (8).................................. (relate) counsellors and an astrologer. As well as each other,
they won a free honeymoon in the Bahamas, a sports car and a luxury two bedroom apartment.

ANIMALS

Most cat ansd dog owners would swear their pet was virtually human. It’s pleased to see you and
shows its disapproval when they go. It may not be particularly (9).................................... (talk) or a
genius mathematics but it sees grass as green and inhabits as rich world of smells
(10).............................. (imagine) to us. Until recently such notions of a pet’s inner life, with
(11)................................... (similar) to our own in some ways would have been met with a
(12).................................... (know) sneer in many respected (13)................................. (science)
circles. Nowdays in fact, claiming (14)................................ (conscious) for your pets is
commonplace. The problem now is providing an adequate (15)............................... (define) of
what this actually means. Is it about having sensations like hunger and pain, or is it more about the
ability to be aware that you are experiencing something?

SECTION C – READING
I. Read the following passage, and then choose the best answer from A, B, C,D.
Printers use the term broadside to refer to a large piece of paper printed on one side. In military
language, it means an attack with all one’s forces. Dudley Randall invoked both these senses of the
word when he established the Broadside Press in 1965. Randall was a librarian and poet in Detroit
when he began the Press with his personal savings as a way to copyright the words to his ballad about
a 1963 racial incident in which Whites killed three Black children. The poem was printed as a
broadside.
“By creating the Broadside Press, the most successful poetry institution in the history of African
American literature. Randall created something that had previously not existed in the United States -
an organization that would publish the works of Black poets,” explains Professor Melba Boyd, a
poet and former Press editor. Historically, work by Black poets had been criticized for emphasizing
political issues and not using the traditional poetic forms of the White literary establishment. Thus,
Black poets had found it difficult to get published.
Boyd is producing a film documentary that will present Randall’s biography as well as his poetry.
Randall served as general editor of the Press from 1965 to 1977. In the mid-seventies sky-rocketing
printing costs and the closing of many small bookstores to whom he had extended credit left the Press
in financial straits. Randall then sold the Press and slumped into a depression, but in the 1980’s, he
revived community support for the Press through the Broadside Poets Theater. Boyd hopes her
documentary on Randall will introduce more people to African American literature.
VI. According to the passage, the Broadside Press is the most famous as a publisher of …………
A. criticism of traditional White poetry.
B. biographies of famous African American poets.
C. poetry written by African Americans.
D. African American documentaries.
VII. Who paid the cost to start the Press?
A. An organization Black writer B. Dudley Randall
C. Professor Boyd D. Many small bookstores
VIII. According to Professor Boyd, what significant change occurred because of the Broadside Press?
A. Black poets returned to traditional poetic forms.
B. Historical works about African Americans began to appear in print.
C. The Black literary establishment began to emphasize political issues.
D. It became easier for Black poets to get their work in print.
IX. What happened to the Broadside Press in the 1980’s?
A. It was renamed the Broadside Poets Theater.
B. It moved into a different community.
C. It regained popular support.
D. It helped support small bookstores during a depression.
X. What did the Broadside Poets Theater do?
A. helped get support for the Broadside Press.
B. led Randall into a personal depression.
C. led the Broadside Press into financial difficulties.
D. supported many bookstores in the community.
II. Choose the best answer from A, B, C or D to fill in the gaps in the following passage.

HELP ALWAYS AT HAND:


A MOBILE IS A GIRL’S BEST FRIEND
If it fits inside a pocket, keeps you safe as well as in touch with your office, your mother and your
children, it is (1)……….. worth having. This is the (2)……….…. of the (3)…………
ranks of female mobile-phone users who are beginning to (4)……………. the customer market.
Although Britain has been (5)…….…… to be one of the most expensive places in the world
to (6)………. a mobile phone, both professional women and (7)…...….. mothers are underterred.
At first, the mobile phone was a rich man plaything, or a businessman’s (8)………........ symbol.
Now women own almost as telephones as men do - but for very different reasons. The
main (9)…...…. for most women customers is that it (10)……….. a form of
communications back-up, wherever they are, in case of (11)……..… . James Tanner of Tancroft
Communications says: ‘The (12)…….… of people buying phones from us this year were women
often young women - or men who were buying for their mothers, wives and girlfriends. And it
always seems to be a question of (13)………….of mind. ‘Size is also (14)…………. for women.
They want something that will fit in a handbag,’ said Mr Tanner, ‘The tiny phones coming in are
having a very big (15)…………. . This year’s models are only half the size of your hand.’

1. A. totally B. certainly C. absolutely D. completely


2. A. vision B. vista C. view D. panorama
3. A. swelling B. increasing C. boosting D. maximising
4. A. master B. dominate C. overbear D. command
5. A. demonstrated B. shown C. established D. seen
6. A. function B. drive C. work D. run
7. A. complete B. total C. full-time D. absolute
8. A. prestige B. fame C. power D. status
9. A. attraction B. enticement C. charm D. lure
10. A. supplies B. furnishes C. provides D. gives
11. A. urgency B. emergency C. predicament D. contingency
12. A. most B. preponderance C. majority D. bulk
13. A. tranquility B. calmness C. serenity D. peace
14. A. crucial B. necessary C. urgent D. essential
15. A. impact B. impression C. perception D. image
III. Fill in each numbered gap with one suitable word.
In a village on the east coast of Scotland, people were waiting anxiously for news. Two of their
fishing-boats (1)………..…. been caught in the storm (2)……….…… had blown up during
the night. In the cottages round the harbour, people stood by their door, (3)…………….. worried
to talk.
The rest of the fishing fleet had (4)……….…. The harbour before dark, and the men from
these ships waited and watched with the wives and families of (5)……….… missing men. Some
had brought thick blankets and some flasks of hot drinks, knowing that the men (6)……………..
be cold and tired. As dawn began to break over in the east, a small point of light was spotted in
the darkness of the water and a (7)….……… minutes later, (8)…………… was a shout. Before
long, the two boats were turning in, past the lighthouse, to the inside of the harbour. The men
(9)………... helped out of their boats, and although they were stiff (10)…….. cold and tiredness,
they were all safe.
SECTION D – WRITING
I. Finish each of the following sentences in such a way that it is as similar as possible in
meaning to the sentence printed before it.
1. Melissa’s father was very busy, but he still played with her.
Busy .....................................................................................................................
2. Mrs Wilson says she’s sorry she didn’t attend the meeting yesterday morning.
Mrs Wilson sends ................................................................................................
3. It’s almost nine months since I stopped subscribing to that magazine.
I cancelled ............................................................................................................
4. For further information, please send a self-addressed envelope to the above
address.
Further information can .......................................................................................
5.Richard only took over the family business because his father decided to retire early.
But for hi............................................................................................................
6. I have called this meeting in order to present the latest sales figures.
My purpose ..........................................................................................................
7. Skyscrapers in the USA are on average taller than anywhere else in the world.
The average.........................................................................................................
8. I was surprised at how easy he was to talk to.
I hadn’t expected .................................................................................................
9. Experts think that all dogs evolved from wolves.
All dogs are ..........................................................................................................
10. The two sides never looked likely to reach an agreement.
At no time ............................................................................................................

B. Some people think that the use of computers should be restricted to


reduce its harmful effect on children. Do you agree or disagree? Write
a composition of 200 words, giving your opinion on the topic.

ĐÁP ÁN VÀ HƯỚNG DẪN CHẤM MÔN TIẾNG ANH-KHỐI 12

SECTION A – PHONETICS
I. Choose the word whose underlined part is pronounced differently from that of the others. (1/1)
1. C 2. C 3. C 4. A 5. D
II. Identify the word whose stressed pattern is different from that of the others.(1/1)
1. D 2. B 3. A 4. A 5. B
SECTION B – VOCABULARY AND GRAMMAR
I. Choose the best answer from A, B, C or D. (1/1)
1. C 2. C 3. D 4. D 5. B
6. B 7. B 8. C 9. D 10. C
11. C 12. C 13. D 14. D 15. B
16. A 17. D 18. D 19. C 20. A
C. Identify the mistakes in the lines and correct them. Write your answer in the space
provided. (1/1)

Mistake Correction Mistake Correction


1.is they is that they 6. introducing introduced
2.like as 7. depends depend
3.regularly in regularly of 8. which that
4.aspect aspects 9. the a
5.to tell telling 10. a good teamwork good teamwork
D. Use the correct form of each of the words given in parentheses to fill in the blank in
each sentence. (1/1)

1. criticism 2. surprisingly 3. dismissive 4. commitment 5. expectation


6. defiantly 7. hopeful 8. relationship 9. talkative 10. unimaginable
11. similarities 12. knowing 13. scientific 14. consciousness 15. definition

SECTION C – READING

I. Read the following passage, and then choose the best answer from A, B, C or D. (2/1)
1. C 2. B 3. D 4. C 5. A

II. Choose the best answer from A, B, C or D to fill in the gaps in the following passage. (1/1)
1. B 2. C 3. A 4. B 5. B
6. D 7. C 8. D 9. A 10. C
11. B 12. C 13. D 14. A 15. A

III. Fill in each numbered gap with one suitable word. (1/1)
1. had 2. which/that 3. too 4. reached 5. the
6. would 7. few 8. there 9. were 10. from/with

SECTION D – WRITING

I. Complete the second sentence so that it has similar meaning to the first one. (1/1)
E. Busy as/though he was, Melisa’s father still played with her.
F. Mrs Wilson sends her apologies for not having attended the meeting yesterday
morning.
G. I cancelled my subscription to that magazine nine months ago.
H. Further information can be obtained by sending a self-addressed envelope to the
above address.

I. But for his father’s early retirement, Richard would not have taken over the
family business.
J. My purpose in calling this meeting is to present the latest sales figures.
K. The average skycraper in the USA is higher than anywhere else in the world.
L. I hadn’t expected (that) he would be/ him to be so easy person to talk.
M. All dogs are thought to have evolved from wolves.

10.At no time did the two sides look likely to reach agreement.
N. Composition. ( 20 points )
a. Vocabulary and Grammar: 30 %
b. Style: 40 %
c. Content: 30 %
KỲ THI CHỌN HSG CẤP TRƯỜNG
ĐỀ THI MÔN: TIẾNG ANH 12
(Thời gian làm bài 150 phút, không kể thời gian giao đề)
A. LISTENING
Part 1: You will hear people talking in eight different situations. Choose the best answer (A, B or C)
and write your answers in the corresponding numbered boxes.
01. You hear the beginning of a lecture in a university. What is the lecture going to be about?
A. the history of a place B. social problems C. a person's life and work
2. You hear part of a radio programme about an island. Why are there so few trees on the island
now?
A. because of urban development
B. because of the expansion of agriculture
C. because of the action of the sea
3. You hear a teenage boy talking on the radio about his family's efforts to earn money. What
will the money be used for?

4. You hear an announcement at a railway station. What should you do if you want to go to London?
A. await further instructions
B. travel from a different platform
C. get on the next train to arrive
5. You hear a man talking about newspapers. What does he say about the newspaper he reads?
A. It is an essential part of life.
B. It is an important source of information.
C. It is useful for passing the time.
6. You hear a conversation on the radio. What is the programme about?
A. solving traffic problems in cities
B. studying nature and wildlife
C. finding part of an ancient town
7. You hear two people talking about a music festival they have been to. What do they agree
about?
A. the quality of the performances
B. the fairness of the prices
C. the standard of the accommodation
8. You hear a travel writer speaking on a radio programme. What aspect of his travels is he talking
about?
A. what he does to keep healthy
B. how he reduces the risk of accidents
C. illnesses from which he has suffered
Part 2: For question 09-15, complete the form below. Write NO MORE THAN TWO WORDS AND /
OR A NUMBER for each answer
Moving Company Service Report
Phone number: (09)________________
USA Address: 509 (10)________________1137 (11)________________in Seattle
Packing day: (12)________________ Date: 11th March
Clean-up by: (13)________________ Day: (14)________________
About the Price: Rather expensive Storage time: (15)________________
B. LEXICO-GRAMMAR
I. Choose the best option A, B, C, or D to complete each of the following sentences.
16. In the hotel lobby the detective caught_________ of the man he had been hired to follow.
A. glance B. view C. vision D. sight
17. ‘Go on, finish the food. It needs_________ up because it won’t stay fresh until tomorrow.
A. eat B. eating C. to eat D. eaten
18. Before he left on his trip to America, the young man promised his parents he_________
them regularly.
A. is writing B. will write C. should write D. would write
19. Of course I didn’t break it_________ purpose- it was an accident!
A. by B. with C. from D. on
20. Andrew couldn’t_________ himself laughing at the expression on Maggie’s face.
A. help B. stop C. escape D. avoid
21. _________ young, chimpanzees are easily trained.
A. When are B. When C. They are D. When they
22. “What’s your proposal?” “I propose that the meeting_________.”
A. is postponing B. be postponed C. to be postponed D. postpones
23. Most of the courses at the banquet were completely consumed, but there_________ food still remaining.
A. were few B. was little C. were a few D. was a little
24. The four-storey house_________ on that hill is still new.
A. be built B. building C. built D. being
building
25. Nam Cao devoted most of his time_________
A. to having written B. to write C. to writing D. to have written
26. I don’t think he will join us, _________?
A. doesn’t he B. won’t he C. will he D. don’t I
27. The young generation must contribute to_________ the country.
A. building of B. the building C. the building of D. build
28. I cannot help feeling anxious_________ the exam results.
A. of B. with C. about D. for
29. In some vocational schools, the training quality is worse than_________ used to be.
A. they B. it C. them D. that
30. When Elvis Presley died, her daughter came_________ a fortune.
A. into B. up with C. by D. across
24. The following passage contains ten errors. Find and correct them.

25. Read the text below, using the word given in brackets to form a word that fits in the space.
Congratulations to all involved with the school (41. PRODUCE)________ of The Woman Next
Door. The (42. ADVERTISE)________ was carried out by the Art Department, and the posters were
very (43. IMAGINE)________. We certainly have some very (44. ART)________ students in our
school! Many people helped with building and painting the (45. SCENE)________ and the play was
written by the English Department, who managed to create an (46. AMUSE)________ story, with
excellent songs. The music was written by Sue Porter, who also (47. COMPANY)________ the singers
on the piano. Everyone enjoyed a thoroughly (48. ENTERTAIN)________ evening, and there was a
long round of (49. APPLAUD)________ at the end. Jim Barrett gave a brilliant performance as
Sergeant Moss, and Liz Aiken was a (50. DELIGHT)________ Mrs. Jump. Well-done everyone!
V. For each question, write one word which can be used in all three sentences.
For example: - I was so tired that I couldn’t even think________. Key: STRAIGHT
I will come________ to the point – you are fired.
It’s time to set the record________ about what really happened that night.
51. - I was just________ to go home, but that’s okay.
a. It’s________ time you got home – We’ve been worried sick!
b. We’re definitely going to have to do something________ increasing our market share.
52. - I should get a reply from them any now.
a. It will only take them a________ to update the website.
b. Now is the________ of truth!
53. - We’re definitely living in the________ of information.
a. This particular wine doesn’t really mature with________.
b. It’s time you started acting your________, young man!
54. - I do hope we’re going to arrive________ time.
a. How anyone could work in that office for years________ end I just don’t know.
b. Just bet me that Doug was going to get the sack so I said: “You’re________.”
55. - To my________, the boss said that I could have an extra day off work.
a. Martin always takes great________ in talking to anyone about fishing.
b. When Erin finally started to move downhill on the skis for the first time, she laughed in
________.
56. - The local council should start a building program to meet the________ for leisure activity.
a. The Major said he had no________ of advice from people who didn’t know the situation.
b. This area is badly in________ of financial investment.
57. - Oh, tell Carrie that I send my________ when you see her.
a. When my mum and dad met, my mum says it was________ at first sight.
b. My________ for travelling comes from the fact that we moved around a lot.
58. - We don’t go out during the week as a________, but we’ll make an exception tonight.
a. People must be made to respect the________ of law, or we’ll have anarchy.
b. A good________ of thumb is that you should allow 15 minutes per exercise in the exam.
59. - Did you hear that the guy who lives in the flat upstairs has been________ with burglary?
a. I couldn’t believe they________ me three euros for a bottle of water!
b. The children ran out of the school gate and________ down the hill.
60. - The head teacher punished Aaron severely in order to make an________ of him.
a. Why can’t you follow your brother’s________ and go to university.
b. Give me one________ of a place round here where young people can go in the evenings.
C. READING
I. Read the text below and decide which answer A, B, C, or D, best fits each space.
Everyone wants to reduce pollution. But the pollution problem is (61)_________ complicated
as it is serious. It is complicated (62)_________ much pollution is caused by things that benefit
people.
(63)_________, exhaust from automobiles causes a large percentage of air pollution. But the
automobile (64)_________ transportation for millions of people. Factories discharge much
(65)_________ the material that pollutes the air and water, but factories give employment to a large
number of people.
Thus, to end (66)_________ greatly reduce pollution immediately, people would have to
(67)_________ using many things that benefit them. Most of the people do not want to do that, of
course. But pollution can be (68)_________ reduced in several ways. Scientists and engineers can
work to find ways to lessen the (69)_________ of pollution that such things as automobiles and
factories cause. Governments can pass and enforce laws that (70)_________ businesses and traffic to
stop, or to cut down on certain polluting activities.

61. A. as B. more C. less D. like


62. A. so B. while C. though D. because
63. A. Specific B. For example C. Such as D. Like
64. A. takes B. affords C. carries D. provides
65. A. about B. for C. of D. with
66. A. or B. and C. as well D. then
67. A. start B. continue C. stop D. go on
68. A. carefully B. unexpectedly C. gradually D. little
69. A. way B. figure C. number D. amount
70. A. forbid B. prevent C. request D. require
61. Read the passage and choose the correct answer A, B, C, or D for each question.
Bill Jarvis took over our village news agency at time of life when most of us only want to relax.
He just thought he would like something but not too much to do, and the news agency was ready- made.
The business produced little enough for him, but then Bill was a chap who only wanted the simplicity
and order and regularity of the job. He had been a long-serving sailor, and all his life had done
everything by the clock.
Every day he opened his shop at six a.m. to catch the early trade; the papers arrived on his door-
step before that. Many of Bill's customers were city workers, and the shop was convenient for the
station. Business was tailing off by ten o'clock, so at eleven sharp Bill closed for lunch. It was hard
luck on anybody who wanted a paper or magazine in the afternoon, for most likely Bill would be down
on the river bank, fishing, and his nearest competitor was five kilometres away. Sometime in the
afternoon, the evening papers landed on the door-mat, and at four o'clock Bill reopened. The evening
rush lasted till seven, and it was worthwhile.
He lived in a flat above the shop, alone. Except in very bad weather, you always knew where to
find him in the afternoons, as I have said. Once, on a sunny afternoon, I walked home along the river
bank from a shopping trip to the village. By my watch it was three minutes past four, so I was
astonished to see Bill sitting there on his little chair with a line in the water. He had no luck, I could see,
but he was making no effort to move.
"What's wrong, Bill?" I called out from the path. For answer, he put a hand inside his jacket and
took out a big, golden object. For a moment I had no idea what it could be, and then it suddenly went
off with a noise like a fire engine. Stopping the bell, Bill held the thing up and called back: "Ten to four,
you see, and this is dead right." He stood up then and began to wind in his line. I had never known
anyone carry a brass alarm clock round with him or her before.
71. Bill Jarvis became a newsagent when__________
A. He needed the money. B. He was quite an old man.
C. He decided to take things easy. D. He gave up clock repairing.
72. What does the passage tell us about the news agency?
A. It was an easy job with fixed hours.
B. It was a very profitable business
C. It was opened specially for Bill Jarvis
D. It belonged to the railway and was part of the station.
73. Why did Bill open the shop so early in the day?
A. He liked to do as much as possible before he went to work.
B. Bill was never sure of the time.
C. The shop had to be open when the morning papers came.
D. It was then that he did a lot of business.
74. We understand from the passage that the shop closed for lunch__________
A. At eleven o'clock more or less. B. At exactly eleven o'clock.
C. Before eleven o'clock. D. Always after eleven o'clock.
75. You might say "hard luck" to someone who__________
A: has just heard some very good news. B. puts great effort into whatever he or she tries.
C: is less fortunate than he or she ought to be. D. fails through his or her own fault entirely.
76. Why was the writer on the riverbank that afternoon?
He was going to do some shopping in the village.B. He was fishing.
C. He was going to get the evening paper. D. He was on his way home from the village
77. Why was the writer surprised when he saw Bill Jarvis?
He thought it was late for Bill to be still fishing.
Bill had not caught anything, and that seemed strange.
He thought Bill was ill, because he was not moving at all
He was surprised because Bill stayed in his flat in the afternoons.
78. From the information given in the passage, who- or what - do you think was wrong?
A. The bell was; it must have gone off at the wrong time. B. The writer's watch was fast.
C. Bill was; he had dropped off to sleep. D. Bill's clock was wrong; it was very old.
79. All of the following are true about Bill Jarvis EXCEPT__________
A. he lived alone. B. he had ever worked as a sailor.
C. he was a newspaper man. D. fishing was his past time.
80. What did Bill Jarvis often bring with him when he went fishing?
A. a clock B. a gold C. a newspaper D. a fire engine
D. WRITING
I. Complete the second sentence so that it has a similar meaning to the first sentence. Use the word
given and other words to complete each sentence.
79. Mr. Holton only operated because he knew the rumor was malignant.
Mr. Holton wouldn't.......................................................................................................
80. All that stood between John and a gold medal was Jim's greater speed.
But for .................................................................................................................................
81. Immediately after their arrival, things went wrong.
No sooner………………………………………………………………………………………..
82. He was so tired that he fell asleep before the end of the film.
He was too..........................................................................................................................
83. Hearing that an earthquake had occurred was a great shock to us.
We were ..............................................................................................................................
84. The decorators have finished our first floor.
We have ..............................................................................................................................
85. Her hobby is one thing that she does not intend to give up.
She has ................................................................................................................................
86. That is the best meal I have ever eaten.
I have never ............................. ............................... ............................... ........................
87. Home computers used to be much more expensive.
Home computers aren’t................... ............................... ............................... ...........
88. It was a mistake of mine to park outside the police station.
I shouldn’t................... ......................................................................................................
4. Essay: There has been said that “students are unequally occupied by homework, and they learn
nothing from the around life”.
What do you think about that saying?
Write an essay of around 250 words to illustrate your ideas.

__________THE END___________
ĐÁP ÁN
A. LISTENING (15 points)
B. LEXICO-GRAMMAR (45 points)

I. (15 points; 1 point/each correct answer)


16. D 17. B 18. D 19. D 20. B 21. B 22. B 23. D 24. C 25. C
26. C 27. C 28. C 29. B 30. A

2, (10 points; 1 point/each correct finding and correcting)


31. wrongly → wrong 32. exhausting → exhausted 33. annoyed → annoying
34. apologizing → apologized 35. an interest → interest 36. fell → felt
37. of → for 38. pinning → pinned 39. said → saying 40. that → which
III. (10 points; 1 point/each correct answer)

41. PRODUCTION 42. ADVERTISING 43. IMAGINATIVE 44. ARTISTIC


45. SCENERY

46. AMUSING 47. ACCOMPANIED 48. ENTERTAINING 49. APPLAUSE


50. DELIGHTFUL

IV: (10 points; 1 point/each correct


answer)
51. about 52. moment 53. age 54. on 55. delight
56. need 57. love 58. rule 59. charged 60. example
C. READING (20 points)

I. (10 points; 1 point/each correct answer)


61. A 62. D 63. B 64. D 65. C 66. A 67. C 68. C 69. D 70. D

II: (10 points; 1 point/each correct


answer)
71. C 72. A 73. D 74. C 75. B 76. D 77. A 78. C 79. B 80. A
D. WRITING (20 points)
I. (10 points; 1 point/each correct answer)
80. → Mr. Holton wouldn't have operated if he hadn’t known (that) the rumour was malignant.
81. → But for Jim's greater speed, John would have won a gold medal.
82. → No sooner had they arrived than things went wrong.
83. → He was too tired to see/watch the end of the film.
84. → We were (greatly/very) shocked to hear/when we heard that an earthquake had occurred.
85. → We have had our first floor decorated (by the decorators).
86. → She has no intention of giving up her hobby.
87. → I have never eaten such a good meal as that/a better meal than that.
88. → Home computers aren’t as expensive as they used to.
89. → I shouldn’t have parked outside the police station.
II. Essay: (10 points)
Total: 100 points

___________THE END___________
TRƯỜNG THPT LƯU NHÂN CHÚ MÔN THI: TIẾNG ANH LỚP 12
KỲ THI CHỌN HỌC SINH GIỎI LỚP 12 CẤP
TRƯỜNG Ngày thi: 22/9/2018
NĂM HỌC 2018 - 2019 Thời gian làm bài: 180 phút
(không kể thời gian giao đề)
ĐỀ CHÍNH THỨC
(Thí sinh làm bài trực tiếp vào bài thi theo hướng dẫn dưới mỗi câu)

Điểm Giám khảo số 1 Giám khảo số 2 Số phách


(họ tên & chữ ký) (họ tên & chữ ký) (do chủ tịch Hội đồng thi)
Bằng số Bằng chữ

Đề thi này gồm 9 trang được đánh dấu từ 1 - 9 Thí sinh kiểm tra số trang trước khi làm bài.

Part 1: Question 1-6


HOMESTAY APPLICATION
Example Answer
Surname: Yuichini
First name: 1. _______________________
Sex: female Nationality: Japanese
Passport number: 2. _______________________ Age: 28 years
Present address: Room 21C, Willow College
Length of homestay: Approx 3.______________________
Course enrolled in 4._________________________
Family preferences: No 5.___________________________
No objection to 6._______________________
Part 2: question 7-12
Choose the correct letter A, B or C.
7. What kind of tour is Sally leading?
A. a bus tour B. a train tour C. a walking tour
8. The original buildings on the site were__________
A. houses B. industrial buildings C. shops
9. The local residents wanted to use the site for ______________.
A. leisure B. apartment blocks C. a sport centre.
10. The tower is at the centre of the _____________.
A. nature reserve. B. formal gardens C. Bicantennial park
7. 8. 9. 10.
SECTION 2: LEXICO – GRAMMAR.
I. Complete the following sentences by choosing option A, B, C or D.
11. David: Would you like fish or meat? Mary: I __________fish, please.
A. would rather B. would prefer C. suppose D. believe
12. Many teenagers show signs of anxiety and ___________when being asked about their future.
A. depress B. depression C. depressed D. depressing
13. There was a huge decline ___________the number of tigers.
A. in B.for C. of D. about
14. I’d rather you _________anything about the garden until the weather improves.
A.don’t make B. didn’t do C. don’t do D. didn’t make
15. A part-time job gives me the freedom to _________my interests.
A. pursue B. chase C. seek D. catch
16. The new road currently under____________will solve the traffic problems in the town.
A. design B. progress C. construction D. work
17. Daisy: What a lovely house you have! Mary: ___________
A. Lovely, I think so B. Thank you. I hope you will drop in.
C. of course not, it’s so costly. D. No problem.
18. Brian asked Judy________ to dinner and a movie.
A. out B. on C. for D. of
19. No news ___________ good news.
A. is B. are C. were D. have been
20. Yesterday he missed the lesson. He wishes he______ it.
A. didn't miss B. hadn't miss C. hasn't missed D. hadn't missed
21. Such ______ that he would stop at nothing.
A. his ambition was B. did his ambition C. does his ambition D. was his ambition

11. 12. 13.


14. 15. 16.
17. 18. 19.
20. 21.

II. Give the correct form of the verbs in brachkets.


22. Bi Rain, together with 58 members of the South Korean National Military Symphony Orchsetra
and 17 traditional musicians, (come) _______to Viet Nam since last week.
23. He suggested that his son (be) ________on time for the interview.
24. Tom will come home as soon as he (finish) _______his test .
25. ASEAN (found)________in 1967 in Bangkok, Thailand.
26. Hardly our teacher (enter)_______the classroom when it started to rain.
27. I must just go and wash, I (garden)___________ .
28,29. I (never, forget)___________ the time when I first (work)_________as a teacher.

30. They thought that he (play)________________football for 10 years.


31. It’s high time you (concentrate) ___________________on your study.

22. 23. 24. 25.


26. 27. 28. 29.
30. 31.

III. Give the suitable form of the word in parentheses.


32. On my salary, we have to live as _________as possible. ECONOMY
33. Different consevationefforts have been made in order to save ________species. DANGER
34. It is reported that humans are the main reason for most species’ declines and habitat
________DESTROY
35. Her son is always mischievous and _______, which annoys her very much. OBEY
36. The Americans are much more concerned than the Indian and Chinese with physical
________when choosing a wife or a husband. ATTRACT
32. 33. 34.
35. 36.
IV. In each of the following sentences, there is ONE mistake. Find and correct these mistakes.
37. The package containing books and records were sent last week.
A B C D
39. He didn’t forget filling petrol on the way home yesterday.
A B C
40.Helen has never met such good person who is ready to help others.

A B C D
41. Everybody in class has to choose a topic of your own to write an essay of 500 words.
A B C D
42. After driving for twenty miles, he suddenly realized that he has been driving in the wrong direction.
A B C D
43. It is likely that all people in Hanoi live in skyscrapers by 2050.
A B C D
44. In 1961, America’s first manned spacecraft launched.
A B C D
45. Paris has been well-known about its famous monuments, beautiful music, and wonderful restaurants for
over 100 years. A B C D
46. In France people drive on the left, so making sure you go on the right side.
A B C D
47. Up to now he wrote five novels and over sixteen short stories.
A B C D
Mistake Correction Mistake Correction
38. 43.
39. 44.
40. 45.
41. 46.
42. 47.

SECTION THREE: READING


I. Fill each of the numbered blanks in the following passage with a suitable given word.
able invitations firm impolite common punctual polite reschedule location set
Americans tend to be very (48) ____________ people. This is often expressed in Conversations. It is
(49)___ for an American to end a conversation by saying: “Let’s get together sometime,” “Come by for a visit
when you have a chance,” or “Let’s meet for coffee.” However, these (50)_____ are usually not intended to be
taken literally. An Invitation is not (51)_____ unless a time and place is (52).
If you have accepted an invitation or if a meeting has been set, Americans usually expect you to arrive at the
agreed (53)_____at the right time. It is considered (54)_____ to accept an invitation and not show up or to
arrive more than 10 to 20 minutes late. Americans tend to be quite (55)_____. If you have to cancel an
appointment or know that you will not be (56) to be on time, you should call your friend or host to cancel or
(57)_____

48. 49. 50.


51. 52. 53.
54. 55. 56.
57.

II. Read the following passage and choose the best answer A, B, C or D to fill in each of the blanks.
Everybody want to have a happy family. It is a (58) ______ of all people in this earth to have a happy
family. Comforts in life are (59) ______ when your family is not happy. Here’s the ingredients of a happy
family:
Unconditional love - Unconditional love means showing love (60) _______may happens. Expressing
love through actions and words is a very important factor to have a happy family life.
Understanding - A big room of understanding is a great (61) ______ also to have a happy family life.
Understand the need of each family members. Wife should understand the need of her husband and vice versa.
Parents should understand their kids (62) _______ they are growing up.
Quality time - This is very important to have a happy family life. Most happy families have time for
63_____. Parents give priority to spending time with their kids during (64) ______ hours from work rather
than having fun with their friends and associates.
Honesty - Being honest to each other would also (65)________to a happy family life.
Care - A (66) _______ family is a happy family. Care for the need of each other.
Patience and consideration - Consider minor differences, don’t make it a big deal. Have patience to (67)
_____ with it and this will contribute to a happy family life.
58. A. right B. dream C. way D. priority
59. A. hopeless B. meaningful C. helpful D. useless
60. A. whatever B. whenever C. wherever D. however
61. A. equipment B. fact C. tool D. result
62. A. since B. if C. because D. as
63. A. other B. each other C. each D. others
64. A. all B. for C. off D. out of
65. A. contribute B. dispense C. distribute D. concentrate
66. A. caring B. careless C. cared D. careful
67. A. manage B. apply C. divide D. deal

58. 59.60. 61. 62.


63. 64.65. 66. 67.

3, Read the following passage and choose the letter A, B, C or D to indecate the correct answer to each
of the questions.
A.
Sometimes people add to what they say even when they don’t talk. Gestures are the “silent language” of every
culture. We point a finger or move another part of the body to show what we want to say. It is important to
know the body language of every country or we may be misunderstood. In the United States, people greet each
other with a handshake in a formal introduction. The handshake must be firm. If the handshake is weak, it is a
sign of weakness or unfriendliness. Friends may place a hand on the other’s arm or shoulder. Some people,
usually women, greet a friend with a hug.
Space is important to Americans. When two people talk to each other, they usually stand about two and a half
feet away and at an angle, so they are not facing each other directly. Americans get uncomfortable when a
person stands too close. They will move back to have their space. If Americans touch another person by
accident, they say, “Pardon me.” or “Excuse me.” Americans like to look the other person in the eyes when they
are talking. If you don’t do so, it means you are bored, hiding something, or are not interested. But when you
are stare at someone, it is not polite. For Americans, thumbs-up means yes, very good, or well done. Thumbs
down means the opposite. To call a waiter, raise one hand to head level or above. To show you want the check,
make a movement with your hands as if you are signing a piece of paper. It is all right to point at things but not
at people with the hand and index finger. Americans shake their index finger at children when they scold them
and pat them on the head when they admire them. Learning a culture’s body language is sometimes confusing. -
If you don’t know what to do, the safest thing to do is to smile.
68. From the passage we can learn that __________
A. gestures don’t mean anything while talking
B. gestures can help us to express ourselves
C. American people often use body language in communication
D. It’s confusing to understand a culture’s body language
69. If you are introduced to a stranger from the USA, you should _________
A. greet him with a hug B. place a hand on his shoulder
C. shake his hand weakly D. shake his hand firmly
70. American people often __________ .

71. When your friend give you a thumbs-up, he; in fact, _________
A. shows his rudeness to you B. shows his anger to you
C. expresses his worries about you D. expresses his satisfaction to you
72. Which of the following is NOT true about the culture of the United States?
A.Pointing at someone is usually considered rude.
B. It’s rude to stare at the other person.
C. It’s impolite to look the other person in the eyes while talking.
D. It’s all right to raise your hand slightly when you want to attract the waiter’s attention.
69. 70. 71. 72.

Before the mid-nineteenth century, people in the United States ate most foods only in season. Drying,
smoking, and salting could preserve meat for a short time, but the availability of fresh meat, like that of fresh
milk, was very limited; there was no way to Line prevent spoilage. But in 1810 a French inventor named
Nicolas Appert

a. developed the cooking-and-sealing process of canning. And in the 1850's an American named
Gail Borden developed a means of condensing and preserving milk. Canned goods and
condensed milk became more common during the 1860's, but supplies remained low because
cans had to be made by hand. By 1880, however, inventors had fashioned stamping and
soldering machines that mass-produced cans from tinplate. Suddenly all kinds of food could
be preserved and bought at all times of the year.
Other trends and inventions had also helped make it possible for Americans to vary their daily diets. Growing
urban populations created demand that encouraged fruit and vegetable farmers to raise more produce.
Railroad refrigerator cars enabled

b. growers and meat packers to ship perishables great distances and to preserve them

for longer periods. Thus, by the 1890's, northern city dwellers could enjoy southern and western strawberries,
grapes, and tomatoes, previously available for a month at most, for up to six months of the year. In addition,
increased use of iceboxes enabled families to store perishables. An easy means of producing ice
commercially

38. had been invented in the 1870's, and by 1900 the nation had more than two thousand
commercial ice plants, most of which made home deliveries. The icebox became a fixture in most
homes and remained so until the mechanized refrigerator replaced it in the 1920's and 1930's.
Almost everyone now had a more diversified diet. Some people continued to eat

39. mainly foods that were heavy in starches or carbohydrates, and not everyone could
afford meat. Nevertheless, many families could take advantage of previously (25) unavailable fruits,
vegetables, and dairy products to achieve more varied fare.

73. What does the passage mainly discuss?


A. Causes of food spoilage B. Commercial production of ice
C. Inventions that led to changes in the American diet D. Population movements in the nineteenth
century
74. The phrase “in season” in line 2 refers to
A. a kind of weather B. a particular time of year
C. an official schedule D. a method of flavoring food
75. The word “prevent” in line 4 is closest in meaning to
A. estimate B. avoid C. correct D. confine
76. During the 1860's, canned food products were
A. unavailable in rural areas B. shipped in refrigerator cars
C. available in limited quantities D. a staple part of the American diet
77. It can be inferred that railroad refrigerator cars came into use
A. before 1860 B. before 1890 C. after 1900 D. after 1920
78. The word “them ” in line 15 refers to
A. refrigerator cars B. perishables C. growers D. distances
79. The word “fixture” in line 22 is closest in meaning to
A. luxury item B. substance C. commonplace object D. mechanical
device
80. The author implies that in the 1920's and 1930's home deliveries of ice
A. decreased in number B. were on an irregular schedule
C. increased in cost D. occurred only in the summer
81. The word “Nevertheless” in line 26 is closest in meaning to
A. therefore B. because C. occasionally D. however
82 Which of the following types of food preservation was NOT mentioned in the passage?
A. Drying B. Canning C. Cold storage D. Chemical additives
83. Which of the following statements is supported by the passage?
A. Tin cans and iceboxes helped to make many foods more widely available.
B. Commercial ice factories were developed by railroad owners.
C. Most farmers in the United States raised only fruits and vegetables.
D. People who lived in cities demanded home delivery of foods.

73. 74. 75. 76. 77.


78. 79. 80. 81. 82.
83.
SECTION FOUR: WRIITNG

I. Finish the second sentence so that it means the same as the first one. (84-88)
85. We couldn’t relax until all the guests had gone home.
- Only after ________________________________________________________________________
86. House prices have risen sharply this year.
There has__________________________________________________________________________
87. Henry regretted buying the second-hand car.
Henry wishes_______________________________________________________________________
88. Experts think that all dogs evolved from wolves.
All dogs ___________________________________________________________________________
89. Would you mind not smoking in my house?
I would____________________________________________________________________________

II. Write a passage of about 250 words to show your opinion on the following statement:
“Smart devices have put all of the world’s information at our fingertips.” What are the drawbacks of this
development?
- The end –
TRƯỜNG THPT LƯU NHÂN CHÚ MÔN THI: TIẾNG ANH LỚP 12
KỲ THI CHỌN HỌC SINH GIỎI LỚP 12 CẤP
TRƯỜNG Ngày thi: 22/9/2018
NĂM HỌC 2018 - 2019 Thời gian làm bài: 180 phút
(không kể thời gian giao đề)
ĐÁP ÁN
(Thí sinh làm bài trực tiếp vào bài thi theo hướng dẫn dưới mỗi câu)

SECTION ONE: LISTENING


Part 1: Question 1-6
HOMESTAY APPLICATION
Example Answer
Surname: Yuichini
First name: 1. ___Keiko_______________
Sex: Female Nationality: Japanese
Passport number: 2. ___J06337______________ Age: 28 years
Present address: Room 21C, Willow College
Length of homestay: Approx 3._____4 months__________
Course enrolled in 4.____(advanced) English (studies) ______
Family preferences: No 5.___ children_________
No objection to 6.__ pets__________
Part 2: question 7-12
Choose the correct letter A, B or C.
7. What kind of tour is Sally leading?
A. a bus tour B. a train tour C. a walking tour
8. The original buildings on the site were__________
A. houses B. industrial buildings C. shops
9. The local residents wanted to use the site for ______________.
A. leisure B. apartment blocks C. a sport centre.
10. The tower is at the centre of the _____________.
A. nature reserve. B. formal gardens C. Bicantennial park
7. C 8. B 9. A 10. B
SECTION 2: LEXICO – GRAMMAR.
I. Complete the following sentences by choosing option A, B, C or D.

12. B 13. B 14. A


15. B 16. A 17. C
18. B 19. A 20. A
21. D 22. D
II. Give the correct form of the verbs in brachkets.
23. has come 24. be 25. has finished/finishes 26. was founded
27. had our teacher 28. have been gardening 29. will never forget 30. worked
entered
31. had played 32. concentrated

III. Give the suitable form of the word in parentheses.


33. economically 34. endangered 35. destruction
36. disobedient 37. attractiveness
IV. In each of the following sentences, there is ONE mistake. Find and correct these mistakes.
Mistake Correction Mistake Correction
38. C was 43. C Will have lived
39. A To fill 44. D Was launched
40. B Such a good 45. B For
41. C Their own 46. C. make sure
42. C Had been driving 47. B has written

SECTION THREE: READING


I. Fill each of the numbered blanks in the following passage with a suitable given word.
48. polite 49. common 50. invitations
51. firm 52. set 53. location
54. impolite 55. punctual 56. able
57. reschedule
II. Read the following passage and choose the best answer A, B, C or D to fill in each of the blanks.
58. B 59. D 60. A 61. C 62. D
63. B 64. B 65. A 66. A 67. D
Read the following passage and choose the letter A, B, C or D to indecate the correct answer to each of
the questions.
A.
68. B 69. D 70. C 71. D 72.C
B.
73. C 74. B 75. B 76. C 77. B
78. B 79.C 80. A 81. D 82. D
83. A

We couldn’t relax until all the guests had gone home.


Only after had all the guests gone home, we could relax.

House prices have risen sharply this year.

93. There has a sharp rise/ increase in house prices this year.

94. Henry regretted buying the second-hand car.

95. Henry wishes he had bought the secod-hand car.

96. Experts think that all dogs evolved from wolves.

97. All dogs are thought to have evolved from wolves.

98. Would you mind not smoking in my house?

99. I would rather you didn’t smoke in my house.

II. Write a passage of about 250 words to show your opinion on the following statement:

“Smart devices have put all of the world’s information at our fingertips.” What are the drawbacks of this
development?

Marking scheme

The impression mark given is based on the following scheme:

70. Content: 50% of total mark: a provision of all main ideas and details as appropriate

71. Language: 30% of total mark: a variety of vocabulary and structures appropriate to the
level of English language gifted upper-secondary school students

72. Presentation: 20% of total mark: coherence, cohesion, and style appropriate to the
level of English language gifted upper-secondary school students.
THPT CHUYÊN VĨNH PHÚC ĐỂ THI CHỌN ĐỘI TUYỂN HSG 12 CẤP TỈNH LẦN 1
NĂM HỌC: 2018-2019
(ĐỀ THI CHÍNH THỨC) Môn thi: Tiếng Anh
Thời gian làm bài: 150 phút (Không kể thời gian giao đề)
I. LISTENING
I. Listen to the recording and complete the note with NO MORE THAN TWO
WORDS. FOUR BUSINESS VALUES
Many business values can result in (1)_____________________
Senior managers need to understand and deal with the potential (2)_________ that may result.
Collaboration
During a training course, the speaker was in a team that had to build a (3)____________
Other teams experienced (4)___________ from trying to collaborate.
The speaker’s team won because they, reduced collaboration.
Sales of a (5)______________ were poor because of collaboration.
Industriousness
Hard work may be a bad use of various company (6)______________
The word ‘lazy’ in this context refers to people who avoid doing tasks that are (7)____________
Creativity
An advertising campaign for a (8)____________ was memorable but failed to boost sales.
Creativity should be used as a response to a particular (9) ____________
Excellence
According to one study, on average, pioneers had a (10) ____________ that was far higher than that of
followers.
Companies that always aim at excellence may miss opportunities.
II. You will hear two people speaking about their fondness for trains. For questions 1-5, choose the
correct answer (A, B, C or D) which fits best according to what you hear.
1) What point does Philip make about the people who are involved in the locomotive side of trains?
A) Some avoid doing it because its too dirty. B) Some worry about the dangers involved.
C) Some find it to be an instinctive activity. D) Some only do it for the financial rewards it
brings.
II.When Mike discusses why he enjoys the world of trains, what contrast does he identify between
the different groups of people he interacts with?
A) A difference in social status. B) A difference in ages.
C) A difference in technical ability. D) A difference in motivation.
3) What view is stated by Mike about how the railway has helped him in his role as a parent?
A) It has helped him develop a closer relationship B) It has taught him patience.
with his daughter.
C) It has helped him focus on his daughters career D) It has given him stability in his life.
possibilities.
4) When discussing different aspects of the railway, both speakers agree that ___________.
A) Age does not need to be a handicap. B) Both men and women are equally welcome.
C) It can be quite demanding. D) Cleanliness is not the first adjective that springs
to mind.
5) What final conclusion do both speakers reach about volunteering on the railway?
A) The time they have spent has been difficult but B) They have both learnt a lot.
worth it.
C) They are better people for having been D) It has helped them grow while at the same time
involved with it. giving them a direction for the future.
B. LEXICO AND GRAMMAR
I. Choose the correct answer A, B, C or D for each question.
1. If the work-force respected you, you wouldn't need to _________ your authority so often.
A. assert B. affirm C. maintain D. inflict
2. The factory is working below _________ because of the shortage of essential materials.
A. range B. scope C. capacity D. denstiy
3. His new designs are ______ the trend in women’s fashion right now.
A. aiming B. setting C. building up D. following
4. She wants to give full ______ to her anger about discrimination.
A. manifestation B. vent C. wear D. barometer
5. Without telling her directly, they ______ that she’d got the job.
A. prophesied B. inferred C. insinuated D. intimated
6. Athletes need to have a higher _________ of protein and vitamins in order to stay heathy
A. intake B. increase C. infection D. production
7. He claimed _____ from military service because he was a foreign national
A. liability B. exception C. demobilization D. exemption
8. It’s Prime Minister’s right to ______ an election at any time he likes
A. summon B. nominate C. call D. submit
9. The footballer never really recovered from the injury ______ at the beginning of the season.
A. got B.struck C.endured D.sustained
10. The drought ________ considerable problems for farmers.
A. instigated B. posed C. flexed D. pressed
11. At the start of the course, everyone is assigned a(n) ________ of studies.
A. advisor B. consultant C. counsellor D. guide
12. Jerry has been burning the midnight ________ over the last few days; his final exam is soon.
A. lamp B. light C. candle D. oil
13. In my first year at university I lived in the halls of ________ .
A. abode B. residence C. dwelling D. accommodation
14. Books taken from the short ________ section are due to be returned the next day.
A. borrowing B. credit C. loan D. return
15. You are bound to find information on the stock market crash of 1987 in the newspaper ________ .
A. files B. archives C. records D. collections
16. The accused confidently ________ that he was innocent.
A. preserved B. maintained C. conserved D. defended
17. We did our best to fix the broken computer but our efforts bore no ________.
A. success B. fruit C. luck D. end
18. I knew my mother would ________ a face the minute she saw my new haircut.
A. drag B. lift C. pull D. raise
19. There are a lot of computer programmes nowadays, but really good ones are few and far ________.
A. between B. apart C. away D. amongst
20. He is such a kind and caring young boy - he wouldn’t hurt a ________ .
A. bird B. worm C. fly D. bug
II. Supply the correct form of the word in bracket to complete the passage.
The signal broke a seven-hour wait of (1. AGONY) intensity and sparked scenes of (2. JUBILEE) at the
European Space Agency’s mission control in Darmstadt. The team in charge of the Rosetta mission
achieved what at times seemed an impossible task by landing a (3. ROBOT) (4. CRAFT) on a comet for
the first time in history. The moment the tension broke came shortly after 1600 GMT when the Philae
called home. “We are there. We are sitting on the surface. Philae is talking to us,” said a (5. JUBILEE)
Stephan Ulamec, Philae lander manager at the DLR German space centre. “We are on the comet.” Andrea
Accomazzo, the Rosetta flight operations director, added: “We cannot be happier than we are now.”
But celebrations were tempered by the later discovery that the probe’s two harpoons had not fired to fasten
the craft down in the (6. LOW) gravity. Scientists now think the probe may have bounced after first
coming into contact with the surface. Ulamec said: “Maybe today we didn’t just land once, we landed
twice.” The safe, if (7. CARE), (8. TOUCH) of the lander gives scientists a unique chance to ride (9.
BOARD) a comet and study from the surface what happens as its activity ramps up as it gets closer to the
sun. The first images beamed back from the lander’s descent revealed a dramatic (10. SCAPE) of pits and
precipices, craters and boulders. However, there have been gaps in its radio link with the orbiting Rosetta
mothership.
III. There are 10 mistakes in the passage. Find and correct them.
There are numerous reason behind the choice of clothing we make, ranging from the practice to the
bizarre, but in every likelihood humans began wearing clothes in order to shield themselves of the
elements.
‘The Iceman’, found mummifying in the ice of a glacier on the Italian-Austrian border in 1991, was still
clad with remarkably well-preserved garments which would obviously have protected him from the rain
and cold five thousand years ago.
The clothing worn by people living in warmer climates further illustrates this basic function of clothing. In
hot, dry desert regions, people tend to wear cool, flowing materials which cover much of body, including
the head. This helps keep them cool during the heat of the day and helps to keep the wearer warm at night
when temperatures increase dramatically. On the other hand, many peoples living in tropical regions, with
their constant high temperatures and humidity, are sparse clothed.
Not all clothing is functional, moreover, and at different times throughout history people, especially
women, have worn clothes which are impractical – restricting the wearer’s movements and in some cases
cause physical discomfort. Examples of such restrictive clothing can be seen in the hooped and corseted
gowns worn by wealthy women in the 18th century, and in the opulent, heavy robes traditionally worn by
royalties. Such customs send a clear message that the wearer does not need to work but occupies a
position in society that others can be commanded to work for them.
Example: 0. reason
→ reasons
IV. Fill in the blank with ONE suitable preposition.
1. The school examination for eleven-year-olds was done_________ with some years ago.
2. Tuck your shirt ________your trousers.
3. At first Tim insisted he was right, but then began to back ________
4. He is a solicitor________ profession.
5. A huge crowd turned _________ in the pouring rain to cheer the president.
PART C. READING
I. Read the passage and choose the correct answer that best fill in the blank.
OFFICE STRESS
Stressful atmospheres, (1) _______ of deadlines and long hours dominate office life, according to a
survey (2) _______ recently.
The majority of those questioned said a good salary and career (3) _______ were their main reason for
working. But (4) _______ numbers did not believe their employers offered either. In general the survey
found that most felt that (5) _______ of life was more important than (6) _______and company perks.
Most would prefer employers to offer (7) _______ hours, challenging tasks and job (8) _______ rather
than perks such as company cars and private health care. Many employers’ (9) _______ to understand this
meant more than a third worried about their work on holiday, and 40 per cent took days off (10)
_______when not ill.
Workers were also (11) _______ by the conditions they had to work in. A fifth struggled with (12)
_______ technology, badly lit offices and chairs which caused backache. Half said their (13) _______
would increase if their environment improved.
On the plus side, the biggest (14) _______ was the friendship offered by colleagues, and it appears that
the office also affords the chance to flirt with colleagues, make (15) _______ calls to friends abroad, steal
stationery and play computer games.
1. A. weight B. force C. heaviness D. pressure
2. A. published B. printed C. publicised D. proclaimed
3. A. outlooks B. odds C. prospects D. views
4. A. important B. impressive C. heavy D. significant
5. A. quality B. calibre C. excellence D. worth
6. A. rank B. status C. degree D. grade
7. A. pliable B. elastic C. amenable D. flexible
8. A. safety B. security C. sanctuary D. protection
9. A. failure B. defeat C. deficiency D. lack
10. A. indisposed B. unwell C. injured D. sick
11. A. pestered B. inflamed C. irritated D. ruffled
12. A. behind the times B. expired C. out-of-date D. invalid
13. A. fertility B. capacity C. value D. productivity
14. A. compensation B. damages C. reimbursement D. atonement
15. A. idiosyncratic B. unique C. personal D. individual
Read the text and choose the correct answer A, B, C or D for each question.

THE FUTURE OF NEWSPAPERS


Anybody who says they can reliably forecast the future of newspapers is either a liar or a fool. Look at the
raw figures, and newspapers seem doomed. Since 2000, the circulation of most UK national dailies has
fallen by between a third and a half. The authoritative Pew Research Center in the USA reports that
newspapers are now the main source of news for only 26 percent of US citizens as against 45 percent in
2001. There is no shoratge of prophets who confidently predict that the last printed newspaper will be
safely buried within 15 years at most.
Yet one of the few reliable facts of history is that old media have a habit of surviving. An over-exuberant
New York journalist announced in 1935 that books and theatre ‘have had their day’ and the daily
newspaper would become ‘the greatest organ of social life’. Theatre dully withstood not only the
newspaper, but also cinema and then television. Radio has flourished in the TV age; cinema, in turn, has
held its own against videos and DVDs. Even vinyl records have made a comeback, with online sales up
745 percent since 2008.
Newspapers themselves were once new media, although it took several centuries before they became the
dominant medium for news. This was not solely because producing up-to-date news for a large readership
over a wide area became praticable and economic only in the mid-19th century, with the steam press, the
railway and the telegraph. Equally important was the emergence of the idea that everything around us is in
constant movement and we need to be updated on its condition at regular intervals- a concept quite alien in
the medieval times and probably also to most people in the early modern area. Now, we expect change. To
our medieval ancestors, however, the only realities were the passing of the seasons, punctuated by
catastrophes such as famine, flood or disease that they had no reliable means of anticipating. Life, as the
writer Alain de Botton puts it, was ‘ineluctably cyclical’ and ‘the most important truths were recurring’.

Journalism as a full-time trade from which you could hope to make a living hardly existed before the 19th
century. Even then, there was no obvious reason why most people needed news on a regular basis, whether
daily or weekly. In some respects, regularity of newspaper publication and rigidity of format was, and
remains, a burden. Online news readers can dip in and out according to how they perceive the urgency of
events. Increasingly sophisticated search engines and algorithms allow us to personalise the news to our
own priorities and interests. When important stories break, internet news providers can post minute-by-
minute updates. Error, misconception and foolish speculation can be connected or modified almost
constantly. There are no space restrictions to prevent narrative or analysis, and documents or events cited
in news stories can often be accessed in full. All this is a world away from the straitjacket of newspaper
publication. Yet few if any providers seem alive to the new medium’s capacity for spreading understanding
and enlightenment.
Instead, the anxiety is always to be first with the news, to maximise reader comments, to create heat and
sound and fury and thus add to the sense of confusion. In the medieval world what news there was was
usually exchanged amid the babble of the market place or the tarven, where truth competed with rumour,
mishearing and misunderstanding. In some respects, it is to that world that we seem to be returning.
Newspapers have never been very good- or not as good as they ought to be- at telling us how the world
works. Perhaps they now face extinction. Or perhaps , as the internet merely adds to what de Botton
discribes as our sense that we live in ‘ an improvable and fundamentally chaotic universe’, they will
discover that they and they alone can guide us to wisdom and understanding.

1. In the first paragragh, the writer is presenting __________.


A. his interpretation of a current trend.
B. evidence that supports a widespread view.
C. his prediction on the future of print journalism.
D. reasons for the decline in newspaper readership.

2. What point is the writer making in the second paragraph?


A. Existing media are not necessarily replaced by new ones.

B. The best media technologies tend to be the most long-lasting.v


C. Public enthusiasm for new types of media is often unpredictable.
D. It is inevitable that most media technologies will have a limited life.

3. Which phrase in the second paragraph has the same meaning as ‘held its own against’?
A. ‘had their day’ B. ‘withstood’ C. ‘flourished’ D. ‘made a comeback’
4. In the third paragraph, the writer stresses the importance of __________.
A. a shift in people’s attitudes towards the outside world.
B. certain key19th- century advances in mechanisation.
C. the challenges of news distribution in the pre-industrial era.
D. the competition between newspapers and more established media.
5. What does the writer suggest is the main advantage of online news sites?
A. the flexibility of the medium B. the accuracy of the reporting
C. the ease of access for their users D. the breadth of their potential readership
III. Read the passage and think of ONE word that best fits in the numbered blank.
A BELATED CONFESSION
“I have been having a crush on you for the past 4 years, I never had the (1)______ to tell you, but I can’t
be so cowardly anymore.” said Peter as he caught Helen hand in the crowd watching fireworks.
Helen was taken (2)_______ by Peter’s words, both because he appeared out of nowhere and also never
had she expected a boy, so silent and seemed uninterested in class’ business, would like her.
“Peter? Why are you here? I thought you said you wouldn’t come.”
“-Listen, we don’t have much time (3)_________, I’m moving to another country, this will be the last
(4)________ we can be together until a few more years”
“Why didn’t you tell-“
“-I realized that I had made a mistake, a huge one. I had always been such an (5)________, never made an
effort to make friends, but you always tried to (6)________ me, you truly changed me, and so I don’t
want to lose you.”
No (7)________ words were said, they slowly turned to the sky. Little does Peter know, Helen had also
had something for him, but she had been afraid that her friends would’ve teased her. Ever since her
boyfriend (8)________ her and her friends turned their back on her, Peter has always been there for her,
he never talked much, mostly listen, but that’s enough for her.
At that (9)_______ moment, she knew she didn’t want to let him go, but then it was too late. He’d always
been stoic, never wanted others to worry about him, so he decided to keep it a secret before he leaves.
That night, there wasn’t (10)________ his first kiss but also her first time feeling truly cared about and
loved.
IV. In the following text, five paragraphs have been removed. Above the extract you will find the five
removed paragraph PLUS one paragraph which doesn't fit. Choose from the paragraphs (A-F) the one
which fits each gap (1-5). There is one extra sentence you do not need to use.
THE INDUSTRIAL REVOLUTION
The Industrial Revolution is the name given to the massive social, economic, and technological change in
18th century and 19th century Great Britain. It commenced with the introduction of steam power (fuelled
primarily by coal) and powered, automated machinery (primarily in textile manufacturing).
The technological and economic progress of the Industrial Revolution gained momentum with the
introduction of steam-powered ships, boats and railways. In the 19th Century it spread throughout
Western Europe and North America, eventually impacting the rest of the world.
CAUSES
The causes of the Industrial Revolution were complex and remain a topic for debate, with some historians
seeing the Revolution as an outgrowth of social and institutional changes wrought by the final end of
feudalism in Great Britain following the English Civil War in the 17th century.
The Enclosure movement and the British Agricultural Revolution made food production more efficient
and less labor-intensive, forcing the surplus population who could no longer find employment in
agriculture into the cities to seek work in the newly developed factories.
The colonial expansion of the 17th century with the accompanying development of international trade,
creation of financial markets and accumulation of capital is also cited as a set of factors, as is the scientific
revolution of the 17th century.
The importance of a large domestic market should also be considered an important cause catalyst of the
Industrial Revolution, particularly explaining why it occurred in Britain. In other nations (e.g. France),
markets were split up by local regions often imposing tolls and tariffs on goods traded among them. The
restructuring of the American domestic market would trigger the second Industrial Revolution over 100
years later.
EFFECTS
The application of steam power to the industrial processes of printing supported a massive expansion of
newspaper and popular book publishing, which reinforced rising literacy and demands for mass political
participation. Universal white male suffrage was adopted in the United States, resulting in the election of
the popular General Andrew Jackson in 1828 and the creation of political parties organized for mass
participation in elections.
1. ________________
In France, the July Revolution widened the franchise and established a constitutional monarchy. Belgium
established its independence from the Netherlands, as a constitutional monarchy, in 1830. Struggles for liberal
reforms in Switzerland's various cantons in the 1830s had mixed results. A further series of
attempts at political reform or revolution would sweep Europe in 1848, with mixed results, and initiated
massive migration to North America, as well as parts of South America, South Africa, and Australia.
TEXTILE MANUFACTURE
In the early 18th century, British textile manufacture was based on wool which was processed by individual
artisans, doing the spinning and weaving on their own premises. This system is called a cottage industry. Flax
and cotton were also used for fine materials, but the processing was difficult because of the pre-processing
needed, and thus goods in these materials made only a small proportion of the output.
Use of the spinning wheel and hand loom restricted the production capacity of the industry, but a number
of incremental advances increased productivity to the extent that manufactured cotton goods became the
dominant British export by the early decades of the 19th century. India was displaced as the premier
supplier of cotton goods.
Step by step, individual inventors increased the efficiency of the individual steps of spinning (carding,
twisting and spinning, and subsequently rolling) so that the supply of yarn fed a weaving industry that
itself was advancing with improvements to shuttles and the loom or 'frame'. The output of an individual
labourer increased dramatically, with the effect that these new machines were seen as a threat to
employment, and early innovators were attacked and their inventions wrecked. The inventors often failed
to exploit their inventions, and fell on hard times.
2. ____________
He created the cotton mill which brought the production processes together in a factory, and he developed
the use of power - first horse power, then water power and finally steam power - which made cotton
manufacture a mechanised industry.
WHY EUROPE?
One question that has been of active interest to historians is why the Industrial Revolution occurred in
Europe and not in other parts of the world, particularly China. Numerous factors have been suggested
including ecology, government, and culture. Benjamin Elman argues that China was in a high level
equilibrium trap in which the non-industrial methods were efficient enough to prevent use of industrial
methods with high capital costs.
Kenneth Pommeranz, in The Great Divergence, argues that Europe and China were remarkably similar in
1700, and that the crucial differences which created the Industrial Revolution in Europe were sources of
coal near manufacturing centres and raw materials such as food and wood from the New World which
allowed Europe to economically expand in a way that China could not. Indeed, a combination of all of
these factors is possible.
WHY GREAT BRITAIN?
The debate around the concept of the initial startup of the Industrial Revolution also concerns the thirty to
hundred year lead the British had over the continental European countries and America. Some have
stressed the importance of natural or financial resources the United Kingdom received from its many
overseas colonies or that profits from the British slave trade between Africa and the Caribbean helped fuel
industrial investment.
Alternatively, the greater liberalisation of trade from a large merchant base may have been able to utilise
scientific and technological developments emerging in the UK and elsewhere more effectively than other
states with stronger monarchies, such as China's Emperors and Russia's Tzars.
The UK's extensive exporting cottage industries also ensured markets were already open for many forms
of early manufactured goods. The nature of conflict in the period resulted in most British warfare being
conducted overseas, reducing the devastating effects of territorial conquest impacting much of the rest of
Europe.
Another theory believes that Great Britain was able to succeed in the Industrial Revolution due to its
dense population for its small geographical size, and the availability of natural resources like copper, tin
and coal, giving excellent conditions for the development and expansion of industry.
IV. ___________________________
THE "PROTESTANT WORK ETHIC"
Another theory is that the British advance was due to the presence of an entrepreneurial class which
believed in progress, technology and hard work. The existence of this class is often linked to the
Protestant work ethic and the particular status of so-called Dissenter Protestant sects that had flourished
with the English revolution.
Reinforcement of confidence in the rule of law, which followed the establishment of the prototype of
constitutional monarchy in Great Britain in the Glorious Revolution of 1689, and the emergence of a
stable financial market there based on the management of the National Debt by the Bank of England,
contributed to the capacity for, and interest in, private financial investment in industrial ventures.
The Dissenters found themselves barred or discouraged from some public offices when the restoration of
the monarchy took place and membership in the official Anglican church became, once more, an
important advantage.
Historians sometimes consider this social factor to be extremely important, along with the nature of the
national economies involved. While members of these sects were excluded from certain circles of the
government, they were considered as fellow Protestants, to a limited extent, by many in the middle class,
such as traditional financiers or other businessmen.
V. ________________
This argument has, on the whole, tended to neglect the fact that several inventors and entrepreneurs were
rational free thinkers or "Philosophers" typical of a certain class of British intellectuals in the late 18th
century, and were by no means normal church goers or members of religious sects.
Examples of these free thinkers were the Lunar Society of Birmingham (which flourished from 1765 to
1809). Its members were exceptional in that they were among the very few who were conscious that an
industrial revolution was then taking place in Great Britain. They actively worked as a group to encourage
it, not least by investing in it and conducting scientific experiments which led to innovative products.
5. _______________
Industrialization also led to the creation of the factory. One of the earliest reformers of early factory
conditions was Robert Owen. Josiah Wedgwood was another prominent early industrialist. The factory
system was largely responsible for the rise of the modern city, as workers migrated into the cities in
search of employment in the factories.
PARAGRAPH
A. Given this relative tolerance and the supply of capital, the natural outlet for the more enterprising
members of these sects would be to seek new opportunities in the technologies created in the wake of
the Scientific revolution of the 17th century.
B. To capitalize upon these advances, it took a class of entrepreneurs, of which the most famous is
Richard Arkwright. He is credited with a list of inventions, but these were actually the products of such
as Thomas Highs and John Kay; Arkwright nurtured the inventors, patented the ideas, financed the
initiatives, and protected the machines.
C. In the United Kingdom, the Reform Act 1832 addressed the concentration of population in districts
with almost no representation in Parliament, expanding the electorate, leading to the founding of
modern political parties and initiating a series of reforms which would continue into the 20th century.
D. Furthermore, the stable political situation, in addition to the greater receptiveness of the society (as
compared to other European countries) are reasons that add to this theory, enhancing its plausibility. E.
The transition to industrialization was not wholly smooth, for in England the Luddites — workers who
saw their livelihoods threatened — protested against the process and sometimes sabotaged factories.
F. It was this process which started the urbanisation of areas in around the west of the country. As
agricultural workers lost their jobs on the farm, they turned in increasing numbers to the city to provide
them with employment. Many new cities sprung up as factories were built close to raw supplies such as
coal and wool - and the people moved near the factories to find work.

PART D. WRITING
I. Complete the sentences without changing the meaning of the given one.
VI. I was not surprised to hear that Harry had failed his driving test.
→Itcame
_______________________________________.
VII. She wore a hearing-aid, even though she could hear the phone ring perfectly well.
→She wasn’t so _______________________________________.
VIII. The sales man told me that my new car would be delivered next Wednesday.
According _______________________________________.
4. Their chances of success are small.
It is not _______________________________________.
5. This affair does not concern you.
This affair is no _______________________________________.
Complete the sentences without changing the meaning of the given one. Do not change the form of
the given word in any way.
1. My cat has lost its appetite. (OFF)
→My cat _______________________________________.
2. Gerald never had enough to live on until he married that rich businesswoman. (SHORT)
→Gerald _______________________________________.
3. His reactions are quite unpredictable. (KNOWS)
→One _______________________________________.
4. There are several categories of people who do not have to pay the new tax. (EXEMPT)
→There are _______________________________________.
5. Ours is the only company allowed to import these chemicals. (MONOPOLY)
→Our ______________________________________.
III. Nowadays, there are many couples chosing to live together without having a baby.
Is it a good thing or bad thing? What’s your point of view on this issue? Write in an essay of at least
300 words.

THE END
BEST OF LUCK

THPT CHUYÊN VĨNH PHÚC ĐỂ THI CHỌN ĐỘI TUYỂN HSG 12 CẤP TỈNH LẦN 1
NĂM HỌC: 2018-2019
(ĐÁP ÁN CHÍNH THỨC) Môn thi: Tiếng Anh

A. LISTENING (10 points)


I. (5 points)
1. damage 2. side effects 3. bridge 4. confusion 5. smartphone
6. resources 7. unnecessary/ 8. chocolate bar 9. problem 10. market share
not necessary
II. (5 points)
1. C 2. D 3. A 4. D 5. D
B. LEXICO AND GRAMMAR (30 points)
I. (10 points)
1. A 2. C 3. B 4. B 5. C
6. A 7. D 8. C 9. D 10. B
11. A 12. D 13. B 14. C 15. B
16. B 17. B 18. C 19. A 20. C
II. (10 points)
1. agonising 2. jubilation 3. robotic 4. spacecraft 5. jubilant
6. ultra-low 7. precarious 8. touchdown 9. onboard 10. landscape
III. (5 points)
There are numerous reason behind the choice of clothing we make, 0. reasons
ranging from the practice to the bizarre, but in all likelihood humans began 1. practical
wearing clothes in order to shield themselves of the elements. 2. from
‘The Iceman’, found mummifying in the ice of a glacier on the Italian- 3. mummified
Austrian border in 1991, was still clad with remarkably well-preserved 4. in
garments which would obviously have protected him from the rain and cold
five thousand years ago.
The clothing worn by people living in warmer climates further
illustrates this basic function of clothing. In hot, dry desert regions, people
tend to wear cool, flowing materials which cover much of the body,
including the head. This helps keep them cool during the heat of the day
and helps to keep the wearer warm at night when temperatures increase 5. plummet /drop/fall
dramatically. On the other hand, many peoples living in tropical regions,
with their constant high temperatures and humidity, are sparse clothed. 6. sparsely
Not all clothing is functional, moreover, and at different times 7. however
throughout history people, especially women, have worn clothes which are
impractical – restricting the wearer’s movements and in some cases cause 8. causing
physical discomfort. Examples of such restrictive clothing can be seen in
the hooped and corseted gowns worn by wealthy women in the 18th
century, and in the opulent, heavy robes traditionally worn by royalties. 9. royalty
Such customs send a clear message that the wearer does not need to work
but occupies a position in society that others can be commanded to work for 10. where
them.
IV. (5 points)
1. away 2. in 3. down 4. by 5. out
C. READING (30 points)
I. (10 points)
1. D 2.A 3.C 4.D 5.A 6.B 7.D 8.B
9.A 10.D 11.C 12.C 13.D 14.A 15.C
II. (5 points)
1. B 2. A 3. B 4. A 5. A
III. (10 points)
1. guts/courage 2. aback 3. left 4. night 5. introvert
6. befriend 7. more 8. dumped/left 9. very 10. only
IV. (5 points)
1. C 2. B 3. D 4. A 5. E
D. WRITING (30 points)
I. (5 points)
IX. It came as no surprise to me that Harry had failed his driving test.
X. She wasn’t so deaf/hard of hearing that she couln’t hear the phone.
XI. According to the salesman my new car would/will be delivered next Wednesday.
XII. It is not (very) likely (that) they will succeed.
XIII. This affair is no concern/business of yours.
II. (5 points)
XIV. My cat has gone/is OFF its food.
XV. Gerald was (always) SHORT of money until/before he married that rich
businesswoman.
XVI. One never KNOWS how he is going to/will/may/might react.
XVII. There are several categories of people who are EXEMPT from (paying) the new tax.
XVIII. Our company has got (the/a) MONOPOLY of/on/over the importation of these
chemicals.
III. (20 points)
XIX. Content: (35% of total mark)
a. Providing all main ideas and details as required
b. Communicating intentions sufficiently and effectively
XX. Organization & Presentation: (30% of total mark)
a. Ideas are well organized and presented with coherence, cohesion, and clarity
b. The essay is well-structured
XXI. Language: (30% of total mark)
a. Demonstration of a variety of vocabulary and structures appropriate to the level
of English language gifted upper-secondary school students
b. Good use and control of grammatical structures
XXII. Handwriting, punctuation, and spelling
(5% of total mark) a. Intelligible handwriting
b. Good punctuation and no spelling mistakes
Markers should discuss the suggested answers and the marking scale thoroughly before marking the
paper.
THE END
SỞ GD&ĐT VĨNH PHÚC KỲ THI CHỌN HSG THPT NĂM HỌC 2018 – 2019
TRƯỜNG THPT LIỄN SƠN ĐỀ THI MÔN: TIẾNG ANH
(Thời gian làm bài 150 phút, không kể thời gian giao đề)

. PHONETICS (10 points)


Question 1: Pick out the word whose underlined part is pronounced differently from those of
the other words. Write your answers in the numbered blanks provided below (5p)
01. A. random B. canal C. many D. explanatory
02. A. good B. moon C. food D. balloon
03. A. provide B. product C. production D. procedure
04. A. kite B. bite C. Christian D. Christ
05. A. voltage B. voyage C. massage D. dosage
Question 2: Choose the word whose stress pattern is differently from those of the other words.
Write your answers in the numbered blanks provided below (5p)
06. A. argumentative B. psychological C. contributory D. hypersensitive
07. A. atomic B. brilliant C. determined D. ambitious
08. A. trigonometry B. explanatory C. immediately D. democracy
09. A. photograph B. payroll C. accent D. regretful
10. A. majority B. ceremony C. astronomy D. investiture
B. GRAMMAR & VOCABULARY (60 points)
Question 3: Give the correct tense/ form of the verbs in the brackets. Write your answers in the
numbered blanks provided below (30p)
a. It’s time we (11. go). If we (12. not leave) now, we (13. miss) the train.
b. I know I (14. not write) to you before, but I (15. be) so busy recently that I (16. not have) time for
writing letters. I (17. telephone) you instead, but I (18. forget) your number.
c. Why didn’t you tell me you could lend me the money? I (19. not borrow) it from the bank.
d. He is walking (20. hurry) to the place where his ex-wife, a famous naturalist now (21. lie) (22. bury)
among the animals she (23. love) so much.
e. We will see Alice at the corner, she (24. wait) for us when we (25. arrive).
f. He resents (26. be) (27. tell) what (28. do).
g. Her mother (29. go) abroad last month, so it (30. not be) her you see at the theater last Sunday.
Question 4: The passage below contains 10 errors. IDENTIFY and CORRECT them. (0) has been
done as an example. Write your answers in the numbered blanks below (20p)

Question 5: Fill in each blank with a suitable PREPOSITION or PARTICLE. Write your answers in
the numbered blanks provided below the passage. (10p)
XXIII. We’re all very obliged_________ you
XXIV. When the wound has healed_________ you can remove the plaster.
XXV. Without a fridge, fresh food will go_________ very quickly.
XXVI. I’m faithful_________ my principle.
XXVII. I haven’t heard_________ Mandy since she wrote in July.
XXVIII. This service is free_________ charge.
XXIX. They went ahead contrary_________ my advice.
XXX. The weather was fine, and everyone was making_________ the coast.
XXXI. We’re going_________ Tom’s car tomorrow.
XXXII. Have a card_________ your sleeve.

C. READING (40 points)


Question 6: Read the following passage and choose the best answer. Write your answers in the
numbered blanks provided below the passage. (20p)
In addition to the great ridges and volcanic chains, the oceans conceal another form of undersea
mountains: the strange guyot, or flat-topped seamount. No marine geologist even suspected the
existence of these isolated mountains until they were discovered by geologist Harry H. Hess in 1946.
He was serving at the time as naval officer on a ship equipped with a fathometer. Hess named these
truncated peaks for the nineteenth-century Swiss-born geologist Arnold Guyot, who had served on the
faculty of Princeton University for thirty years. Since then, hundreds of guyots have been discovered in
every ocean but the Arctic. Like offshore canyons, guyots present a challenge to oceanographic theory.
They are believed to be extinct volcanoes. Their flat tops indicate that they once stood above or just
below the surface, where the action of waves leveled off their peaks. Yet today, by definition, their
summits are at least 600 feet below the surface, and some are as deep as 8,200 feet. Most lie between
3,200 feet and 6,500 feet. Their tops are not really flat but slope upward to a low pinnacle at the center.
Dredging from the tops of guyots has recovered basalt and coral rubble, and that would be expected
from the eroded tops of what were once islands. Some of this material is over 80 million years old.
Geologists think the drowning of the guyots involved two processes: The great weight of the volcanic
mountains depressed the sea floor beneath them, and the level of the sea rose a number of times,
especially when the last Ice Age ended, some 8,000 to 11.000 years ago.
XXXIII. What is the author's main purpose in writing this passage?
I. To trace the career of Arnold Guyot.
II. To describe feature of the undersea world.
III. To present the results of recent geologic research.
IV. To discuss underwater ridges and volcano chains
XXXIV. The word “conceal” is closest in meaning to which of the following?
A. Contain B. Erode C. Hide D. Create
XXXV. The passage implies that guyots were first detected by means of________.

A. a fathometer B. computer analysic C. a deep-sea diving expedition D.


research submarines
XXXVI. The author indicates that Arnold Guyot________.
A. was Harry Hess's instructor B. invented the fathometer
C. named the guyed after himself D. taught at Princeton University
XXXVII. What does the passage say about the Arctic Ocean?
I. The first guyot was discovered there.
II. No guyots have ever been found there.
III. There are more guyots there than in any other ocean.
IV. It is impossible that guyots were ever formed there.
XXXVIII. The author states that offshore canyons and guyots have which of the
following characteristics in common?
I. Both are found on the ocean floor near continental shelves.
II. Both present oceanographers with a mystery.
III. Both were formed by volcanic activity.
IV. Both were, at one time, above the surface of the sea.
XXXIX. According to the passage, most guyots are found at a depth of________.
A. less than 600 feet. B. between 600 and 3,200 feet.
C. between 3,200 and 6,500 feet D. more than 8,200 feet
XL. Which of the following is closest in meaning to the word “rubble”?
A. Fragments B. Mixture C. Columns D. Core
XLI. According to the passage, which of the following two processes were involved in
the submersion of guyots?
I. Erosion and volcanic activity.
II. The sinking of the sea floor and the rising of sea level.
III. Mountain building and the action of ocean currents
IV. High tides and earthquakes
XLII. According to the passage, when did sea level significantly rise?
A. In 1946 B. In the nineteenth century
C. From 8,000 to 11,000 years D. 80 million years ago
Question 7: Read the following passage and choose the best answer. Write your answers in the
numbered blanks provided below the passage. (20p)
As Christmas evolved in the United States, new customs were adopted and many old ones were
reworked. The legend of Santa Claus, for example, had origins in Europe and was brought by Dutch
settlers to New York in the early 18th century. Traditionally, Santa Claus - from the Dutch Sinter Klaas
- was depicted as a tall, dignified, religious figure riding a white horse through the air. Known as Saint
Nicholas in Germany, he was usually accompanied by Black Peter, an elf who punished disobedient
children. In North America he eventually developed into a fat, jolly old gentleman who had neither the
religious attributes of Saint Nicholas nor the strict disciplinarian character of Black Peter.
Santa’s transformation began in 1823, when a New York newspaper published the poem A Visit
from Saint Nicholas, which Clement Clark Moore had written to amuse his daughter. The poem
introduced many Americans to the story of a kindly saint who flew over housetops in a reindeer-drawn
sleigh. Portraits and drawings of Santa Claus by American illustrator Thomas Nast further strengthened
the legend during the second half of the 19th century. Living at the North Pole and assisted by elves, the
modern Santa produced and delivered toys to all good children. By the late 19th century he had become
such a prominent figure of American folklore that in 1897, when Virginia O’Hanlon wrote to the New
York Sun newspaper asking if Santa was real, she received a direct answer: “Yes, Virginia, there is a
Santa Claus”.
XLIII. Who brought the legend of Santa Claus to the USA according to the passage?
A. Sinter Klaas B. Saint Nicholas C. A German D. Dutch settlers
XLIV. Santa Claus was traditionally described as a________.
A. tall man who could walk through the air. B. fat, jolly, old man.
C. religious figure. D. fat man riding a white horse.
XLV. Santa Claus in North America was depicted as________.
I. a man with the strict disciplinarian character of Black Peter.
II. a good old man with less religious character.
III. one with religious attributes of Saint Nicholas.
IV. a jolly man on horseback.
XLVI. Who was Black Peter?
A. an elf accompanying Saint Nicholas. B. an elf who rode a white horse.
C. one of the disobedient children. D. a popular traditional figure.
XLVII. What word is closest in meaning to “attributes”?
A. symbols of a person B. natural qualities C. effects D. outer appearance
XLVIII. Where did the legend of Santa Claus come from?
A. the North Pole B. Europe C. North America D. the City of New York
XLIX. 1823 was mentioned as a year when________.
I. Clement Clark Moore wrote his first poem
II. Clement Clark Moore’s poem made him popular
III. Saint Nicholas visited New York
IV. the image of Santa Claus was transformed
L. According to Clement Clark Moore’s poem
I. Santa Claus had nothing different in appearance from the traditional one.
II. Santa Claus had wings and could fly.
III. Santa Claus liked poetry.
IV. Santa Claus was a kindly saint who flew over housetops in a sleigh.
LI. The answer “Yes, Virginia, there is a Santa Claus” is an illustration for the fact
that________.
I. the New York Sun was popular with children.
II. Santa Claus was a prominent figure at that time.
III. newspapers are unreliable.
IV. Virginia O’Hanlon was a reader of the New York Sun
LII. Which of the following statements is TRUE?
I. Santa Claus was an imaginary old man created by artists based on traditional figures.
II. Living in the North Pole, Santa Claus visited children at Christmas.
III. Santa Claus was a real figure living in northern America.
IV. Santa Claus was a story based on Saint Nicholas and Black Peter.
D. USE OF ENGLISH (30 points)

Question 8: Choose the word that best fits each of the blanks in the following passage. Write your
answers in the numbered blanks provided below the passage. (0) has been done as an example. (10p)
Vietnamese generally shake hands when greeting and parting. Using both hands shows respect
as does a (71)________ bow of the head. In rural areas, elderly people do not extend their hand are
greeted with a bow. Women are more (72)________ to bow the head than to shake hands. Vietnamese
names begin with the family name and are (73)________ by a given name. For example, in the name
Nguyen Van Due, Nguyen is the family name. People address (74)________ by their given names, but
add a title that indicates their perceived (75)________ to the other person. These titles are family
related rather than professional. Among colleagues, for example, the (76)________ of the two might
combine the given name with the title of Anh ("Older Brother"). A(n) (77)________ greeting
combined with the given name and title is Xin chao ("Hello."). Classifiers for gender and familiarity are
also combined with the greeting. In formal meetings, business cards are sometimes (78)________ on
greeting.
Vietnamese people have a strong (79)________ of hospitality and feel embarrassed if they
cannot show their guests full respect by preparing for their arrival. Therefore, it (80)________ to visit
someone without having been invited. Gifts are not required, but are appreciated. Flowers, incense, or
tea may be proper gifts for the hosts. Hosts also appreciate a small gift for their children or elderly
parents
71. A. light B. slight C. lightly D. lighted
72. A. possible B. capable C. probable D. likely
73. A. continued B. chased C. followed D. taken
74. A. each one B. one other C. one the other D. one another
75. A. relation B. relationship C. relation D. relatives
76. A. younger B. most young C. youngest D. young
77. A. easy B. basic C. fundamentally D. elementary
78. A. changed B. transferred C. reserved D. exchanged
79. A. sense B. sensation C. sensitivity D. sensible
80. A. inactive B. inaccurate C. inappropriate D. inexact
Question 9: Fill each blank with ONE suitable word. Write your answers in the numbered blanks
provided below the passage. (0) has been done as an example. (10p)
Cell Phones
Cell phones have been popular in Japan (0. since) the early 1990s, but it was (81)________ until
1999 that their use really took off. The age of cell phones has emerged, but with it come problems.
Cell phones are used on buses and trains, in restaurants, and in all areas of (82)________. They
cause problems when they (82)________ during meetings, concerts, weddings, or even funerals.
What's more, people speak loudly in public, and students read and text messages during lessons.
(83)________ seriously, when a cell phone is used near a person (84)________ a pacemaker to fegulate
his heartbeat, its radio waves may interfere with the functioning of the pacemaker.
Now, something is being done to solve these (86)________. In many places, new technology
is being used to block cell phone calls. Airline (87)________ are requested to stop using cell phones
while on board. Concert halls ask their audience to switch their phones to the (88)________ mode.
However, phone users fear that if they do not (89)________ their phones, they will lose valuable
business opportunities. That's why many do not (90)________ off their phones even when they are
asked to.
Question 10: Write the correct FORM of each bracketed word. Write your answers in the numbered
spaces provided below. (10p)
39. Those (theatre) group has never lost its appeal.
40. Dolphins, (mammal) species, sometimes jump above the surface of the water.
41. The situation is so (chaos) in some countries now that it is difficult to see any solution
42. He’s the most (please), ill-mannered person I’ve ever met.
43. Good friends can enrich the quality of our lives (measure).
44. This famous singer had two (enter) in the Guinness Book of World Records.
45. Beauty is in the eye of the (hold).
46. Computers are now considered (dispense) in the business world.
47. Due to (electric) the difference between urban life and rural life is more and more reduced.
48. A doctor may prescribe (biotic) if the patient has an infection.
E. WRITING (30p)

Question 11: Finish each of the following sentences in such a way that it means the same as
the sentence printed before it (10p).
LIII. Someone stole the old lady’s handbag.
I. The old lady was .........................................................................................
LIV. A true story forms the basis of Mary’s new novel.
I. Mary’s new novel .........................................................................................
LV. If you have completed your test, you can go home.
I. Get ......................................................................................................................
LVI. Absolute secrecy was crucial to the success of the mission.
I. Without ............................................................................................................
LVII. Something must be done to solve this problem.
I. Urgent .......................................................................................... ....................

Question 12: Finish each of the following sentences in such a way that it is as similar its possible in
meaning to the original sentence. Use the word given and other words as necessary. Do not change
the form of the given word (10p).
106. The job received over a hundred applications. (APPLIED)
→ .......................................................................................................................
107. Alan prides himself on his punctuality. (TAKES)
→ .......................................................................................................................
108. Many people attended this year’s festival. (TURNOUT)
→ .......................................................................................................................
109. He is becoming quite famous as an interviewer. (NAME)
→ .......................................................................................................................
110. The whole team was in a happy mood. (SPIRITS)
→ .......................................................................................................................

Question 13: Writing an essay (40p)

It is widely believed nowadays that social knowledge and the ability to adapt in a changing
society make a school-leaver succeed in getting a job.
What do you think about the saying?
Write an essay of around 250 words to prove your points.

_____________THE END_____________
ĐÁP ÁN
A. PHONETICS (10p)
Question 1: (5p) 01. C 02. A 03. B 04. C 05. C
Question 2: (5p) 06. C 07. B 08. A 09. D 10. B
B. GRAMMAR & VOCABULARY (20p)
Question 3: (10p)
11. went 12. don’t leave 13. will miss 14. shouldn’t/ oughtn’t have written
15. have been 16. haven’t had 17. should/would have telephoned 18. have forgot(ten)
19. needn’t have borrowed 20. hurrying 21. lies 22. buried
23. loved 24. will be waiting 25. arrive 26. being 27. told
28. to do 29. went 30. can’t/ couldn’t have been
Question 4: (10p)
31. visiting → to visit 32. no → not 33. right address → the right address
34. their → its 35. up → down 36. had → had been
37. risen → raised 38. extending → extended
39. previous → previously 40. untidy gravel yard → an untidy gravel yard
Question 5: (5p)
41. to 42. over 43. off 44. to 45. from
46. of 47. to 48. for 49. in 50. Up
C. READING (20p)
Question 6: (10p)
51. B 52. C 53. A 54. D 55. B 56. B 57. C 58. A 59. B 60. C

Question 7: (10p)
61. D 62. C 63. B 64. A 65. A 66. B 67. D 68. D 69. B 70. A D. USE OF
ENGLISH (20p)
Question 8: (10p)
LVIII. B 72.D 73.C 74.D 75.B 76.A 77.B 78.D 79.A 80.C
Question 9: (10p)
81. not 82. life 83. ring 84. More 85.using
86. problems 87. passengers 88. silent 89. answer 90. turn/switch
Question 10: (10p)
91. theatrical 92. mammalian 93. chaotic 94. unpleasant 95. immeasurably 96.
entries 97. beholder 98. Indispensable 99. Electrification 100. antibiotics
E. WRITING (30p)
Question 11: (10p)
LIX. The old lady was robbed of her hand bag.
LX. Mary’s new novel is based on a true story.
LXI. Get your test completed/ finished and you can go home.
LXII. Without absolute secrecy this mission would not have succeeded/ been
successful/ been a success/ would have failed.
LXIII. Urgent action is/ measures are necessary/ essential to solve this problem.
Question 12: (10p)
LXIV. Over a hundred people applied for the job.
LXV. Alan takes (a) pride in his punctuality.
LXVI. This year’s festival attracted a record/ high turnout.
LXVII. He is making quite a name for himself as an interviewer.
LXVIII. The whole team was in good spirits.
Question 13: (20p) ESSAY WRITING

___________THE END__________
SỞ GIÁO DỤC VÀ ĐÀO TẠO HÀ NỘI
KÌ THI CHỌN HỌC SINH GIỎI THÀNH PHỐ
ĐỀ CHÍNH THỨC LỚP 12 NĂM HỌC 2016 - 2017
Điểm GK1 GK2 Môn thi: TIẾNG ANH
Ngày thi: 14 tháng 9 năm 2016
Bằng số: Thời gian làm bài: 180 phút

Bằng chữ: Số phách:

I. LEXICO-GRAMMAR (10.8 pts)


a. Choose the best option A, B, C or D to complete the sentences.
1. She swam strongly and_______ cross the river easily, even though it was swollen by the heavy rain.
A. used to B. was able to C. could D. was supposed to
2.The government has spent one million pounds on an advertising ______ to encourage energy
conservation.
A. campaign B. promotion C. operation D. competition
3. Toxic chemicals are one of the factors that lead wildlife to the ______ of extinction.
A. wall B. fence C. verge D. bridge
4. ______ the best of my knowledge, this system cannot work automatically.
A. For B. To C. Within D. In
5. The aim of ASEAN is to promote closer economic_______.
A. delegation B. integration C. reputation D. migration
6. The International Committee of the Red Cross is a private _______ institution founded in 1863 in Geneva,
Switzerland.
A. human B. humanity C. humanization D. humanitarian
7, Affected by the Western cultures, Vietnamese young people's attitudes _______ love and marriage have
dramatically changed.

A. for B. with C. through D. towards


8. How do you speak the fraction 2/5?
A. two-five B. two-fifths C. second-fifths D. two-fifth
LXIX. Don’t be late for the interview, _______ people will think you are a disorganized person.
A. unless B. otherwise C. if not D. or so
LXX. You may borrow as many books as you like, provided you show them to ______ at the
desk.
A. whoever B. who C. whom D. which
11. It is essential that your luggage _______ thoroughly before delivery.
A. to be checked B. being checked C. be checked D. should be checking
LXXI. The reason why this game attracts so many youngsters is that ___________ other video
games, this one is far more interesting.
A. comparing to B. in compared with
C. on comparison to D. in comparison with
13. Take the shortcut round the church _______ late for school.
A. in order not be B. in order that we won't be
C. so that not to be D. so that we couldn't
14.A cooperative program between two companies building a famous ancient city, into
a(n) _______ city has proceeded smoothly since it started in September last year.
A. friendly ecology B. ecology-friendly
C. friendly-ecological D. ecological-friendly
15. Doctor Pike ______ the hospital after he ______an uneventful evening on duty. He
______ of his day of rest.
A. was leaving / has had / thought B. left / had had / was thinking
C. will leave / had / will think D. is leaving / will have / thinks
16. Something _____ immediately to prevent teenagers from _____ in factories and mines.
A. should be done /being exploited B. we should do /exploiting
C. should do / be exploited D. should have done /exploited
b. Choose the best option A, B, C or D to indicate the word or phrase that is CLOSEST in meaning to the
underlined part in each of the following questions.
1.I knew he was only flattering me because he wanted to borrow some money.
A. making me impatient B. praising me too much
C. making me feel worse D. elevating me
2.We spent the entire day looking for a new apartment.
A. all day long B. all long day C. the long day D. day after day
LXXII. He testified that he had seen the man leaving the building around the time of the
murder.
A. said B. provided evidence C. disapproved D. denied
4.She has a deep aversion to getting up in the morning so she usually goes to work late.
A. intense dislike B. feeling C. provocation D. trouble
5.Those who transgress the laws of society will be punished.
A. disagree with B. disperse C. violate D. interfere with
c. Fill a suitable preposition/ particle into each gap to complete the sentences.
LXXIII. She felt a bit dizzy and had to lean _________ the wall before walking on.
LXXIV. Next year, we intend to bring __________ several new products. But at the
moment, we are still testing them.
LXXV. I’m not sure if this is a suitable topic for a research paper. I need to talk it ______ with
my teacher before I start writing.
LXXVI. The store had to lay _______ a number of clerks because sales were down.
LXXVII. “Have you ever heard _________ William Carlos Williams?”
“I believe so. He was a poet, wasn’t he?”
LXXVIII. “Mitchell looked pale and tired.”
“ He told me he was feeling a little _______ the weather.
LXXIX. “Are you going to take that job?”
“No, I decided to pass it _________ because I don’t want to relocate.”
d. Choose and give correct form of the verbs in column A with particles in column B to make phrasal
verbs to complete the sentences. There are some extra.
A B
give fall step go get hold come off down in up through on

LXXX. Let’s hope the rain will ___________ long enough for them to finish the cricket match.
LXXXI. He twisted my arm to make me tell what I knew, but I refused to ___________.
LXXXII. Some people think that the Queen should ___________ and allow the Prince to become King.
LXXXIII. Whenever the subject of holidays _______________, Ruth and her husband have an argument.
LXXXIV. They were planning to hold a Pop Concert in one of the parks but it ___________ owing to
opposition from the local residents.
e. Give the correct form of the words given to complete the sentences.
LXXXV. One is left with the ______________ problem of larger populations requiring more and
more food. (solve)
LXXXVI. The ______________ of the trains and buses causes frustration and annoyance. (frequent)
LXXXVII. Women who are slimming can never enjoy a meal without being afraid
of______________ their diet. (organize)
LXXXVIII. They frequently ______________ the traffic as they march through the streets.
(mobile)
LXXXIX. There will sometimes be silences and pauses in a TV commentary, although these are
becoming ______________ rare. (increase)
XC. More women than men emphasized ______________ and trust; more men than women emphasized
pleasure in a friend’s company, going out with a friend and having a friend in one’s home. (confidence)
XCI. Among the middle-aged men and women, the lack of contact with relatives, even those
who lived nearby, was notable, suggesting that friends and relatives are indeed
____________. (change)
XCII. There is __________ dissatisfaction with the government’s policies. (wide)
XCIII. Welcome to another edition of the club newsletter. A list of ___________ events for
the autumn is being prepared. (come)
XCIV. One of the ____________ of this car is its high petrol consumption. (draw)
f. Fill a suitable word in each space to complete the passage.
The Countryside Agency began the process of designating the South Downs as a National Park in April last year. We
believe that being a National Park is the best way to protect the Downs, build on the achievements of the past to (1)
______________ and enhance the area
in its widest sense for future (2) ______________ . The Downs are under huge and
increasing pressure. The South East is one of the busiest and most pressurised regions (3)
______________ Europe. This means there is (4) ______________ demand for the
development of new homes, roads and industry. There are also more people living and working in the region, which means
more need for people to be able to access beautiful and peaceful countryside to get (5) ______________ from it all. The
Downs already receives
around 35 million visits a year: this number is likely to increase as more and more people live and work in the area.
Designation will bring a body with new ideas and resources specifically focused on (6)
______________ and visitor management, working with others across the whole of the
Park to encourage co-ordination and joint action, and taking action itself where needed. It will be able to manage the
increasing number of visitors so that the Downs themselves are not (7) ______________, but are still a resource everyone
can enjoy. It will also be best
placed to protect and enhance the Downs, so that the qualities so many people love in this special area remain for future
generations. We have two (8) ______________: to identify
a boundary for the proposed National Park and to prepare advice to the Government on the arrangements needed for a
South Downs National Park Authority.
The Agency is launching a widespread public consultation in November 2015 which will
last for three months and give all interested organisations and individuals the (9)
_________ to comment in detail on our initial proposals for the boundary and the
administrative options for the National Park Authority. If you would like to receive a copy of the consultation document
once it is published then please contact us (10)
_____________ the feedback form.
II. READING (5.2 pts)
a. Read the passage and choose the best option A, B, C or D to answer the questions.
Most forms of property are concrete and tangible, such as houses, cars, furniture, or anything else that is included in one's
possessions. Other forms of property can be intangible, and copyright deals with intangible forms of property. Copyright is
a legal protection extended to authors of creative works, for example, books, magazine articles, maps, films, plays,
television shows, software, paintings, photographs, music, choreography in dance, and all other forms of intellectual or
artistic property.
Although the purpose of artistic property is usually public use and enjoyment, copyright establishes the ownership of the
creator. When a person buys a copyrighted magazine, it belongs to this individual as a tangible object. However, the
authors of the magazine articles own the research and the writing that went into creating the articles. The right to make and
sell or give away copies of books or articles belongs to the authors, publishers, or other individuals or organizations that
hold the copyright. To copy an entire book or a part of it, permission must be received from the copyright owner, who will
most likely expect to be paid.
Copyright law distinguishes between different types of intellectual property. Music may be played by anyone after it is
published. However, if it is performed for profit, the performers need to pay a fee, called a royalty. A similar principle
applies to performances of songs and plays. On the other hand, names, ideas, and book titles are excepted. Ideas do not
become copyrighted property until they are published in a book, a painting, or a musical work. Almost all artistic work
created before the 20th century is not copyrighted because it was created before the copyright law was passed.
The two common ways of infringing upon the copyright are plagiarism and piracy. Pla-giarizing the work of another
person means passing it off as one's own. The word plagiarism is derived from the Latin plagiarus, which means
"abductor." Piracy may be an act of one person but, in many cases, it is a joint effort of several people who reproduce
copyrighted material and sell it for profit without paying royalties to the creator. Technological innovations have made
piracy easy, and anyone can duplicate a motion picture on videotape, a computer program, or a book. Video cassette
recorders can be used by practically anyone to copy movies and television programs, and copying software has become
almost as easy as copying a book. Large companies zealously monitor their copyrights for slogans, advertisements, and
brand names, protected by a trademark.

I. What does the passage mainly discuss? A. Legal rights of property


owners
B. Legal ownership of creative work C. Examples of copyright piracy work D.
Copying creating work for profit
49. Which of the following properties is NOT mentioned as protected by
copyright?
A. music and plays B. paintings and maps C. printed medium D. scientific discoveries
3. It can be inferred from the passage that it is legal if ___________.
A. two songs, written by two different composers, have the same melody B. two books, written by two different
authors, have the same titles
C. two drawings, created by two different artists, have the same images D. two plays, created by two different
playwrights, have the same plot and characters
XCV. With which of the following statements is the author most likely to agree? A.
Teachers are not allowed to make copies of published materials for use by their students.
B. Plays written in the 16th century cannot be performed in theaters without permission.
C. Singers can publicly sing only the songs for which they wrote the music and the lyrics.
D. It is illegal to make photographs when sightseeing or traveling.
XCVI. The phrase "infringing upon" is closest in meaning to _________.

A. impinging upon B. inducting for C. violating D. abhorring


XCVII. The purpose of copyright law is most comparable with the purpose of which
of the following?

A. A law against theft B. A law against smoking C. A school policy D. A household rule 7. According to the
passage, copyright law is _____________. A. meticulously observed B. routinely ignored C. frequently debated D.
zealously enforced

b. Read the passage and do the tasks followed


At any given time, more than a million international students around the world are engaged in the study of the English
language in a predominantly English-speaking country. The five most popular destinations, in order of popularity, are the
U. S., Britain, Australia, New Zealand, and Canada. The reasons for choosing to study English abroad differ with each
individual, as do the reasons for the choice of destination.
Numerous studies conducted in Britain and the United States show that the country of choice depends to a large extent on
economic factors. While this should not provoke much surprise, careful analysis of the data suggests that students and their
parents are most influenced by the preconceptions they have of the countries considered for study abroad, which, in turn,
influence the amount they or their parents are prepared to outlay for the experience. The strength of international business
connections between countries also gives a good indication of where students will seek tuition. In the main, students tend
to follow the traditional pattern of study for their national group. The United States attracts the most diverse array of
nationalities to its English language classrooms, this heterogeneity being largely due to its immense pulling
power as the world's foremost economy and the resulting extensive focus on U.S. culture. Furthermore, throughout
the non-European world, in Asia and North and South America especially, the course books used to teach English in most
elementary and high schools introduce students to American English and the American accent from a very early age.
Canada also benefits from worldwide North American exposure, but has the most homogenous group of students - most
with French as their first language. Before furthering their English skills, students in Europe study from predominantly
British English material; most Europeans, naturally, opt for neighboring Britain, but many Asian, Middle-Eastern, and
African students decide upon the same route too.

Australia and New Zealand are often overlooked, but hundreds of thousands of international students have discovered the
delights of studying in the Southern Hemisphere. The majority are Asian for reasons that are not difficult to comprehend:
the proximity of the two countries to Asia, (Jakarta, the capital of Australia's closest Asian neighbor, Indonesia, is only
5506 kilometers from Sydney), the comparatively inexpensive cost of living and tuition, and, perhaps of most importance
to many Asian students whose English study is a prelude to tertiary study, the growing awareness that courses at
antipodean universities and colleges are of an exceptionally high standard. In addition, revised entry procedures for
overseas students have made it possible for an increasing number to attend classes to improve their English for alternative
reasons.
Australia and New Zealand have roughly the same mix of students in their language classrooms, but not all students of
English who choose these countries are from Asia. The emerging global consciousness of the late twentieth century has
meant that students from as far as Sweden and Brazil are choosing to combine a taste for exotic travel with the study of
English 'down under' and in 'the land of the long white cloud'. But even the Asian economic downturn in the 1990s has
not significantly altered the demographic composition of the majority of English language classrooms within the region.
Nor have the economic problems in Asia caused appreciable drops in full-time college and university attendances by
Asian students in these two countries. This is partly because there has always been a greater demand for enrolment at
Australian and New Zealand tertiary institutions than places available to overseas students. In addition, the economic
squeeze seems to have had a compensatory effect. It has clearly caused a reduction in the number of students from
affected countries who are financially able to study overseas. However, there has been a slight but noticeable shift
towards Australia and New Zealand by less wealthy Asian students who might otherwise have chosen the United States
for English study.
The U.S. and Britain will always be the first choice of most students wishing to study the English language abroad, and it
is too early to tell whether this trend will continue. However, economic considerations undoubtedly wield great influence
upon Asian and non-Asian students alike. If student expectations can be met in less traditional study destinations, and as
the world continues to shrink, future international students of English will be advantaged because the choice of viable
study destinations will be wider.
1-3. Complete the missing information in the table below
U.S Britain Australia New Canada
Zealand
order of popularity 1st 2nd 3rd 4th 5th
type of English in American (1)……………… (2)……………. not given not given
course books used
in this country
student 1 2 (3)……………… equal 3 5
heterogeneity
(1 = most
heterogenous
5 = least
heterogenous)
4-9. Tick (√ ) in the right column if the statement is True, False or Not Given in the text

True False Not Given


4. Study destination choices are mostly influenced by proximity to home.

5. Students who wish to study business will probably study English


overseas.
6. Students of the same nationality usually make similar study choices.

7. English language classrooms in the U.S. have the widest range of student
nationalities.
8. Standards at Australian and New Zealand tertiary institutions are
improving.
9. Despite the 1990s Asian economic crisis, Asian students still dominate
the English language classrooms of Australia and New Zealand.
c. Read the following passage and choose the best option A, B, C, or D to indicate the correct word(s) for each of
the numbered blanks.
The expression on your face can actually dramatically alter your feelings and perceptions, and it has been proved that
XCVIII. ____ smiling or frowning can create corresponding emotional responses. The idea was
first (2) _______ by a French physiologist, Israel Waynbaum, in 1906. He believed that different
facial (3) _______ affected the flow of blood to the brain, and that this could create positive or
negative feelings. A happy smile or irrepressible laughter increased the blood flow and contributed to
joyful feelings. But sad, angry expressions decreased the flow of oxygen- carrying blood, and created
a vicious (4) _______ of gloom and depression by effectively (5) _______ the brain of essential fuel.

Psychologist Robert Zajonc rediscovered this early research, and (6) ____ that the temperature of the brain could
affect the production and synthesis of neurotransmitters which definitely influence our moods and energy levels. He
argues that an impaired blood flow could not only deprive the brain of oxygen, but create further chemical imbalance by
inhibiting these vital hormonal messages. Zajonc goes on to propose that our brains remember that smiling is associated
with being happy, and that by deliberately smiling through your tears you can (7) ____ your brain to release uplifting
neurotransmitters – replacing a depressed condition with a happier one. People suffering from psychosomatic illness
depression and anxiety states could (8) _____ from simply exercising their zygomatic (9) ____ which pull the corners of
the mouth (10) ____ to form a smile, several times an hour.
1. A. desperately B. determinedly C. deliberately D. decidedly
2. A. put off B. put down C. put by D. put forward
3. A. aspects B. looks C. expressions D. appearances
4. A. cycle B. spiral C. circle D. vortex
5. A. cutting B. starving C. removing D. eliminating
6. A. advises B. wants C. demands D. suggests
7. A. make B. persuade C. give D. decide
8. A. recover B. improve C. benefit D. progress
9. A. muscles B. nerves C. veins D. bones
10. A. to and from B. up and back C. now and then D. up and down
III. WRITING (4.0 pts)
a. Finish each of the sentences in such a way that it means exactly the same as the sentence printed before it.
1. I am having a lot of trouble now because I lost my passport last week.
If I ________________________________________________________________________________________.
2. The hurricane blew the roof off the house.
The house __________________________________________________________________________________.
3. He was sentenced to six months in prison for his part in the robbery.
He received a ________________________________________________________________________________.
4. That rumor about the politician and the construction contract is absolutely false.
There is _____________________________________________________________________________________.
5. I thought about what had happened all those years before.
I cast ______________________________________________________________________________________.
6. There aren’t many other books which explain this problem so well.
In few other books ___________________________________________________________________________.
7. Don’t you think we should ask the price?
Hadn’t _____________________________________________________________________________________?
8. Doris tiptoed up the stairs because she didn’t want to wake anyone up.
To _________________________________________________________________________________________.
9. I’m sure it wasn’t Mr. Bill you saw because he’s in New York.
It can’t _______________________________________________________________________________________.
10. The collision didn’t damage my car much.
Not a great ___________________________________________________________________________________.
b. Complete the second sentence so that it has a similar meaning to the first sentence, using the word given. Do
not change the word given in any way.
1. Most people regard him as the best man for the job. (widely)
_____________________________________________________________________________________________.
2. The suspect could not explain why he had sand on his boots. (account)
_____________________________________________________________________________________________.
3. The best solution was thought of by Sally. (came)
_____________________________________________________________________________________________.
4. You should consider the fact that he hasn’t spoken French for years. (allowances)
_____________________________________________________________________________________________.
5. He stood no chance of passing his driving test. (inevitable)
_______________________________________________________________________________________.

______The end ______


HƯỚNG DẪN CHẤM
ĐỀ THI CHỌN HỌC SINH GIỎI THÀNH PHỐ LỚP 12
NĂM HỌC 2017-2018
MÔN: TIẾNG ANH
Tổng toàn bài: 20 điểm và làm tròn điểm đến 0,25
Nếu thí sinh có câu trả lời khác với đáp án nhưng đúng ngữ pháp, ngữ nghĩa, giám khảo thống nhất
và cho điểm.

I. Lexico – Grammar ( 54 x 0.2 pt = 10.8 pts) d. Choose and give correct form of the
a. Choose the best option A, B, C or D verbs in column A with particles in column
to complete the sentences. B to make phrasal verbs to complete the
XCIX. B. was able to sentences.
C. A. campaign CXXVIII. hold off
CI. C. verge CXXIX. give in
CII. B. To CXXX. step down
CIII. B. integration CXXXI. comes up
CIV. D. humanitarian CXXXII. fell through
CV. D. towards
CVI. B. two-fifths e. Give the correct form of the words
CVII. B. otherwise given to complete the sentences.
CVIII. A. whoever CXXXIII. insoluble/unsolved
CIX. C. be checked CXXXIV. infrequency
CX. D. in comparison with CXXXV. disorganizing
CXI. B. in order that we won't be CXXXVI. immobilize
CXII. B. ecology-friendly CXXXVII. increasingly
CXIII. B. left / had had / was CXXXVIII. confidentiality
thinking CXXXIX. interchangeable
CXIV. A. should be done / being CXL. widespread
exploited CXLI. forthcoming/ upcoming
CXV. C. two exciting new spy CXLII. drawbacks
novels
f. Fill a suitable word in each
b. Choose the best option A, B, C or D to space to complete the passage
indicate the word or phrase that is closest 5. conserve/ protect
in meaning to the underlined part in each 6. generations
of the following questions. 7. in
CXVI. B. praising me too much 8. increasing/ greater
CXVII. A. all day long 9. away
CXVIII. B. provided 10. conservation
evidence 11. damaged
CXIX. A. intense dislike 12. tasks
CXX. C. violate 13. opportunity
14. using
c. Fill a suitable preposition/ particle
into each gap to complete the
sentences.
CXXI. against
CXXII. out
CXXIII. over
CXXIV. off
CXXV. of
CXXVI. under
CXXVII. up
II. READING ( 26 x 0.2 pts = 5.2 pts)
a. Read the passage and choose the best option A, B, C or D to answer the questions.
CXLIII. B. Legal ownership of creative work
CXLIV. D. scientific discoveries
CXLV. B. two books, written by two different authors, have the same titles
CXLVI. A. Teachers are not allowed to make copies of published materials for use by their
students.
CXLVII. C. violating
CXLVIII. A. A law against theft
CXLIX. B. routinely ignored

b. Read the passage and do the tasks followed


l. British 2. not given (NG) 3. (equal) 3
4. F 5. NG 6. T 7. T 8. NG 9. T

c. Read the following passage and choose the best option A, B, C, or D to indicate the correct word(s)
for each of the numbered blanks.
CL. C. deliberately CLV. D. suggests
CLI. D. put forward CLVI. B. persuade
CLII. C. expressions CLVII. C. benefit
CLIII. C. circle CLVIII. A. muscles
CLIV. B. starving CLIX. B. up and back

III. WRITING (4.0 pts)


a. Finish each of the sentences in such a way that it means exactly the same as the sentence printed before
it. (10 x 0.2 pt = 2.0 pts)
CLX. If I hadn’t lost my passport last week, I wouldn’t be having so much trouble now.
CLXI. The house had its/the roof blown off by/in/during/because of the hurricane.
CLXII. He received a six month sentence (in prison) for his part in the robbery.
CLXIII. There is absolutely no truth in that rumor about the politician and the construction
contract.
CLXIV. I cast my mind back to what had happened all those years before.
CLXV. In few other books is this problem explained so well./ will we see this problem so well
explained.
CLXVI. Hadn’t we better ask the price?
CLXVII. To avoid waking anyone up, Doris tiptoed up the stairs.
CLXVIII. It can’t have been Mr. Bill you saw because he’s in New York.
CLXIX. Not a great deal of damage was done to my car in/by the collision./
Not a great amount of damage was caused to my car in/by the collision.
b. Complete the second sentence so that it has a similar meaning to the first sentence, using the word
given. Do not change the word given in any way. (5 x 0.4 pt = 2.0 pts)
CLXX. He is widely regarded as the best man for the job.
CLXXI. The suspect could not account for the sand on his boots /
The suspect could not account for the fact (that) he had sand on his boots.
CLXXII. Sally came up with the best solution.
CLXXIII. You should make allowances for the fact that he
hasn't spoken French for years. You should make allowances for
his not speaking French for years.
CLXXIV. It was inevitable that he would fail his driving test.

______The end ______


SỞ GD&ĐT VĨNH PHÚC KỲ THI CHỌN HSG LỚP 12 - THPT NĂM HỌC 2017 - 2018
TRƯỜNG THPT LIỄN SƠN ĐỀ THI MÔN: TIẾNG ANH
(Thời gian làm bài 150 phút, không kể thời gian giao đề)
Part I: Choose the best answer among A, B, C or D.
1. Our holiday was_________ by the weather.
A. spoilt B. damaged C. overcome D. wasted
2. The_________ charged by the architect for the plans of the new building were unusually high.
A. hire B. price C. fees D. sum
3. He_________ his son of the dangers of driving too fast in his new car
A. warned B. remembered C. threatened D. concerned
4. The child was_________ by a lorry on the safety crossing in the main street.
A. knocked out B. run across C. run out D. knocked down
5. When Ali arrived in London he spent all his time_________ and visited all the important
museums and buildings.
A. sight-seeing B. traveling C. looking D. touring
6. If you want a cheap air ticket you must_________ well in advance.
A. book B. engage C. reserve D. buy
7. His sister was full of_________ for the way in which he had so quickly learned to drive a car.
A. pride B. admiration C. surprise D. jealousy
8. He asked if we would_________ to share the room.
A. accept B. consider C. agree D. approve
9. I wondered whether you would like to_________ to the theater tomorrow.
A. visit B. go away C. go out D. walk out
10._________ I would like to say how pleased I am to be here.
A. Primarily B. Foremost C. Earliest D. First
11, The independent arbitrator managed to_________ the confrontation between the union and
the employers.
A. refuse B. confuse C. refute D. defuse
12. When I heard the footsteps behind me I was_________ that I would be attacked.
A. horrified B. terror-struck C. terrorized D. terrified
13. His illness made him_________ of concentration.
A. incompetent B. unable C. incapable D. powerless
14. Has the committee_________ a decision yet?
A. done B. made C. arrived D. voted
15. I am a bit hungry. I think_____________ something to eat.
A. I’ll have B. I’ll be having C. I’m going to have D. I’m having
16. What do you plan to do when you_____________ your course at college?
A. finish B. will finish C. have finished D. is going to finish
17. Where_____________? Which hairdresser did you go to?
A. did you cut your hair B. have you cut your hair
C. did you have cut your hair D. did you have your hair cut
18, ‘Shall I stay here?’ ~ ‘I’d rather_____________ with us’.
A. you come B. you to come C. you would come D. you came
19. I_____________ saying what I think.
A. believe B. believe in C. believe for D. believe when
20. Somebody ran in front of the car as I was driving. Fortunately I_____________ just in time.
A. could stop B. could have stopped C. managed to stop D. must be able to stop
Part II: The passage below contains 10 mistakes. Underline the mistakes and write their
correct forms in the space provided in the column on the right. (0) has been done as an
example.
Traditional, mental tests have been divided into two types. 0. traditional → traditionally
Achievement tests are designed to measure acquiring skills and 1. _____________________
knowledge, particularly those that have been explicitness taught. 2. _____________________
The proficiency exams required by few states for high school
17. __________
graduation are achievement tests. Aptitude tests are designed
___________
and measure a person’s ability to acquire new skills but 18. __________
___________
knowledge. For example, vocation aptitude tests can help you
19. __________
decide whether you would do better like a mechanic or musician. ___________
20. __________
However, all mental tests are in some sense achievement tests
___________
because they assumption some sort of past learning or 21. __________
___________
experience with certainly objects, words, or situations. The
22. __________
difference between achievement and aptitude tests is the degree ___________
23. __________
and intention use.
___________
24. __________
___________
Part III: Give the correct form of the words in brackets to fill in the blanks.
Dark black clouds in a dull sky meant one thing and one thing only: there was going to be a
(1.thunder)….. Not one of us had brought an umbrella, or even a (2.rain)……. So when Jack
suggested we should go to a museum, we all agreed immediately. As we had been (3.
shop)………all morning we were now feeling very tired, it would be a (4. pleasant)………...to sit
down. We took a bus and arrived just as large shops of rain were beginning to fall.
The museum was quite (5.desert)……and very peaceful. We sat down in the main hall and listened
to the rain (6. beat)…….against the windows.
Suddenly, there was a great (7. disturb)……..at the (8. enter)……… a large party of schoolboys were
(9. lead) ………in by a teacher. The poor man was trying to keep them quiet and threatening to
punish them, but they did not pay the (10.slight)……. attention.
Section B: Reading
Part I: Read the following passage and choose the best answer to fill in the blanks. Fill each
numbered blank with one suitable word from the list given below.
The shark is a meat- eating fish and one of the most feared animals of the sea. Scientists
(1)………… about 250 species of fish as sharks. These fish live in oceans (2)………..the world, but
they are most common in warm seas.
Sharks (3)………….greatly in size and habits. Whale sharks, the largest kind of shark, may grow
60 feet long. A whale shark weighs up to 15 tons, more than twice (4)…………….much as an African
elephant. The smallest shark may (5)…………..only 4 inches long and weigh less than 1 ounce.
Some kinds of sharks live in the depths of the ocean, but (6)……………are found near the surface.
Some species live in coastal waters, but others (7)……………far out at sea. A few species can even
live in (8)…………..water.
All sharks are carnivores (meat- eaters). Most of them eat (9)…………fish, including other
sharks. A shark’s only natural enemy is a large shark. Sharks eat their prey whole, or they tear off
large chunks of flesh. They also (10)………………..on dead or dying animals.
1. A. classify B. divide C. organize D. arrange
2. A. all B. through C. throughout D. over
3. A. grow B. rise C. evolve D. vary
4. A. as B. so C. very D. exactly
5. A. stretch B. measure C. develop D. expand
6. A. some others B. others C. different kinds D. some sharks
7. A. dwell B. exist C. emigrate D. migrate
8. A. fresh B. sweet C. light D. clear
9. A. uncooked B. live C. lively D. alive
10. A. eat B. swallow C. exist D. feed
Part II: Fill in each blank with a suitable word to fill in the blanks.
Vitamins are substances required for the proper functioning of the body. In this century, thirteen
vitamins have been (1)…………
A lack of any vitamins in a person’s body can cause illness. In some cases, an excess of vitamins
can also (2)…………to illness. For example, sailors in the past were prone to (3)…………from scurvy
that is a disease resulting from the lack of vitamin C. It causes bleeding of the gum, loss of teeth
and skin rashes. Sailors suffer from scurvy because they did not eat fruits and vegetables. Fruits
and vegetables (4)…………vitamin C which is necessary for good (5)…………
Vitamin B complex is composed of eight different vitamins. A lack of any of these vitamins will
lead to different (6)…………For instance, a person who has too little vitamin B1 will suffer
from beri-beri, a disease that causes heart problems and mental (7)…………A lack of vitamin B2
results in eye and skin problems while deficiency of vitamin B6 causes problems of the nervous
system. Too little vitamin B12 will cause anemia. The (8)…………that vitamin deficiencies caused
certain diseases led doctors to cure people suffering from these illnesses by giving them doses of
the (9)……...vitamins.
Today, vitamins are (10)…………in the form of pills and can easily be bought at any
pharmacy.
Part III: Read the following passage and answer the questions by choosing the best
answer among A, B, C or D.
Over the past 600 years, English has grown from a language of few speakers to become the
dominant language of international communication. English as we know it today emerged around
1350, after having incorporated many elements of French that were introduced following the
Norman invasion of 1066. Until the 1600s, English was, for the most part, spoken only in England
and had not extended even as far as Wales, Scotland, or Ireland. However, during the course of the
next two centuries, English began to spread around the globe as a result of exploration, trade
(including slave trade), colonization, and missionary work. That small enslaves of English
speakers became established and grew in various parts of the world. As these communities
proliferated, English gradually became the primary language of international business, banking,
and diplomacy. Currently, more than 80 percent of the information stored on computer systems
worldwide is in English. Two thirds of the world’s science writing is in English, and English is the
main language of technology, advertising, media, international airports, and air traffic controllers.
Today there are 700 million English users in the world, and over half of these are nonnative
speakers, constituting the largest number of nonnative users of any language in the world.

1. What is the main topic of the passage?


A. The number of nonnative users of English.
B. The French influence on the English language.
C. The expansion of English as an international language.
D. The use of English for science and technology.

2. English began to be used beyond England approximately.............................


A. in 1066 B. around 1350 C. before 1600 D. after 1600

3, According to the passage, all of the following contributed to the spread of English around
the world EXCEPT .....................................
A. the slave trade B. the Norman invasion
C. missionaries. D. colonization

4. Which of the following statements is NOT true?


A. Most of the information stored on computer systems is in English.
B. Only one thirds of the world’s science writing is in languages other than English.
C. English is the only language used in technology, and advertising.
D. International airports and air controllers use mostly English.

5. According to the passage, approximately how many nonnative users of English are there in the
world today?
A. A quarter million B. Half a million
C. 350 million D. 700 million.
Part IV: Read the passage then choose the best sentences A-K to fill in each gap. There
is one extra sentence which you do not need to use:
BITTER WATER HITS THE BIG TIME
Chocolate, which has its origins in South America, is now part of a multi-million pound worldwide
business.
At Easter, British people spend over $230 million on chocolate. A massive eight per cent of all
chocolate is bought at this time.
(1)____. Although the large scale industrial production of chocolate began in the last
century, the cacao plant was first cultivated by the Aztec, Toltec and Mayan civilizations of
Central America over three thousand years ago.
The cacao tree is an evergreen, tropical plant which is found in Africa, South and Central America,
the West Indies and South East Asia. The fruit of this tree is melon-sized and contains 20-40
seeds. (2)____. In English – speaking countries, they are called cocoa beans. This is a misspelling
from the 17th century when they were also called cacoa and cocao beans.
The Aztecs used cocoa beans as money. (3)____. This is from the world in the Aztec
language, Nahuatl, meaning “bitter water”. (4)____. The Spanish found the drink more palatable
mixed with cinnamon and sugar, but the recipe did not spread to the rest of Europe for another
century. In the late 17th century, chocolate houses were set up in Europe’s capital cities, where
people gathered to drink chocolate.
(5)____. But in 1826, CJ van Houten of the Netherlands invented chocolate powder. (6)____.
The age of the chocolate bar as we know it began in 1847 when a Bristol company, Fry and Sons,
combined cocoa butter with pure chocolate liquor and sugar to produce a solid block that you
could eat. (7)____.
At the turn of the century, the British chocolate market was dominated by French companies. In
1879 the English company Cadbury even named their Birmingham factory Bournville (ville is the
French word for town) in the hope that a little glamour would rub off. But then came Cadbury’s
famous Dairy Milk bar which began life as a Dairymaid in 1905. (8)____.
It seems that, for the time being at least, chocolate intake in Britain has established at about four
bars each week. (9)____. The latest market trick is the so-called “extended line”. This is
when the humble chocolate bar becomes an ice cream, a soft drink or a dessert, to tempt
chocoholics who have grown tired of conventional snacks.
At the other end of the production process, cacao farmers are still feeling the effects of a crash in
cocoa bean prices at the end of 1980s. (10)____. Perhaps you could spare a thought for
them as you munch your next chocolate bars.

A. This was made by extracting most of the cocoa butter from the crushed beans.
B. A Swiss company then introduced milk solids to the process which gave us milk chocolate.
C. They also used them to make a drink called xocoatl.
D. Until the last century, the chocolate drink was made from solid blocks of chocolate which had
to be melted down in hot water.
E. When dried they become cacao beans, which can be used to make chocolate.
F. Clever advertising which associated it with the healthy qualities of milk from the English
countryside quickly established the bar as a rival to the more decadent French brands.
G. British manufacturers include up to 5 per cent vegetable fat in their chocolate, something
forbidden elsewhere.
H. As most cacao farmers operate on a very small scale, many were forced out of business.
I. This has forced manufacturers to look for new ways to attract customers.
J. In Aztec times the chocolate drink was flavored with spices and used on ceremonial occasions
and for welcoming visitors.
K. Only at Christmas do people eat more of the cocoa-based foodstuffs.
Section C: Writing

Part I: Rewrite the following sentences so that they have a similar meaning with the
first one.
1. “Don’t forget to phone the police”, she said
She reminded him .................................................. .................................................. .........
2. It is believed that the man escaped in a stolen car.
The man............................................... .................................................. ...................
3. A small church lies at the foot of the hill.
At the foot .................................................. .................................................. .....................
4. If you changed your mind, you’ll be welcome to join our club .
Were you .................................................. .................................................. ......................
5. We don’t have to do so many things to please him.
It is .................................................. .................................................. .................................
6. I’m sure he didn’t do it by himself.
He................................................ .................................................. ........................................
7. He can’t afford to go to America this summer.
He doesn’t .................................................. .................................................. ......................
8. Timmy has become confident as a result of his success .
Timmy’s success has turned............................................ ................................................
9. I haven't seen my uncle for a long time.
It's a long time .................................................. .................................................. ....................
10. She dances beautifully and she sings sweetly, too.
Not only .................................................. .................................................. .............................
Part II: Finish the second sentence so that it means the same as the first one , using
the word in capital letters which must not be altered in any way :
1. You should think about the price before you decide whether to buy it or not. (consideration)
→………………………………………………………………………………………….
2. People don’t want to buy cars with large engines any more. (call)
→………………………………………………………………………………………….
3. Twenty years ago this region produced twice as much coal as it does now.(halved)
→………………………………………………………………………………………….
4. The prime minister in unlikely to call an early general election. (likelihood)
→………………………………………………………………………………………….
5. The policeman acted quickly and averted an accident (prompt)
→………………………………………………………………………………………….

Part III: Write a composition (300 words) about the following topic:

How do movies and television influence people’s behavior? Use reasons and specific
examples to support your answer.
SỞ GD&ĐT VĨNH PHÚC KỲ THI CHỌN HSG LỚP 12 - THPT NĂM HỌC 2017 - 2018
TRƯỜNG THPT LIỄN SƠN ĐỀ THI MÔN: TIẾNG ANH
(Thời gian làm bài 150 phút, không kể thời gian giao đề)

HƯỚNG DẪN CHẤM


Section A: Grammar & Vocabulary (40điểm)
Part I (1x 20= 20 điểm)
Câu 1 2 3 4 5 6 7 8 9 10
A C A D A A B A C D
Câu 11 12 13 14 15 16 17 18 19 20
D D C B A A D D B C
Part II (1 x 10 =10 điểm)
1. acquiring → acquired 6. vocation → vocational
2. explicitness → explicitly 7. like → as
3. few → a few 8. assumption → assume
4. and → to 9. certainly → certain
5. but → to 10. intention → intended
Part III (1 x 10 = 10 điểm)
1. thunderstorm 2. raincoat 3. shopping 4. pleasure 5. deserted
6. beating 7. disturbance 8. entrance 9. led 10. slightest
Section B: Reading (35 điểm)
Part I ( 1x 10 = 10 điểm)
1. A 2. C 3. D 4. A 5. B 6.B 7.A 8. A 9.B 10.D
Part II (1x 10 = 10 điểm)
1.discovered 2. lead 3. suffer 4. contain 5. health
6. diseases 7. disorders 8. knowledge 9. necessary 10. available
Part III (1x 5= 5 điểm)
1. C 2. D 3. B 4. C `5. C
Part IV (1x 10= 10 điểm)
1.K 2.E 3.C 4.J 5.D 6.A 7.B 8.F 9.I 10H
Section C: Writing (25 điểm)
Part I (1x 10 = 10 điểm)
II. She reminded him to phone the police.
III. The man is believed to have escaped in a stolen car
IV. At the foot of the hill lies a small church
V. Were you to change you mind, you’ll be welcome to join our club.
VI. It is unnecessary to do so many things to please him.
VII. He can’t have done it by himself.
VIII. He doesn’t have enough money to go to America this summer.
IX. Timmy’s success has turned him into a confident person.
X. It’s a long time since I last saw my uncle/ I saw my uncle for the last time.
XI. Not only does she dance beautifully but she also sings sweetly.
Part II (1x 5= 5 điểm)
VII. You should take the price into consi…..
VIII. there is no(longer any)/ there isn’t much call for cars……..
3 .coal production/ the coal produced in this region has been halved in the 20 years.
XII. there is little likelihood. The likelihood………is small of the PM calling an…/that the PM
will call a…….
5. The policeman’s prompt action averted/ the prompt action of the policeman averted……..
Part III (10 điểm) – Giám khảo tự cho điểm, yêu cầu:
KỲ SỞ GIÁO DỤC VÀ ĐÀO TẠO THI HỌC SINH GIỎI LỚP 12 THPT CẤP TỈNH
QUẢNG NAM NĂM HỌC 2017-2018
ĐỀ CHÍNH THỨC Môn thi: TIẾNG ANH
Mã đề thi: 180 Thời gian: 90 phút (không kể thời gian phát đề)
Ngày thi: 29/3/2018
(Đề gồm có 13 trang)

SECTION I: LISTENING
PART 1: You are going to hear a new student, Stefan, talking to an assistant, Anna, at the Student
Union about his membership. Mark the letter A, B, C, or D on your answer sheet to indicate the correct
answer to each of the following questions from 1 to 10.
Question 1: Stefan’s degree programme is _____.
A. a course for international students B. a short course C. postgraduate D. undergraduate
Question 2: He is studying ______.
A. Natural Science B. Marine Construction
C. Road Construction D. Advanced Engineering
Question 3: One of his leisure activities is ______.
A: playing football B. playing computer games C. watching films D. joining a club

Question 4: Stefan’s native language is ______.

C. Italian B. Spanish C. German D. French


Question 5: At present, he is living in ______.
C. a flat B. a house C. a bed-sit D. Hall
Question 6: The students who want to contact Stephan can call him through _______.
A. his own phone number B. the Union’s phone number
C. his Department’s phone number D. his Hall’s phone number
Question 7: According to Anna, students can use the photocopier ______.
A; in the afternoon B. after 1 pm C. before 1 pm D. in the evening

Question 8: Events are usually organised _______.

A. in the hall B. in the Round Theatre

C. in the Conference Centre D. in the Union building


Question 9: The Union officer who is responsible for van hire is _______.
A. the Transport Security B. the Transport Student
C. the Transport Chairman D. the Transport Secretary
Question 10: Union members will be able to get a discount on ______.

A. clothes B. newspapers C. books D. music


PART 2: You will hear part of an interview with a student called Ben Broadley, who is talking about his

idea for creating more open spaces for people to use in cities. Mark the letter A, B, C, or D on your

answer sheet to indicate the correct answer to each of the following questions from 11 to 16.

Question 11: Ben’s research project on local parks focused on______.


A. how much they have changed B. who makes use of them nowadays

C; how they have developed D. why they were created

Question 12: What does Ben tell us about UK city parks in the 20th century?

A. houses were often built on them B. they grew very fast

C; there was less need for them D. no new ones were created

Question 13: Ben was surprised to discover that in the UK in the 21st century______.

A. fewer homes with gardens are being built

B. only richer people can afford to have gardens

C. people prefer to live in apartment blocks

D. More and more people use the underground railway to work

Question 14: The aim of Ben's 'garden-sharing' scheme is to______.

A. ensure younger families have the biggest gardens

B. help younger families financially so that they can have gardens

C. make better use of private gardens

D. force owners of large gardens to let other people use them

Question 15: In order to use a garden in Ben's scheme, families have to______.

A. pay the owners for time spent in it B. help the owners to look after it

C. buy some equipment to use in it D. rent someone to tidy the gardens

Question 16: What has surprised Ben about the scheme?

A. how many garden owners wanted to take part

B. how little damage there has been done to gardens

C. how few problems there have been overall

D. how many people in other areas have taken up the idea

SECTION II: WRITING

Mark the letter A, B, C, or D on your answer sheet to indicate the sentence that best combines each

pair of sentences in the following questions from 17 to 32.

Question 17. You don’t obey the regulations. You may be disqualified.
A. As you are disqualified, you may disobey.

B. Disqualifications may lead to your disobedience of the regulations.

C. Failure to obey the regularions may result in disqualifications.

D. Unless you obey the regulations, you may not be disqualified.

Question 18. The prospects of them surviving are slim. I have my doubts.

A. I’m sure they are likely to survive.

B. Fears are growing as to their chances of survival.

C. There is no doubt about their survival.

D. If they were still alive, I wouldn’t think about it.

Question 19. Mike was offered the job. He was unqualified to do it.

A. Mike was offered the job by reason of being unqualified to do it.

B. Mike was offered the job regardless of his incompetence.

C. If Mike had been offered the job, he would have been qualified to do it.

D. That Mike was offered the job was a compensation for his unqualified to do it.

Question 20. His academic record at high school was poor. He failed to apply to that prestigious institution.

A. He failed to apply to that prestigious institution on account of his poor academic record at high school.

B. His academic record at high school was poor as a result of his failure to apply to that prestigious

institution.

C. Failing to apply to that prestigious institution resulted in his poor academic record at high school.

D. His academic record at high school was poor because he didn’t apply to that prestigious institution.

Question 21. Our computers crashed. This caused all the trouble.
A. That our computers crashed resulted from all the trouble.

B. Owing to all the trouble, our computers crashed.

C. The trouble all stemmed from our computers crashing.

D. Our computers crashing was really annoying.

Question 22. Anna is Ken’s boss. His criticism of her may have a bad effect on him A.

Ken’s criticism of Anna may be rooted in the fact that she is his boss.

B. Ken’s criticism of Anna may be a matter of concern to her now that she’s his boss.

C. Ken’s criticism of Anna may be put into effect because she’s his boss.
D. Ken’s criticism of Anna may rebound on him now that she’s his boss.

Question 23. The Prime Minister set up a committee of financial experts. They were to help him

discuss and formulate new policies.

A. The committee of financial experts set up by the Prime Minister was aimed at helping him discuss

and formulate new policies.

B. The Prime Minister, who is a financial expert, set up a committee to discuss and formulate new policies.

C. A committee consisting of financial experts who were helped by the Prime Minister discussed and

formulated new policies.

D. The Prime Minister set up a committee so that he could help financial experts with new policies.

Question 24. I’m sick of that programme. I’ve watched it too often.

A. I’m not taken aback at that programme because I’ve watched it too often.

B. That programme is out of bounds although I’ve watched it too often.

C. I’ve gone off that programme because I’ve watched it too often.

D. I’m thrown off balance by the programme although I’ve watched it too often.

Mark the letter A, B, C, or D on your answer sheet to indicate the sentence that is closest in

meaning to the sentence given in each of the following questions from 65 to 72.

Question 25. I travel by bus only when I have no alternative.

A. It’s my only alternative to travel by bus.

B. I travel by bus as a last resort.

C. I resort to travel by bus only when I have no alternative.

D. Travelling by bus is my only alternative.

Question 26. Steve came across some high school friends while he was waiting outside the ice cream shop.

A. He met some of his friends by chance while he was waiting outside the ice cream shop.

B. He passed some of his friends while waiting outside the ice cream shop.

C. While he was waiting outside the ice cream shop, some of his friends went past.

D. He met some of his friends while they were outside the ice cream shop.

Question 27. It was your assistance that enabled us to get achievements.

A. If you had assisted us, we could have got achievements.


B. Your assistance discouraged us from getting achievements.

C. We could get achievements with a view to having your assistance.

D. But for your assistance, we could not have got achievements.

Question 28. Twenty years ago, this region produced twice as much coal as it does now.

A. Coal production in this region has doubled in the last twenty years.

B. More coal is produced now in this region than twenty years ago.

C. Coal production in this region has been halved in the last twenty years.

D. This region has produced more coal than twenty years ago.

Question 29. Impressed as we were by the new cinema, we found it rather expensive.

A. The new cinema was more expensive than we had expected.

B. We were not impressed by the new cinema at all because it was rather expensive.

C. The new cinema impressed us because it was rather expensive.

D. We were very impressed by the new cinema, but we found it rather expensive.

Question 30. I was not surprised to hear that Harry had failed his driving test.

A. By having failed his driving test, Harry made no surprise.

B. It came as no surprise to me that Harry had failed his driving test.

C. Harry’s having failed his driving test is not my surprise.

D. If Harry hadn’t failed his driving test, I wouldn’t have been surprised.

Question 31. Martin missed his flight because he had not been informed of the change in flight schedule.

A. Not having missed his flight, Martin was informed of the change in flight schedule.

B. Martin missed his flight, though he had been informed of the change in flight schedule.

C. Not having been informed of the change in flight schedule, Martin missed his flight.

1. Martin had been informed of his flight delay, which was due to the change in flight schedule.

Question 32. Slightly more than twenty-five percent of the students in the class come from Spanish-

speaking countries.

A. Seventy-five percent of the students in the class speak Spanish.

B. A small minority of the students in the class are Hispanic.

C. A considerable proportion of the students in the class are Spanish.


D. The percentage of the students speaking Spanish fell by twenty-five percent.

SECTION III: LEXICO-GRAMMAR

Mark the letter A, B, C, or D on your answer sheet to indicate the underlined part that needs

correction in each of the following questions 35 to 36.

Question 33: The principles used in air-conditioning are basically the same as those used by A B Cthe
human body to cool himself.D

Question 34: Until the invention of the telephone, skyscrapers were not considered very A B
Cpractically.D

Mark the letter A, B, C, or D on your answer sheet to indicate the word(s) CLOSEST in meaning to

the underlined word(s) in each of the following questions from 33 to 34.

Question 35: If I say something odd, it’s because I didn’t sleep at all last night. When I get extremely

tired, I can sometimes get a bit weird.

A. unable to participate B. setting strangely or unusually

C. being too clever D. unable to concentrate

Question 36:The Rev. Dr. Martin Luther King fought to put an end to racial segregation in the

United States.

A. integration B. education C. torture D. separation

Mark the letter A, B, C, or D on your answer sheet to indicate the word(s) OPPOSITE in meaning to

the underlined word(s) in each of the following questions from 39 to 40.

Question 37: He provides the committee with the record of his work and clears himself of the charge of

plagiarizing from Plautus and Nevus.

A. stealing B. searching C. copying D. creating

Question 38: When her boss broke his leg, Julie had to represent the company at the congress, and

she rose to the occasion extremely well.

A. made a bad job of it B. made it a success

C. showed sympathy D. got up the chance

Mark the letter A, B, C, or D on your answer sheet to indicate the sentence that best completes each

of the following exchanges from 37 to 38.

Question 39: John offers to give Mary a lift and they are talking on the street.
John: “______”

Mary: “OK, John. Thank you for giving me a ride home.”


A. Would you like to drive home, Mary?

B. Hey Mary, throw your stuff in the back and ride up front with me.

C. Okay, let’s take me home Mary.

D. Safe trip home, Mary! Catch you later.

Question 40: Ralph is seeing Anna off at the airport.

Ralph: “I hope you’ll have an enjoyable trip. Make sure to bundle up. It’s freezing there.”

Anna: “______”

A. Thank you for talking down on me! B. Thank you, Ralph. I’ll sure do it!

C. It’s too cold to wait. Thank you anyway. D. It sounds like a great idea. I like snow.

Mark the letter A, B, C, or D on your answer sheet to indicate the correct answer to each of the

following questions from 17 to 32.

Question 41:______ warfare duties primarily to males was imperative when combat was hand-to-hand.

A. Assigned B. Assigning

C. They who assigned D. That they were assigning

Question 42:I believe that judges should be independent______ the government.

A. to B. from C. with D. on

Question 43: General Custer was confident of victory despite being vastly______ by the enemy.

A. outnumbered B. outclassed C. overcome D. overtaken

Question 44: The minister refused to______ the figures to the press.

A. leak B. release C. show D. add

Question 45: Steve ______ his chances of passing by spending too much time on the first question.

A. threw out B. threw off C. threw away D. threw in

Question 46: Mr. Nixon refused to answer the questions on the ______ that the matter was confidential.

A. reasons B. excuses C. grounds D. foundations

Question 47: The soldiers walked cautiously through the ______ deserted streets.
A. obviously B. apparently C. probably D. hopefully
Question 48: Since he spoke about the subject so indirectly, it was difficult to see what he was______.
A. getting at B. getting on C. getting in D. getting up

Question 49: ______ that we went swimming.

A. Being a hot day B. Due to a hot day

C. It was a hot day D. So hot was the day

Question 50: Less moderate members of Congress are insisting that changes in the social security

system ______ made.

A. will be B. be C. must be D. are

Question 51: They changed their plan at the last moment. Otherwise, they______ in the plane crash.

A. would be killed B. had been killed

C. were D. would have been killed

Question 52: Only because she had to support her family ______ to leave school.

A. that Alice decides B. does Alice decide

C. did Alice decide D. Alice decided

Question 53: There are very large rooms with______ in this house.

A. beautiful decorated walls B. beautifully decorated walls

C. beautiful wall decorating D. beautiful decorating walls.

Question 54: All but two of the injured were______ from hospital within twenty- four hours.

A. discharged B. released C. sent D. allowed

Question 55: He was awarded a medal in ______ of his services to the Queen.

A. view B. recognition C. regard D. light

Question 56: It’s amazing how Jenny acts as though she and Darren______ serious problems at the
moment.
A. are not having B. hadn’t had C. haven’t had D. weren’t having

SECTION IV: READING

Read the following passage and mark the letter A, B, C, or D on your answer sheet to indicate the

correct answer to each of the questions from 57 to 64.

CAN GIFTS BUY LOVE?

Every day, millions of shoppers hit the stores in full force—both online and on foot—searching frantically

for the perfect gift. Last year, Americans spent over $30 billion at retail stores in the month
of December alone. Aside from purchasing holiday gifts, most people regularly buy presents for other

occasions throughout the year, including weddings, birthdays, anniversaries, graduations, and baby

showers.This frequent experience of gift-giving can engender ambivalent feelings in gift-givers. Many

relish the opportunity to buy presents because gift-giving offers a powerful means to build stronger

bonds with one’s closest peers. At the same time, many dread the thought of buying gifts; they worry

that their purchases will disappoint rather than delight the intended recipients. Anthropologists describe

gift-giving as a positive social process, serving various political, religious, and psychological functions.

Economists, however, offer a less favorable view. According to Waldfogel (1993), gift-giving

represents an objective waste of resources. People buy gifts that recipients would not choose to buy on

their own, or at least not spend as much money to purchase (a phenomenon referred to as ‘‘the

deadweight loss of Christmas”). To wit, givers are likely to spend $100 to purchase a gift that receivers

would spend only $80 to buy themselves. This ‘‘deadweight loss” suggests that gift-givers are not very

good at predicting what gifts others will appreciate. That in itself is not surprising to social

psychologists. Research has found that people often struggle to take account of others’ perspectives—

their insights are subject to egocentrism, social projection, and multiple attribution errors.

What is surprising is that gift-givers have considerable experience acting as both gift-givers and gift-

recipients, but nevertheless tend to overspend each time they set out to purchase a meaningful gift. In the

present research, we propose a unique psychological explanation for this overspending problem— that

is, that gift-givers equate how much they spend with how much recipients will appreciate the gift (the

more expensive the gift, the stronger a gift-recipient’s feelings of appreciation). Although a link

between gift price and feelings of appreciation might seem intuitive to gift-givers, such an assumption

may be unfounded. Indeed, we propose that gift-recipients will be less inclined to base their feelings of

appreciation on the magnitude of a gift than givers assume.

Why do gift-givers assume that gift price is closely linked to gift-recipients’ feelings of appreciation?

Perhaps givers believe that bigger (that is, more expensive) gifts convey stronger signals of thoughtfulness

and consideration. According to Camerer (1988) and others, gift-giving represents a symbolic ritual,

whereby gift-givers attempt to signal their positive attitudes toward the intended recipient and their

willingness to invest resources in a future relationship. In this sense, gift-givers may be motivated to spend
more money on a gift in order to send a “stronger signal” to their intended recipient. As for gift-recipients,

they may not construe smaller and larger gifts as representing smaller and larger signals of thoughtfulness

and consideration.

(Source: The S A T®Copyright 2015 by the College Board)

Question 57. What is the main idea discussed in the passage?

A. Gift-giving, despite its uneconomical downsides, cultivates a positive social process.

B. Gift-recipients are widely acknowledged as considerably experienced in gift-giving.

C. Gifts can serve as implicit signals of thoughtfulness and consideration.

D. Gift-giving may have certain drawbacks alongside its positive qualities.

Question 58. The authors most likely use the examples of gift-giving occasions in paragraph 1 to

highlight the __________.

A. regularity with which people shop for gifts

B. recent increase in the amount of money spent on gifts

C. anxiety gift shopping causes for consumers

D. number of special occasions involving gift-giving


Question 59. In paragraph 1, the word “ambivalent” most nearly means __________.

A. unrealistic B. conflicted C. apprehensive D. supportive

Question 60. The author indicates that people value gift-giving because they feel it ___________.

A. functions as a form of self-expression B. is an inexpensive way to show appreciation

C. requires the gift-recipient to reciprocate D. can serve to strengthen a relationship

Question 61. In paragraph 3, the author indicates that the assumption made by gift-givers may be__________.

A. insincere B. unreasonable C. incorrect D. substantiated

Question 62. The word “convey” in paragraph 4 most nearly means __________.

A. transport B. counteract C. exchange D. communicate

Question 63. In paragraph 4, the author refers to work by Camerer and others in order to ______.

A. offer an explanation B. introduce an argument

C. question a motive D. support a conclusion

Question 64. Which of the following best characterizes the tone of the author?

A. authoritative B. pessimistic C. ironic D. informative


Read the following passage and mark the letter A, B, C, or D on your answer sheet to indicate the

correct answer to each of the questions from 65 to 72.

THE IRISH LANGUAGE TODAY

Until about the seventeenth century, Irish was the normal everyday language of Ireland. At that time, though,

the English who had colonised the country began to impose their own language and the Irish people

gradually accepted this, mostly for economic reasons. All official business was conducted in English and the

British economy dominated the country. This practical motivation to use English grew even stronger when

people began to leave Ireland in vast numbers in the nineteenth century. Families thought that it would

be difficult to move to an English - speaking country such as America, Australia or England if they

knew nothing but Irish.

Nowadays it is estimated that little more than one per cent of the population of the country use Irish as

their daily first language. Even in the areas of the country which are supposed to be Irish speaking, the

use of the language is decreasing. These areas, known officially as the 'Gaeltacht', are mainly in the

remote far west of the country and have a total population of 83,000, of whom nearly all can speak Irish,

although only about 30,000 use it as their normal language of communication. All children in Ireland
have to learn Irish. Until 1973 pupils had to gain a pass in Irish if they were to be awarded their school

leaving certificate. This rule was very unpopular and was dropped. Pupils still have to take Irish for

these examinations but it no longer seems to matter very much if they fail.

Most children in primary schools seem to enjoy their Irish lessons but in secondary schools the situation

is often different. As examination pressure mounts, pupils often find Irish to be boring and irrelevant, as

compared with French or German, which can at least be useful for getting a job. The most surprising

development of recent years has been the rapid rise in the number of state schools in towns that do all

their main teaching in Irish. This is not a scheme imposed by the state but one that has grown up in

response to the demands of parents. Some parents send their children to these schools for patriotic

reasons, believing them to be a defense against the country being swamped with American and English

culture. Many other parents choose these schools simply because they see them as being better than the

English-language schools. They tend to be newer and to have smaller classes and better motivated

teachers. One criticism made of these schools is that they could be distracting pupils from learning

'more useful' modern languages. In fact, in modern languages, as in most subjects, these schools have

results which are better than the national average.

Only time will tell whether the new rise in Irish in the towns will compensate for its decline in the

Irish-speaking areas of the rural west.

( Source: How to pass FCE by Brian Orpet)


Question 65. What do we learn in this passage about Ireland at the beginning of the seventeenth

century?

A. Many people left for America, Australia and England.

B. English had always been the language used in business.

C. The British began to invade the country.

D. People spoke Irish as their everyday language.

Question 66. People who left Ireland thought_______.

A. they needed nothing more than Irish. B. they would find life easy in an English-speaking country.

C. they ought to know some English. D. they should pretend not to be Irish.

Question 67. In the 'Gaeltacht'_______.

A. 83,000 people use Irish as their normal language of communication.


B. almost everybody uses Irish as their first language.

C. 30,000 people do not understand Irish.

D. there is a fall in the number of people using Irish as their first language.

Question 68. Since 1973 pupils in Irish schools_______.

A. have not needed to pass Irish in order to gain their school leaving certificate.

B. have had to drop Irish.

C. have needed to pass Irish in order to gain their school leaving certificate.

D. have not had to study Irish.

Question 69. Why are French and German often more popular than Irish in secondary schools?

A. They ensure that pupils can find a job. B. They are not boring.

C. They can help pupils find work. D. They ease examination pressure.

Question 70. The government introduced schools where teaching was done in Irish because_______.

A. they have better results. B. parents wanted them.

C. pupils in towns do not learn Irish. D. the teachers are more committed

Question 71. Some people have criticized these Irish language schools because_______.

A. they prevent pupils from learning modern languages.


B. they have classes which are too small.
C. their results are only a little better than average.

D. they do not teach any useful subjects.

Question 72. Which of the following would make a suitable title for this passage?

A. Irish - End in Sight. B. Irish - Signs of Hope in the Towns.

C. Irish - A Dead Language. D. Irish - Increasingly Important in the Villages.

Read the following passage and mark the letter A, B, C, or D on your answer sheet to indicate the

correct word or phrase that best fits each of the numbered blanks from 73 to 80.

WARWICKSHIRE – SHAKESPEARE’S ENGLAND

From Romeo and Juliet to Hamlet and Macbeth, the world-renowned Royal Shakespeare Company performs

all year (73) _________ in Stratford-upon-Avon, the birthplace of our most famous literary
figure. With its rolling hills, meandering rivers and canals, picturesque Warwickshire-Shakespeare’s

England is the ideal place for a country escape.

The Tudor house where the great playwright grew up is a shrine for Shakespeare fans all over the world.

You can wander around the rooms and (74) ______ a glimpse of the world that shaped the man. Other

preserved Shakespeare family houses in Stratford-upon-Avon open to the public (75)

________ the homes of his wife, Anne Hathaway, and his mother. A stroll through the pretty town

will take you to the River Avon, where you can take a relaxing boat cruise and let your mind (76)

________ the past.

Nearby, the magnificent Warwick Castle is one of the country’s (77) ______ medieval fortresses. A

lavish interior of state rooms and a great hall is complemented by beautifully landscaped gardens. You

can climb to the top of towers and ramparts to see breathtaking views and watch birds (78)

_____, jousting tournaments and fireball launching.

The market town of Warwick offers a mixture of old and new, with antique sellers, tea shops, fine dining,

and literary and folk festivals. After a day’s sightseeing, you could unwind (79) ____ style at

the Ardencote Manor Hotel and Spa or Wroxall Abbey Hotel and Estate, once (80) _______ to Sir

Christopher Wren.
Question 73. A. up B. about C. round D. down

Question 74. A. give B. put C. make D. get

Question 75. A. inclusive B. include C. including D. included

Question 76. A. grow into B. drift into C. grow on D. drift on

Question 77. A. better-kept B. best-kept C. better-keeping D. best-keeping

Question 78. A. of prey B. of predator C. on prey D. on predator

Question 79. A. on B. at C. for D. in

Question 80. A. house B. homage C. home D. housing


THE END

SỞ GIÁO DỤC VÀ ĐÀO TẠO ĐÁP ÁN ĐỀ THI HỌC SINH GIỎI TIẾNG ANH LỚP 12
QUẢNG NAM NĂM HỌC: 2017-2018

Mã đề: 180

CÂU ĐÁP ÁN CÂU ĐÁP ÁN CÂU ĐÁP ÁN CÂU ĐÁP ÁN


1 C 21 C 41 B 61 C

2 B 22 D 42 B 62 D

3 B 23 A 43 A 63 A

4 C 24 C 44 B 64 D

5 D 25 B 45 C 65 D

6 B 26 A 46 C 66 C

7 C 27 D 47 B 67 D

8 B 28 C 48 A 68 A

9 D 29 D 49 D 69 C

10 B 30 B 50 B 70 B

11 B 31 C 51 D 71 A

12 C 32 B 52 C 72 B

13 B 33 D 53 B 73 C

14 C 34 D 54 A 74 D

15 B 35 B 55 B 75 B

16 C 36 D 56 D 76 B

17 C 37 D 57 D 77 B

18 B 38 A 58 A 78 A

19 B 39 B 59 B 79 D

20 A 40 B 60 D 80 C

SỞ GIÁO DỤC - ĐÀO TẠO ĐỀ THI CHON HỌC SINH GIỎI


NAM ĐỊNH MÔN: TIẾNG ANH LỚP 12 THPT
Thời gian làm bài: 180 phút
ĐỀ CHÍNH THỨC Đề thi gồm 09 trang

Điểm của bài thi: Họ, tên và chữ kí của 2 giám khảo: Số phách
Ghi bằng số: …………….. Giám khảo 1: ............................................ (Do chủ tịch hội đồng
chấm thi ghi)
Ghi bằng chữ: …………… Giám khảo 2: ............................................
Lưu ý: Học sinh làm bài vào đề thi này
PART A - LISTENING (3/20 points)
HƯỚNG DẪN PHẦN THI NGHE HIỂU
Bài nghe gồm 2 phần, mỗi phần được nghe 2 lần, mỗi lần cách nhau khoảng 15 giây, mở đầu và kết thúc mỗi
phần nghe có tín hiệu.
Mở đầu và kết thúc bài nghe có tín hiệu nhạc. Thí sinh có 3 phút để hoàn chỉnh bài trước tín hiệu nhạc kết
thúc bài nghe.
Mọi hướng dẫn cho thí sinh (bằng tiếng Anh) đã có trong bài nghe.
SECTION 1. QUESTIONS 1 - 10
Questions 1 - 5
Write NO MORE THAN THREE WORDS for each answer.
Example Answer
How long has Caroline been waiting? five minutes
1. What does Matt suggest they do?
…………………………………………………………………
2. In which month is the party going to take place?
…………………………………………………………………
3. Where is Nikki?
…………………………………………………………………
4. How long is Matt going to be away for?
…………………………………………………………………
5. What day of the week is the party going to be held?
…………………………………………………………………
Questions 6 - 10
Circle the correct

letters A - C N

6. The location of
Nikki’s house B A
on the map is River
C
A33

A. The house has


B. a bridge over the river
C. a large swimming pool
D. gardens leading down to the river.
8, The address is
A. 93 Western Road B. 39 West Road C. 93 West Road
9. The telephone number is
A. 477113 B. 447130 C. 477130
10. Each person has to give
A. Ten pounds B. Refreshments C. A barbecue
SECTION 2. QUESTIONS 11 - 20
Questions 11 - 14
Circle FOUR letters A - F
The Mystery Personality
11, is a French speaker 14. is very rich
15. has played for his national team
12. plays for a well - known club 16. is a famous footballer
13. has got a famous wife
Questions 15 - 19
Use NO MORE THAN THREE WORDS or A NUMBER for each space below
aimed at young people
15.……………………20

connected with sports which involve some


16.…………………………………

after appeal many people offered their


services as 17. ………………………
John Tebbit’s group

Received many offers to use facilities free


18 .…………………………………

some donations over


19.………………………

Question 20.
Circle the appropriate the letter A - D
20. Patrick
A. had an accident three years ago. B. used to excel in archery.
C. has been sponsored by one manufacturer. D. is not able to walk, but is very good at archery.

PART B – PHONETICS (1/20 points)


I. Circle the word whose underlined part is differently pronounced from that of the others. (0.5
points)
1.A. famous B. nervous C. favorite D. mountain
2.A. African B. Japanese C. partner D. Canada
3.A. through B. enough C. rou gh D. tough
4.A. sacred B. crook ed C. plough edD. blessed
5.A. club B. subtle C. climb D. doub t
II. Circle the word that has the stress pattern different from that of the other words. (0.5 points)
1.A. celebration B. reputation C. machine D. scientific
2.A. attend B. dinosaur C. substantial D. cartoon
3.A. intensive B. original C. mature D. injure
4.A. comprehend B. ability C. incredible D. efficiency
5.A. average B. agricultural C. signature D. supervise
PART C – GRAMMAR AND VOCABULARY (5/20 points)
I. Choose the word or the phrase which best complete each sentence. Circle A, B, C, or D to indicate
your option. (3.5 points)
1.You had your house decorated, .............. ?
A. hadn’t you B. didn’t you C. weren’t you D. haven’t you
2. ..............1920 that the cinema really became an industry.
A. It is not until B. It was not until C. Not until D. Until in
3. ..............are a form of carbon has been known since the late eighteenth century.
A. Diamonds B. Because diamonds C. That diamonds D. Diamonds, which
4.“ Sorry, do I know you?”
“ ...........................”
A. No, you don’t B. I’ll be in touch
C. We’ve met before, haven’t we? D. All the best
5.The Masters, one of the most important golf tournaments, .............. every year in Augusta, Georgia.
A. are held B. is held C. held D. hold
6.Because bone loss occurs earlier in women.............., the effect of osteoporosis are more apparent in women.
A. than men do B. than in men C. as men D. similar to men
7.Fast food is very popular. .............., a diet of burgers, pizzas and fried chicken is not very healthy.
A. Consequently B. Moreover C. Unfortunately D. In contrast
8, The type of plant and animal life living in and around a pond depends on the soil of the pond, ..............,
and the pond’s location.
A. what the quality of the water is B. how is the water quality
C. the quality of water D. what is the water quality
9.............
.. the advent of the Industrial Revolution, pollution was virtually unheard of.
A. Previously B. Earlier than C. Formerly D. Prior to
10. If a food label indicates that a food is.............. carbonhydrate, it doesn’t mean that it is good food to eat.
A. mostly B. most C. almost D. the most
11. It was so foggy that the drivers couldn’t.............. the traffic signs.
A. make out B. break out C. keep out D.take out
12. Psychologists have found that the number of social contacts we have.............. only reason for loneliness.
A. are not the B. is not the C. are not an D. is not an
13. Have I told you about .............. the government is now dealing?
A. the problem B. the problem with which
C. the problem that D. the problem to which
14. I was born in Scotland but I .............. in Northern Ireland.
A. grew up. B. raised C. brought up D. rose

15.Some young children suffer from feeling of.............. when they first start school.
A. secure B. security C. insecurity D. insecure
16.The company’s website was overloaded with an unexpected surge of emails .............. information on its
new product line.
A. request B. in which requests C. requesting D. requested
17. I don’t think .............. likes the film. It is so boring.
A. one B. someone C. anyone D. no one
18.Although they are twins, they don’t look..............
A. similarly B. likely C. like D. alike
19.Julie was upset because her job application was .............. twice.
A. turned off B. turned down C. turned out D. turned over
20.Only by working hard .............. your goal.
A. you can achieve B. can you achieve C. you will achieve D. will help you achieve
21.“Shall we eat out tonight?” ”
“..............

A. That’s a great idea. B. That’s understandable


C. It’s very kind of you to invite me. D. You are very welcome.
22...............learning a foreign language, John is smarter than Tim.
A. Comparing B. As far as C. Instead of D. In terms of
23. Thomas is.............. because he can speak both French and Spanish.
A. bilingual B. bilateral C. monolingual D. linguistic
24.Some fresh herbs can greatly change the way food..............
A. smell B. smelly C. smells D. has smell
25.It’s essential that he.............. the contract which they have given him.
A. accept B. accepts C. to be accepting D. accepting
26.It was.............. that I couldn’t put it down.
A. such interesting book B. so interesting a book
C. interesting book D. so interesting
27.I see you have a saw in your hands. .................... you .................... the dry tree branches?
A. Will ... cut off B. do ... cut off
C. Are ... going to cut off D. Will... be cutting off
28.To promote him so quickly, you must have a very high .................... of his ability.
A. view B. opinion C. idea D. feeling
29.Jo was shocked when I disagreed with her. She’s so used to getting her own ...................
A. mind B. way C. opinion D. views
30.All people said that they would come to the annual meeting. However, nobody ................ at last.
A. came up with B. turned up C. took over D. get off
31.Only three of the students in my class are girls, ..............are all boys .
A. others B. other students C. the others D. the other
32...............two fingerprints have ever been found to be exactly the same.
A. No B. Never C. Not D. None
33.............. Bill wanted to have dinner in a Mexican restaurant, the rest of his family prefered to eat at a
pizzeria.
A. Besides B. However C. While D. In spite of
34 ..............at 2 a.m, he was thought to be the suspect.
A. Seeing and leaving B. Seen and left C. Seen leaving D. To be seen leaving
35.If only I ............... about this service before.
A. had known B. could know C. knew D. could have known
B. Circle the letter A, B, C, or D to indicate the word or phrase that is CLOSEST in meaning to the
underlined part in each of the following questions. (0.5 points)
36. Helen Keller, blind and deaf from an early age, developed her sense of smell so finely that she
could identify friends by their personal odors.
A. classify B. communicate with C. describe D. recognize
37.The life boat rescued the crew of the sinking ship.
A. picked up B. provided food for C. saved the life of D. looked for
38.The two bombs exploded simultaneously.
A. accidentally B. all of the sudden C. violently D. at the same time
39.You must apply yourself to your work more.
A. improve B. carried out C. did quickly D. cocentrate on
40.They have modern notions about raising children.
A. opinions B. rules C. standards D. plans
C. Circle the letter A, B, C, or D to show the underlined part that needs correction in
each of the following questions. (0.5 points)
41. The book that you see laying on the table belongs to the teacher.
A B C D
42. Because of refraction, the water in a tank ever looks as deep as it actually is.
A B C D
43. In the early days of jet development, jet engines used great numbers of fuel.
A B C D
44. The principles used in air - conditioning are basically the same as those used by the human body
A B C
to cool himself.
D
45. Some underground water is enough safe to drink, but all the surface water must be treated.
A B C D
IV. Circle the letter A, B, C, or D to indicate the correct answer to each of the following question
(0.5 points)
46. .................. the answer, I would have got full mark.
A. Had I known B. If I have known C. If I knew D. if I would have known
47. .................. theoretical information or practical use.
A. Either the two directions are too vague for B. The directions either are too vague for
C. The directions are too vague either for D. The directions are too vague for either
48. Weakened by successive storms, ..................
a. they find the bridge not safe any more.
b. many people thought the bridge had not been safe.
c. the bridge can be no longer safe.
d. it is the bridge that is no longer safe.
49. Such was the demand..................
e. the book had to be reprinted immediately.
f. so that they had to reprint the book immediately.
g. when they had to be reprint the book immediately.
h. that they had to reprint the book immediately.
50..................., driving fast is dangerous.
A. You are experienced B. However experienced you may be
C. Although it is your experience D. As experienced as you are
PART D - READING (6/20 points)
I. Read the following passage and circle the most suitable answer (A, B, C or D) to fill in each gap.
(2.0 points)
MONEY IN SPORT
Today, the distinction between the professional and the (1)…………….is purely a matter of sporting
success. Most athletes start their career by winning competitions for non - professionals before deciding to
(2)………….themselves to their chosen sport. However, for a long time, it used to be believed that getting
paid for a sporting (3)…………..destroyed the ancient Olympic (4)………….of people simply trying to
do
their best for the love of the sport.
In fact, even participants in ancient Olympic (5)………….were able to make large amounts of money
from winning. At the games themselves, only a laurel wreath was (6)………….to the winner, but back in
his
(7)…………..town, he could become very rich.
During most of the 20th century, professionals were disallowed from entering the Olympics. This gave the
wealthy an advantage since they could afford to train and compete without needing to earn money. In
1988, the IOC (which (8) …………..for International Olympic Committee) decided to allow professionals
to
take (9)………..in the Olympics. Only boxing and football still restrict the (10)………….of professionals
allowed to compete. Boxing does not allow professionals at all, while Olympic football teams are allowed
up
to three professionals on the side.
1. A. champion B. umpire C. amateur D. volunteer
2. A. concentrate B. focus C. devote D. aim
3. A. action B. performance C. activity D. recreation
4. A. suggestion B. thought C. belief D. ideal
5. A. sessions B. acts C. plays D. events
6. A. awarded B. earned C. gained D. rewarded
7. A. birth B. native C. original D. home
8. A. stands B. takes C. means D. goes
9. A. place B. account C. part D. sides
10. A. amount B. number C. degree D. quantity
D. Read the following passage and circle the letter (A, B, C or D) to indicate the correct answer to
each of the following questions. (2.0 points)
American Online is one of the big names on the Internet, and unlike many other digital companies, it
actually makes a profit. But the company which its rivals call the “Cyber –cockroach” was launched only
in 1992. Before that, it was a small firm called control Video Corporation, and it made video games. Then
Steve Case, a former Pizza Hut marketing executive arrived and took the company online, innovative, fast
moving, and user - friendly, American online appeals to people who want to search the Internet, but who
do not have a lot of experience. For the same reason “teachies”, people who think they are more expert
with computers, look down on American Online and its users. Recently, American Online (or AOL, as it
calls itself) joined with Time Warn – a multi - million dollar movie and magazine company - to create a
multi media giant.
Now, AOL has begun to expand abroad. In many European countries, including the United Kingdom, it is
hard to buy a computer magazine that does not have a free AOL introductory offer. The company also
puts advertisements onto the television, and employs people to hand out its free introductory disks at
places like train stations. As the Internet gets faster, AOL is changing. With many homes getting high -
speed connection through fiber optic cables, or the new ADSL technology, the “Cyber - cockroach” will
have to show that like real cockroaches, it can survive in almost any environment.
1.What is the passage about?
A. a computer company B. a software company
C. an Internet company D. a video company
2.The word “it” in line……..refers to:
A. American Online B. Cyber - cockroach
C. Control video corporation D. digital company

3. Who does Steve Case work for?


A. Pizza Hut B. AOL
C. Control video Corporation D. None of these
4, How do “teachies” feel about American Online?
A. They think it is a Cyber - cockroach
B. They think it is for experts
C. They think it is a movie and a magazine company
D. They feel superior to its users
5, American Online has the following characteristics EXCEPT:
A. innovative B. experienced C. user - friendly D. fast moving
6. People who use American Online are probably:
A. video game players B. “teachies”
C. movie fans D. people new to the Internet
7. American Online is an unusual digital company because:
A. it used to make video games B. it is innovative
C. it makes money D. is has joined with another company
8. Which marketing idea is NOT mentioned?
A. advertisements on the Internet B. advertisements on TV
C. free disks in journals D. people giving disks away
9. What does the article say about AOL’s future?
A. it will do well B. it will do badly
C. it will face challenges D. The article doesn’t say
10. What is this passage mainly concerned with?
A. technology B. a history of the Internet
C. computer users D. a successful business
3.Read the following passage and circle the letter (A, B, C or D) to indicate the correct answer to
each of the following questions. (2.0 points)
No one can calculate the quantity of solid waste that has been dumped in the world’s oceans, but the total
certainly exceeds many millions of tons. For example, from 1880 to 1895, 75 percent of the solid waste
from New York City was dumped untreated into the Atlantic Ocean. Although it is now prohibited by law,
the dumping of solid waste, including wastewater sludge, industrial waste, and high - level radioactive
waste were common in the United States until 1970. Cruise ships and huge floating fishing factories still
regularly dispose of their solid waste products directly into the ocean.
The earth naturally recycles water and refreshes the land in what is called the hydrological cycle. The
hydrological cycle not only renews the supply of water, but cleans it as well. The process begins as heat
from the sun causes sea water, 97 percent of the earth’s total water reserve, to evaporate and form clouds.
Because water evaporates at lower temperatures than most pollutants, the water vapor that rises from the
seas is relatively pure and free of the contaminants, which are left behind. Next, water returns to us as
rain, which drains into streams and rivers and rushes toward the sea.
Chemicals, petroleum products, and other dangerous substances such as radioactive materials remain in
the ocean, polluting it permanently. The polluted ocean water kills fish or makes them dangerous to eat,
posing health problems for those who consume them. It kills the tiny sea creatures that are the source of
food for larger fish, sharks, and whales. It also spoils a source of great beauty and pleasure when some
solid waste is thrown onto beaches during storms. Discharged petroleum products are frequently found on
beaches and they not only ruin the beach, the petroleum residue kills hundreds of shore birds. Nonpoint
pollutants are dumped into lakes, rivers, and streams that may be far away from any ocean. However,
these pollutants flow, eventually, into the oceans. They can come from a variety of sources, from road salt
to agricultural pesticides. One source of nonpoint pollution is runoff from farming, including fertilizers,
manure, and pesticides. Another source is industrial runoff, including heavy metals, phosphorous, and
many other chemicals. Urban runoff (oils, salts, various chemicals) and atmospheric fallout of airborne
pollution are other sources of nonpoint pollutants that reach the oceans. This includes water and waste
from sinks, toilets, washing machines and bathtubs. The problem with this type of waste is that it provides
massive amounts of nutrients for water plants such as algae, so that they grow rapidly. This sudden growth
causes concentration or algae blooms, which use up the oxygen in the water. As the oxygen level of the
water declines, many organisms suffer and die, and the ocean ecosystem is radically altered. This can be
prevented by the installation of waste treatment plants that prevent waste from entering the sea, but such
facilities do not exist in many poorer countries.

1.What can be inferred about the waste dumped into the world’s oceans?
A. It’s mainly household waste B. Much of it was not treated
C. It is billions of tons D. It is primary industrial waste
2.As can be inferred from the passage, which of the following was NOT disposed into the ocean?
A. waste from dead plants B. waste from cruise ships
C. wastewater sludge D. radioactive waste
3.Which of the following is NOT referred to as use of hydrological cycle?
A. refreshing the land B. separating water from used liquid
C. cleaning water D. renewing the water supply
4.Which of the following has similar meaning to the word “contaminants” in the second paragraph?
A. pollutants B. rushes C. sea water D. vapor
5.What does the word “which” in the second paragraph refer to?
A. rain water B. the earth C. sea water D. water vaporizer
6.Why are chemicals, petroleum products and radioactive materials mentioned in the passage?
A. as untreated waste B. as industrial substances
C. as raw sewage D. as dangerous pollutants
7.Which of the following is NOT referred to as a result of polluted ocean water?
A. killing fish B. harming people who eat sea food
C. spoiling beach beauty D. killing shore birds
8.Which of the following is NOT an example of farming runoff?
A. manure B. pesticide C. road salt D. fertilizer
9. Which of the following can be inferred from the passage?
A. It is expensive to build waste treatment plants
B. developing countries do not need waste treatment plant yet
C. the environment of develop countries is more polluted than that of poor countries
D. the environment of industrial countries is more polluted than that of agricultural countries
10. Where is the passage most probably found?
A. in a geography book B. in a bibliography
C. in a tourism book D. in a social science report

PART E - WRITING (5/20 points)


I. Rewrite each of the following sentences in such a way that it has the same meaning as the first
sentence, using the cues at the beginning of each sentence . (0) has been done as an example.(2.0
points)
0. She was not used to driving on the left.
=> She found it strange to drive on the left.
1. “You damaged my bicycle, John” Mary said.
Mary ..................................................................................................................................................................
2. We’d prefer you not to smoke.
We’d rather .......................................................................................................................................................
3. His jealousy increases with his love for her.
The more ...........................................................................................................................................................
4. The result of the entrance exam was never in doubt.
At no time .........................................................................................................................................................
5. He is very strong but he still can’t lift that box.
Although ...........................................................................................................................................................
6. Most students ignored what the teacher was saying.
Few ....................................................................................................................................................................
7. They said that the explosion had been caused by mine.
The explosion
....................................................................................................................................................
8. They tried hard so that they would pass the exam.
With
...................................................................................................................................................................
9. She discovered eight new comets in the course of her work.
Her work resulted
..............................................................................................................................................
10. She became interested in wildlife conservation, so she joined Greenpeace.
But
.....................................................................................................................................................................
II. Finish the second sentence so that it has similar meaning to the first sentence, using the words given
in the brackets. The words must not be altered in any way. (0) has been done as an example.(1.0
point)
0. I never have enough money. (SHORT)
=> I am always short of money.
1. Their chances of success are small.(UNLIKELY)
It
........................................................................................................................................................................
2. The police ended the fighting between two gangs by arresting the leaders.(CAME)
The fighting
.......................................................................................................................................................
3. The concert wasn’t as good as we expected. (MEET)
The concert
........................................................................................................................................................
4. The boss postponed the meeting for an hour.(PUT)
The boss
............................................................................................................................................................
5. She thought it was too difficult for her to come to the class on time.(FOUND)
She
.....................................................................................................................................................................
III. Paragraph writing (2.0 points)
You should spend about 30 minutes on this task.
Write a paragraph (between 180 to 200 words) about the following topic:
Many students choose to attend school or university outside their home countries. Why do they prefer to
study abroad?
Give reasons for your answer and include any relevant examples from your own knowledge or experience.

THE END
SỞ GIÁO DỤC - ĐÀO TẠO KÌ THI CHỌN HỌC SINH GIỎI
NAM ĐỊNH NĂM HỌC 2014 - 2015
MÔN: TIẾNG ANH LỚP 12
ĐỀ CHÍNH THỨC HƯỚNG DẪN VÀ BIỂU ĐIỂM CHẤM

PART A: LISTENING (3/20 points)

Section 1.
1. have a coffee 2. September 3. on holiday 4. 2/two weeks
5. Saturday 6. A 7. B. 8. C
9. C 10. A

Section 2.
11 - 14: A, B, E and F in any order 15. up to 16. (kind of) expense
17. volunteers 18. of charge 19. £5,000/ 5,000 pounds 20. D

PART B: PHONETICS (1/20 points)


I. 1. D 2. C 3. A 4. C 5. A

II. 1. C 2. B 3. D 4. A 5. B

PART C : VOCABULARY AND GRAMMAR (5/20 points)

I. 0.1 x 35 = 3.5 points


1. B 2.B 3. C 4. C 5.B 6. B 7. C
8. C 9. D 10. A 11. A 12. B 13. B 14. A
15. C 16. C 17. C 18. D 19. B 20. B 21. A
22. D 23. A 24. C 25. A 26. B 27. C 28. B
29. B 30. B 31. C 32.A 33. C 34. C 35.A

II. 0.1 x 5 = 0.5 points


36. D 37. C 38. D 39. D 40. A

III. 0.1 x 5 = 0.5 points


41.C 42. B 43. C 44. D 45. B

IV. 0.1 x 5 = 0.5 points


46. A 47. D 48. C 49. D 50. B

PART III : READING (6/20 points)


Mỗi câu đúng = 0.2 points
I. 1. C 2. C 3. B 4. D 5. D
6. A 7. D 8. A 9. C 10. B

II. 1. C 2. A 3. B 4. D 5. B
6. D 7. C 8. A 9. C 10. D

III. 1. B 2. A 3. B 4. A 5. A
6. B 7. D 8. C 9. A 10. A

PART E: WRITING (5/20 points)


I. 0.2 x 10 = 2.0 points
11. Mary accused John of damaging her bicycle.
12. We’d rather you didn’t smoke.
13. the more he loves her, the more jealous he is.
14. At no time was the outcomeof the entrance exam in doubt.
15. Although he is very strong, he can’t lift the that box.
16. Few students paid attention to/ took notice of what the teacher was saying.
17. The explosion was said to have been caused by mine.
18. With a view to passing the exam, they tried hard.
19. Her work resulted in the discovery of eight new comets.
20. But for her interest in wildlifeconservation, she wouldn’t have joined Greenpeace.
II. 0.2 x 5 = 1.0 point
21. It is unlikely that they will succeed.
22. The fighting between two gangs came to an end when the police arrested the leaders.
23. The concert didn’t meet our expectations.
24. The boss put off the meeting.
25. She found it too diffcult to come to the class on time.
III. Paragraph writing (2.0 points)
The impression mark is based on the following scheme:
50. Format (0.2 points) correct format of a paragraph.
51. Content (1.0 point) provision of main ideas and details as appropriate to
support the argument.
52. Language (0.5 points) a variety of vocabulary and structures appropriate to the
level of English language gifted secodary school students.
53. Presentation(0.3 points)coherence, cohesion, and style appropriate to the level of
English language gifted secodary school students.

You might also like